Section II
Reports

Introduction

The major purpose of this book is to illustrate the use and interpretation of the WJ IV for understanding an individual's learning abilities and disabilities. The fields of school psychology and special education require trained, competent professionals who can integrate qualitative and quantitative information, make accurate diagnoses that capture and explain difficulties, as well as clarify strengths, and propose the most efficacious interventions.

This section provides a variety of sample diagnostic reports that illustrate how the Woodcock-Johnson IV may be used and explained in assessments. The purposes of these diagnostic reports are to help the reader (a) increase understanding of the use and interpretation of WJ IV results, both alone and in combination with other assessment instruments; (b) tailor written reports to the purposes of the evaluation; and (c) use the assessment results to generate recommendations for appropriate supports, adjustments, and interventions. Although the WJ IV can provide helpful information for vocational planning, the focus of this section is mostly on students enrolled in school settings from first grade to postsecondary.

In building a new skill or polishing a frequently used one, a model is often helpful. Rather than “starting from scratch,” one may adopt what fits and adapt what does not. The diagnostic reports are based on real people evaluated by the authors and a few colleagues. For the sake of confidentiality, all names and identifying information have been changed. The diagnostic reports were chosen to illustrate a wide variety of reasons for referral, student ages and backgrounds, educational situations, and diagnoses. Although many different diagnoses are represented here, the types of cases are far from exhaustive.

Occasionally, it is appropriate to report a person's race or ethnicity in a diagnostic report, especially when cultural factors might affect the interpretation of the results. Many school districts indicate a student's race or ethnicity by an ethnic code placed with the identifying information. In the following reports, race or ethnic background has been noted only when this information might have some bearing on the interpretation of the test results.

Much of the diagnostic information for these reports was derived from WJ IV assessments, student work samples, and interviews with parents and/or teachers. In instances in which the evaluator wanted more information, supplementary testing was conducted using other assessment instruments. Where contributing factors outside the expertise of the evaluator were suspected, such as a severe language impairment, attention deficit/hyperactivity disorder, or clinical depression, recommendations are made for further evaluation by an appropriate professional. In some cases, these additional evaluations were incorporated into the initial assessment process. Although this arrangement is often not possible in a school setting, it is preferable to have all pertinent information before writing the final report so that all findings may be integrated. Comprehensive diagnosis of conditions affecting a person's ability to function efficiently in the appropriate setting, such as the school or workplace, makes it possible to write more useful recommendations for enhancing performance. A list of examinees, with their ages, their diagnoses, and the tests used, is presented, to facilitate locating types of cases in which the reader may have specific interest.

Types of Evaluations

Evaluations may be requested for a variety of reasons, leading to different types of assessments and recommendations. Likewise, the setting in which the evaluation takes place, such as a school, hospital, private clinic, postsecondary setting, or vocational training center, may dictate the scope and type of testing done. For a child or adult without a previous diagnosis, a full battery of tests may be administered with language, occupational therapy, medical, and psychological evaluations as indicated. Where continued eligibility for special education services is the primary issue, administration of selected cognitive, oral language, academic clusters of the WJ IV, supplemented by informal testing and work samples, may be sufficient. Where evaluation or documentation of progress in academic skills is needed, the evaluator might decide to administer only the WJ IV Tests of Achievement: Standard Battery, in conjunction with progress monitoring procedures, such as the use of curriculum-based measurements (CBM).

Depth of Information

The purpose and setting of an evaluation also might influence the levels of information sought through testing and interpretation, as well as the amount of detail included in the report. A lengthy report with detailed information does not necessarily signal the complexity of the case, although a person with many difficulties and/or an atypical background may certainly warrant more extensive evaluation and a more complex report. In a private setting, more time may be allotted to each case and more in-depth information presented in the report. In a school setting, practicality may dictate less time for testing, error analyses, and writing reports. An advantage of the school setting, however, is the ongoing opportunity for diagnostic teaching and observation.

The setting in which a person is evaluated, as well as the purpose for evaluation, also might influence the level of diagnosis or the manner in which it is worded. In a clinical or private setting, the principal evaluator may diagnose a specific learning disability or some other disability. In the school setting, the evaluator would be more likely to suggest that the student met certain criteria for a disability and then make a referral to the multidisciplinary team for the final decisions.

Types of Scores

Depending on the purpose of the evaluation and specific information offered within the report, the evaluator might report different types of scores. For example, the purpose of the Grade Band Profiles is to provide information concerning the level of instructional materials to be used for a variety of tasks and represent the range of easy to difficult tasks and materials for each skill area. The grade equivalent (GE) score expresses an approximate instructional level. In using the Grade Band Profile as a guideline, use caution in equating the GE score with the level of instructional materials. Although the GE score shows a person's developmental standing within the normative sample, it may not directly match the level of performance among peers in a specific educational setting. Considering the variability of schools and individual classes across the nation, a third-grade child could receive a GE score 1 year below his or her actual grade level, but be at the same level of instruction as the average third-grader in the specific school. Moreover, different materials intended for a certain grade level may vary considerably from each other in the initial level of student skill and knowledge assumed. The Relative Proficiency Index (RPI) is useful for describing how successful a person will be on similar tasks.

If the evaluation is mainly for purposes of diagnosing a disability and determining eligibility for special education services, common practice is to use standard scores or percentile ranks because these scores express a person's relative standing in a group of age- or grade-peers. Some of the diagnostic reports in this section include both standard scores and percentile ranks. Although both types of scores provide the same information, percentile ranks are easier for many people to interpret, whereas standard scores are necessary for inter-test comparisons and eligibility decisions. CALP scores represent Cognitive-Academic Language Proficiency. They are particularly useful with examinees who are English language learners (ELL) to indicate the proficiency with which they can understand the language used in an educational setting. Some schools or institutions may require stanine scores, normal curve equivalents, or T scores, all of which are available from the WJ IV.

Some of the sample reports have scores embedded in the narrative. Other reports in this section include a notation stating “All test scores are attached to the end of this report.” The actual scores from the WJ IV Online scoring program are included for a few of the reports. Students and evaluators who are new to the WJ IV may want to study the obtained scores and form hypotheses before reading the report.

Recommendations

Reports also differ in scope and specificity of recommendations. In some reports, whether private or school-based, recommendations may be written to cover all areas: cognitive, academic, and behavioral. Other reports might include recommendations only for areas seen as critical for immediate intervention. Reports may refer to general interventions, include suggestions for specific techniques and materials, or defer to recommendations made by the educational team when the evaluation results are discussed.

Discrepancy and Variation Interpretation

The reader may note that some of the children in the reports who were diagnosed as having learning disabilities do not show a significant ability/or achievement discrepancy. The lack of an ability and/or achievement discrepancy does not necessarily indicate the absence of a learning disability. This discrepancy may not be found, because the person's performance on tests of cognitive abilities, particularly related to the academic areas in question, may be low. It is precisely this poor performance on some cognitive abilities relative to others that suggests the presence of a learning disability. In this case, it would be more appropriate to look at the student's intra-individual variations and the Gf-Gc composite rather than the General Intellectual Ability (GIA) composite.

Sources and Uses of Information

The information in these diagnostic reports was based on many types of data gathered from a variety of sources. Information regarding the home environment; physical, emotional, social, and cognitive development; sensory faculties; functioning in the educational setting; previous educational opportunities; results of previous evaluations; and the person's description of the problem provide a basis for interpreting and judging the validity of the new test data. Additionally, full use of information from formal and informal tests includes consideration of scores, behavior during testing, and analysis of error patterns. In some cases, as with some school reports, this information is collected by different people and integrated at the multidisciplinary conference.

Ultimately, the quality and validity of evaluation results for any specific individual depend on the skill of the evaluator in (a) selecting and administering the tests; (b) evaluating the validity of individual test scores in light of possible interfering factors (e.g., fatigue, inattention, reluctance to test, cultural differences, sensory impairments); (c) interpreting and integrating test results; (d) analyzing error patterns; (e) discerning the person's strategies and compensations; and (f) deriving appropriate recommendations. The following sections contain an outline for report writing, as well as an assessment rubric that can be used to evaluate reports of students enrolled in assessment courses.

Outline for Reports

An outline and guidelines are often helpful in structuring reports, although the nature of each assessment will ultimately determine the organization and information you will include. You may use the following suggestions as a starting point:

Identifying Information

You may provide the following in the heading:

  1. Student's name and date of birth
  2. Student's age and grade
  3. Testing and report dates
  4. Examiner's name
  5. School, parents' names, teachers' names (optional)
  6. Ethnicity (optional)
  7. Primary language of the home
  8. Primary language of the student
  9. Parent contact information
  10. Referral source

Some evaluators use a heading format such as this:

Name __________________ School __________________
Date of Birth_______________ Grade __________________
Age __________________ Evaluator__________________
Parents ________________ Report Date __________________
Date(s) of Evaluation _________________________________
  1. Others place the information in one column:
  2. Name
  3. Parents
  4. Date of Birth
  5. Age
  6. Grade
  7. Test Dates
  8. School
  9. Evaluator

Reason for Referral

  1. State the name and role of the referring person or agency (unless there is a reason for not doing so) and the concerns that prompted the referral.
  2. State the specific purpose(s) of the evaluation and write them so that they can be easily interpreted as questions. Your assessment results should answer the referral question(s).
  3. Be concise. This section can usually be written in one short paragraph.

Background Information

  1. Describe current family and living situation.
  2. Summarize information from school records and from outside evaluations. Include information from a variety of sources, including previous educational history (e.g., special services, retentions), the results from prior evaluations and reports (psychological, educational), and relevant interventions.
  3. Include relevant information about medical and developmental history. Include relevant family history (e.g., family history of learning or behavioral problems.) Before scheduling testing, make sure that the student has had recent vision and hearing screenings. If not, request that these be done. This allows the student to have any needed treatment before testing, allows you the opportunity to provide accommodations or modifications, if necessary, and prevents invalid results caused by unrecognized vision or hearing problems that may influence the student's performance.
  4. Describe current behaviors and behavioral history in school and at home (if the reason for referral includes behavioral or social concerns).

Classroom Observations

For elementary and secondary school, classroom observations may be required by the local educational agency).

  1. Schedule your observation during activities in which the student is seen to have problems as well as during an activity (preferably on the same day) in which the student functions well.
  2. State number of students, teachers, and instructional aides in classroom.
  3. Describe class activity and general class demeanor.
  4. Describe the task on which the student was working.
  5. Discuss student's attention and effort, task success, general behavior.
  6. Depending on the referral concerns, you may need to take interval data of time on task and off task.

Assessment Procedures Used

  1. List all of the formal and informal assessments administered. State the full name of the assessment, including the form or edition, followed by the initials or shortened name in parentheses. You can then refer to the test by its initials throughout the remainder of the report. If you administer only selected tests or subtests within a battery, list their names. Alternatively, if there are too many to name, list their numbers.
  2. State the purpose for administering the test used. For example: “Jane was administered selected tests from the Woodcock-Johnson IV (WJ IV) to assess her cognitive abilities related to mathematics and her math skills.”
  3. State adherence to or exceptions to best assessment practices (e.g., ethnicity and race considered in test selection; educational disadvantage or other exclusionary factors were not primary factors in test results; tests were administered in the student's primary language).
  4. State adherence to or deviations from standardized administration (e.g., accommodations, modifications) and the reasons for any deviations.
  5. Describe any cautions or limitations regarding findings (e.g., student's performance may have been negatively affected by impulsivity and attention difficulties).
  6. State the type of norms used for scoring (age or grade).
  7. Explain how the scores reported are interpreted (e.g., Relative Proficiency Index indicates the student's proficiency in similar tasks when grade- or age-peers would achieve 90% success). Or you may provide a table that explains the scores used, such as the following. (Optional)
Standard Score <70 70–79 80–89 90–109 110–119 120–129 >130
Range Very Low Low Low Average Average High Average Superior Very Superior
% of people 2% 7% 16% 50% 16% 7% 2%
Percentile 2 & below 3 to 8 9 to 24 25 to 74 75 to 90 91 to 97 98 & above
RPI range 0/90–3/90 3/90–24/90 24/90–67/90 67/90–82/90 82/90–95/90 95/90–98/90 98/90–100/90 100/90
Proficiency Extremely limited Very Limited Limited Limited to Average Average Average to Advanced Advanced Very Advanced
Instructional Implications Nearly impossible Extremely difficult Very difficult Difficult Manageable Easy Very easy Extremely easy

Behaviors during Testing

  1. Confine the comments in this section to general observations concerning the individual's reaction to the testing situation, as well as his or her general involvement, attention, and motivation throughout the testing. You may discuss his or her reaction to the assessment, general response style, activity level, general attitude, receptive and expressive language, and responses to feedback.
  2. Cite specific examples if possible, such as “he rocked back and forth in his chair several times during the assessment procedure,” or “she asked multiple times, ‘Am I getting the answers wrong?’
  3. Do not label the behavior (e.g., “John was hyperactive during testing”).
  4. Save comments relevant to performance on specific subtests or tasks for the “Results” section.

Intellectual/Cognitive Abilities, Oral Language, and/or Achievement Results and Interpretation

  1. Report scores in text, include in a table within the body of this section, or attach at the end of the report.
  2. Interpret and integrate data rather than discussing each test and score.
  3. Consider findings from a variety of sources (e.g., classroom work samples; diagnostic teaching; teacher, parent, and student interviews).
  4. Separate paragraphs for each cognitive, oral language, and academic area and use bolded subheadings to delineate one from another (e.g., Oral Language, Reading, Mathematics). Use subheadings under each of these, as necessary, to separate long discussions of different aspects of a skill, such as Reading Decoding, Reading Fluency/Rate, and Reading Comprehension.
  5. Analyze responses and errors and provide specific examples to document clinical interpretation and impressions.
  6. Draw conclusions related to the referral question(s).
  7. Include a statement regarding a diagnosis (or no diagnosis) or defer to the school multidisciplinary team for a decision regarding eligibility.
  8. In this section, discuss test results and be sure to include qualitative information. For each area of performance to be discussed: (a) Make a general statement about the student's performance followed by more specific information. (b) Include results from error analyses, particularly if the errors are related to the reason for referral, indicate a problem not previously introduced, or suggest remedial implications.

Organize the discussion as follows: total test scores; composites, clusters, or factors; tests and subtests; and qualitative information (comments, behavior, information from error analyses). You do not have to discuss every test and subtest as long as the information you have obtained from them is included. For example, you might discuss particular strengths and weaknesses and then state that all other areas assessed (list them) were within the expected range or the average range for the examinee. When test scores within a cluster differ significantly, discuss the reason for the difference. Discuss narrow abilities rather than the broad ability represented by the cluster. If working with a multidisciplinary team within a school system, do not state whether a student does or does not have a specific educational classification (e.g., learning disabilities, intellectual disability, autism spectrum disorder) unless the student has been previously classified as having a disability. Instead, make a recommendation that the multidisciplinary team consider eligibility for special services.

Summary (optional)

  1. Briefly summarize test results.
  2. State implication of results (conclusions).
  3. Do not include new information.
  4. Include a statement of the diagnosis (if one has been made) or refer to the multidisciplinary team for consideration of the next step.

Recommendations

  1. Make recommendations specific to the referral question or findings, if different from the referral question.

Further Testing

  1. Recommend further testing to yield more specific information, if necessary.
  2. Recommend further testing for problems that have become evident as a result of the evaluation but were not included in the referral concern (e.g., a student referred for reading problems performs poorly on oral language tests).

Accommodations and Instruction

  1. Develop a list of appropriate accommodations and/or modifications and explain the justification (e.g., the student requires extended time on tests because of a slow reading rate).
  2. List suggestions for instructional approaches and materials.
  3. Base your suggestions on information gathered in this assessment as well as from other sources.

General Stylistic Suggestions

  1. Use the third person (i.e., “the examiner” or “the evaluator”) or first person (“I”). Whichever style is selected, stay consistent.
  2. Use formal language but avoid jargon. Attempt to use terminology that will be familiar to the reader(s).
  3. Capitalize the names of tests, subtests, and clusters. You may capitalize verbal score descriptors (e.g., High Average) or not (e.g., high average), but whichever style is chosen, stay consistent.

Assessment Report Rubric

img img img

Diagnostic Reports

Table 2.1 is a guide to the names of the tests and their acronyms. Table 2.2 provides an overview of the diagnostic reports contained in Section II.

Table 2.1 Test Acronyms

ABAS-II Adaptive Behavior Assessment System, Second Edition
BASC-2 Behavior Assessment System for Children, Second Edition
VMI-6 Beery-Buktenica Developmental Test of Visual-Motor Integration, Sixth Edition
CAS2 Cognitive Assessment System, Second Edition
CBM Curriculum-Based Measures
CTOPP-2 Comprehensive Test of Phonological Processing, Second Edition
CVLT-C California Verbal Learning Test—Children's Version
ERA Early Reading Inventory
GORT-5 Gray Oral Reading Tests—Fifth Edition
KeyMath3 KeyMath-3 Diagnostic Assessment
NEPSY-II Developmental NEuroPSYchological Assessment, Second Edition
QRI-5 Qualitative Reading Inventory, Fifth Edition
RAN/RAS Rapid Automatized Naming and Rapid Alternating Stimulus Tests
RCFT Rey Complex Figure Test
TEC Tasks of Executive Control
TOC Test of Orthographic Competence
TOSWRF-2 Test of Silent Word Reading Fluency, Second Edition
TOVA Test of Variables of Attention
TOWL-4 Test of Written Language, Fourth Edition
WISC-V Wechsler Intelligence Scale for Children—Fifth Edition
WJ IV ACH Woodcock-Johnson IV Tests of Achievement
WJ IV COG Woodcock-Johnson IV Tests of Cognitive Abilities
WJ IV OL Woodcock-Johnson IV Tests of Oral Language
WRAML2 Wide-Range Assessment of Memory and Learning, Second Edition

Table 2.2 Table of Reports by Age of Examinee

Name Age Grade Tests Used Referral Concern Diagnosis Scores Included
Jorge Lopez 6–5 1 WJ IV COG
WJ IV ACH
VMI-6
Attention and behavior problems with lower academic performance. Fatigue has been a problem at school. Attention deficit/hyperactivity disorder—Combined type Yes
Mun Panza 6–6 1 WJ IV COG
WJ IV OL
WJ IV ACH
WISC-V
TOVA
CAS-2
CTOPP-2
ERA
VMI-6
Slow progress in reading and writing Phonological dyslexia
Writing disorder
Attention deficit/hyperactivity disorder, combined presentation
Yes
Kendra Stover 6–8 1 WJ IV ACH
WISC-V
KeyMath 3
Dislikes school; math instruction is insufficient Gifted with possible dyslexia No
Sabrina Adams 7–9 1 WJ IV COG
WJ IV OL
WJ IV ACH
Slow progress in reading and writing Phonological dyslexia
Mathematics disorder
Yes
Delia Sanchez 7–10 2.7 WJ IV COG
WJ IV OL
WJ IV ACH
CVLT-C
NEPSY-II
RCFT
Legal referral: Present level of cognitive functioning and whether current difficulties are related to head injury No diagnosis. TBI caused current difficulties. Yes
Kevin Reid 8–3 2 WJ IV COG
WJ IV OL
WJ IV ACH
WRAML2
Problems with memory and learning Severe deficit in memory along with low performance in all areas assessed. Criteria for SLD is up to the school district. No
Michael Ambrose 8–4 3 WJ IV COG
WJ IV OL
WJ IV ACH
Reevaluation to determine cognitive decline secondary to rare form of epilepsy Overall cognitive and academic decline No
Emma Sanchez 8–11 2.8 WJ IV COG
WJ IV OL
WJ IVI ACH
TOC
TOSWRF-2
BASC-2
Difficulty with all basic academic skills and concentration Orthographic dyslexia, ADHD No
Dustin Greystone 9–0 4.0 WJ IV COG
DAS-II
RCFT
WCST
D-KEFS
VMI-6
TEC
Difficulty regulating attention and emotions; socially immature; problem with writing skills Gifted, ADHD, dysgraphia, developmental coordination disorder No
Simon Schneider 9–4 3.9 WJ IV ACH Three-year review of special education services. Concern regarding ongoing problems in motor development and handwriting. Developmental dysgraphia Yes
Anthony Hall 9–8 3.8 WJ IV COG
WJ IV OL
WJ IV ACH
Limited progress in reading and request for instructional recommendations Limited attention
Reading disability
No
Martha Downing 11–4 5.6 WJ IV ACH
KeyMath 3
Request for updated evaluation to determine current factors interfering with educational development and present academic achievement levels Significant weaknesses in basic math skills and concepts
Fred Armstrong 11–5 6 WJ IV COG
WJ IV ACH
WISC-V
BASC-2
Intelligent student with school placement issues. Parent request for recommendations for educational planning. Intelligent student who is poorly matched with his school Yes
Skylar Swanson 11–11 6.8 WJ IV COG
WJ IV OL
WJ IV ACH
RAN-RAS
TOWL-4
Spelling list—irregular words
Parent requested information regarding current academic needs and recommendations to increase school success Dyslexia, dysgraphia, attention deficit/hyperactivity disorder No
Jennifer Challis 12–4 6.9 WJ IV COG
WJ IV OL
WJ IV ACH
Update cognitive abilities, language, academic skills to document need for special education or 504 plan Gifted, deficits in processing/perceptual speed; speed of lexical access; dysgraphia residuals; continued need for reading decoding and spelling intervention Yes
Brandon Goldberg 12–9 6.9 WJ IV COG
WJ IV OL
WJ IV ACH
Request for current status of cognitive and language abilities and academic skills subsequent to disastrous middle school year. Recommendations for accommodations and instruction. Deficits in processing/perceptual speed and speed of lexical retrieval contribute to continued deficits in basic reading and writing skills, slow math facts. Superior range Gf-Gc. Yes
Glenn Van Houten 13–2 7.4 WJ IV COG
WJ IV OL
WJ IV ACH (Form B)
QRI-5
Diagnostic teaching
Reading comprehension Does not have learning disabilities. Needs reading comprehension strategies. No
Gilbert Welch 14–6 8.7 WJ IV COG
WJ IV OL
WJ IV ACH
RAN/RAS
QRI-5
Low test scores in comparison with peers, especially on standardized tests. Works slowly on homework. No LD but weakness in perceptual speed. No academic weakness. Previous standardized test scores are commensurate with WJ IV scores. No
Annette Hernandez 15–11 9.9 WJ IV COG
WJ IV OL
WJ IV ACH
TOSWRF-2
CBM passages
Request info on level of academic performance and needs Weakness in processing/perceptual speed, phonetic coding, vocabulary No
Anita Velasquez 18–1 10.9 WJ IV COG
WJ IV ACH
ABAS-II
Neuropsychological evaluation to determine cognitive and adaptive abilities, and educational plan Intellectual disability? Yes
Robert Maddox 22 College senior WJ IV COG
WJ IV OL
WJ IV ACH
GORT-5
Update on previous evaluation to apply for extended time on Graduate Record Exam Gifted. Deficits in long-term retrieval and speed of word retrieval affect reading comprehension and speed. Yes
Damon Seldon 44 College sophomore WJ IV COG
WJ IV ACH
Difficulty with math Specific learning disability in math No

Reason for Referral

Jorge was referred for an evaluation to determine his present level of cognitive functioning as well as his cognitive and strengths and weaknesses. His academic performance is low, and he is reported to have problems in attention and behavior. Fatigue at school has also been a concern.

Background Information

Jorge is a 6-year-old boy with a history of hydrocephalus and extremely rare neurological malformations. He has also exhibited problems with academics and behavior. He lives half-time with his mother, stepfather, and baby sister and with his father for the other half. Jorge takes the bus to school, and when he is with his father, he is dropped off at the same bus stop for consistency.

Developmental, Social, and Educational History

Jorge is the product of a healthy pregnancy and birth with a weight of 7 pounds 1 ounce at 39 weeks' gestation. His mother reported that a large brain cyst was discovered at 20 weeks via ultrasound. She also reported that Jorge spent 5 days in the neonatal intensive care unit (NICU) (for monitoring). Dr. Andrea Levin, pediatric neurologist, summarized Jorge's medical history in a recent medical report (4/18/2015): Jorge has “a complex history involving a left occipital porencephalic cyst and left parietal closed lip schizencephaly. He has had a VP shunt that was placed by Dr. Schuster in neurosurgery. His MRIs have been stable, including one in February 2015. A detailed MRI in the past raised concerns regarding the possibility of dysgenesis of the corpus callosum. He has generally done well developmentally and has not had a history of seizures.”

Developmental milestones were reached later than typical peers. He received early intervention from birth until he reached the age of 3 years. This included speech/language, physical therapy, and occupational therapy. Jorge exhibits problems separating from his parents each time there is a change in caregivers, school environment, and classrooms. Jorge sleeps well except when nasal congestion and allergies interfere. Current medication are for allergies (e.g., Claritin or Zyrtec).

Jorge enjoys playing outside and music. He plays the drums, piano, and guitar. He also enjoys Legos, video games, reading, dancing, and being silly. He reportedly has no problems playing with or relating to other children. When asked what role he takes in peer groups, his mother described Jorge as both a leader and a follower. She stated that Jorge would like to make the rules but others do not always follow him. She also reported that Jorge frequently challenges rules and questions them. The following behaviors were noted on a developmental history form: “has a short attention span, at times is easily overstimulated in play, seems overly energetic in play, overreacts when faced with a problem, and requires a lot of parental attention.”

Jorge attended preschool and kindergarten. He struggled with attention in preschool, had difficulty staying seated during circle time, and was behind his peers in learning letters and corresponding sounds. Problems continued in kindergarten, where he struggled to follow directions and pay attention. Jorge's mother reported that his teacher felt like Jorge needed more assistance than other students. He was more successful in the classroom when volunteer aides were available.

Jorge currently attends Grindall Elementary School in the Mesa Heights School District. He is in a multi-age classroom with first and second graders. His mother reported that Jorge has no problems in reading; in fact, he loves to read and demonstrates good fluency and prosody. He exhibits poor handwriting and hates to write. He fatigues easily when writing tasks are required. His mother was unable to report Jorge's current abilities in math, because he has difficulty working independently. Jorge's mother has received numerous emails and phone calls regarding Jorge's behavior. Jorge has cried at school, wanting to take naps, but he does not ask for naps on the weekends. He struggles with transitions and sustained attention. When asked to be at the rug with peers, Jorge will lower his body to the floor, then get up and leave the group after two minutes of “sitting.” Jorge has also exhibited defiant behaviors at school and has been sent to the principal's office more than ten times during the first-grade year.

Observations and Impressions

Jorge was seen for one formal testing session (morning). He refused to come to the testing office without his mother. He then refused to work with the examiner without his mother in the room. Although the room is a comfortable environment with a testing table, couch and chair, games, toys, and a big stuffed bear, Jorge did not want to work without his mother in attendance. The evaluation was conducted as Jorge's mother sat on the couch. Jorge was at the testing table with his back to his mother. Throughout the evaluation, he exhibited significant problems attending to and completing the tasks. He required frequent breaks and much encouragement to complete each task presented. On many occasions, the examiner had to bargain with Jorge to get him to comply. Even with breaks and “bargains,” Jorge often walked away from the testing table while in the middle of a task. During tasks, Jorge fidgeted with pencils and kept handling the testing materials (e.g., playing with test easel). He made noises with his mouth, whistled, stood rather than sat, and generally fidgeted the entire testing time. Even when he was on a break, he moved from game to game and had difficulty sustaining attention on tasks he chose. When tasks became too hard and required effort, he quit, stating, “I can't do that one.” On other challenging tasks, he refused to continue. During one of the breaks, Jorge needed to use the restroom. The examiner offered to go with him and he wanted her to go into the bathroom with him. She refused, stating she would stand right outside the door while he was in the bathroom. He refused to go to the bathroom without “in-the-bathroom” company. His mother attempted to apply the same rule; however, after Jorge refused to use the bathroom without company, his mother went into the bathroom with him.

Jorge is left-handed and has adequate hearing and vision (with glasses, when needed). Overall, his conversational language appeared to be within typical age limits.

Tests Administered

  1. Beery-Buktenica Developmental Test of Visual-Motor Integration, Sixth Edition (VMI)
  2. Woodcock-Johnson IV Tests of Cognitive Ability (WJ IV COG)
  3. Woodcock-Johnson IV Tests of Achievement (WJ IV ACH), Form A (Scores Attached)
  4. Parent Interview
  5. Review of Records

Score Ranges

The following descriptors are used to describe Jorge's performance within the percentile rank (PR) ranges.

Descriptor Percentile Rank Descriptor Percentile Rank
Very Superior 98 or higher Low Average 9 to 23
Superior 91 to 97 Low 2 to 8
High Average 85 to 90 Very Low Below 8
Average 76 to 84

Test Results and Interpretation

Jorge's score in overall intellectual ability fell in the Low Average to Average range compared with his age-peers (WJ IV COG GIA SS = 90; PR = 25). On the seven tests of the WJ IV COG battery, all scores but one were in the Average range, between the 44th and 61st percentiles. The task demands of the tests were: (a) numerical reasoning: figuring out what number was missing in a series of four numbers, (b) receptive and expressive vocabulary: giving synonyms and antonyms for the word the examiner said, (c) phonological processing: demonstrating the ability to recognize the individual sounds in spoken words and rapidly retrieve words based on their sounds, (d) visualization: visualizing how pieces of an abstract design would fit the whole design and how an object would look if shown from a different perspective, (e) short-term working memory: holding a list of words in mental awareness and selecting the one word the evaluator asked for, and (f) long-term retrieval: listening to a story and retelling the key details.

Even though Jorge's performance on the working memory and retrieval tasks were in the Average range, he required frequent encouragement to continue with the tests. Because of his lack of sustained attention to these tasks, it is likely that these scores are an underestimate of his abilities in these areas.

In contrast to these average scores, Jorge's performance on a time-limited test of cognitive processing speed was in the Very Low range (Letter-Pattern Matching: PR = 1). On a page of rows of letters and letter combinations, he was asked to circle the two matching letters or letter combinations in each row. Rather than sustaining attention to the task, he kept looking up. On the few items he completed, he just circled letters at random. Consequently, the results of this test are invalid. As well, because this score was averaged into the GIA, that score should be considered spuriously low.

Jorge also performed in the Low Average range on a test of visual perception in which he was required to identify the exact match for as many shapes as possible (VMI Visual Perception: PR = 18). Jorge's visual-motor integration skills were also in the low average range (VMI Visual Motor Integration: PR = 18). Visual motor integration is the degree to which the hand and fingers can move in coordination with what the person sees. In contrast, Jorge's ability to do a simplified task of drawing from dot to dot to create the shapes he had previously seen was in the Very Low range (Motor Coordination: PR = <1). This weakness in motor skills is most likely attributable to the porencephalic cyst in his left occipital lobe and the left parietal closed lip schizencephaly. Both are known to cause motor deficits, and parietal anomalies are known to cause difficulties with spatial relationships and orientation.

Jorge's handwriting is difficult to read. Letters and words are formed with inadequate spacing. Sizing of letters is essentially the same (e.g., lower case p was the same height as lower case h). Overall, Jorge's ability in visual processing is in the Low Average range, with motor coordination less well developed.

Academic Achievement

All of Jorge's academic scores were solidly in the Average range, between the 41st and the 58th percentiles. The reading tests included reading a list of words, some of which were regular and some irregular; demonstrating comprehension of one- to two-sentence statements by filling in the missing word; and reading a brief expository passage aloud. The writing test involved Jorge writing meaningful sentences, each in response to different instructions. As with other academic areas, Jorge demonstrated average abilities in math computation and word problems.

Attention, Social, and Emotional Functioning

Jorge's mother reported significant difficulties with attention, particularly at school. He has also exhibited some defiant behaviors resulting in visits to the principal. He is unable to sit during circle time for more than two minutes. Transitions are problematic as well. He is unable to follow directions, work independently, and attend to academic tasks. Jorge's mother reported that Jorge does not simply accept rules; rather, he consistently questions things. He overreacts when faced with a problem, and cries, “this isn't fair.” No reports of difficulties with peers were evident from parent or school reports.

During the evaluation, Jorge attempted to cooperate with the examiner; however, when tasks became too difficult or he was bored with the task, he walked away from the testing table or refused to complete the task. Jorge negotiated his desires throughout the evaluation period (e.g., mother in testing room, mother with Jorge in restroom). When Jorge did attempt tasks, he often required frequent prompting to help him maintain attention.

Summary and Conclusions

Jorge is a 6-year-old boy with a history of hydrocephalus secondary to rare brain malformations. In addition, he has a history of attention problems, noncompliance, and fatigue. These test results place his overall cognitive ability in the Average range, which is probably an underestimate because of his limited attention on many of the tests. Of the cognitive abilities assessed, his only scores that were below average (other than the one on which he paid scant attention) were on a task of identifying matching geometric shapes (Low Average) and on a motor coordination task on which he had great difficulty (Very Low). Jorge's handwriting is difficult to read. Words and letters are formed inconsistently. In this evaluation, most scores may be affected by his limited attention. Additionally, his difficulties with matching shapes, and especially with visual-motor coordination, are most likely affected by his neurological anomalies.

Compared with his age-peers, Jorge demonstrated average achievement in reading, writing, and mathematics.

Even in an optimal environment (quiet, one-to-one), Jorge had difficulty attending to a variety of tasks, despite incentives, cues, and breaks. Parent reports and evaluator observation indicate an area of concern regarding noncompliance. The reported difficulties in attention, distractibility, and self-regulation warrant intervention. These difficulties will continue to impact his ability to fulfill academic requirements without the aid of accommodations (e.g., one-to-one assistance or instruction). These characteristics are consistent with attention deficit/hyperactivity disorder (ADHD). There is evidence that these symptoms significantly impair his abilities to concentrate in academic settings, as well as in his home environment. Jorge meets the diagnostic criteria for ADHD, Combined Presentation.

Impressions

  1. 314.01 Attention Deficit/Hyperactivity Disorder, Combined Presentation

Recommendations

  1. Because Jorge is currently under the care of his physician, Dr. Albert Goldstein, as well as his pediatric neurologist, Dr. Andrea Levin, provide them with a copy of this report. The attentional, distractibility, and behavioral concerns noted by his parents, teachers, and this evaluator may help in designing a treatment plan for ADHD.
  2. Given Jorge's diagnosis of ADHD and previous medical diagnoses of hydrocephalus, left occipital porencephalic cyst, and left parietal closed lip schizencephaly, Jorge's home school may be contacted to request a multidisciplinary meeting to address his need for special education services. Although a multidisciplinary evaluation team (MET) will recommend services as they deem appropriate, these evaluation results indicate the need for occupational therapy for his handwriting under the category of other health impaired (OHI).
  3. If the MET does not provide special education services for Jorge, at the very least, he requires a “504 plan” (from Section 504 of the Rehabilitation Act and the Americans with Disabilities Act) to address his needs because of his attentional problems. Problems with attention impact his ability to manage and complete school tasks with the same level of success as his peers.
  4. Refer Jorge to an educational occupational therapist to evaluate fine-motor skills, muscle tone, and general coordination, and to then design an appropriate treatment plan.

Attention Deficit/Hyperactivity Disorder (ADHD)

  1. As noted in the evaluation report, Jorge exhibits several characteristics consistent with individuals with attention deficit/hyperactivity disorder (ADHD), which carries with it some difficulties in attention and distractibility. To help Jorge experience success, the following modifications can be made:
    • To offset Jorge's difficulty with attention, write instructions on the board. If possible, do so before explaining them orally. Make oral instructions clear and concise. If he forgets multiple-step instructions, write them on an index card as a numbered list.
    • Keep the daily schedule at home and school consistent. Routine sets up a structure in which Jorge can develop a sense of inner organization and control rather than feeling as if the day's activities and schedule happen haphazardly.
    • Students with attention problems experience significantly more difficulties completing homework than do their peers. These children are more likely to forget their assignments, lose their homework, and make careless mistakes. Increased discipline or withdrawal of privileges are not the ways to address incomplete schoolwork. Attempt to determine a successful homework plan that meets the needs of the Jorge and his family.
    • Because Jorge may have difficulty attending in the classroom as well as other settings, to the extent possible, vary the types of tasks, shorten work periods, and provide frequent reinforcers for staying on task or working carefully.
    • Work with a case manager/advocate or 504 specialist to assist in the management and implementation of accommodations necessary for Jorge to be successful. Jorge may experience difficulty in communicating his needs, clarifying school requirements, and interacting with others (e.g., teachers, peers, project-based groups).
  2. Jorge would benefit from a class placement incorporating the following components:
    • a highly organized teacher with a structured and systematic teaching style and calm, respectful manner of interacting with students
    • a behavioral program with clear rules, frequent and immediate positive reinforcement for target behaviors, and immediate, mild consequences for previously specified negative behaviors
    • a consistent daily schedule so that areas of academic instruction, recess, and routines (e.g., passing out daily work, assigning homework) are done in the same manner and order daily
    • a minimum of classroom noise and confusion (visual and auditory) in which students are aware that a transition is coming, when the current activity will end, what will happen next, and what they are expected to do to be ready
    • an emphasis on interactive and participatory instructional activities in which students have minimal wait time

Behavior

Jorge exhibits difficulties in complying with parental requests. The following behavioral interventions are suggestions to minimize Jorge's oppositional behaviors and to teach him alternatives to his current response (e.g., tantrums).

  1. When making a request of or a directive to Jorge, be specific in describing what you want him to do. Be concise and include only the most important details.
  2. Tell, do not ask, Jorge what you want him to do. Make only one request at a time. Wait until Jorge has completed what you have asked before making the next request.
  3. When correcting a behavior or redirecting Jorge, remain calm. If Jorge perceives anger or disapproval, he will react with an emotional response and be unable to process the content of your statement.
  4. For both praise and reprimands, keep statements brief and specific to the behavior. Do not allow or become engaged in lengthy verbal discussions regarding inappropriate or disruptive behavior. Stand near Jorge and obtain eye contact. Redirect or give consequences calmly, firmly, and immediately.

Counseling, Parent Training, Support Groups

  1. The parents are strongly advised to seek counseling or a class that focuses on parenting difficult children. Effective techniques for behavior management are available but typically require training, reinforcement, and feedback. These techniques succeed or fail depending on the consistency with which the parents can carry out the procedures, so support is often a necessary component. Parent groups with this focus are also helpful if led by a professional with expertise in parenting difficult children.
  2. A different type of family counseling is often helpful to help the child deal with the sense of failure and frustration that often accompanies this condition, and to help siblings cope with the child's behavioral differences and need for special attention. Also, counseling can provide support and education for the parents in a frustrating and puzzling situation, while helping them to develop realistic expectations for their child. This is not, however, a substitute for learning behavior management techniques.
  3. For information about joining a support group for parents of children with ADHD and for finding mental health professionals who specialize in this area, contact the local CHADD (Children and Adults with Attention Deficit Disorder) group at http://chaddoffresno.org.
  4. At least once a quarter, request a meeting with the teachers. Ask them to show you the materials they are using with Jorge, how they are being used, how Jorge is progressing, the skills with which he is having difficulty, and how you can help.

Resources for Attention Deficit/Hyperactivity Disorder

Websites:

Books

  • Barkley, R. A. (2013). Taking charge of ADHD: The complete authoritative guide for parents. (3rd ed.). New York, NY: The Guilford Press.
  • Kurcinka, M. S. (2000). Kids, parents, and power struggles: Effective strategies that will help you. New York, NY: Harper.
  • Shapiro, L. E. (2010). The ADHD workbook for kids: Helping children gain self-confidence, social skills and self-control. Oakland, CA: Lawrence E. Shapiro Instant Help Books.

Videos

  • The following video is highly recommended for Mr. and Mrs. Lopez to help them learn about ADHD: “The 30 Essential Ideas Everyone Needs to Know about ADHD,” by Dr. Russell Barkley, is a presentation specifically for parents of children with ADHD: https://www.youtube.com/watch?v=SCAGc-rkIfo. Any of Dr. Barkley's videos on www.youtube.com are both interesting and based on sound research. Be aware, however, that the web and YouTube are rife with sites and videos that sound professional but provide information that is not founded in good research or is incorrect.

Table of Scores

Woodcock-Johnson IV Tests of Cognitive Abilities

CLUSTER/Tests AE RPI Proficiency SS (68% Band) PR
Gen Intellectual Abil 5–11 75/90 Limited to average 90 (86–93) 25
Oral Vocabulary 6–6 91/90 Average 102 (95–108) 55
Number Series 6–4 86/90 Average 98 (94–102) 44
Verbal Attention 6–3 88/90 Average 98 (93–103) 46
Letter-Pattern Matching 4–4 0/90 Extremely limited 65 (54–76) 1
Phonological Processing 6–4 89/90 Average 99 (93–104) 47
Story Recall 6–9 92/90 Average 104 (98–110) 61
Visualization 6–2 88/90 Average 98 (93–103) 45

Woodcock-Johnson IV Tests of Achievement Form A and Extended

CLUSTER/Tests AE RPI Proficiency SS (68% Band) PR
READING 6–5 88/90 Average 99 (97–101) 48
Letter-Word Identification 6–4 83/90 Average 97 (95–100) 43
Passage Comprehension 6–6 92/90 Average 101 (98–105) 54
[Oral Reading]* 6–6 93/90 Average 102 (99–106) 56
MATHEMATICS 6–3 84/90 Average 96 (94–99) 41
Applied Problems 6–0 79/90 Limited to average 93 (88–99) 33
Calculation 6–4 88/90 Average 99 (96–102) 47
ACADEMIC APPLICATIONS 6–5 90/90 Average 100 (98–102) 50
Applied Problems 6–0 79/90 Limited to average 93 (88–99) 33
Passage Comprehension 6–6 92/90 Average 101 (98–105) 54
Writing Samples 6–7 94/90 Average 103 (97–109) 58
*Oral Reading is not part of the Reading cluster.

Variations and Comparisons

Standard Scores Discrepancy
Variations Actual Predicted Difference PR SD Interpretation at ±1.50 SD (SEE)
Intra-Cognitive [Extended] Variations
Oral Vocabulary 102 94 8 76 +0.72
Number Series 98 95 3 60 +0.25
Verbal Attention 98 95 3 60 +0.24
Letter-Pattern Matching 65 100 −35 0.3 −2.77 Weakness
Standard Scores Discrepancy
Intra-Cognitive [Extended] Variations
Phonological Processing 99 94 5 66 +0.41
Story Recall 104 95 9 76 +0.70
Visualization 98 96 2 57 +0.17
Standard Scores Discrepancy
Variations Actual Predicted Difference PR SD Interpretation at ±1.50 SD (SEE)
GIA/Achievement Discrepancy Procedure
READING 99 92 7 78 +0.76 No
MATHEMATICS 96 91 5 70 +0.53 No
ACADEMIC APPLICATIONS 100 91 9 86 +1.10 No

Beery VMI

Tests Standard Score Percentile Rank
Visual Perception 86 18
Motor Coordination 53 0.1

Background Information and Reason for Referral

Six-year-old Mun is currently in first grade. His parents are concerned that he is not progressing at the rate they would expect. According to his teacher, Ms. Bardot, Mun has difficulties with phonics and paying attention in class. Mun is struggling to identify letter patterns within words and “flips his letters” when reading and writing. Mun's parents are interested in an evaluation to determine how Mun learns and whether Mun has a learning disability.

Although English is Mun's first language, he spent a great deal of time during his early years with his grandparents, who speak Vietnamese to Mun. Mun is able to understand Vietnamese and can express a few vocabulary words, but he typically responds to his grandparents' conversations by speaking in English. There are no indications that Mun's bilingual heritage has negatively affected his academic skills development. Mun entered first grade last fall at Saint Maria Chiresi Primary School, a small program that emphasizes the nurturance of creativity and the arts, with an eye toward academic excellence. Over the past year, Ms. Bardot has observed that Mun is starting to learn how to decode words by sounding out individual phonemes; however, he is not yet competent in whole word recognition or reading fluently. Mun continues to misidentify letters, for example reading /bag/ as /dag/. Also, when writing letters of the alphabet, Mun will often reverse the letters /s/ and /b/. Beginning this past January, Mun was evaluated for mild articulation concerns by a speech-language pathologist from the local public school district, who has initiated after-school speech therapy for Mun twice per week. This process is of note because research indicates that young children with speech sound disorders (formerly articulation disorder) often develop later reading difficulties.

Review of Mun's family and developmental/medical history reveals that Mun's mother and his paternal grandmother are reported to have type II diabetes. Mun's father reported that he may have had difficulty reading when he was younger, but does not clearly remember, and he ultimately graduated from college. Mr. Panza reports no current learning difficulties, although Ms. Panza helps by proofreading everything he writes. Ms. Panza also attended college and reports no personal history of learning difficulties. She reported that her pregnancy with Mun was complicated by gestational diabetes and elevated blood pressure. She stated that she had to be induced 3 weeks early; however, there were no complications during labor. Mun experienced jaundice at birth and was kept in the hospital for one week. He was subsequently treated with phototherapy at home, did not need blood transfusion, and was medically cleared after a couple of weeks. Mun's parents reported that he met all of his early developmental milestones on time but noted that he did a side crawl before he crawled on hands and knees. Mun has continued to have trouble with gross motor coordination; for instance, he just learned how to pump himself on the swing and is still riding a tricycle. His parents reported that they tried to get him to play soccer but he did not do well. Now Mun is in an anti-bullying class in which he learns jiu-jitsu. He has participated in this class for the past 2 years, and he enjoys it. Mun does well and is very focused in this class, which Mun's parents describe as being very structured. Mun's medical history is noncontributory to his current challenges. He is currently healthy, eats and sleeps well, and is not taking any prescription medications. His vision and hearing have recently been screened as being unimpaired.

Mun's parents describe Mun as creative and as having a great imagination. As an only child, Mun often plays alone and is able to come up with creative play scenarios, especially with his Legos. Mun is reportedly not able to follow the instructions provided with Lego building sets, but he uses the pieces to make his own creations. Mun is also nice, caring, and complimentary towards others. He enjoys playing computer games such as Minecraft, and he watches cartoons and reads comic books with his father.

Behavioral Observations

Mun presents as a right-handed boy who was fully cooperative with the current evaluation. He was polite and talkative, with noticeable articulation errors, such as with the letter sounds /r/ and /l/, and pronouncing “trap” as “twap.” Mun occasionally needed directions or prompts repeated to him during the evaluation, and he needed more frequent breaks during the second four-hour day of testing. Also, during the second half of the evaluation, Mun was fidgety, frequently moved around in his seat, and regularly touched things within his reach. On several tasks, Mun was observed to say the stimulus items' names to himself before he could perform the task—in other words, he was not able to simply listen to the examiner's question, but needed to repeat the stimuli within the question to himself before he could answer. Completion of the evaluation required two half-days so as to maintain Mun's alertness and cooperation; however, he appeared to put forth a strong level of effort on all administered tasks, and embedded measures of performance validity suggested adequate levels of reliability and validity for the obtained results. The current evaluation is therefore considered a fair assessment of Mun's current status.

Test Results and Interpretations

Mun was administered a comprehensive psychoeducational battery that included measures of cognitive abilities, academic achievement, tests of executive functioning, and tests of reading and phonological processing. Specific scores from these test instruments are appended to this report, but they should be interpreted only by individuals who have been trained in test administration and scoring and who have a good understanding of the scientific literature on learning abilities and disorders. The following represents an interpretation of these findings and hopefully presents an integrated picture of Mun's abilities and disabilities.

Psychoeducational testing typically involves the administration of a large number of individual measures of specific cognitive and academic abilities. It is normal for individuals to have strengths and weaknesses in their profiles; however, Mun's profile of test scores has a greater than typical degree of “scatter,” suggesting that he has areas of significant strength combined with areas of notable weakness. As such, summarizing Mun's abilities in any single number or descriptive category would be misleading and inaccurate.

Mun's strengths seem to involve his processing of familiar material and its retrieval from his long-term memory. His overall listening comprehension is strong, and Mun seems to use meaningfulness within presented information to help him to learn. He is adept at rapidly naming items from or within various categories, is able to repeat back short stories that involve realistic life situations, and is fully capable of following oral instructions or directions. Mun's visual perceptual skills are also intact, and he is able to visualize shapes in different orientations and grasp the concept underlying parts to whole relationships. Consistent with these skills, Mun has relatively stronger skills in mathematics, visual memory, and visual-motor integration. He seems to have no difficulties with rapid visual perceptual analysis and closure, which allows him to accurately and rapidly scan familiar information such as numbers.

In contrast to these strong skills, Mun also demonstrates fairly significant difficulties in his processing of the sounds and symbols associated with learning to read. Mun has a poorly developed understanding of the phonemic building blocks within words, as demonstrated in his struggling to isolate phonemes within words, blend phonemes to create words, or change the forms of words by deleting phonemes within the word. When parts of words are read to Mun by phonemes (e.g., f—ar—m), Mun often misperceives the word the sounds make, and he guesses on the basis of only parts of the word (e.g., “rim”). He has trouble identifying the sounds that are at the beginning, middle, or end of a word, and he could not create new words by substituting new phonemes within a presented word. For example, if asked to change the word “buy” in “buying” to “play,” Mun had difficulty understanding that the new word would be “playing.” Although he had excellent skills in remembering and naming items within certain provided categories (e.g., “Tell me as many sports as you can think of”), Mun had great difficulty in coming up with words that begin with a certain letter (e.g., “Tell me as many words that start with /r/ as you can think of”). Collectively, this pattern represents inadequately developed phonological awareness, which has been linked to troubles with the development of early reading skills. Added to this finding is the fact that Mun seems to have a limited capacity memory span for language-based information that he hears.

When presented sequences of nonmeaningful information (e.g., strings of numbers or made up words such as “racklidom”), the information seems to fade rapidly from Mun's immediate memory. This is made even more difficult when he needs to do something with the information, for example, if he is asked to repeat a series of numbers in reverse order or to associate patterns of letters with certain sounds (e.g., N says /n/, I-G-H says /ī/, T says /t/ to make “night”). The combination of poor phonological awareness and poor phonological/working verbal memory thus makes the process of learning to read very difficult. Rather than just “hearing” the sounds associated with letter sequences in his mind, Mun has to “figure out” what the word is. Indeed, Mun's ability to read common words or sound out nonsense words that conform to common English spelling patterns is significantly lower than that of others his age and with his ability levels. Mun has not yet “broken the code” of reading, which in turn leads to his also having difficulty with spelling and written expression. This makes sense, because his difficulty with the “input” side of written language—reading—is directly linked to his difficulty with the “output” side—writing. Troubles with initial language processing and working memory are also associated with the fact that Mun's level of vocabulary knowledge is lower than expected, whether measured by asking him to define words or simply having him name pictures of objects. In summary, Mun is found to have a specific learning disability in reading and writing, also known as “phonological dyslexia.”

Success in school is dependent not only on developing competence in basic information processing, but also in developing the “executive functions” needed to demonstrate one's abilities and use them for daily problem solving. This was found to be a difficulty for Mun, who seems capable of paying attention to fairly brief and simple tasks but becomes increasingly overwhelmed as task demands increase. Mun's performance on a computerized test of focused and sustained attention was not within normal limits and indicated difficulties with both inattentiveness and impulsivity. Mun's difficulties increase as he is asked to “put the brakes on” what are the more automatic or instinctual responses to presented information. Mun thus seems to struggle with using rule-governed behavior to guide his responses to environmental demands. He tends to respond impulsively rather than to stop and think, suggesting that he has difficulty managing conflicting demands and requirements in tasks. To his credit, he is often aware of his mistakes and self-corrects; however, this causes Mun to take longer than most to get through presented activities. In part, this may be tied to his poor working memory in that he does not use instructions or directions to guide his behavior, and he may be developing a habit of trying to “just get it over with” rather than performing accurately.

On questionnaire measures, Mun's teacher endorsed items suggesting that Mun experiences anxiety, learning problems, and difficulties with functional communication. Ms. Bardot observes that Mun often seems nervous and worried. She comments that Mun often has trouble keeping up in class, has problems with mathematics, and almost always struggles with reading and spelling. Mun is reported to have trouble asking for and getting information when needed, and he can be unclear when he has to present his ideas. Although Ms. Bardot reports that Mun has a short attention span, she does not indicate that attention problems are a primary concern. Completion of similar questionnaires by Mun's mother emphasized concerns consistent with both internalizing problems (anxiety, somatic complaints) as well as attention problems. Ms. Panza suggests that Mun is “almost always” easily distracted and he regularly demonstrates a short attention span. Mother sees Mun as prone to impulsive interruptions of others when they are speaking and as having an elevated activity level. Both of Mun's parents completed a questionnaire assessing Mun's executive functioning, the results of which again suggest that Mun tends to lack inhibitory control. Mun reportedly gets out of control more so than his friends, and he can become too silly. Mun's parents also indicate that Mun tends to have difficulty getting started on tasks, often has trouble finishing tasks, has a short attention span, and is easily distracted. Mun needs help from an adult to stay on task, and if given three things to do he often remembers only the first or last. His parents believe that Mun has good ideas but cannot get them on paper, and he has trouble carrying out the actions needed to reach his goals. He is rather messy, is prone to making careless errors, and doesn't notice when his behavior has affected or caused a negative reaction from individuals in his life.

Impressions and Recommendations

Mun was referred for an evaluation of possible reasons for why he is not progressing as expected in school. The findings of our evaluation indicate that Mun has many cognitive abilities that are well within the Average range but evidence of a specific learning disability with impairments in reading and writing. Mun's profile is consistent with what is termed phonological dyslexia. Mun demonstrates incomplete development of the skills of phonological awareness and phonological working memory, which are further complicated by a speech-sound (articulation) disorder. Mun's learning disability represents an unexpected pattern of impairment, and he does not show a generalized learning disorder in that he has well-developed skills in mathematical reasoning and calculation. There is no evidence that Mun's learning difficulties are primarily the result of an emotional or behavioral disorder, although he is described as demonstrating symptomatology consistent with attention and executive functioning difficulties. Research evidence suggests a 20 to 40 overlap between attention deficit/hyperactivity disorders (ADHD) and dyslexia, and that children with poor articulation often develop reading disorders. Mun's vision and hearing have been tested as unimpaired, and although Mun may have greater difficulties learning a new language (e.g., Spanish), we do not think that Mun's Vietnamese heritage and language exposure is a primary cause of his challenges, and we do not believe it relevant or important to ask the household to be monolingual.

Diagnostic Impression

  1. 315.00 (F81.0) Specific Learning Disorder with impairment in reading
  2. 315.2 (F81.1) Specific Learning Disorder with impairment in written expression
  3. 314.01 (F90.2) Attention-Deficit/Hyperactivity Disorder, combined presentation
  4. 315.39 (F80.0) Speech Sound Disorder (per history)
  5. Rule out Developmental Coordination Disorder (per history)

Recommendations

The following recommendations are offered:

  1. Mun's parents are encouraged to read the book Overcoming Dyslexia, by Dr. Sally Shaywitz (New York: Alfred A. Knopf, 2004), which presents scientific understanding of dyslexia and its remediation in an easy-to-read, understandable format. This book covers what is known about the biological basis of reading disorders, as well as what can be done to help children and adults become confident readers. Another option is the book From ABC to ADHD by Eric Q. Tridas (Baltimore: The International Dyslexia Association, 2007). An audiobook version of Overcoming Dyslexia is available through the following link: http://www.audiobooks.com/audiobook/overcoming-dyslexia-a-new-and-complete-sciencebased-program-for-reading-problems-at-any-level/141783
  2. Commonly recommended commercial programs for reading remediation include the Lindamood-Bell Learning Process, the Wilson Reading System, Orton-Gillingham, and certain other specific approaches, depending upon the age of the individual. Current understanding suggests that effective programs are explicit, teach to mastery, are oriented to academic content, provide scaffolding and emotional support, and monitor progress.
  3. Research into the effectiveness of various forms of reading intervention has been reported by the Florida Center for Reading Research. A summary of research and brief for educators and parents regarding dyslexia is available on their website, at the following link: http://www.fcrr.org/TechnicalReports/Dyslexia_Technical_Assistance_Paper-Final.pdf
  4. Many children with dyslexia benefit from the use of bypass strategies to acquire academic content and knowledge about the world. Although we wish to emphasize teaching Mun to read competently, it will be essential for him to use technologies such as audiobooks, text-to-speech software, and multimedia presentations such as YouTube and other video and hands-on learning strategies until his reading ability catches up with the academic content. Mrs. Panza will be provided with an application for services from LearningAlly.org, an agency from which resources may be obtained.
  5. Research has shown that children can improve their working memory capacity through systematic training. The program Cogmed Working Memory training is discussed at www.cogmed.com, and has solid scientific support. This computer-based program can be administered over the summer months and will help Mun with both his attention and memory encoding abilities.
  6. Investigate computer software that enhances the development of phonological awareness and early reading skills. Consider programs such as Earobics, LeapPad, Jump Start Learning, and Reader Rabbit—all of which make learning to read fun and emphasize the core building blocks of phonological awareness, working memory, and rapid naming. Investigate applications for mobile devices that he can play with when out and about.
  7. Mun's parents may wish to seek a developmental motor evaluation, through either a physical or occupational therapist.
  8. It is important to address both Mun's academic difficulties and his attention concerns. Parents are encouraged to review information contained on the website www.chadd.org.
  9. Treatment options for ADHD typically include a combination of behavioral techniques, consideration of stimulant medication, environmental organization, and the maintenance of a long-term perspective of ADHD as a chronic condition. Mun's parents may wish to discuss with his pediatrician whether medical treatment of his symptoms is indicated, as well as a medical examination to rule out medical factors that may influence Mun's diagnosis (e.g., thyroid dysfunction).
  10. Mun's presentation is consistent with a model of ADHD that has been posed by Dr. Russell Barkley and is discussed in the book Taking Charge of ADHD. This book provides a comprehensive explanation of the nature of ADHD, its impact on child development and behavioral regulation, and the strategies parents can use to intervene in an effective manner.
  11. Mun needs external organization of his efforts, in the form of someone previewing what he is to do, providing commentary and feedback as to what he is doing, and review of his actions after he has finished. Providing external feedback to Mun will assist him to become more self-aware and reflective.
  12. Mun's progress in learning to read should be closely monitored and regularly reevaluated. Interventions following the guidelines should result in substantial and fairly rapid improvement in his skills. Should Mun not respond quickly, a more comprehensive evaluation for the reasons he continues to struggle should be sought.

Thank you for the opportunity to perform this evaluation.

Test Scores

Note: These scores are provided for professional use only, by persons trained in psychological test theory, development, administration, scoring, and interpretation. A significant risk of misinterpretation of these scores exists. Please refer to the body of the report for interpretive information. If you have any questions about these scores, please speak with your psychologist.

Woodcock-Johnson iv Tests of Cognitive Abilities, Tests of Oral Language, and Tests of Achievement

Scaled Scores by Age
CLUSTER/Test SS 1 2 3 4 5 6 7 8 9 10 11 12 13 14 15 16 17 18 19
GIA 89 X
GF-GC COMPOSITE 90 X
COMPREHENSION-KNOWLEDGE 85 X
COMP-KNOWLEDGE 3 84 X
FLUID REASONING 96 X
SHORT-TERM WORKING MEMORY 88 X
VOCABULARY 88 X
COGNITIVE EFFICIENCY 84 X
Cognitive Abilities
Oral Vocabulary 94 X
Number Series 98 X
Verbal Attention 100 X
Letter-Pattern Matching 87 X
Phonological Processing 67 X
Story Recall 108 X
Visualization 93 X
General Information 80 X
Concept Formation 94 X
Numbers Reversed 82 X
Academic Achievement
READING 78 X
BASIC READING 73 X
MATH 98 X
WRITTEN LANGUAGE 69 X
BROAD WRITTEN LANGUAGE 69 X
ACADEMIC SKILLS 86 X
ACADEMIC APPLICATIONS 72 X
BRIEF ACHIEVEMENT 82 X
Letter-Word Identification 78 X
Applied Problems 94 X
Spelling 82 X
Passage Comprehension 78 X
Calculation 100 X
Writing Samples 61 X
Word Attack 65 X
Oral Reading 67 X
Oral Language
ORAL LANGUAGE 95 X
BROAD ORAL LANGUAGE 95 X
ORAL EXPRESSION 84 X
LISTENING COMPREHENSION 102 X
PHONETIC CODING 87 X
SPEED OF LEXICAL ACCESS 118 X
VOCABULARY 88 X
Picture Vocabulary 82 X
Oral Comprehension 108 X
Segmentation 95 X
Rapid Picture Naming 117 X
Sentence Repetition 88 X
Understanding Directions 95 X
Sound Blending 82 X
Retrieval Fluency 111 X
Sound Awareness 99 X

Test of Variables of Attention

Condition SS 1 2 3 4 5 6 7 8 9 10 11 12 13 14 15 16 17 18 19
RT* Variability—First Half 93 X
RT Variability—Second Half 85b X
RT Variability—Total 85b X
Commission Errors—First Half 81b X
Commission Errors—Second Half 80** X
Commission Errors—Total 77** X
Omission Errors—First Half 85b X
Omission Errors—Second Half 98 X
Omission Errors—Total 93 X
Attention Performance Index = −.28

b = borderline result. *Response Time
**Significantly deviant result.

Cognitive Assessment Scale—Second Edition

Scale/Subtest SS ScS 1 2 3 4 5 6 7 8 9 10 11 12 13 14 15 16 17 18 19
Planning Scale 74 X
Planned Codes 4 X
Planned Connections 7 X
Attention Scale 91 X
Expressive Attention 7 X
Number Detection 10 X

Comprehensive Test of Phonological Processing—Second Edition

Scale/Subtest SS ScS 1 2 3 4 5 6 7 8 9 10 11 12 13 14 15 16 17 18 19
Phonological Awareness 71 X
Elision 6 X
Blending Words 4 X
Phoneme Isolation 6 X
Phonological Memory 70 X
Memory for Digits 7 X
Nonword Repetition 3 X
Rapid Symbolic Naming 98 X
Rapid Digit Naming 10 X
Rapid Letter Naming 9 X
Rapid Nonsymbolic Naming 113 X
Rapid Color Naming 12 X
Rapid Object Naming 12 X

Early Reading Assessment

Scale/Subtest SS ScS 1 2 3 4 5 6 7 8 9 10 11 12 13 14 15 16 17 18 19
Written Word Vocabulary 7 X
Rapid Orthographic Naming 8 X
Silent Orthographic Efficiency 7 X
Early Reading Index 82 X
Phonological Awareness 7 X
Receptive Vocabulary 5 X

Beery Visual-Motor Integration

Scale SS 1 2 3 4 5 6 7 8 9 10 11 12 13 14 15 16 17 18 19
VMI 97 X

Wechsler Intelligence Scale for Children, Fifth Edition

Scale/Subtest ScS 1 2 3 4 5 6 7 8 9 10 11 12 13 14 15 16 17 18 19
Working Memory
Digit Span 4 X
Digit Span Forward 7 X
Digit Span Backward 3 X
Picture Span 9 X

Reason for Referral and Background Information

Kendra is a 6-year-old girl who lives with her family in Huron, Ohio. She is in the first grade at Walter Elementary School. She does well in school and is obviously a bright child; her parents note that her ability to learn is well above that of her age-peers. Her parents are highly educated. They teach her two older siblings at home, because they do not feel that their needs are met in public school. They are considering doing the same for Kendra. They sought an evaluation to better understand her cognitive profile and how to guide her as a learner.

Academically, Kendra has always been a very strong student, with school-based testing showing her at the 99th percentile in both reading and mathematics, when tested on grade-level skills. She reportedly loves learning but does not like school, because the work offers so little challenge. Even so, she is cooperative and well-behaved. She excels in all subjects, but math is her passion. The school has been providing approximately fifteen minutes per day for her to work on mathematics, but she is being taught by a special education teacher who is not a math specialist.

Behavioral Observations

Kendra was brought to the testing site by her parents. She was dressed and groomed appropriately. She presented as quiet and somewhat shy, but personable and curious, with appropriate oral language and social pragmatics. Her language was clear, concise, and to the point.

She was easily redirected from informal chat to the testing tasks and was focused, cooperative, and hardworking throughout testing, manifesting a level of maturity and stamina unusual for her age. Her level of motor activity was within normal limits, and she showed no sign of impulsivity. Her work pace was consistently slow and careful—not sluggish, but thorough.

She was cooperative with all testing tasks and highly attentive to task requirements, but was clearly the most enthusiastic when engaged in mathematical and problem-solving tasks. She persevered on difficult items but was able to recognize when she was unlikely to be successful with further effort. She often used verbal mediation to coach herself through difficult tasks. When she was given time to play independently, she set challenging goals for herself and followed through, troubleshooting problems that came up along the way. She also cleaned up without having to be asked. These results are considered to be a valid assessment of Kendra's performance.

Tests Administered

  1. Cognitive: Wechsler Intelligence Scale for Children, Fifth Edition (WISC-V)
  2. Academic: Woodcock-Johnson Tests of Achievement, Fourth Edition (WJ IV ACH),
  3. KeyMath Diagnostic Assessment, Third Edition (KeyMath-3)

Scores Reported in this Assessment

Standardized tests provide scores based on a normative comparison with other individuals of the same age. For the tests used in this report, composite standard scores (SS) of 100 and subtest scaled scores (ScS) of 10 represent average performance. Approximately 50% of the population receives standard scores between 90 and 110 (scaled scores of 8–12), which is considered to be the “average” range. Percentile ranks (PR) do not represent “percent correct,” but rather the percentage of the population who performed below the level that Kendra did. Standard scores and scaled scores are linear and thus can be compared with each other more meaningfully.

Some of the WJ IV ACH tests are also reported with the Relative Proficiency Index (RPI) or the Generalized Relative Proficiency Index (GRPI). The RPI shows Kendra's success rate on skills on which an average child her age would have a 90% success rate (considered mastery), whereas the GRPI shows the success rate of the average child of Kendra's age if the task difficulty were increased to the level that Kendra would demonstrate success only 90% of the time.

Within the score tables, scores in brackets [] provide supplementary information but are not used in composites. A double dagger ‡ before a cluster score indicates that it is composed of subtests with widely varying scores and should be interpreted with caution. An asterisk (*) indicates that Kendra's raw score was above the upper limits of the norming sample (ceiling effects) and thus may be an underestimate of her ability.

Authors' Note: For more information about the GRPI, see the introduction to Dustin Greystone's report.

Cognitive Assessment

Wechsler Intelligence Scale for Children, Fifth Edition (WISC-V)

Index/Subtest/[Supplemental] SS* ScS* PR
FULL-SCALE IQ (SI, VO, BD, MR, FW, DS, CD) 137 99
NONVERBAL INDEX (BD, MR, FW, VP, CD, PS) 139 99.5
GENERAL ABILITY INDEX (SI, VO, BD, MR, FW) 137 99
COGNITIVE PROFICIENCY INDEX (DS, PS, CD, SS) 117 87
VERBAL COMPREHENSION INDEX 118 88
Similarities (SI) 13 84
Vocabulary (VO) 14 91
[Information (IN)] [17] [99]
[Comprehension (CO)] [18] [99.6]
VISUAL SPATIAL INDEX 135 99
Block Design (BD) 15 95
Visual Puzzles (VP) 17 99
FLUID REASONING INDEX 151 > 99.9
Matrix Reasoning (MR) *19 99.9
Figure Weights (FW) 18 99.6
[Picture Concepts (PC)] [13] [84]
WORKING MEMORY INDEX (DS, PS) ‡ 125 95
[AUDITORY WORKING MEMORY INDEX (DS, LN)] [136] [99]
Digit Span (DS) 17 99
Picture Span (PS) 12 75
[Letter-Number Sequencing (LN)] [16] [98]
[Arithmetic (AR)] [*19] [99.9]
PROCESSING SPEED INDEX 103 58
Coding (CD) 11 63
Symbol Search (SS) 10 50
[Cancellation (CA)] [7] [16]
[QUANTITATIVE REASONING INDEX] [152] [> 99.9]
[Figure Weights (FW)] [18] [99.6]
[Arithmetic (AR)] [*19] [99.9]
**SS: M = 100, SD = 15; ScS: M = 10, SD = 3.

Kendra's Cognitive Proficiency Index (SS 117, PR 87), which is composed of simple and automatic skills, was weaker than the composite of her mediated skills, but still above average. Differences this large are quite common in the high-ability population. Her ability to hold and manipulate information in her mind is strong enough to support the complexity of her thought (Working Memory Index SS 125, PR 95; Auditory Working Memory Index SS 136, PR 99). Although her speed of perceptual and clerical tasks is only in the average range (Processing Speed Index SS 103, PR 58), this relative weakness is still quite common in the gifted population and does not appear at this time to reflect an underlying learning disability or any cognitive sluggishness. Rather, it reflects Kendra's slow, careful, and methodical working style. However, she may get bogged down easily, not able to do simple parts of complex tasks efficiently. As observed in the classroom, Kendra tends to work slowly enough that her ability to get tasks done in a reasonable amount of time can be compromised.

Academic Assessment

Woodcock-Johnson IV Tests of Achievement (WJ IV ACH), Form A

WJ IV SS** PR RPI GRPI
BROAD ACHIEVEMENT ‡ 117 88 90/39
BROAD READING 99 47 88/90
READING 101 52 90/89
BASIC READING SKILLS 104 60 90/82
READING FLUENCY 96 40 81/90
Letter-Word Identification 101 53 90/88
Passage Comprehension 100 51 90/89
Sentence Reading Fluency 96 40 77/90
Word Attack 108 70 90/75
Oral Reading 97 43 84/90
BROAD WRITTEN LANGUAGE ‡ 114 82 90/58
WRITTEN LANGUAGE ‡ 111 78 90/61
WRITTEN EXPRESSION 121 92 90/37
Spelling 100 51 90/89
Writing Samples 119 90 90/23
Sentence Writing Fluency 119 90 90/50
PHONEME-GRAPHEME KNOWLEDGE 107 69 90/79
Word Attack 108 70 90/75
Spelling of Sounds 106 65 90/81
BROAD MATHEMATICS 141 99.7 90/2
MATHEMATICS 146 99.9 90/2
MATH CALCULATION SKILLS 139 99.5 90/1
Calculation 133 99 90/2
Applied Problems 147 > 99.9 90/3
Math Facts Fluency 135 99 90/1
CROSS-DOMAIN COMPOSITES
ACADEMIC SKILLS ‡ 112 79 90/53
ACADEMIC FLUENCY ‡ 114 83 90/37
ACADEMIC APPLICATIONS ‡ 124 94 90/29
**M = 100, SD = 15.

Kendra's above-average Broad Achievement score obscures significant differences between her superior math achievement and her average achievement in reading and writing. The same is true for all of the cross-domain composites.

Kendra's overall achievement in the domain of reading was typical for her age, consistent with her reported independent reading at the advanced picture-book level. Although she appeared to make a solid effort toward sounding out words, and her knowledge of phoneme-grapheme correspondences is within age expectations, she did not appear to know the necessary phonics rules to approach unfamiliar words. Furthermore, she was largely unwilling to guess, skipping almost one third of the items rather than risking mistakes. On the fluency measure, she took the time to read each item but declined to answer many, which substantially weakened her performance. When she was asked to read short text passages, she benefited greatly from picture support, not using grammatical or context cues effectively without them.

Supplementary testing of her ability to read and spell phonologically regular nonsense words and her oral reading showed largely age-appropriate development. However, she manifested a number of problems in simple letter-sound correspondence (e.g., e/i, s/sh, long/short vowels), and declined to try to sound out words when she was unsure of them. Most gifted children Kendra's age learn to read well and fluently with little external encouragement, because reading is highly self-reinforcing. However, Kendra shows little enthusiasm for the simple picture books she can currently read independently; she greatly prefers the more sophisticated ideas and information she can access when her parents read to her. Her cognitive profile suggests that she will be capable of easily learning and coordinating the multiplicity of rules needed for accurate decoding. However, these findings suggest that Kendra also may have a subtle phonologically based reading disability (dyslexia) that has thus far been masked by her high intelligence and the relatively low demands of the academic environment. This possibility bears watching going forward.

On the tests of writing, Kendra's ability to generate and write meaningful sentences according to a variety of demands, as well as to generate and write brief, simple sentences rapidly, were above average to superior, and significantly above her spelling achievement, which was average for her age. Her pencil grip and handwriting were largely age appropriate, with some immature features (four-fingered grip, wrapped thumb, directional confusion), which should be addressed but will probably resolve without difficulty. She generally used invented spelling.

Kendra's performance on the tests of Mathematics, by contrast, was in the Very Superior range for her age. She showed a solid mastery of basic arithmetic facts and a range of skills including multiple-digit addition and subtraction with regrouping, multiplication, and simple division. Furthermore, when she had to model a mathematical problem implied in a real-world context, her conceptual understanding and problem solving were extremely strong. Her Generalized Relative Proficiency Index scores in mathematics suggest that the overwhelming majority of typical classroom enrichment tasks would be inappropriately easy for her. To provide a meaningful challenge for her, she would need complete alternative curricula targeted to the level of her actual conceptual and skill development.

KeyMath-3 Diagnostic Assessment, Form B (KeyMath-3) ‡COMPOSITE/Subtest SS** ScS** PR GE**
TOTAL TEST COMPOSITE * 145 > 99 5.8
BASIC CONCEPTS * 144 > 99 5.6
Numeration 18 99.6 4.4
Algebra 19 99.9 4.8
Geometry 19 99.9 6.0
Measurement *19 99.9 9.0
Data Analysis & Probability 18 99.6 5.0
OPERATIONS * 145 > 99 5.2
Mental Computation & Estimation 19 99.9 6.5
Addition & Subtraction 19 99.9 5.2
Multiplication & Division (see note) 4.4
APPLICATIONS * 145 > 99 8.7
Foundations of Problem Solving *19 99.9 7.7
Applied Problem Solving 19 99.9 9.5
**SS: M = 100, SD = 15; ScS: M = 10, SD 3; GE = Grade Equivalent.

Because of Kendra's passion for mathematics, her conceptual and skill development were further explored using the KeyMath-3. This test is specifically designed to match the typical elementary and middle school curriculum as designed by the National Council of Teachers of Mathematics in their Principles and Standards for Mathematics, on which most state educational standards (including the Ohio State Standards) are based.

Although grade equivalent scores are problematic because of the nonlinear nature of development, they are reported here for qualitative information. They show that Kendra's performance was similar to that of a student several years older, not necessarily that she has learned the skills taught in those grade levels. The numerous ceiling scores indicate not that she completed all items on the test, but rather that she was substantially above the highest raw score in the norms chart for her age.

The tests of Basic Concepts address the five major content strands. Kendra's performance was largely limited only by her knowledge of standard terminology and notation. She was able to correctly use concepts of place value, variables, equations, spatial relationships, coordinate graphs, data representation, probability, and translating between real-world situations and their symbolic representations. She was markedly skilled at figuring out how to handle novel situations.

The domain of Operations covers both written and mental computational skills. Kendra was able to make reasonable estimates and solve multidigit and multistep problems correctly in her head. On paper, she was able to add and subtract whole and decimal numbers with regrouping, using standard algorithms. She also showed solid early skills in multiplication and division; no scaled score could be calculated for the Multiplication subtest, because there are no norms for children this young.

The Applications domain explored Kendra's skills in approaching mathematical problems: identifying the key elements, operations, and strategies necessary to solve the problem and modeling a story problem as a mathematical relationship. These subtests involve considerably more ambiguity and metacognitive awareness than the other subtests on this instrument. Her reasoning on these subtests was highly sophisticated.

Summary

Kendra is a highly gifted child with an excellent work ethic and a broad array of cognitive strengths. She has a solid fund of general knowledge, strong ability to concentrate and to manage complex tasks in both the visual and verbal domains, as well as excellent fluid reasoning, problem-solving, and metacognitive awareness. Kendra's greatest strength is as a problem-solver. She is able to develop skills and approach tasks that are not typical for a child her age, much more thoroughly even than many older children can. Furthermore, she has the self-discipline and drive to engage in the practice needed to internalize and retain new information.

Her working speed is markedly slow, although well within the average range for her age. Her deliberative and careful style will stand her in good stead when a task merits depth of thought, but she can bog down and is vulnerable to perfectionist anxiety. At present, this difference does not appear to indicate a disability but rather a stylistic tendency to prioritize “getting it right” over “getting it done.”

Kendra shows a very strong command of fundamental skills in mathematics and excellent conceptual understanding. It will be crucial for her to receive consistent support from educators familiar with higher mathematics; age-typical classroom materials are completely inappropriate for her at present. Whether she progresses more quickly in the traditional arithmetic curriculum or whether she does in-depth study of topics lateral to the curriculum, she should not be forced to mark time in the subject in which she excels and that she most loves.

Her development in reading and writing, as well as the phonics skills underlying them, are age-typical at this time. Likely this difference reflects her strong interest in mathematics and her frustration with the simplistic content of the picture books she can currently read independently. However, the issue bears watching, because most children at her cognitive level do read well by her age. If she does not develop into a strong and fluent reader within two years, it would be important to follow up on the possibility of a specific reading disorder (dyslexia).

Recommendations

Academic Recommendations

  1. Kendra will learn best when she is presented with opportunities to figure out new concepts and skills in an inquiry format.
  2. Help her develop her repertoire of problem-solving strategies, as well as her skill in recognizing situations in which those strategies will be most useful.
  3. When learning content, Kendra will benefit from instructional strategies that tap into her ability to learn and apply general rules (e.g., studying etymology rather than arbitrary word lists in spelling).
  4. When she encounters situations she is unsure how to handle, give her the time to identify what is going on, consider options, preview likely outcomes, and decide on a course of action.
  5. Teach her how to think about what it is that she does not yet know and how she could formulate questions that would help her find out more.
  6. Provide Kendra with a flexible pace of instruction, allowing her to move rapidly through skills she masters readily while spending more time to work in depth on what does not come as easily to her.
  7. Do not hold back instruction appropriate to her level of knowledge, skill, and understanding as a reward for finishing work that is inappropriately easy or repetitious for her. Substitute materials designed for older and more sophisticated learners.
  8. Use curriculum compacting to avoid unnecessary reteaching of skills Kendra has already mastered. Use formative assessments during new skill instruction, because she is likely to learn and retain new skills quite rapidly. Too much repetition will encourage the development of maladaptive beliefs about the importance of hard work. On the other side of the coin, incorrect assumptions that she probably already knows something are likely to provoke anxiety and self-criticism.
  9. In content-focused areas of instruction (e.g., science, social studies, literature), provide tiered assignments, including reading materials and learning activities on the same topic but at a higher level of sophistication. School and local librarians are often a good resource for locating appropriate resources.
  10. If her teachers are uncomfortable making these choices on their own, collaborating with Kendra's parents in choosing a topic and materials will be helpful. A great many options are compiled on the website http://www.hoagiesgifted.org.
  11. The book Teaching Gifted Kids in Today's Classroom by Susan Winebrenner is a good resource for classroom teachers to understand how to work with children like Kendra in a practical way. It also includes a CD with customizable forms.
  12. Recognize, however, that, as with many gifted children, Kendra's executive functioning may not develop precisely in line with her cognitive and academic development. When developing work for her to do independently, provide appropriate guidance and scaffolding so that she can learn how to manage sophisticated projects.
  13. Kendra's strong work ethic and her anxiety over imperfection create the risk of perfectionism. She needs explicit guidance (e.g., access to grading rubrics while she works) in terms of what constitutes “good enough.”
  14. There is a substantial difference between Kendra's skill development in mathematics and her skill development in language arts. It is important to help her develop in both areas, without either letting mathematics dominate her day or holding it hostage to her more age-typical level of development in reading.
  15. Provide her with reading materials that appeal to her high interest in mathematics and science but provide substantial picture support, such as the DK Visual Encyclopedia books. These may help her “get over the hump” to being able to read materials that are interesting enough to her to encourage her to continue developing her reading skills.
  16. While reading to Kendra, make sure she can see the page and follow along with the reader's finger. Periodically encourage her to decode words and sentences.
  17. Administrators are strongly encouraged to adopt a cluster grouping model, putting a small group (typically 4–6) of gifted children within a single classroom instead of spreading them throughout all classrooms. This strategy has been shown to raise achievement for all students, not just for the gifted students, as well as supporting their social and emotional growth and reducing the burden on teachers. The Cluster Grouping Handbook (Winebrenner & Brulles, 2008) provides concrete guidance on how to implement this system within a larger school community.
  18. Whether now or after her reading skills begin to develop, Kendra may be a good candidate for whole-grade acceleration. She has a broad array of strengths and a high level of social maturity. Longitudinal research indicates clearly that most children find this intervention to be a positive force in their lives. Both the choice to accelerate and the choice not to accelerate are actions, each with its own risks and benefits. The Iowa Acceleration Scales, available through http://www.accelerationinstitute.org/Resources/IAS.aspx, can inform the conversation and place various kinds of concerns in research-based context.
  19. The research on all forms of academic provision for gifted students is summarized in the comprehensive report, A Nation Deceived (Colangelo, Assouline, & Gross, 2004) and its update, A Nation Empowered (Assouline, Colangelo, VanTassel-Baska, & Lupkowski-Shoplik, 2015), available at http://www.accelerationinstitute.org. As with whole-grade acceleration, all forms of service to gifted children are strongly supported by research.
  20. A subject-acceleration or partial-homeschooling model may be most appropriate for Kendra, allowing her to remain with age-mates for the subject areas in which her development is age-typical and providing her with the challenge she needs in subjects in which the materials that would challenge her would be inaccessible to them.

Mathematics

  1. Kendra needs to be guided in mathematics by school personnel who are themselves sophisticated mathematical thinkers. Often, children like Kendra spontaneously come up with ideas that should be recognized and fostered.
  2. As a whole curriculum, the Singapore Math series (http://www.singaporemath.com) is highly structured while emphasizing problem-solving; it is likely to be a good match for Kendra's learning style.
  3. The supplementary materials from the Art of Problem Solving (http://www.artofproblemsolving.com), a set of textbooks and online courses for students who are advanced in mathematics, as well as their online interactive forum, are excellent. They have released a number of books aimed at younger readers, the “Beast Academy” series.
  4. Ed Zaccaro's Challenge Math series is another excellent set of structured math materials covering the K–10 curriculum with various levels of sophistication. It is not an independent curriculum in itself, but it is a good supplement.
  5. To help Kendra move outside of her comfort zone, consider books by Key Curriculum Press (http://www.keypress.com) and others who specialize in mathematical enrichment and recreational mathematics topics. In particular, the book Crossing the River With Dogs (K. Johnson, Herr, & Kysh, 2012) specifically teaches strategies for solving nonroutine problems.
  6. Harold Jacobs's Mathematics: A Human Endeavor was originally written for college liberal arts majors, to introduce methods of mathematical thinking through a problem-set approach; it is, however, often a good match for young gifted mathematical thinkers. It covers topics lateral to the typical K–12 number-strand curriculum.
  7. The Committee on Mathematics and its Applications (http://www.comap.org) primarily focuses on materials for older learners (e.g., they have a high school curriculum emphasizing the use of mathematical modeling), but the book For All Practical Purposes (COMAP, 2011) is a very nice book to use as a follow-up to Human Endeavor (Jacobs, 1994) This book also takes a problem-set approach, teaching topics about how mathematics applies to real-world situations.
  8. Seek out local opportunities for Kendra to interact with other children who are curious about mathematics. Math Circles are interactive groups that work together to solve nonroutine and complex mathematical problems, with an emphasis on being able to explain one's reasoning to others. A directory of these groups can be found at http://www.mathcircles.org.
  9. Books on recreational math or special topics, such as those written by Dale Seymour, Martin Gardner (the Aha! books are appropriate for her age), Theoni Pappas, and Hans Enzensberger (The Number Devil is delightful), can provide space for her to explore.
  10. Kendra's scores and her general presentation suggest that she would be a good match for the Davidson Institute's Young Scholar program (http://www.davidsongifted.org/youngscholars/). This organization provides free educational advising and other resources for highly gifted students and their families, as well as providing venues where kids can socialize with each other.
  11. Kendra should also be eligible to participate in the courses offered by the Johns Hopkins University Center for Talented Youth (http://cty.jhu.edu). These programs enable gifted children to make social contact with each other and explore their shared areas of passion.
  12. Kendra's test scores and her approach to mathematics suggest that she may be a good fit for Epsilon and Delta Camps (8–12-year-olds: http://www.epsiloncamp.org; 6–7-year-olds: http://www.deltacamp.org). These two-week residential camps (a parent accompanies the child and lives with them on a college campus) pair children who love math and are highly talented mathematical thinkers with college professors who enjoy sharing complex mathematical ideas with young learners.
  13. As Kendra gets older, she may wish to seek out mathematics competitions. However, many of these competitions reinforce the idea that speed of basic skills is what is truly important. Help her seek out programs that emphasize sophisticated mathematical and conceptual thought over pure rote skills.
  14. Similarly, she is likely to enjoy the FIRST robotics competition (http://www.usfirst.org), which has divisions for students ages 6–18. These contests challenge children to work in teams to design and build robots to carry out specific tasks. These competitions are also likely to be of interest to other bright-to-gifted kids, and can serve as a positive social venue.
  15. Involve her in the process of thinking through real-world situations in which mathematics can inform decision-making but in which multiple factors must be balanced against each other (e.g., planning a fundraising bake sale or a family trip).
  16. Many bright students resist showing their work because they believe that part of being smart is being able to do the work in your head. However, Kendra will soon reach the level of the curriculum where this is no longer possible or advisable. Help her understand that good mathematicians are very careful about showing their work and explaining their reasoning to each other.

Approach to Giftedness

  1. It will be important to explore the meaning of high intelligence to Kendra and to her family, and consider how this idea can be talked about, both within the family and to others, in a positive and affirming way. There are many pitfalls; for example, the messages, “You are valuable and loved primarily because you are smart,” “If you are this intelligent, everything should come easily to you, you should never make mistakes, and you should not have to work hard at anything,” “You are a better human being than other people are because you are smart,” “Intelligent people do not need to learn basic skills or do anything else that is not constantly stimulating and fun,” and “You should only socialize with other people who are as smart as you,” are all examples of inappropriate messages that are sometimes inadvertently sent.
  2. On the other side of the coin, however, failing to directly address the issue of giftedness with a child is also highly problematic. Despite what many parents and teachers might wish to believe, children are typically intensely aware of the differences among them. When adults refuse to acknowledge and celebrate a child's strengths, children often interpret this message to mean that their high intelligence is a bad thing, literally “unspeakable.” Many adults report having felt strongly that there must be something wrong with them because they were not like other children and because they struggled at times to find common ground with them.
  3. Kendra's parents are encouraged to think through how their own intelligence and their daughter's intelligence has affected their lives, how they want to understand the very real differences among people, how they can accept what is good about themselves without devaluing others, how those differences create different needs in both the academic and social realms, and how they can continue to address those needs in a constructive way.
  4. As Kendra moves through the school-age years into adolescence, her needs are likely to change, and her parents are encouraged to be aware that solutions should themselves change to respond to those changing needs.
  5. Kendra's parents should continue to educate themselves about giftedness and connect with other families to provide mutual support and companionship. Many parents of gifted children find themselves socially isolated, because it can be hard to find places where they can ask questions and receive ideas and support without being seen as “bragging.” The website www.hoagiesgifted.org provides links to these communities, as well as to a host of other articles and resources for gifted kids.
  6. Probably the best single all-around book for understanding the needs of gifted children is A Parent's Guide to Gifted Children, by James Webb, Janet Gore, Edward Amend, and Arlene DeVries. This book answers many common questions about how to help gifted children navigate school, social, and family experiences.
  7. Parents may find a parent support group, such as those facilitated by Supporting the Emotional Needs of the Gifted (http://www.sengifted.org), to be helpful to them. SENG also provides articles and other resources for families.
  8. The online group Gifted Homeschoolers Forum (http://giftedhomeschoolers.org) is a very good resource for families who provide full-time or part-time homeschooling for their highly intelligent children. They offer online courses of their own, as well as extensive lists of links to other organizations and useful information.
  9. It is a common misconception that gifted children can either receive appropriate academic support or fit in socially. In fact, both goals can and should be met simultaneously. The most important social-emotional needs of a gifted child like Kendra are real academic challenge and a peer group who can keep up with her. Research indicates that gifted children develop a healthy self-concept, social skills, and positive attitudes toward other children of all levels of ability if these needs are reliably met. It is likely to be confusing and upsetting to Kendra when few other children around her understand what she is interested in, or if she is always either working alone or waiting for others to catch up.
  10. Whether these peers are found in a traditional school environment or in a series of communities found through organized or casual activities, it is important to help Kendra find those peers and learn to connect with them. Support her participation in extracurricular activities that allow for cross-age contact and self-directed learning, that celebrate achievements without ambivalence, and that attract a similarly engaged and driven population.
  11. Find ways to help Kendra learn to relax, to enjoy unstructured time without guilt. She is a very hardworking child, and it is often difficult for children like her to accept that self-care is a valid part of life.
  12. Recognize that an essential need of all people is to be recognized and appreciated for their achievements, and to have authority figures whom they can accept as valid judges look at their accomplishments with approval. Over the course of development, the people who serve those needs should gradually shift from parents to outsiders (e.g., teachers, coaches, professors, colleagues) who are seen as domain experts and thus able to judge at a higher level. Gifted children, particularly at young ages, often struggle to find these outside authorities. Help Kendra connect with those who bring out the best in her.
  13. The article “Play Partner or Sure Shelter,” by Miraca Gross, describes her research into the developmental trajectory of friendships. Most gifted children do well when they have access to a variety of social environments, some that play to their strengths and some that allow them to be more “normal” in ability.
  14. The family should continue to cultivate an attitude that emphasizes the role of hard work and persistent effort in combination with native abilities in solving difficult problems, rather than the role of native talent in making problems easy. This approach is described well in Carol Dweck's book Mindset. People of all ages are found in research to work harder on more difficult challenges when they are cued to think about how hard they have been working, rather than about their innate talents. However, the emphasis on praise for what a child does should not require one to ignore or denigrate who a child is. Celebrate the ways in which hard work potentiates the expression of natural talent, and the joy that is found when one is able to express one's uniqueness effectively.
  15. Behavior contracts, sticker charts, rewards, token-economy, and response–cost strategies are rarely if ever effective for a child at Kendra's cognitive level. Rather, recognize that Kendra has the capacity to be a true partner in her own learning, with adults serving as valued resources. A collaborative approach has been found to be far more effective with gifted children, because it respects their desire for autonomy and leverages their strengths.
  16. Charts that document improvement over time can be used, not to earn a reward, but as a way to help Kendra notice her actual successes. She should be the one to record her own progress.
  17. The most effective types of responses are those in which the adult cues the child to recall the structures or skills she is learning and to reorient her behavior accordingly. Rather than telling her what to do or pointing out a mistake, ask her questions that help her notice her own mistakes or the aspects of the situation that she may have overlooked. If she cannot remember, then use progressively more specific questions to guide her recall or to reteach what has been forgotten.
  18. Additional resources on authoritative (rather than authoritarian) parenting, which has been consistently shown to have the most positive outcomes, include the books How to Talk So Kids Will Listen and Listen So Kids Will Talk, by Adele Faber and Elaine Mazlich, and Between Parent and Child, by Haim Ginott.

Reason for Referral

Sabrina is a 7-year-old girl currently repeating the 1st grade. She was referred for an evaluation because of concern about her progress in the areas of reading and writing. She has experienced persistent difficulty with both reading and spelling.

Background Information

Developmental, Health, and Family History

Sabrina is the product of a healthy, full-term pregnancy and weighed 7 pounds 4 ounces at birth. Her mother reports that the cord was wrapped around Sabrina's neck at the time of delivery and that she had slight jaundice. There were no other complications, and Sabrina and her mother were discharged from the hospital together. All developmental milestones, such as crawling, walking, and talking, were met within normal limits.

Sabrina's current health is described as excellent. She has seasonal allergies. Her mother reports that Sabrina is color blind and that she has frequent nightmares. Sabrina is described as a very active child. Her mother reports that she has always had a slight problem with staying focused. She has always done well with change and transition. She has no difficulties with expressing her feelings and is described as always happy and playful. Her activity levels, sleeping, and eating patterns are all described as regular.

Sabrina lives in Cedarville with her mother, father, and 5-year-old brother. In the home, Sabrina has regular bedtime and wakeup times and is reported to sleep well. Her home responsibilities include setting the table and cleaning up the yard. She performs these tasks regularly but needs frequent reminders. Her mother reports frequent problems with completing homework and occasional behavior problems when Sabrina is told to do something that she does not want to do.

There is a family history of learning difficulties with Sabrina's father, great aunt, and two cousins.

Psychosocial and Behavioral History

Sabrina's mother described her as fidgety, with a tendency to engage in impulsive behaviors. She is easily frustrated and sometimes feels fearful. As well, she can be defiant and tends to blame others (e.g., her younger brother) for mistakes. She has low self-esteem in relation to school and gets frustrated and irritable with school work, especially reading. She has told her mother that she is “stupid” and that her classmates say mean things to her. She fears sleeping alone and going into her room or the bathroom alone.

Sabrina's mother reports that Sabrina has two to five “BFFs” but that she is friends with everyone. Sabrina is described as outgoing and both a leader and follower. She has no difficulties with helping or sticking up for a peer.

Educational History

Sabrina is reported to like school on some days and hate it on others. She is very motivated to learn and works on homework for 20 minutes each night. She is currently seen by a Title I tutor for reading support for 30 minutes per day, five days per week. Sabrina attended Ash Hill Elementary School for preschool, Westview Elementary School for kindergarten, and Hunter Elementary for 1st grade. She is currently repeating 1st grade at Hunter Elementary because of her limited progress in reading and writing. Sabrina's mother reports that Sabrina is extremely hard on herself and cries frequently when she struggles with homework assignments.

Tests Administered

  1. Woodcock-Johnson Tests of Cognitive Ability, Fourth Edition (WJ IV COG)
  2. Woodcock-Johnson Tests of Oral Language, Fourth Edition (WJ IV OL)
  3. Woodcock-Johnson Tests of Achievement, Fourth Edition (WJ IV ACH)

Testing was conducted over three separate sessions. During the first session, Sabrina was administered tests 1 through 8 of the WJ IV ACH; the session lasted approximately 1½ hours. During the second session, which lasted 1½ hours, she was administered tests 1 through 10 of the WJ IV COG. The third session lasted approximately two hours and involved administration of selected sections of the WJ IV ACH, WJ IV COG, and WJ IV OL.

Behavioral Observations

Sabrina was polite and cooperative throughout the three testing sessions. She engaged in casual conversation, maintained eye contact, and responded appropriately to questions about school and her family. Sabrina worked steadily during the testing sessions, at times making comments that the items were getting more difficult. She completed written activities with her right hand. Sabrina maintained her attention throughout testing; however, a few times, she appeared to not have heard or understood a prompt or direction. Overall, these results are considered to be an accurate representation of Sabrina's cognitive, academic, and oral language abilities at this time.

Test Results

The WJ IV COG, WJ IV ACH, and WJ IV OL were scored according to age norms. Because these three batteries are co-normed, direct comparisons can be made among her cognitive, oral language, and achievement scores. These comparisons can help to determine the presence and significance of any strengths and weaknesses among her abilities. These tests provide measures of Sabrina's specific cognitive and oral language abilities, as well as her academic achievement.

Sabrina's performance on the WJ IV COG is compared with her same-age peers and is described using standard score (SS) ranges:

Standard Score Range Qualitative Descriptor
>130 Very Superior
121–130 Superior
111–120 High Average
90–110 Average
80–89 Low Average
70–79 Low
<69 Very Low

Sabrina's proficiency on the WJ IV ACH and WJ IV OL is described by Relative Proficiency Index (RPI) levels and ranges. The top number of the index indicates the level of performance the she would achieve when typical same-age peers were 90% proficient on the same task. For example, an RPI of 75/90 would indicate that Sabrina performing at 75% proficiency on an item which same-age peers would perform at 90% proficiency. This score would fall in the Limited to Average range.

RPI Range Qualitative Descriptor
100 Very Advanced
98–100 Advanced
95–98 Average to Advanced
82–95 Average
67–82 Limited to Average
24–67 Limited
3–24 Very Limited
0–3 Negligible

Based on the tests of WJ IV COG, Sabrina's overall intellectual ability, as measured by the General Intellectual Ability (GIA) measure, was in the Average range (SS = 97). The Gf-Gc Composite, a combination of verbal and nonverbal ability, is an additional measure of her cognitive abilities. Sabrina's Gf-Gc composite was in the Average range (SS = 101) and is likely the better measure of her overall intellectual ability because it does not include measures of auditory processing, which is an area of relative weakness for her. Sabrina's abilities with Comprehension Knowledge, as well as her Fluid Reasoning (nonverbal reasoning and problem solving), Short-Term Working Memory, Long-Term Memory, and Visual Processing were all within the Average range. Cognitive Efficiency, which reflects the ability to work quickly and accurately, was also in the Average range (SS = 92). Sabrina's Quantitative Reasoning cluster, which encompass nonverbal reasoning and an understanding of quantity and numerical symbols, was also in the Average range (SS = 106). Her abilities with Auditory Processing were in the Low Average range (SS = 84). Auditory processing involves processing and manipulating sounds, and skill in this area is critical for efficient reading and spelling.

Oral Language Abilities

According to Sabrina's RPIs, her performance on oral language tasks ranged from Limited to Average-to-Advanced. She demonstrated language comprehension by completing sentences that had a word missing by following oral directions. She was also able to blend orally presented syllables into whole words. Sabrina had the most difficulty with segmenting words; she was unable to separate words of more than two syllables into separate syllables.

Academic Achievement

Reading

On reading tasks, Sabrina had difficulty applying phonics skills, recognizing sight words, and reading simple sentences fluently. Her reading comprehension abilities were inconsistent; on one measure her score was in the average range, and on another, similar task, her score was in the limited range. On one test, in which Sabrina was asked to read a short passage and then retell it, she could not recall any details, even after rereading it to herself. On a more difficult passage, however, Sabrina easily remembered several details after reading the story to herself. Sabrina demonstrated relative strength in word attack skills. Overall, it is clear that reading is an area of significant weakness for Sabrina.

Written Language

Sabrina's writing abilities were mixed. Sabrina fluently composed simple sentences when given a printed set of words to use, and could write simple sentences according to a variety of directions, with accompanying pictures. She consistently used correct capitalization and punctuation. She also consistently reversed the letters b and d in her writing. Spelling is an area of relative weakness for Sabrina. Her spelling errors were indicative of difficulties with understanding both phonics and orthographic spelling rules. For example, she spelled might as “mite” and job as “gob.” She would benefit from direct instruction in spelling rules, starting with the most basic spelling patterns (words of one closed syllable) and working her way up to master more complex spelling patterns.

Mathematics

All of Sabrina's math scores were within the Limited range. On the Applied Problems test, she was able to count small quantities and could correctly answer a simple addition story problem accompanied by a picture. She was unable to answer a simple subtraction problem presented in a written story problem format. She was also unable to count money. On the Calculation test, Sabrina answered three problems incorrectly because of mixing up addition and subtraction symbols. She was unable to answer a simple two-digit subtraction problem and was unable to add three two-digit numbers without renaming. On the Math Facts Fluency test, Sabrina mixed up addition and subtraction symbols on several problems. Overall, Sabrina demonstrated limited abilities with math calculation and math reasoning.

Clinical Clusters

On the clusters of Academic Skills, Academic Fluency, and Academic Applications, Sabrina demonstrated abilities in the Limited range. She will likely find it difficult to perform at the same level as her classmates because of her overall academic difficulties, particularly in the areas of reading, spelling, calculation, and applied math. She also struggles with completing reading and math tasks with speed and efficiency, which further impacts her overall performance. Sabrina does demonstrate strength in sentence writing fluency. When given a few printed words as a prompt, she can write quickly and efficiently, and with correct spelling, capitalization, and punctuation. Sabrina's abilities in using phonics and orthography for reading and spelling were in the Average to Limited-to-Average range.

WJ IV Variation and Discrepancy Procedures

On the WJ IV, intra-individual variations are computed to show the likelihood of a person obtaining a particular score, given the average of their other core cognitive, oral language, and achievement test scores. Large variations indicate areas of significant strength or weakness. Sabrina's intracognitive abilities were consistent. She demonstrated average performance across most cognitive abilities, with no statistically significant variations among them. When examining Sabrina's intra-achievement abilities, written expression is a statistically significant strength. When comparing Sabrina's Fluid Reasoning (Gf) and Comprehension- Knowledge (Gc) composite (i.e., a combination of fluid reasoning, language, and general knowledge) with her achievement scores, reading is a statistically significant weakness. This means that her performance in reading is unexpectedly weak given her knowledge and reasoning abilities.

Summary and Diagnostic Impressions

Sabrina is a 7-year-old girl currently repeating the 1st grade, who was referred for an evaluation because of concern about her slow progress in the areas of reading and writing. She has experienced persistent difficulty with both reading and spelling. Sabrina was evaluated with the WJ IV COG, WJ IV ACH, and the WJ IV OL tests to assess and identify strengths and weaknesses within her abilities.

Sabrina demonstrates characteristics consistent with a specific learning disability in the areas of basic reading skills, which causes her difficulty in reading fluency and comprehension. Although Sabrina has learned some of the skills required for reading, she continues to demonstrate inconsistent performance as well as difficulties with the foundational skills of reading, such as accurately processing the sounds of language, and the basic skills of sounding out new words and reading sight words. In addition, Sabrina demonstrates characteristics consistent with a specific learning disability in the areas of math calculation and math reasoning. Although she was able to solve basic single-digit addition and subtraction facts and could answer some basic math reasoning questions with visual supports, she frequently mixed up signs, could not perform simple two-digit addition and subtraction problems, and had difficulty answering story problems presented without visual supports.

Sabrina has the specific learning disabilities of phonological dyslexia and mathematics disorder.

Recommendations

  1. Sabrina will benefit from direct instruction in a highly structured phonics-based reading program. Such instruction should be frequent (at least three times per week) and consistent. Such lessons should start by teaching the building blocks of reading, including sound segmenting, sound blending, basic phonics, and sight words. A multisensory approach will be most effective.
  2. Sabrina's instruction in spelling should be matched to the skills she is learning in her reading lessons. For example, spelling words should contain the phonics elements and syllable structures that she is learning in her reading. Spelling should be practiced daily. A brief practice test of three or four words given daily will be more conducive to mastery than a test of 20 words once per week.
  3. Irregular sight words (i.e., those that do not follow the rules of the English language) should be taught directly. Words such as “said” and “of” should be practiced in small sets daily. One or two irregular words from a high-frequency word list should also be included in her daily spelling practice.
  4. Sabrina will benefit from reading fluency drills to increase her reading speed. Simple one-minute passages can be practiced repeatedly, and her progress can be graphed so Sabrina can see her progress over time. Repeated reading passages should be given at her instructional reading level, and the level should be increased as she progresses.
  5. Sabrina will benefit from direct instruction in reading comprehension strategies, such as monitoring for meaning, forming questions, and developing summarizing skills. However, until Sabrina's reading skill approaches her oral language level, these skills should be taught orally. Later, teach her how to apply these skills to her reading.
  6. Sabrina will benefit from a decreased or modified homework load. Homework should entail brief practice of known skills rather than long drills or work that is disconnected from what she is learning in class.
  7. Sabrina would benefit from being given many opportunities to write and should be acknowledged for her skills in this area. She can be taught simple editing strategies such as the error monitoring strategy, which uses the acronym “COPS,” to review her work. (See Error Monitoring Strategy.) She should be taught how to identify her misspellings and to find resources to fix spelling errors.
  8. Sabrina can be taught simple strategies to keep track of math symbols. For example, she can be taught to circle and say the sign of each new addition or subtraction problem on a math facts worksheet.
  9. Sabrina also can be taught strategies for solving different types of story problems. Highlighting key words, using charts, or drawing pictures can help Sabrina to understand how to approach and solve math story problems.
  10. Sabrina will benefit from seeing her successes. Visual representations of progress in spelling, sight words, math problems, and so on can be charted so that Sabrina can see that she is improving in the areas that are difficult for her.

Thank you for the opportunity to work with Sabrina. She is a kind, funny, and thoughtful student, and I enjoyed our time together. If you have any questions, please feel free to contact me.

Woodcock-Johnson IV Score Report

Cognitive Abilities

Cluster/Tests SS* (95% CI*) PR* Descriptor
General Intellectual Ability (GIA) 97 (91–103) 42 Average
Oral Vocabulary 98 (87–108) 45 Average
Number Series 107 (95–118) 68 Average
Verbal Attention 95 (84–105) 37 Average
Letter-Pattern Matching 95 (81–108) 37 Average
Phonological Processing 89 (79–100) 23 Low Average
Story Recall 102 (92–111) 55 Average
Visualization 96 (86–107) 39 Average
Gf-Gc Composite 101 (94–108) 53 Average
Oral Vocabulary 98 (87–108) 45 Average
Number Series 107 (95–118 68 Average
General Information 98 (89–108) 45 Average
Concept Formation 100 (91–108) 50 Average
Comprehension-Knowledge (Gc) 98 (90–105) 45 Average
Oral Vocabulary 98 (87–108) 45 Average
General Information 98 (89–108) 45 Average
Fluid Reasoning (Gf) 104 (95–112) 61 Average
Number Series 107 (95–118 68 Average
Concept Formation 100 (91–108) 50 Average
Short-Term Working Memory (Gwm) 91 (82–100) 27 Average
Verbal Attention 95 (84–105) 37 Average
Numbers Reversed 89 (77–101) 23 Low Average
Auditory Processing (Ga) 84 (77–92) 14 Low Average
Phonological Processing 89 (79–100) 23 Low Average
Nonword Repetition 85 (77–94) 16 Low Average
Long-Term Retrieval (Glr) 97 (91–103) 42 Average
Story Recall 102 (92–111) 55 Average
Visual-Auditory Learning 94 (88–99) 34 Average
Visual Processing (Gv) 97 (88–106) 42 Average
Visualization 96 (86–107) 39 Average
Picture Recognition 99 (87–110) 47 Average
Quantitative Reasoning 106 (98–114) 66 Average
Number Series 107 (95–118) 68 Average
Analysis-Synthesis 104 (95–112) 61 Average
Cognitive Efficiency 92 (81–102) 30 Average
Letter-Pattern Matching 95 (81–108) 37 Average
Numbers Reversed 89 (77–101) 23 Low Average
*SS = Standard Score; CI = confidence interval; PR = Percentile Rank.

Oral Language

Cluster/Tests AE RPI Proficiency SS (95% CI) PR
Listening Comprehension 8–0 92/90 Average 103 (93–112) 58
Oral Comprehension 8–11 96/90 Average to Advanced 110 (98–123) 75
Understanding Directions 7–0 84/90 Average 94 (85–103) 34
Phonetic Coding 6–9 78/90 Limited to Average 93 (86–99) 32
Segmentation 6–1 30/90 Limited 82 (75–89) 12
Sound Blending 12–4 97/90 Average to Advanced 112 (100–124) 79

Reading

Cluster/Tests AE RPI Proficiency SS (95% CI) PR
Reading 6–8 25/90 Limited 83 (79–87) 13
Letter-Word Identification 6–9 24/90 Very Limited 85 (81–90) 16
Passage Comprehension 6–7 26/90 Limited 83 (76–89) 13
Broad Reading 6–7 17/90 Very Limited 83 (78–87) 13
Letter-Word Identification 6–9 24/90 Very Limited 85 (81–90) 16
Passage Comprehension 6–7 26/90 Limited 83 (76–89) 13
Sentence Reading Fluency 6–7 8/90 Very Limited 84 (76–93) 14
Basic Reading Skills 6–10 54/90 Limited 89 (85–93) 23
Letter-Word Identification 6–9 24/90 Very Limited 85 (81–90) 16
Word Attack 7–3 81/90 Limited to Average 94 (86–103) 34
Reading Comprehension 6–11 60/90 Limited 88 (84–93) 21
Passage Comprehension 6–7 26/90 Limited 83 (76–89) 13
Reading Recall 7–6 87/90 Average 97 (92–103) 42
Reading Fluency 6–9 31/90 Limited 86 (79–92) 18
Oral Reading 7–0 70/90 Limited to Average 92 (87–98) 30
Sentence Reading Fluency 6–7 8/90 Very Limited 84 (76–93) 14

Writing

Cluster/Tests AE RPI Proficiency SS (95% CI) PR
Written Language 7–6 85/90 Average 97 (92–102) 42
Spelling 7–0 65/90 Limited 90 (82–98) 25
Writing Samples 8–3 94/90 Average 105 (105–114) 63
Broad Written Language 7–9 90/90 Average 100 (94–106) 50
Spelling 7–0 65/90 Limited 90 (82–98) 25
Writing Samples 8–3 94/90 Average 105 (105–114) 63
Sentence Writing Fluency 8–8 96/90 Average to Advanced 110 (98–121) 75
Written Expression 8–5 95/90 Average 108 (99–116) 70
Writing Samples 8–3 94/90 Average 105 (105–114) 63
Sentence Writing Fluency 8–8 96/90 Average to Advanced 110 (98–121) 75

Mathematics

Cluster/Tests AE RPI Proficiency SS (95% CI) PR
Mathematics 6–10 53/90 Limited 87 (82–93) 19
Applied Problems 6–8 57/90 Limited 85 (75–96) 16
Calculation 6–10 50/90 Limited 88 (81–94) 21
Broad Mathematics 6–10 52/90 Limited 87 (80–93) 19
Applied Problems 6–8 57/90 Limited 85 (75–96) 16
Calculation 6–10 50/90 Limited 88 (81–94) 21
Math Facts Fluency 6–11 49/90 Limited 89 (79–95) 23
Math Calculation Skills 6–10 49/90 Limited 87 (79–95) 19
Calculation 6–10 50/90 Limited 88 (81–94) 21
Math Facts Fluency 6–11 49/90 Limited 89 (79–95) 23

Clinical Clusters

Cluster/Tests AE RPI Proficiency SS (95% CI) %ile
Academic Skills 6–10 45/90 Limited 87 (83–91) 19
Letter-Word Identification 6–9 24/90 Very Limited 85 (81–90) 16
Spelling 7–0 65/90 Limited 90 (82–98) 25
Calculation 6–10 50/90 Limited 88 (81–94) 21
Academic Fluency 7–0 55/90 Limited 89 (82–96) 23
Sentence Reading Fluency 6–7 8/90 Very Limited 84 (76–93) 14
Sentence Writing Fluency 8–8 96/90 Average to Advanced 110 (98–121) 75
Math Facts Fluency 6–11 49/90 Limited 89 (79–95) 23
Academic Applications 7–0 66/90 Limited 91 (86–95) 27
Applied Problems 6–8 57/90 Limited 85 (75–96) 16
Passage Comprehension 6–7 26/90 Limited 83 (76–89) 13
Writing Samples 8–3 94/90 Average 105 (105–114) 63
Phoneme-Grapheme Knowledge 7–6 87/90 Average 98 (92–104) 45
Word Attack 7–3 81/90 Limited to Average 94 (86–103) 34
Spelling of Sounds 8–0 92/90 Average 102 (91–113) 55

Reason for Referral

Delia was referred for an evaluation to determine her present levels of cognitive and academic functioning secondary to a head injury. The family's insurance company is seeking an opinion regarding which, if any, of her present difficulties are related to the head injury. A separate evaluation is being conducted by school staff to determine appropriate services and educational recommendations for Delia. The results of these evaluations will be shared at a multidisciplinary team meeting in April.

Background Information

Delia is a 7-year-old Native American and Hispanic girl in the second grade. She is attending Sam Watson Elementary School in the Allen Independent School District in Allen, Texas. She lives with her father, mother, and two older sisters. Delia, her sisters, and her mother are registered members of the Coushata tribe.

Developmental History

Delia is the product of an unremarkable pregnancy and birth with a weight of 8 pounds, 0 ounces. Developmental milestones were reportedly reached within normal limits. According to Delia's Individualized Education Program (IEP), written on May 25, 2014, at the end of first grade, Delia's fine and gross motor skills are appropriate for her age. She writes with a variety of instruments with ease. Large motor skills, such as running, jumping, skipping, throwing a ball, and jumping rope, are within normal limits. The IEP also states that her interaction with peers is also age-appropriate. Her current teacher wrote: “Delia gets along well with her peers, and she is well liked. She has a great sense of humor and can build relationships with peers and adults. She is open and friendly with all of her teachers. Delia is also very kind and considerate.” In contrast, her mother reports that, at home, she has been emotionally closed off since her head injury. She has been receiving counseling services and has shown improvement in sharing emotions.

Brief Medical History

Dr. Calvin Moats, a developmental pediatrician at Franklin Medical Center, summarized Delia's accident as follows: At age 6–2 (July 2013), Delia was diagnosed with a mild traumatic brain injury (TBI) after she was struck by a car at a shopping mall. Delia was walking with her mother when a dog darted up. Startled, Delia ran into the street and was hit by an oncoming car. She flew ten feet and hit her head as she landed. Since the accident, she has had recurrent headaches and fatigue, and has demonstrated behavioral changes, anxiety, and possible cognitive deficits. At Dr. Moats's recommendation, Delia participated in counseling sessions from age 6–7 to age 7–3, specifically addressing symptoms related to anxiety.

Educational History

Kindergarten

Delia has attended Sam Watson Elementary School since kindergarten. On enrollment, she primarily spoke Spanish, although her mother reported that the family speaks both Spanish and English at home. In kindergarten, she was considered an English language learner (ELL). According to the results of the Texas English Language Proficiency Assessment System (TELPAS), her English was “limited,” and her academic achievement was “below grade level.” At the second quarter, she was placed in the Structured English Immersion (SEI) Program, a daily 4-hour class in which she and other Spanish speakers are pulled out for English Language Development (ELD), a program in which children move up through levels as their proficiency increases.

At the end of her kindergarten year, her teacher reported, “Delia is one of the sweetest students I have had the pleasure of having in my class. She had a smile and hug for me every day. I've been amazed by her desire to learn everything about everything. She has grown so much this year, and I am so proud of all her accomplishments. She tried her best in all she did and that is why she has become a good reader and writer.”

Delia's most recent progress report prior to her injury was her end-of-the-year report card in kindergarten:

  1. Reading Process: Meets the Standard (all three subtests)
  2. Writing Process: Meets the Standard (both subtests)
  3. Math Strands: Meets the Standard (all three subtests)
  4. Science Strands: Meets the Standard (both subtests); Approaches (both subtests)
  5. Content Standards: Meets the Standard (all four subtests); Exceeds (one subtest)
  6. Social and Emotional Growth and Learner Qualities: Meets the Standard (all four subtests); Exceeds (one subtest)

First Grade

Delia suffered her head injury during the summer before first grade. At the beginning of first grade, the SEI teacher observed that Delia was having some difficulty communicating in Spanish, but because the purpose of the class was to develop English, the team determined that it was still an appropriate placement for her. Consequently, throughout the remainder of the day, in first grade and now in second grade, she receives sheltered content instruction, a method of teaching in which language development is integrated with grade-level content in mainstream classrooms. Delia's school attendance from kindergarten through second semester of second grade has been between 89% and 94%.

In October of Delia's first-grade year, her parents received a letter from Delia's school reporting that she was not reading on grade level, needed strategic and intensive intervention, and would be working three times per week with the school reading specialist. In March of that school year, a Child Study meeting was conducted to address Delia's difficulties at school. Despite the special help, her TELPAS scores had dropped in reading and writing since the previous quarter. During the meeting, Delia's mother reported that Delia had been experiencing many challenges since her head injury. She could no longer speak Spanish, her first and primary language. Also, she sometimes forgot what she was doing and had to ask her mother, and she had difficulty retaining information. She also needed a regular bathroom schedule at home so as to avoid accidents. Her teacher reported that Delia needed to have routines repeated to her and that she consistently needed extra time to complete writing tasks. She had difficulty remembering details of stories that the teacher read to the class and then discussed. Her teacher had also established a regular schedule for bathroom breaks for her; however, on one occasion, when the school schedule was abnormal (because of testing), Delia soiled her clothes. None of these issues were evident in kindergarten. A review of records indicated that, in kindergarten, Delia was consistently performing at least at grade level. The team discussed the difficulty of ascertaining whether Delia's language and learning problems were attributable to her status as an English language learner, which would not explain her loss of Spanish, or if they were caused by the brain injury. The team recommended a language evaluation, for which Mrs. Sanchez granted permission.

At the end of the first grade year (May 2014), the Multidisciplinary Evaluation Team (MET) got together to discuss the results of the testing, which had been conducted only in English, because Delia demonstrated little use of Spanish. No concerns were noted regarding Delia's oral structure, fluency, voice, or articulation. The following standard scores were reported for the Clinical Evaluation of Language Fundamentals, Fourth Edition (CELF-4):

  1. Receptive Language Index: 79 (Low range of functioning);
  2. Expressive Language Index: 69 (Very Low range of functioning);
  3. Language Content Index: 64 (Very Low range of functioning);
  4. Language Structure Index: 79 (Low range of functioning);
  5. Core Language Score: SS = 73 (Low range of functioning).

Delia qualified for special education services under the category of Speech/Language Impairment. Goals included building vocabulary, improving memory, and formulating complete grammatically correct sentences. Delia's results on the Texas State tests at the end of first grade were:

  1. Reading: Falls far below the standard
  2. Writing: Approaches the standard
  3. Math: Falls far below the standard

Second Grade

This year, on Delia's second quarter progress report (12/19/2014), her teacher wrote the following comment: “Delia puts minimal effort into her school work. Her assignments are often incomplete and reflect little care or effort on her part. Cutting her work load in half does not produce different results. Delia is a capable student who is not working to her full potential.”

Observations and Impressions

Delia was seen for two formal testing sessions. She looked around and smiled when she came into the testing office, because it is a warm, comfortable room with a small table, a couch, and a giant stuffed bear. Delia appeared relaxed throughout the evaluation. The first day, she coughed frequently, which her mother reported was because of a medical procedure completed days earlier. Once per hour, she was given the opportunity to take a bathroom break. She was friendly, easily engaged, and fully cooperated. Although she was clearly putting forth her best effort, she sometimes lost attention during the instructions and needed them repeated; at other times, she appeared to be attending to the instructions and was successful on the trial items, but shortly afterward, forgot what she was supposed to do. Delia is right-handed and demonstrated adequate hearing and vision.

Tests Administered

  1. California Verbal Learning Test, Children's Version (CVLT-C)
  2. Developmental NEuroPSYchological Assessment (NEPSY-II)
  3. Rey-Osterrieth Complex Figure Test (RCFT)
  4. Woodcock-Johnson IV Tests of Cognitive Ability (WJ IV COG)
  5. Woodcock-Johnson IV Tests of Oral Language (WJ IV OL)
  6. Woodcock-Johnson IV Tests of Achievement (WJ IV ACH)
  7. Parent Interview
  8. Review of Records

Explanation of Scores

Within this report, scores are reported as percentile ranks, indicating the percentage of children of Delia's age (year and month) in the norming sample who scored lower than she did. The descriptive labels below represent percentile rank ranges.

Percentile Range Percentile Range
Very Superior 98 to 99.9 Low Average 9 to 24
Superior 92 to 97 Low 3 to 8
High Average 76 to 91 Very Low 0.1 to 2
Average 25 to 75 —- —-

Delia's performance is sometimes described by the relative proficiency index (RPI). The RPI indicates the level of success a person is predicted to have on tasks similar to those used in the test. The comparison is always with average age-peers who have achieved mastery, represented by 90% proficiency. For example, Delia's RPI of 15/90 on the Letter-Word Identification test indicates that she has a 15% chance of success on similar word identification tasks when a typical age-peer would have a 90% chance of success. Verbal descriptors for each RPI range indicate proficiency levels and the level of difficulty with which Delia is likely to perceive the task. Because the RPI is derived from the person's initial score differently than the standard score or percentile rank, it often shows a person's difficulty with a task when the other scores do not.

RPI Instructional Implication RPI Instructional Implication
100/90 Extremely easy 67/90 to 82/90 Difficult
98/90 to 100/90 Very easy 24/90 to 67/90 Very difficult
95/90 to 98/90 Easy 3/90 to 24/90 Extremely difficult
82/90 to 95/90 Manageable 0/90 to 3/90 Nearly impossible

Unless otherwise indicated, these descriptors are based on comparison with age-appropriate statistical norms. When adequate group norms are not available, more general terms reflecting behavioral observations and clinical judgment are used (e.g., typical, weak).

Test Results and Interpretation

Cognitive Abilities

Cognitive abilities are different types of mental activities, each of which has its own job to do in helping us to think and function throughout the day. When a task or problem comes up, the relevant abilities start working. The cognitive abilities function like a team; although each has its own job to do, it also relies on the performance of the others to perform successfully and efficiently. A weakness in one impairs, to some degree, the functioning of the others.

Some cognitive abilities are thought of as automatic, in that they happen without conscious thought, such as immediate memory (holding information in mental awareness for a second or two immediately after perceiving it) or visual perception (instant recognition of objects, shapes, or symbols). Other cognitive abilities involve more mental work and are considered higher-level abilities, such as using logical reasoning to solve problems, or working with information in immediate memory so as to transfer it into long-term memory.

This section describes Delia's level of functioning and efficiency in some of the most important cognitive abilities.

Attention and Executive Functioning

Delia's attention and executive functioning abilities were assessed through school record reviews, parent interview, and formal tests. Attention and concentration difficulties have been noted at home and school. Executive functioning is the ability to keep track of and regulate your thoughts and actions, while evaluating the efficacy of what you are doing. It involves a variety of abilities, such as planning, initiating actions when appropriate, inhibiting behaviors that are not appropriate, talking to yourself to guide your own behavior, and problem-solving.

Delia generally demonstrated underdeveloped executive functioning in several areas, including sustaining attention and selective attention, creating strategies to facilitate learning or to make tasks easier, inhibiting impulsive actions, and being flexible in her thinking (i.e., cognitive flexibility: the ability to change the way you think or act depending on the situation, such as switching between one set of rules and another, or perceiving the same situation, task, or problem in multiple ways).

Delia's performance was significantly weak on a test of auditory attention, which called on several executive functions, including sustained and selective attention, inhibiting impulsive responses, and cognitive flexibility (NEPSY II: Auditory Attention, Auditory Attention and Response Set). Delia was required to point to various circles according to verbal instructions. Although instructions were clearly stated at the beginning of each task and Delia was successful on the practice items, during the test, she forgot the instructions or did not attend sufficiently to the prompts she was hearing. Results indicated that Delia's correct responses were in the Low to Low Average range, equal to or above only 9% of her age-peers, indicating, overall, a lack of sustained attention to the task and limited selective attention (i.e., remembering to point when she heard certain words while remembering not to point to any others). Delia demonstrated a high level of impulsivity, responding when she should not, and somewhat fewer omission errors, not responding when she should have. Her scores were in the Very Low and Low Average range, respectively. When the instructions for the task changed, Delia was not able to change her thinking accordingly. She had somewhat fewer impulsive responses but more items to which she did not respond at all. Possibly, the rule changes in the task made it sufficiently difficult that it exceeded her ability to sustain focused attention, increasing her nonresponses.

Delia was also administered a test of learning and memory (California Verbal Learning Test—Children, CVLT-C). This test required her to learn a list of 15 words that fit into three categories, although she was not told about the categories. She was given five trials in which the evaluator read the list (List A) aloud, and then she named all the words she could remember. Her scores on the first and last trials were in the higher part of the Average range, in the 69th percentile. On the other trials, however, her scores were so low that they brought her overall score down to the 10th percentile. She was then given a different set of words to repeat but was unable to separate the second list (List B) from the first in her mind. Her score (2nd percentile) indicated an unusually high degree of proactive interference (i.e., List A stayed active in memory, interfering with learning List B). Yet after a short delay, and without hearing List A again, she brought her score up to the 50th percentile (mid-average). At no point had she recognized that she could group the items and thus make it easier to remember, but when the evaluator suggested a category, Delia recalled more items (e.g., Can you remember any more vegetables?). Following the same routine after a longer delay, Delia eventually brought her score back up to the 69th percentile and with further cuing, to the 84th percentile.

The design of the CVLT-C typically keeps the child actively engaged throughout; still, at times Delia's attention appeared to wander and, not surprisingly, her recall of what she had been learning diminished. She did not demonstrate strategic thinking, such as organizing the items into three groups of five items each, which would have made the list much easier to memorize. Her response to the prompt regarding thinking of the items in groups was helpful to her, suggesting that although she may not create her own strategies on her own, she would benefit from having them taught to her. As well, she benefited from repetition of what she needed to learn and memorize. Finally, her ability to quickly shift her thinking to a new topic or activity was limited. Consequently, when she is learning information or is focused on an assignment, it is important not to switch gears suddenly and introduce an unrelated academic activity. She might need an activity that would act as a buffer.

Intellectual Functioning

General intelligence is just a way of summarizing how well a person's cognitive abilities generally function as an interactive system. The level of a person's intelligence is not unchangeable; it can increase over time with the right instruction and environmental stimulation. Or, if a person is left alone, with little interaction or stimulation, it can decrease. According to the WJ IV General Ability Intelligence composite (GIA), Delia's overall intelligence, at this time, is in the Low Average range, meaning that 15% of other students her age had a score the same or lower. Compared with her GIA standard score of 84, she did not demonstrate significant strengths or weaknesses in any of her cognitive abilities. However, when one considers the relative proficiency indexes (RPI), appreciable differences among her abilities are evident.

Fluid Reasoning

One important cognitive ability that is considered a high-level ability is fluid reasoning. Fluid reasoning is the process of focusing one's attention on a task or problem, activating the other relevant cognitive abilities, and remaining flexible enough to consider various ways to do the task or solve the problem. When using fluid reasoning, the information and procedures a person has learned previously may be useful but are not always sufficient to solve the problem. Consequently, the person has to use whatever information is available and logical reasoning.

Delia took three tests that required fluid reasoning. Her test results indicate that her ability to use reasoning to solve new types of problems and successfully accomplish new types of tasks may depend on how simple they are. Her performance on one test provides a good example: She was supposed to take a set of eight cards and sort them into two groups (four cards each) as many ways as she could (e.g., sort foods into vegetables and meat). Although she created enough correct groupings to score just into the Average range, the number of incorrect groupings and groupings she had already made lowered her score to the 2nd percentile (NEPSY II Animal Sorting). She performed almost as poorly on a test in which she was to figure out the distinguishing feature that separated two groups of colored shapes (WJ IV Concept Formation: PR 8). Her RPI on this test suggested that she would have a 41% chance of success on similar tasks when her average age-peers would have a 90% success rate.

The third test consisted of number patterns—finding the missing number in a series of four. Almost all of the patterns she solved required her to simply start at the first number and count up by ones; two items required her to count backward by ones. For her age, her performance was well into the Average range, 41% of her age-peers had a score the same or lower. Her RPI was also in the Average range. This particular test was one that required some prior knowledge (i.e., starting at different numbers and counting by ones), prior knowledge that, apparently, Delia has.

Delia's scores on these tests and analysis of what she could and could not do indicate that her ability to reason in novel situations is limited. She is unlikely to infer rules of reading or spelling from seeing multiple incidences of them (e.g., figuring out that vowel-consonant-e makes the vowel sound long) or figuring out why the steps in a math algorithm go together. Once she knows a procedure, however, she is likely to be able to follow it, if she knows the situation in which it is appropriate to use it.

Comprehension-Knowledge

Delia's comprehension-knowledge, also referred to as “crystallized or acquired intelligence,” can be described as a combination of her language development and the general knowledge she has acquired through her life experiences and education. As such, it can be strongly influenced by the culture and values of the home. Knowledge and the ability to use language form the basis for new learning. People understand and retain new information more easily when they can relate it to something they already know. So a person who has been to an aquarium or spends time near the ocean has a better foundation for understanding text or a lecture about marine life than the person who lives in a desert area and has no familiarity with marine life. Comprehension-Knowledge is organized, consolidated, and stored in long-term memory, “the mind's warehouse.” Delia's performance on comprehension-knowledge tasks was in the Low range; only 4% of her age-peers had a score the same or lower. Her RPI of 49/90 indicates that when average 7-year-olds are 90% successful on similar tasks, she is likely to be only 49% successful. Because comprehension-knowledge is based on knowledge of language and the mainstream culture, a low score on this cluster (group) of tests is not unusual in a person who comes from another culture and who is learning a new language. It does indicate, however, that Delia needs more experience with and information about her surroundings and a broader English vocabulary.

Memory and Long-Term Retrieval for Verbal Information

Delia was administered multiple tests of memory, assessing different specific abilities within this broad cognitive area. These included memory for unrelated elements, memory for meaningful information (e.g., stories), paired associate learning, short-term working memory, and long-term retrieval.

Meaningful Memory

Delia had difficulty in three tests of meaningful memory. Consequently, one must consider whether her difficulty was in understanding of the material in the first place or remembering it. The results indicate that understanding or remembering individual sentences, such as instructions, with complex structures is likely to be difficult for her, even if accompanied by a related picture. She also had difficulty on two tests requiring the retelling of brief stories she had just been told, indicating, again, that either she did not understand the stories or she was unable to remember the details. Her scores were in the Low to Low Average range. It is likely she did not understand the stories, because she did not remember more when she was asked open-ended questions (e.g., Where were the children going?) or yes-no questions (Was there a squirrel in the story?).

Short-Term Working Memory

Working memory may be defined as “…the ability to hold information in mind while performing complex tasks. It incorporates the ability to draw on past learning and experience to apply to the situation at hand or to project into the future” (Dawson & Guare, 2012, p. 8). Information enters mental awareness through our senses or is retrieved from long-term memory. However, it is only available for conscious thought while we focus our attention on it. If no attention is directed toward it, the information dissipates immediately. If a person can remain focused on the information, and hold it in short-term working memory while other cognitive abilities work on it, then it can be transferred into long-term memory, and thus be available for later retrieval and use. Although short-term working memory is a dynamic process, long-term memory is a storage facility, “the mind's warehouse.”

Delia took four different tests of short-term working memory, in which she worked with unrelated elements. The only one on which she did poorly was repeating series of numbers in reverse order. Her RPI of 68/90 suggests that the task of mentally reordering numbers was Difficult to Very Difficult for her.

In contrast, Delia performed in the Average range on a task of listening to a string of four words and holding them in memory until the evaluator asked a question about the position of one them (e.g., What word came just before rabbit?). Her RPI of 83/90 indicates that this type of task may be somewhat difficult but manageable.

Learning and Long-Term Retrieval

Delia also did well on a test in which she listened to a list of 15 words and repeated those she remembered. (CVLT-C: See Attention and Executive Functions.) Overall, it appears that if she is paying attention, her ability to hold in memory and work with unrelated elements is at least Average compared with her age-peers. She was also able to retrieve this information after a short and then a long delay. She did not develop a strategy to make the task easier but benefited when one was indirectly given her.

A different task of learning and long-term retrieval required her to learn words paired with unfamiliar symbols (e.g., img = book) and then “read” a story written with these symbols. The sentences are all short, with a simple structure. Delia seemed to perceive this test as a game and was particularly engaged. She liked being given the word when she got one wrong or could not remember it, and she learned from repeated feedback. Her score was solidly in the Average range (Visual-Auditory Learning: PR 44, RPI 88/90).

Although Delia's inconsistent attention somewhat limits interpretation of the results of her memory and long-term retrieval abilities, some generalizations can be made. She can learn lists of unrelated elements and hold them in memory for a few seconds, but if she needs to learn them for later repetition or use, she is particularly helped by repetition. She can learn word-symbol associations, store them in long-term memory, and retrieve them over several minutes but, again, repetition is key. She appears to have difficulty understanding brief stories and is thus unable to retell them, and she has difficulty mentally reordering numbers.

Cognitive Processing Speed and Perceptual Speed

Processing speed is the ability to quickly and fluently perform relatively easy or over-learned tasks, especially when focused concentration is required. Processing speed is important because when the simpler or more basic mental processes work quickly and fluently, the mind and memory are available to focus on more complex and higher-level tasks. Rapid performance of basic arithmetic operations, reading speed, and writing speed are academic skills directly related to processing speed. For example, a beginning reader with fast processing speed is likely to become a fluent reader quickly because she will more easily create the automatic associations between letters and sounds.

Perceptual speed is a subskill of processing speed; it involves a person's ability to rapidly and accurately compare visual symbols (e.g., letters, numbers, math symbols) and recognize similarities and differences. The faster one can recognize something, the faster one can use it. Perceptual speed is important in developing automatic identification of symbols for use in higher-level tasks such as reading, writing, and math.

The WJ IV uses different combinations of three tests to measure processing speed and perceptual speed. Because both clusters contain significant discrepancies between the scores of the component tests, consideration of the individual abilities is appropriate. All three tests required Delia to scan rows of letters, numbers, or small pictures to find matches as fast as possible within a time limit. When matching numbers of one to three digits and finding repeated instances of pairs of pictures, Delia's scores were in the Average range. Her RPI, however, indicated that her likelihood of success on similar tasks would be 67% (numbers) and 74% (pictures) when other students of her age have 90% success. Consequently, these tasks are likely to be difficult for her. On the letter matching test, she mismatched letters and letter combinations and in doing so, chose some letter combinations that do not exist in English spelling. This was not because of her status as an English language learner, because those patterns do not exist in Spanish either. Although her score was in the Low Average range, her RPI indicated that when typical age-peers quickly recognize letter patterns with 90% success, Delia's likely success rate would be 38%.

Overall, Delia's cognitive processing speed and her perceptual speed appear to be slow, but the excessive difficulty with letter matching also points to a lack of experience or facility with letters. Similarly, she also may lack experience or facility with numbers, although not to the same extent. Her poor performance on the picture matching test reinforces the likelihood that Delia's cognitive processing and perceptual speed are inefficient. This inefficiency likely makes it more difficult for Delia to make associations between letters/letter combinations and sounds for reading, and in recognizing number combinations for use in learning math facts.

Visual Processing, Visual Fine-Motor Skills, and Visual Memory

Delia was given tests of visualization, visual-motor planning and execution, and visual memory. Visual processing is the ability to use mental imagery for a variety of purposes.

Visualization

Visualization, as used here, is an aspect of visual–spatial relations—the ability to grasp the spatial relationships among objects and to imagine what an object, design, or mental image would look like from a different perspective or if the components were rearranged. Visualization is important for artwork and picturing how the classroom will look if the desks were rearranged. Delia did reasonably working with problems drawn in two dimensions, similar to mentally putting a puzzle together, but had difficulty with the second part of the test, which required identifying the two block patterns that matched the target pattern. Although she scored in the Low Average range, her RPI indicated that this type of task would be difficult for her (WJ IV Visualization: PR 16, RPI 72/90).

Visual Perception, Planning, and Visual Fine-Motor Skills

Delia was shown a complex figure made of multiple, overlapping geometric shapes and was asked to draw it. Her approach to copying the figure was well planned, focusing first on the larger outlines and then filling in the smaller details. She appeared confident while drawing the figure, and her final product was neat. Her execution of this task indicated good perception of the figure, appropriate planning, and good coordination of her vision and small hand movements. Her handwriting is also neat, reinforcing the finding of good visual fine-motor skills. Her production of the figure was in the higher part of the Average range (Rey-Osterrieth Complex Figure Test [RCFT]: PR 61).

Visual Memory

Delia's strongest skills were in visual recall and recognition of visual images. After a delay and without a previous warning, Delia was asked to draw the previous figure from memory. Again, she appeared self-confident, and her drawing was a good reproduction. She also performed well on a test that involved recognizing the picture she had just seen when it was shown again in a larger set of similar pictures (e.g., choose the coat she had seen out of a set of four coats). Neither test involved any language (RCFT: PR 69; WJ IV Picture Recognition: PR 68, RPI 94/90).

Oral Language

Delia's oral language was assessed in both English and Spanish to determine whether a clear difference existed between her competence in these languages. Because her mother reported that she has lost her ability to understand and speak Spanish and she attends a school where English is the primary language, English was tested in more depth.

English Vocabulary and Comprehension

Delia's conversational language appeared to be age-appropriate when speaking about basic information. The tests of the WJ IV and the NEPSY II assess language as required for academic achievement. Results of these tests indicated that Delia's oral language is significantly less well developed than that of her age-peers. Her vocabulary score was in the Low range, equal to or above only 6% of her age-peers. Her RPI indicated that when her average age-peers would be 90% successful in naming pictures of objects and providing synonyms and antonyms for given words, Delia would be only 53% successful. On a test in which she listened to one-sentence verbal instructions of increasing length and complexity and responded by pointing to objects in a picture, her RPI was 75/90, indicating that this type of task would be difficult for her. Additionally, her understanding of one- to two-sentence passages on a variety of topics was in the Low to Low Average range. Her RPI of 60/90 indicated that this type of task would be very difficult for her.

A limitation to the interpretation of low language comprehension is that, except for the test that required following instructions, the other tests required Delia to provide an oral response. Possibly she understood more than she was able to demonstrate but, with such a restricted vocabulary, did not have or could not find the correct word for the response. For example, she may know what the word “invent” means but not know a synonym for it. Time constraints precluded the possibility of adding a test of receptive vocabulary to the evaluation.

English Word Retrieval

Delia also demonstrated difficulty with rapid retrieval of simple words. She was given pages with rows of shapes, colors, letters, or numbers and told to name them as fast as she could. Although she named most of them correctly, she was slow, equaling or exceeding only 5% of others her age. She found it easier to come up with words when she was given a category (e.g., clothing) and could give any word she could think of that fit rather than having to come up with a specific word. This was also a timed task but she scored at the 25th percentile, her highest oral language score.

English Phonological Processing and Retrieval

Delia does not have the understanding that words are composed of individual sounds, a critical aspect of becoming an accomplished reader. She had difficulty retrieving from memory words based on individual sounds, with or without a time limit (e.g., “Tell me a word that begins with the /f/ sound”). On a similar test of rapid retrieval from the NEPSY II, her score was considerably lower. When repeating nonsense words of two syllables, she said the right number of syllables but confused similar sounds, such as /m/ and /n/. She could change the initial sound of a word to make a new word but not the last or middle sound (Say “cake.” Say it again but change /k/ to /m/). Although her scores on the tests of the WJ IV were in the Low Average range, her RPIs on the four tasks were between 72/90 and 75/90, indicating that she will find tasks requiring these skills to be difficult.

Another way of looking at these tests is as metacognitive tasks. To understand them well, a person has to be able to think of words not in terms of a word's meaning, but in terms of their building blocks, the sounds and syllables. Some children develop these understandings on their own; others, like Delia, need explicit teaching.

Spanish Vocabulary, Comprehension, Short-Term Working Memory

A fully fluent Spanish language speaker was present during the administration of Spanish tasks to ensure that Delia was provided a full opportunity to demonstrate her oral language abilities in Spanish. Of all the tests in this evaluation, her performances on the Spanish language tests were by far the lowest. Her scores on all three tests were equal to or higher than only 1 in 1,000 children of her age. Contrary to her upbringing as a bilingual child with Spanish as her primary language, she performed like a child who knows no Spanish. Despite her difficulties, as described previously, her facility in using English is far stronger than her current status in Spanish.

The Comparative Language Index (CLI) provides a comparison of Delia's scores on the parallel English and Spanish Broad Oral Language clusters, which are composed of the Picture Vocabulary, Oral Comprehension, and Understanding Directions tests. This index compares the student's performance on the same underlying abilities and is presented as a ratio between the person's RPI numerators, with the Spanish numerator being presented first. Delia's likelihood of success on similar Spanish tasks would be 1% when her likelihood of success on the same English tasks would be 59% (CLI = 1/59). This means that Delia is predicted to perform oral language tasks in Spanish with only 1% proficiency and 59% proficiency in English, when average 7-year-olds perform those tasks with 90% proficiency.

Academic Achievement

Reading

CLUSTER/Test RPI Proficiency Instructional Implications SS (68% CI) PR
BASIC READING SKILLS 19/90 Very Limited Extremely difficult 77 (74–79) 6
Word Attack 25/90 Limited Very difficult 73 (68–78) 4
Letter-Word Identification 15/90 Very Limited Extremely difficult 80 (77–82) 9
READING COMPREHENSION —- —- —- 81 (78–84) 10
Passage Comprehension 23/90 Very Limited Extremely difficult 76 (71–80) 5
Reading Recall* 82/90 Average Manageable* 93 (90–96) 31
READING FLUENCY 4/90 Very Limited Extremely difficult 73 (69–77) 4
Oral Reading 30/90 Limited Very difficult 78 (75–81) 7
Sentence Reading Fluency 1/90 Ext Limited Nearly impossible 74 (70–79) 4
*Invalid

Overall, the reading assessment indicates that Delia does not have functional reading skills. Compared with other children of her age, all of her scores were in the Low or Low to Low-Average range. Her RPIs, however, indicate that all reading skills will be very difficult to nearly impossible for her. The tasks involved reading real and nonsense words, comprehension of individual sentences, oral reading accuracy, and silent reading speed with comprehension. Delia does not know the sounds of all of the consonant digraphs and does not yet know that closed syllable words have a short vowel sound. She tried to sound out irregular words rather than simply identifying them as whole words (e.g., what, could). Her difficulty with basic decoding made comprehension nearly impossible, although she made some good guesses, and silent reading extremely slow.

Delia's Reading Recall score should be considered invalid. The task is to read silently and then retell the story, but credit is given solely for the number of target words said. The target words happened to be words that Delia could sound out or recognize. Her sentences in retelling the story did not always make sense or indicate that she understood the gist of the stories, but she included enough target words to score in the Average range.

Writing

CLUSTER/Test RPI Proficiency Instructional Implications SS (68% CI) PR
Spelling 23/90 Very Limited Extremely difficult 79 (75–82) 8
Spelling of Sounds 70/90 Limited to Average Difficult 87 (81–92) 19
WRITTEN EXPRESSION 81/90 Limited to Average Difficult 93 (90–97) 33
Writing Samples 84/90 Average Manageable 97 (93–100) 41
Sentence Writing Fluency 77/90 Limited to Average Difficult 90 (85–96) 26

The WJ IV writing tests assess spelling, including knowledge of phoneme-grapheme correspondences (representing each sound with the correct letter or letter combination) and orthography (spelling words and parts of words that are not spelled as they sound, such as the past tense ending “ed” in “jumped”). They also assess the ability to write single sentences according to a variety of task demands (some of which are just fill-in-the-blank) and the ability to generate and write short, simple sentences.

Delia's spelling reflected what she knows about reading. She spelled the nonsense words exactly as they sound, without applying the rules of conventional spelling. Her approach to spelling real words was the same. Sometimes it worked, but mostly it did not. For example, phonetically correct spelling works for cat but not for kick (spelled as kik).

Delia performed in the Average range on the Writing Samples test, with an RPI of 84/90. On this test and Sentence Writing Fluency, spelling, punctuation, and minor grammatical errors do not count as errors. On Writing Samples, her sentences were simple, but most met the criteria necessary to earn some credit. Her RPI of 77/90 on Sentence Writing Fluency reflects her work speed. This is a time-limited test; her slightly lower RPI reflects the time it took for her to ask what some of the words said and then to formulate a sentence. Three words were provided for each item and were to be included in the sentence. On a few sentences, she used only two. Later, when asked why she had left out some words, she said that she just forgot that she was supposed to use them all.

Delia's handwriting is neat, with consistent spacing, size, and formation of letters and words. She applied capital letters at the beginning of each sentence followed by a period. She also used commas appropriately.

Math

CLUSTER/Test RPI Proficiency Instructional Implications SS (68% CI) PR
MATH CALCULATION SKILLS 27/90 Limited Very Difficult 80 (76–84) 9
Calculation 24/90 Limited Very Difficult 78 (73–82) 7
Math Facts Fluency 31/90 Limited Very Difficult 84 (78–90) 14
Applied Problems 63/90 Limited Very Difficult 87 (83–92) 20
Number Series 85/90 Average Manageable 97 (92–101) 41

On the tests of Calculation and Math Facts Fluency, Delia appeared to know most of the simple addition facts, although occasionally she seemed to be counting on from the top number. On both tests, she had a tendency to add despite the subtraction sign. She did not complete any subtraction problems correctly. As shown on the table, her math computation skills are in the Low to Low Average range, but her RPI of 27/90 provides a better reflection of her skills. Basic math calculation on the second-grade level is likely to be very difficult for her. Her understanding of the word problems was relatively better, because the earlier problems had pictures associated with them. She did subtraction problems that had a picture, but for those without one, she looked at the evaluator with a puzzled expression and then said, “Don't know.”

As described previously, she did well on the Number Series test because all of the items at her level required counting by ones or twos, which she can do.

Summary

Delia is currently age 7–10 and is in the third quarter of second grade. When she started kindergarten, her primary language was Spanish, and she spent that year in Spanish immersion and pull-out classes. Subsequent to an accident at age 6–2 in which she hit her head on the ground and suffered a mild traumatic brain injury, she began to lose her Spanish. Currently, she understands a few of the most common Spanish phrases that her mother speaks to her, but she no longer can speak any Spanish. She is also having significant difficulty in aspects of all school subjects. The purpose of this evaluation was to try to ascertain her current levels of functioning and whether any of her present difficulties are a result of the head injury.

Attention and Executive Functions

Review of records, discussion with Delia's teacher and mother, and test results show that Delia has significant difficulties in sustained and selective attention to task and in class discussions. Results of executive functions tests indicate that despite apparent effort to please and put forth her best effort, she is impulsive, has inconsistent attention, and has difficulty shifting her mental set when rules or the situation change. She is able to memorize apparently unrelated elements but needs repetition. She does not develop learning strategies on her own but benefits from having them shown to her. All of her executive functions are made less efficient by her inconsistent attention, over which she does not appear to have control.

Cognitive Abilities

Currently, Delia's General Intellectual Ability is in the Low Average range. Delia has significant difficulty with tasks requiring fluid reasoning unless they are fairly simple. Memory is good for recalling elements in order but not if rearranging is involved. With sufficient repetition, she is able to learn and retrieve from long-term memory apparently unrelated information (e.g., names of planets). The use of pictures and symbols is also helpful for her in learning verbal information.

Delia needs considerably more time than her age-peers in processing simple information and recognizing it visually. This means that it is more difficult for her to learn the higher-level skills for which these are the foundation. Lower-level skills such as letter-sound association, phonics, and math facts need to become automatic before she can focus on reading comprehension and application of math skill in practical problems. At the same time, improving comprehension and knowledge of concepts in both areas will be constrained unless her comprehension of spoken English, especially her vocabulary, increases.

Delia has a notable strength in her ability to accurately perceive pictures and designs. She is also competent in copying, and later recalling and reproducing designs. This demonstration and her handwriting indicate that she has good visual perception, visual memory, and visual-motor skills for nonverbal information.

Oral Language

Delia's oral language was assessed in English and, to a lesser extent, in Spanish. Delia demonstrated no competence in Spanish. She responded as though it were a foreign language to her. Her English is also limited, which could be related to her head injury but could be partially related to the fact that her primary language was Spanish. She had great difficulty recalling the details of brief stories and understanding or remembering instructions given in individual sentences in English. Her vocabulary is considerably limited, and she is slow in retrieving from memory even words she does know. Again, providing her with a strategy, in the form of a category, was helpful to her.

Academic Achievement

Delia's reading is far below grade level. She needs to learn that words are composed of individual sounds so that she can apply basic phonics to sound out words for reading and spelling. She requires intensive, systematic, explicit instruction in phonics to increase these skills. Both her minimal decoding skills and poor comprehension of oral English impair her reading comprehension.

Delia is able to generate grammatically correct simple sentences in English. She was not assessed on writing longer passages. Because she does not appear to understand complex sentences in English, it is unlikely that she can write them. Her handwriting is good, and her letter production is correct.

The finding that Delia has difficulty reordering numbers mentally may be one reason that her arithmetic skills have not developed, although that skill would likely have developed secondary to learning the algorithms. Delia seems to have a good start on memorizing math facts but is delayed in her math achievement overall. She needs to learn the more difficult addition facts and to notice operation signs. She seems to have the concepts of addition and subtraction and was able to tell time to the hour.

Conclusion

The purposes of this evaluation were not only to find Delia's current cognitive, language, and academic abilities but to make a determination as to whether her current difficulties are related to her head injury.

Based on her test scores and her teacher's comments, Delia did not appear to have any learning problems in kindergarten. Her attention was average, or at least not sufficiently out of the ordinary. She was learning as expected for a child with a first language other than English. None of her present weaknesses in cognitive abilities, language, and achievement were apparent prior to the accident. Most obvious, one does not lose her primary language so completely, within a few months, while still living in an environment in which Spanish is continuously spoken.

Based on this evaluation and Delia's medical and educational history, it does appear that her head injury had more severe ramifications than one would expect of a mild TBI. Her current problems started after her head injury, leading to the conclusion that the head injury is the cause.

Summary of Test Results

California Verbal Learning Test, Children's Version (CVLT-C)

CVLT-C Standard Score Percentile
Trial 1 0.5 69
Trial 5 0.5 69
Trials 1–5 Total T = 37 10
List B Free Recall −2.0 2
Short Delay Free Recall 0 50
Short Delay Cued Recall 0.5 69
Long Delay Free Recall 0.5 69
Long Delay Cued Recall 1.0 84
Yes/No Recognition 1.0 84

Rey-Osterrieth Complex Figure Test (ROCF)

Drawing Z Score Percentile
Copy 0.29 61
Delayed Recall 0.50 69

Developmental NEuroPSYchological Assessment (NEPSY-II)

Scaled Score Percentile
ANIMAL SORTING
Novel Sort Errors 6–10
Total Repeated Sort Errors <2
Total Errors <2
Total Correct Sorts 7 16
Combined Score 4 2
AUDITORY ATTENTION
Total Correct 6 9
Commission Errors <2
Omission Errors 11–25
Inhibitory Errors <2
Combined Score 3 1
AUDITORY ATTENTION AND RESPONSE SET
Total Correct 6 9
Commission Errors 2–5
Omission Errors 6–10
Inhibitory Errors 2–5
AUDITORY ATTENTION AND RESPONSE SET
Combined Score 4 2
Contrast Score 6 9
NARRATIVE MEMORY
Free Recall Score 6 9
Free and Cued Recall Score 5 5
Recognition Score 11–25
Free & Cued Recallvs. Recognition Contrast Score 5
SPEEDED NAMING
Completion Time 5 5
Total Correct 51–75
Total Self-Corrections 51–75
Combined Score 7 16
WORD GENERATIONS
Semantic Total Score 8 25
Initial Letter Total Score 4 2
Semantics vs. Initial Letter Contrast Score 4 2

Woodcock-Johnson IV Tests of Cognitive Abilities (Norms based on age 7–10)

CLUSTER/Tests AE RPI Proficiency SS (68% Band) Descriptor PR
GEN INTELLECTUAL ABIL 8–0 74/90 Limited to Avg. 84 (81–88) Low Avg. 15
Oral Vocabulary 7–5 67/90 Limited to Avg. 83 (77–89) Low Avg. 13
Number Series 9–1 85/90 Avg. 97 (92 101) Avg. 41
Verbal Attention 8–7 83/90 Avg. 94 (88–100) Avg. 35
Letter-Pattern Matching 7–8 38/90 Limited 85 (78–92) Low Avg. 16
Phonological Processing 7–6 75/90 Limited to Avg. 86 (81–92) Low Avg. 18
Story Recall 7–2 76/90 Limited to Avg. 85 (79–91) Low Avg. 16
Visualization 7–0 72/90 Limited to Avg. 85 (79–90) Low Avg. 16
Gf-Gc COMPOSITE 7–6 60/90 Limited 81 (78–84) Low Avg. 11
Number Series 9–1 85/90 Avg. 97 (92–101) Avg. 41
Oral Vocabulary 7–5 67/90 Limited to Avg. 83 (77–89) Low Avg. 13
General Information 6–2 41/90 Limited 76 (71–81) Low 6
Concept Formation 6–6 41/90 Limited 79 (74–83) Low 8
COMP-KNOWLEDGE 3 6–7 49/90 Limited 74 (71–78) Low 4
Oral Vocabulary 7–5 67/90 Limited to Avg. 83 (77–89) Low Avg. 13
General Information 6–2 41/90 Limited 76 (71–81) Low 6
Picture Vocabulary 5–10 38/90 Limited 72 (65–78) Low 3
FLUID REASONING (Gf) 8–0 66/90 Limited 87 (84–91) Low Avg. 20
Number Series 9–1 85/90 Avg. 97 (92–101) Avg. 41
Concept Formation 6–6 41/90 Limited 79 (74–83) Low 8
S-TERM WORK MEM (Gwm) 8–1 76/90 Limited to Avg. 90 (84–95) Low Avg. to Avg. 24
Verbal Attention 8–7 83/90 Avg. 94 (88–100) Avg. 35
Numbers Reversed 7–7 68/90 Limited to Avg. 88 (82–95) Low Avg. 22
COG PROCESS SPEED (Gs) 8–1 57/90 Limited 88 (84–93) Low Avg. 22
Letter-Pattern Matching 7–8 38/90 Limited 85 (78–92) Low Avg. 16
Pair Cancellation 8–8 74/90 Limited to Avg. 94 (89–99) Avg. 34
AUDITORY PROCESS (Ga) 7–3 73/90 Limited to Avg. 85 (81–89) Low Avg. 16
Phonological Processing 7–6 75/90 Limited to Avg. 86 (81–92) Low Avg. 18
Nonword Repetition 7–1 72/90 Limited to Avg. 89 (84–93) Low Avg. 23
L-TERM RETRIEVAL (Glr) 7–10 83/90 Avg. 90 (86–94) Low Avg. to Avg. 26
Story Recall 7–2 76/90 Limited to Avg. 85 (79–91) Low Avg. 16
Visual-Auditory Learning 8–9 88/90 Avg. 98 (94–101) Avg. 44
VISUAL PROCESSING (Gv) 8–7 86/90 Avg. 95 (90–100) Avg. 38
Visualization 7–0 72/90 Limited to Avg. 85 (79–90) Low Avg. 16
Picture Recognition 12–8 94/90 Avg. 107 (99–114) Avg. 68
NUMBER FACILITY 8–4 67/90 Limited to Avg. 90 (84–96) Low Avg. to Avg. 24
Numbers Reversed 7–7 68/90 Limited to Avg. 88 (82–95) Low Avg. 22
Number-Pattern Matching 8–8 67/90 Limited to Avg. 93 (86–100) Avg. 33
PERCEPTUAL SPEED 8–2 53/90 Limited 88 (83–94) Low Avg. 22
Letter-Pattern Matching 7–8 38/90 Limited 85 (78–92) Low Avg. 16
Number-Pattern Matching 8–8 67/90 Limited to Avg. 93 (86–100) Avg. 33
VOCABULARY 6–9 53/90 Limited 76 (72–81) Low 6
Oral Vocabulary 7–5 67/90 Limited to Avg. 83 (77–89) Low Avg. 13
Picture Vocabulary 5–10 38/90 Limited 72 (65–78) Low 3
COG EFFICIENCY (Ext) 8–1 65/90 Limited 87 (82–92) Low Avg. 19
Verbal Attention 8–7 83/90 Avg. 94 (88–100) Avg. 35
Letter-Pattern Matching 7–8 38/90 Limited 85 (78–92) Low Avg. 16
Numbers Reversed 7–7 68/90 Limited to Avg. 88 (82–95) Low Avg. 22
Number-Pattern Matching 8–8 67/90 Limited to Avg. 93 (86–100) Avg. 33

Woodcock-Johnson IV Tests of Oral Language (Norms based on age 9–6)

CLUSTER/Tests AE RPI Proficiency SS (68% Band) Descriptor PR
ORAL LANGUAGE 6–8 49/90 Limited 75 (70–80) Low 5
Picture Vocabulary 5–10 38/90 Limited 72 (65–78) Low 3
Oral Comprehension 7–3 60/90 Limited 81 (75–87) Low to Low Avg. 10
BROAD ORAL LANG. 6–11 59/90 Limited 77 (73–81) Low 6
Picture Vocabulary 5–10 38/90 Limited 72 (65–78) Low 3
Oral Comprehension 7–3 60/90 Limited 81 (75–87) Low to Low Avg. 10
Understanding Directions 7–8 75/90 Limited to Avg. 87 (82–92) Low Avg. 19
LISTENING COMP 7–5 68/90 Limited to Avg. 82 (77–86) Low to Low Avg. 11
Oral Comprehension 7–3 60/90 Limited 81 (75–87) Low to Low Avg. 10
Understanding Directions 7–8 75/90 Limited to Avg. 87 (82–92) Low Avg. 19
VOCABULARY 6–9 53/90 Limited 76 (72–81) Low 6
Picture Vocabulary 5–10 38/90 Limited 72 (65–78) Low 3
Oral Vocabulary 7–5 67/90 Limited to Avg. 83 (77–89) Low Avg. 13
LENGUAJE ORAL 2–9 0/90 Ext. limited 14 (9–19) Very low <0.1
Vocabulario sobre dibujos <2–0 0/90 Ext. limited 7 (1–13) Very low <0.1
Comprensión oral 3–9 1/90 Ext. limited 31 (24–39) Very low <0.1
AMPLIO LENGUAJE ORAL 3–2 1/90 Ext. limited 19 (15–24) Very low <0.1
Vocabulario sobre dibujos <2–0 0/90 Ext. limited 7 (1–13) Very low <0.1
Comprensión oral 3–9 1/90 Ext. limited 31 (24–39) Very low <0.1
Comp de indicaciones 4–5 16/90 Very limited 54 (48–60) Very low 0.1
COMPRENSIÓN AUDITIVA 4–0 5/90 Very limited 34 (29–10) Very low <0.1
Comprensión oral 3–9 1/90 Ext. limited 31 (24–39) Very low <0.1
Comprensión de indicaciones 4–5 16/90 Very limited 54 (48–60) Very low 0.1

Comparative Language Index (CLI; Spanish compared with English)

Spanish Cluster CLI English Cluster
LENGUAJE ORAL (Ext. limited) 0/49 ORAL LANGUAGE (Limited)
(AMPLIO LENGUAJE ORAL (Ext. limited) 1/59 BROAD ORAL LANGUAGE (Limited)
COMPRENSIÓN AUDITIVA (Very limited) 5/68 LISTENING COMP (Limited to fluent)

Woodcock-Johnson IV Tests of Achievement Form A and Extended (Norms based on age 9–6)

CLUSTER/Tests AE RPI Proficiency SS (68% Band) Descriptor PR
READING 7–3 18/90 Very limited 76 (73–79) Low 5
Letter-Word Identification 7–4 15/90 Very limited 80 (77–82) Low to Low Avg. 9
Passage Comprehension 7–2 23/90 Very limited 76 (71–80) Low 5
BROAD READING 7–2 6/90 Very limited 76 (73–79) Low 5
Letter-Word Identification 7–1 15/90 Very limited 80 (77–82) Low to Low Avg. 9
Passage Comprehension 7–2 23/90 Very limited 76 (71–80) Low 5
Sentence Reading Fluency 6–11 1/90 Ext. limited 74 (70–79) Low 4
BASIC READING SKILLS 7–1 19/90 Very limited 77 (74–79) Low 6
Letter-Word Identification 7–4 15/90 Very limited 80 (77–82) Low to Low Avg. 9
Word Attack 6–8 25/90 Limited 73 (68–78) Low 4
READING COMPRE. 7–5 54/90 Limited 81 (78–84) Low to Low Avg. 10
Passage Comprehension 7–2 23/90 Very limited 76 (71–80) Low 5
Reading Recall 8–3 82/90 Avg. 93 (90–96) Avg. 31
READING FLUENCY 6–11 4/90 Very limited 73 (69–77) Low 4
Oral Reading 6–11 30/90 Limited 78 (75–81) Low 7
Sentence Reading Fluency 6–11 1/90 Ext. limited 74 (70–79) Low 4
MATHEMATICS 7–11 42/90 Limited 82 (79–85) Low Avg. 12
Applied Problems 8–3 63/90 Limited 87 (83–92) Low Avg. 207
Calculation 7–7 24/90 Limited 78 (73–82) Low 7
BROAD MATHEMATICS 7–10 38/90 Limited 82 (79–85) Low Avg. 11
Applied Problems 8–3 63/90 Limited 87 (83–92) Low Avg. 20
Math Facts Fluency 7–10 31/90 Limited 84 (78–90) Low Avg. 14
Calculation 7–7 24/90 Limited 78 (73–82) Low 7
MATH CALC. SKILLS 7–8 27/90 Limited 80 (76–84) Low to Low Avg. 9
Calculation 7–7 24/90 Limited 78 (73–82) Low 7
Math Facts Fluency 7–10 31/90 Limited 84 (78–90) Low Avg. 14
WRITTEN LANGUAGE 7–10 55/90 Limited 86 (84–89) Low Avg. 18
Spelling 7–4 23/90 Very limited 79 (75–82) Low 8
Writing Samples 8–8 84/90 Avg. 97 (93–100) Avg. 41
BROAD WRITTEN LANG. 7–11 63/90 Limited 87 (84–90) Low Avg. 19
Spelling 7–4 23/90 Very limited 79 (75–82) Low 8
Writing Samples 8–8 84/90 Avg. 97 (93–100) Avg. 41
Sentence Writing Fluency 8–4 77/90 Limited to Avg. 90 (85–96) Low Avg. to Avg. 26
WRITTEN EXPRESSION 8–6 81/90 Limited to Avg. 93 (90–97) Avg. 33
Writing Samples 8–8 84/90 Avg. 97 (93–100) Avg. 41
Sentence Writing Fluency 8–4 77/90 Limited to Avg. 90 (85–96) Low Avg. to Avg. 26
ACADEMIC SKILLS 7–5 20/90 Very limited 76 (74–79) Low 6
Letter-Word Identification 7–4 15/90 Very limited 80 (77–82) Low to Low Avg. 9
Spelling 7–4 23/90 Very limited 79 (75 82) Low 8
Calculation 7–7 24/90 Limited 78 (73–82) Low 7
ACADEMIC FLUENCY 7–5 16/90 Very limited 79 (76–82) Low to Low Avg. 8
Sentence Reading Fluency 6–11 1/90 Ext. limited 74 (70–79) Low 4
Math Facts Fluency 7–10 31/90 Limited 84 (78–90) Low Avg. 14
Sentence Writing Fluency 8–4 77/90 Limited to Avg. 90 (85–96) Low Avg. to Avg. 26
ACADEMIC APPLICATIONS 7–10 58/90 Limited 84 (82–87) Low Avg. 15
Applied Problems 8–3 63/90 Limited 87 (83–92) Low Avg. 20
Passage Comprehension 7–2 23/90 Very limited 76 (71–80) Low 5
Writing Samples 8–8 84/90 Avg. 97 (93–100) Avg. 41
PHONEME-GRAPHEME KNOWLEDGE 7–1 47/90 Limited 78 (74–82) Low 7
Word Attack 6–8 25/90 Limited 73 (68–78) Low 4
Spelling of Sounds 7–8 70/90 Limited to Avg. 87 (81–92) Low Avg. 19
STANDARD SCORES DISCREPANCY Interpretation at ±1.50 SD (SEE)
VARIATIONS Actual Predicted Difference PR SD
Intra-Cognitive [Extended]
COMP-KNOWLEDGE (Gc) 77 89 −12 16 −1.01
COMP-KNOWLEDGE 3 74 90 −16 10 −1.28
FLUID REASONING (Gf) 87 85 2 58 +0.19
S-TERM WORK MEM (Gwm) 90 88 2 57 +0.18
COG PROCESS SPEED (Gs) 88 92 −4 39 −0.28
AUDITORY PROCESS (Ga) 85 88 −3 39 −0.29
L-TERM RETRIEVAL (Glr) 90 91 −1 49 −0.03
VISUAL PROCESSING (Gv) 95 92 3 61 +0.28
PERCEPTUAL SPEED 88 92 −4 40 −0.25
VOCABULARY 76 89 −13 14 −1.08
ORAL LANGUAGE 75 90 −15 11 −1.24
Oral Vocabulary 83 88 −5 32 −0.46
Number Series 97 86 11 81 +0.89
Verbal Attention 94 89 5 65 +0.40
Letter-Pattern Matching 85 92 −7 29 −0.57
Phonological Processing 86 88 −2 43 −0.18
Story Recall 85 92 −7 30 −0.53
Visualization 85 91 −6 31 −0.49
General Information 76 91 −15 14 −1.09
Concept Formation 79 89 −10 21 −0.81
Numbers Reversed 88 89 −1 48 −0.05
Number-Pattern Matching 93 92 1 53 +0.08
Nonword Repetition 89 91 −2 42 −0.20
Visual-Auditory Learning 98 92 6 67 +0.43
Picture Recognition 107 94 13 82 +0.90
Pair Cancellation 94 94 0 51 +0.02
Memory for Words 85 90 −5 34 −0.42
Picture Vocabulary 72 91 −19 7 −1.47
Oral Comprehension 81 90 −9 24 −0.72
Understanding Directions 87 89 −2 45 −0.13
STANDARD SCORES DISCREPANCY Interpretation at ±1.50 SD (SEE)
VARIATIONS Actual Predicted Difference PR SD
Intra-Achievement [Extended]
BASIC READING SKILLS 77 83 −6 21 −0.80
READING COMPREHENSION 81 84 −3 38 −0.30
READING FLUENCY 73 84 −11 11 −1.21
MATH CALCULATION SKILLS 80 86 −6 29 −0.56
WRITTEN EXPRESSION 93 81 12 88 +1.19
Letter-Word Identification 80 83 −3 35 −0.39
STANDARD SCORES DISCREPANCY Interpretation at ±1.50 SD (SEE)
VARIATIONS Actual Predicted Difference PR SD
Intra-Achievement [Extended] Variations
Applied Problems 87 85 2 59 +0.22
Spelling 79 84 −5 28 −0.59
Passage Comprehension 76 84 −8 18 −0.91
Calculation 78 85 −7 24 −0.71
Writing Samples 97 84 13 87 +1.15
Word Attack 73 86 −13 13 −1.14
Oral Reading 78 86 −8 24 −0.71
Sentence Reading Fluency 74 86 −12 13 −1.11
Math Facts Fluency 84 88 −4 36 −0.37
Sentence Writing Fluency 90 83 7 76 +0.72
Reading Recall 93 89 4 64 +0.36
Spelling of Sounds 87 87 0 49 −0.03
STANDARD SCORES DISCREPANCY Interpretation at ±1.50 SD (SEE)
VARIATIONS Actual Predicted Difference PR SD
Academic Skills/Academic Fluency/Academic Applications [Extended] Variations
ACADEMIC SKILLS 76 82 −6 19 −0.88
ACADEMIC FLUENCY 79 83 −4 33 −0.45
ACADEMIC APPLICATIONS 84 80 4 69 +0.51
COG PROCESS SPEED (Gs) 88 89 −1 48 −0.06
PERCEPTUAL SPEED 88 86 2 57 +0.19
STANDARD SCORES DISCREPANCY Interpretation at ±1.50 SD (SEE)
VARIATIONS Actual Predicted Difference PR SD
Gf-Gc Composite/Other Ability Comparisons
S-TERM WORK MEM (Gwm) 90 87 3 57 +0.18
COG PROCESS SPEED (Gs) 88 90 −2 45 −0.12
PERCEPTUAL SPEED 88 89 −1 48 −0.04
AUDITORY PROCESS (Ga) 85 87 −2 43 −0.18
LONG-TERM RETRIEVAL (Glr) 90 89 1 54 +0.11
VISUAL PROCESSING (Gv) 95 90 5 64 +0.37
NUMBER FACILITY 90 87 3 57 +0.17
COGNITIVE EFFICIENCY 84 88 −4 38 −0.32
COG EFFICIENCY (Ext) 87 87 0 51 +0.03
BRIEF ACHIEVEMENT 81 84 −3 38 −0.31
BROAD ACHIEVEMENT 78 83 −5 33 −0.45
READING 76 86 −10 17 −0.95
BROAD READING 76 84 −8 23 −0.74
BASIC READING SKILLS 77 86 −9 22 −0.79
STANDARD SCORES DISCREPANCY Interpretation at ±1.50 SD (SEE)
COMPARISONS Actual Predicted Difference PR SD
Gf-Gc Composite/Other Ability Comparisons
S-TERM WORK MEM (Gwm) 90 87 3 57 +0.18
COG PROCESS SPEED (Gs) 88 90 −2 45 −0.12
PERCEPTUAL SPEED 88 89 −1 48 −0.04
AUDITORY PROCESS (Ga) 85 87 −2 43 −0.18
L-TERM RETRIEVAL (Glr) 90 89 1 54 +0.11
VISUAL PROCESSING (Gv) 95 90 5 64 +0.37
NUMBER FACILITY 90 87 3 57 +0.17
COGNITIVE EFFICIENCY 84 88 −4 38 −0.32
COG EFFICIENCY (Ext) 87 87 0 51 +0.03
BRIEF ACHIEVEMENT 81 84 −3 38 −0.31
BROAD ACHIEVEMENT 78 83 −5 33 −0.45
READING 76 86 −10 17 −0.95
BROAD READING 76 84 −8 23 −0.74
BASIC READING SKILLS 77 86 −9 22 −0.79
READING COMPREHENSION 81 84 −3 38 −0.30
READING FLUENCY 73 85 −12 15 −1.04
MATHEMATICS 82 84 −2 45 −0.13
BROAD MATHEMATICS 82 84 −2 42 −0.19
MATH CALCULATION SKILLS 80 85 −5 34 −0.42
WRITTEN LANGUAGE 86 85 1 55 +0.14
BROAD WRITTEN LANGUAGE 87 84 3 60 +0.26
WRITTEN EXPRESSION 93 86 7 73 +0.60
ACADEMIC SKILLS 76 84 −8 24 −0.69
ACADEMIC FLUENCY 79 86 −7 29 −0.56
ACADEMIC APPLICATIONS 84 83 1 55 +0.13
PHONEME-GRAPHEME KNOWLEDGE 78 87 −9 24 −0.72
STANDARD SCORES DISCREPANCY Significant at ±1.50 SD (SEE)
COMPARISONS Actual Predicted Difference PR SD
GIA/Achievement Discrepancy Procedure
BRIEF ACHIEVEMENT 81 86 −5 28 −0.57 No
BROAD ACHIEVEMENT 78 87 −9 18 −0.93 No
READING 76 87 −11 16 −0.99 No
BROAD READING 76 87 −11 14 −1.10 No
BASIC READING SKILLS 77 87 −10 15 −1.02 No
READING COMPREHENSION 81 87 −6 29 −0.55 No
READING FLUENCY 73 88 −15 9 −1.33 No
MATHEMATICS 82 87 −5 32 −0.46 No
BROAD MATHEMATICS 82 87 −5 27 −0.62 No
MATH CALCULATION SKILLS 80 88 −8 21 −0.81 No
WRITTEN LANGUAGE 86 87 −1 50 −0.01 No
BROAD WRITTEN LANGUAGE 87 86 1 53 +0.09 No
WRITTEN EXPRESSION 93 87 6 71 +0.55 No
ACADEMIC SKILLS 76 86 −10 15 −1.05 No
ACADEMIC FLUENCY 79 89 −10 16 −1.00 No
ACADEMIC APPLICATIONS 84 86 −2 45 −0.13 No
PHONEME-GRAPHEME LEDGE 78 88 −10 18 −0.93 No
ORAL LANGUAGE 75 89 −14 12 −1.19 No
BROAD ORAL LANGUAGE 77 88 −11 15 −1.05 No
LISTENING COMPREHENSION 82 88 −6 28 −0.57 No
STANDARD SCORES DISCREPANCY Interpretation at ±1.50 SD (SEE)
COMPARISONS Actual Predicted Difference PR SD
Oral Language/Achievement Comparisons
READING 76 82 −6 31 −0.50 No
BROAD READING 76 83 −7 29 −0.56 No
BASIC READING SKILLS 77 83 −6 30 −0.53 No
READING COMPREHENSION 81 83 −2 43 −0.18 No
READING FLUENCY 73 85 −12 19 −0.89 No
MATHEMATICS 82 84 −2 44 −0.15 No
BROAD MATHEMATICS 82 84 −2 42 −0.20 No
MATH CALCULATION SKILLS 80 86 −6 34 −0.41 No
WRITTEN LANGUAGE 86 84 2 57 +0.18 No
BROAD WRITTEN LANGUAGE 87 83 4 60 +0.26 No
WRITTEN EXPRESSION 93 85 8 71 +0.55 No
ACADEMIC SKILLS 76 83 −7 29 −0.55 No
ACADEMIC FLUENCY 79 86 −7 30 −0.52 No
ACADEMIC APPLICATIONS 84 82 2 59 +0.22 No
PHONEME-GRAPHEME KNOWLEDGE 78 85 −7 31 –0.51 No
STANDARD SCORES DISCREPANCY Significant at ±1.50 SD (SEE)
COMPARISONS Actual Predicted Difference PR SD
Oral Language [Spanish]/Achievement [English] Comparisons
READING 76 35 41 >99.9 +3.35 Yes (+)
BROAD READING 76 42 34 99.6 +2.65 Yes (+)
BASIC READING SKILLS 77 39 38 >99.9 +3.19 Yes (+)
READING COMPREHENSION 81 41 40 >99.9 +3.28 Yes (+)
READING FLUENCY 73 49 24 96 +1.80 Yes (+)
MATHEMATICS 82 42 40 >99.9 +3.16 Yes (+)
BROAD MATHEMATICS 82 46 36 99.7 +2.76 Yes (+)
MATH CALCULATION SKILLS 80 51 29 98 +2.06 Yes (+)
WRITTEN LANGUAGE 86 38 48 >99.9 +3.73 Yes (+)
BROAD WRITTEN LANGUAGE 87 39 48 >99.9 +3.54 Yes (+)
WRITTEN EXPRESSION 93 45 48 >99.9 +3.33 Yes (+)
ACADEMIC SKILLS 76 39 37 99.8 +2.94 Yes (+)
ACADEMIC FLUENCY 79 42 37 99.7 +2.72 Yes (+)
ACADEMIC APPLICATIONS 84 34 50 >99.9 +4.16 Yes (+)
PHONEME-GRAPHEME KNOWLEDGE 78 44 34 99.7 +2.74 Yes (+)

Woodcock-Johnson IV Tests of Cognitive Abilities Test Session Observations

  1. Level of conversational proficiency: Advanced
  2. Level of cooperation: Cooperative (typical for age/grade)
  3. Level of activity: Typical for age/grade
  4. Attention and concentration: Attentive to the tasks (typical for age/grade)
  5. Self-confidence: Appeared at ease and comfortable (typical for age/grade)
  6. Care in responding: Prompt but careful in responding (typical for age/grade)
  7. Response to difficult tasks: Generally persisted with difficult tasks (typical for age/grade)

Woodcock-Johnson IV Tests of Achievement Form a and Extended Qualitative Observations

  1. Letter-Word Identification: Identified initial items rapidly and accurately but had difficulty applying phoneme-grapheme relationships to latter items
  2. Applied Problems: Solved initial problems with no observed difficulty but demonstrated increasing difficulties solving the latter items (typical)
  3. Spelling: Spelled initial items easily and accurately; spelling of latter items reflected a need for further skill development (typical)
  4. Passage Comprehension: Appeared to read passages very slowly and had difficulty identifying a correct word (struggled with application of syntactic and semantic cues)
  5. Calculation: Solved initial problems quickly with no observed difficulties but demonstrated less automaticity with the latter items (typical)
  6. Writing Samples: Sentences were inadequate (for example, run-ons, incomplete sentences, awkward syntax, or limited content)
  7. Word Attack: Identified the initial items rapidly and accurately but had difficulty applying phoneme-grapheme relationships to latter items
  8. Oral Reading: Errors involving mispronunciation (11) were observed
  9. Sentence Reading Fluency: Appeared to read sentences slowly
  10. Math Facts Fluency: Solved problems slowly
  11. Sentence Writing Fluency: Wrote appropriate sentences at a slow pace

Reason for Referral

Kevin is an 8-year-old boy, in the second grade, who was referred for an evaluation by his mother, Mrs. Sally Reid. She is seeking an evaluation to determine whether he has a specific learning disability and/or memory deficits. Mrs. Reid reported that Kevin is falling behind academically and is functioning at about a first-grade level. His teachers suspect that he has memory difficulties, because he has trouble with retaining and recalling topics that have been taught in class. Mrs. Reid stated Kevin requires “extensive repetition and clarification” of material that is presented to him, and his attention often has to be redirected. This assessment was conducted to explore Kevin's cognitive and academic profile, highlight any factors that appear to impact his scholastic performance, and provide recommendations that may help alleviate the effects of such factors.

Background Information

Kevin currently lives with his biological mother and father and his two older brothers (ages 10 and 12). Mr. Reid is a computer analyst, and Mrs. Reid is lawyer. Mrs. Reid reported that Kevin gets along with all family members. Both of Kevin's older brothers have been diagnosed with cystic fibrosis, autism spectrum disorder, auditory processing disorder, dyslexia, and dyscalculia.

Developmental and Medical History

Kevin was born via emergency C-section 7 weeks premature eclampsia. At birth, he had a critically low Apgar score, and there was difficulty resuscitating him. He was placed on an oscillating ventilator for two weeks. At this time, he also had aspiration, apnea, jaundice, and patent foramen ovale, a hole in the heart that normally closes during infancy, and patent ductus arteriosus, an opening between two blood vessels that normally closes shortly after birth (both of which have since been resolved). He spent two months in the neonatal intensive care unit at a nearby medical center.

Kevin's medical history is significant for central and obstructive apnea. Mrs. Reid reported that four days after coming home from the hospital he turned blue and had no pulse, and cardiopulmonary resuscitation (CPR) was initiated. Approximately one year later, Kevin was admitted to the hospital for an acute life-threatening episode of apnea. During this time, a nurse witnessed him having a seizure and consequently he had a 72-hour video electroencephalogram (EEG). It was concluded that Kevin had had a seizure but it was believed to be secondary to his apnea. Kevin continued to have episodes of central and obstructive apnea up until 24 months of age. Because of this, he was on oxygen on an as-needed basis until he was 2 years old. Kevin also had gastroesophageal reflux disease and eosinophilic esophagitis (EE); however, neither has been a problem since then.

Currently, Kevin has chronic diarrhea, which is treated with Senna as needed. He also has difficulty sleeping. Mrs. Reid reported that he wakes up every night at least once between 1:00 and 4:00 a.m., but he is able to fall back asleep. He also snores very loudly, which Mrs. Reid suspects may be due to obstructive apnea. Kevin takes 1 mg melatonin every night. He was recently genetically tested to determine whether he has cystic fibrosis. He is also currently in the process of being scheduled for a noninvasive melanoma surgery.

Mrs. Reid reported that Kevin's developmental milestones were 1 to 3 months delayed. He qualified for early intervention at approximately 4 months and received speech therapy, physical therapy, and occupational therapy (amount and frequency unknown). When he turned 3, he qualified for a preschool program for children with disabilities. He attended this program for half of the day and a private Catholic preschool for the other half of the day. The school questioned whether he was developmentally ready for kindergarten but his mother felt that being with other children his age would aid his social and language development, so he started on time.

Kevin attended Lincoln Elementary School for kindergarten and first grade. He was initially evaluated by the child study team in 2010 and found to be eligible for special education services under the classification of Other Health Impaired. He was reevaluated in 2013, and his Individual Education Program (IEP) was modified for second grade. In December of 2013, he was diagnosed with central auditory processing disorder (CAPD) by doctors at Boston General Hospital. Kevin was also diagnosed with dyslexia in April of 2014 by his developmental pediatrician and with a verbal learning disorder in February 2014 by the same pediatrician.

Currently, he has started at a new school and is in a general education classroom. Mrs. Reid reported that she believes Kevin has not had difficulty adjusting to the new school, because it is the school where he attended preschool and a few friends from preschool are in his classroom. Kevin has an aide throughout the school day. He goes to the resource room for language arts and math. He also receives multisensory reading sessions once per day for 30 minutes and speech and occupational therapy twice per week for 25 minutes per session.

Current Academic Functioning

Mrs. Reid reported that Kevin is functioning academically at an early first-grade level and receiving grades in the 70–80% range. He reportedly has difficulty in all subjects. Mrs. Reid also noted that Kevin has typically been resistant to completing his homework. His behavior, however, has been improving with the help of a tutor three times per week. Mrs. Reid further explained that lately he has been more compliant and has shown motivation to learn and complete his homework.

Social Functioning

Mrs. Reid reported that Kevin does not have difficulty initiating or maintaining friendships. He has friends at school and gets along with his classmates. Kevin has two friends that Mrs. Reid reports he is very close to and on Mrs. Reid's request, Kevin is now in the same class with them. He also has one to two friends on his ice hockey team. Kevin asks to see his friends outside of school; however, Mrs. Reid explained that it is typically not possible because of the family's busy schedules and health issues. Mrs. Reid reported that Kevin has had three play dates this year.

Tests Administered

  1. Selected tests of the Woodcock Johnson IV Tests of Cognitive Abilities (WJ IV COG)
  2. Woodcock Johnson IV Tests of Achievement, Form A (WJ IV ACH)
  3. Woodcock Johnson IV Tests of Oral Language (WJ IV OL)
  4. Wide Range Assessment of Memory and Learning Second Edition (WRAML2)
  5. Testing was conducted over a two-day period. During the first session, which took approximately two hours, he was administered the core tests of the WJ IV COG. During the second session one week later, which took approximately 2½ hours, he was administered the WRAML2 and then the core tests of the WJ IV ACH. During each session, Kevin took several 5-minute breaks.

Behavioral Observations

Kevin arrived on time to his sessions. He was well groomed and appropriately dressed. His speech was of normal speed, pitch, and rhythm. He was oriented to person, place, and time. For most of the testing, Kevin exhibited good effort and appeared motivated. However, he had difficulty sustaining attention, and on multiple occasions throughout the testing sessions the examiner had to prompt Kevin and redirect his focus. For example, when presenting the cards on the Picture Memory test of the WRAML2, the examiner had to direct Kevin's gaze toward the cards for almost every item because they are removed after 10 seconds.

Throughout the testing sessions, directions often had to be repeated slowly to ensure that Kevin comprehended them. Even so, at times, on a later item of a test, Kevin was unable to recall what he was supposed to do. He also missed some of the directions on the recorded tests. For example, on the Understanding Directions test of the WJ IV OL battery, he completed 16 items, then on the next two, he asked, “What?” He was encouraged to give his best guess. Although it is likely that some of Kevin's scores are lowered because of his difficulty sustaining attention and difficulty recalling information, these behaviors appear to be consistent with the behaviors reported by Mrs. Reid and his teachers. For this reason, the results of this assessment appear to be valid and reliable indicators of Kevin's current functioning.

Assessment Results

Various test publishers use different qualitative descriptors to describe the same types of scores. For consistency and ease of interpretation, Kevin's scores are reported as percentile ranks, with the following descriptors for the percentile ranges.

Descriptor Percentile Rank Range Descriptor Percentile Rank Range
Very Superior 98+ Low Average 9–24
Superior 92–97 Low 2–8
High Average 76–91 Very Low below 2
Average 25–75

Percentile ranks are often misunderstood as percentages. The percentile rank states the percentage of children of Kevin's age (year and month) in the norm sample whose score was equal to or below his. For example, Kevin's Fluid Reasoning score in the 5th percentile means that only 5% of children of age 8–3 obtained a score the same as or lower than Kevin's.

The tests were scored according to age-based norms. On many of the clusters of the WJ IV, if no discrepancy exists between the test scores within a cluster, only the cluster score is reported within the body of the report. A complete set of scores is provided in the Appendix.

Cognitive Functioning

Kevin's cognitive functioning was assessed using the WJ IV COG. Because Kevin's performance on the Short-Term Working Memory cluster was significantly lower than the other cognitive domains assessed, it was determined that the Gf-Gc Composite more accurately reflects Kevin's intellectual abilities. The Gf-Gc Composite is a distillation of the cognitive domains that measure knowledge, language, and reasoning. Kevin's Gf-Gc Composite was equal to or higher than that of 3% of children his age, in the Low range. A more detailed analyses of his scores in individual domains are provided in the following sections.

Comprehension-Knowledge (Gc)

Comprehension-Knowledge, sometimes called crystallized intelligence, includes the breadth and depth of acquired knowledge, as well as the ability to communicate this knowledge to others and the ability to reason and solve problems in new situations, using previously learned experiences and procedures. Comprehension-Knowledge was assessed using two tests, Oral Vocabulary and General Information. On Oral Vocabulary (Synonyms and Antonyms), Kevin listened to a word and then was instructed to provide a synonym or antonym. Despite the examples, Kevin initially gave rhyming words instead of synonyms. For example, when instructed to provide a synonym for “big” he said “wig” and “dig.” However, after the examiner provided corrective feedback, Kevin appeared to understand the directions. On the General Information test, the examiner named an object and Kevin explained where one would find it or what one would do with it. This test also involves vocabulary, because Kevin must know the meaning of the word in order to answer the question. Overall, only 3% of Kevin's age-peers scored the same as or lower than Kevin on Comprehension-Knowledge.

Fluid Reasoning (Gf)

Fluid Reasoning refers to the ability to reason, form concepts, and solve problems using unfamiliar information or novel procedures. Fluid reasoning is assessed by using two tests, Number Series and Concept Formation. The Number Series test involves figuring out the missing number in a series of four numbers. Kevin responded correctly when the missing number was in a series of consecutive numbers (e.g., 12, __, 14, 15). However, when the pattern became more complex, such as subtracting by 2 in reverse order (e.g., 9, 7, 5, __), Kevin was not successful. On the Concept Formation test, Kevin was provided with two groups of colored shapes and was instructed to determine the attribute on which the two groups differed (e.g., shape, size, color). Kevin's fluid reasoning ability was in the 5th percentile, in the Low range.

Additional Tests

Other areas of cognition were measured using only one test. Given this limitation, no in-depth analyses were made. Kevin's results indicated the following: He had great difficulty rapidly seeing the difference between letters and did not quickly recognize common letter combinations (2nd percentile). Kevin also had difficulty thinking of spoken words in terms of their component sounds, recalling known words from memory based on specific sounds, and playing with the sounds in words to make new words (1st percentile). These skills are critical foundations for learning to read and spell.

Visualization was one of Kevin's strongest abilities measured. The Visualization test assesses visual-spatial abilities, specifically the ability to imagine what an object, design, or mental image would look like from a different perspective or if the components were rearranged (11th percentile, Low Average).

Memory, Learning, and Language Comprehension

Kevin's memory was assessed using the WRAML2 core subtests and age-appropriate optional subtests. The WRAML2 includes a measure of general memory ability, the General Memory Index (GMI), which is a distillation of verbal and visual memory, attention, and concentration. Kevin's GMI was the same as or higher than that of only 1% of children his age, falling in the Very Low range. Kevin was also administered the Short-Term Working Memory cluster and the tests of Story Recall and Understanding Directions from the WJ IV COG and WJ IV OL batteries. His scores on these tests ranged from the 1st percentile to the 3rd percentile.

Verbal Memory within a Meaningful Context

The Verbal Memory Index is an estimate of how well Kevin can learn and recall verbal information. It is derived from the Story Memory and Verbal Learning subtests, each of which assesses short-term memory, delayed recall, and recognition. For Story Memory, Kevin was told two stories and after each, was asked to retell it. On the first story, the simpler of the two, his memory of specific details (16th percentile) was significantly better than his ability to relate the gist of the story, using his own words (1st percentile). He was unable to recall any parts of the second story, which was much more complex. When encouraged to remember anything, he provided one detail that was the opposite of what was said in the story. After a 20-minute delay, Kevin retold the first story, and then answered multiple choice questions. Because the test is scored on responses to both stories and Kevin was able to recall only one, all of his Story Memory scores were affected. On Short-Term Memory, his performance was the same as or better than that of only 2% of his age peers; on delayed recall, 16%; and on recognition, 5%.

Kevin was also administered the Story Recall test from the WJ IV OL battery. The task is similar, to listen to and then retell stories, but these stories are shorter than those of the WRAML2 and increase in length and complexity incrementally. Nevertheless, his score was in the 3rd percentile. Kevin was able to give a few details from three of the four stories that he heard, but did not understand the gist of the stories.

The WJ IV OL Understanding Directions test is a robust test of short-term working memory as well as of comprehension of syntactic complexity. The task is to look at a detailed picture while listening to sentence-length directions that increase in complexity and length. The student demonstrates understanding and memory of the instructions by waiting until the sentence is finished before pointing to the details in the picture as instructed. Kevin followed directions accurately when given in simple or compound sentences, when what he was supposed to do followed the sequence of the directions, and when there were few qualifiers regarding the objects (e.g., “Point to the rock and then point to the fish,” but not “Point to the big rock by the stream and then point to the smallest fish on the ground”). Only 1% of Kevin's age peers scored as low or lower than Kevin on this test, placing his performance in the Very Low range.

Non-Contextual Verbal Learning and Short-Term Working Memory

On the Verbal Learning test, over four trials, the evaluator read a list of 13 words and Kevin then said as many of the words as he could recall. Over four trials, Kevin progressed from recalling only one word to recalling seven (9th percentile). On delayed recall, without hearing the list again, Kevin recalled six words (25th percentile). After another, the evaluator read a longer list and Kevin said yes or no, indicating which of the words had been on the original list. It's likely the distractor words on the recognition list confused him, because his performance was only in the 1st percentile. Kevin's recognition errors suggested a weakness in phonological retrieval, because he identified six words that sounded similar to those on the original list as being on the list (e.g., make for lake). Overall, only 3% of 8-year-olds in the norm sample obtained a score as low as or lower than Kevin's on the Verbal Memory Index.

Short-term working memory is responsible for much of the work of learning. It refers to the ability to focus on and hold information in immediate awareness and then use or manipulate it to carry out a task. It is also the process by which new or transformed information is placed in long-term memory, and which searches for and retrieves specific information from long-term storage when one needs it for active thinking. On the WJ IV Short-Term Working Memory cluster, only 1% of Kevin's age-peers would have scored the same as or lower than he did. In the test of Verbal Attention, Kevin listened to a short list of mixed words (animals and numbers) and then was told to say one of them according to its place in the sequence (e.g., What word came before lamb?). The Numbers Reversed test involves listening to a series of numbers and then repeating them in reverse order. On 2-digit items, Kevin answered 3 out of 5 correctly; on 3-digit items, he answered 2 of 5 correctly. His errors were giving the right numbers in the wrong order or including additional numbers. On 4-digit items, Kevin was unable to recall the correct numbers in any order.

Paired Associate Learning

The Sound Symbol test of the WRAML2 and the Visual-Auditory Learning test of the WJ IV COG assess associative memory, the ability to link pairs of inherently unrelated elements and retrieve them over time. Both tests involved learning words to match with abstract symbols. Sound Symbol uses one-syllable nonsense words (e.g., gah, kye) presented on individual cards, without a context. The task is similar to learning the association between sounds and letters. Visual-Auditory Learning matches real words to symbols and then has the student read a simple story “written” in the symbols. Kevin's scores on Sound Symbol and Visual-Auditory Learning were in the 4th percentile and 6th percentile, respectively. It appears that the meaningful context of the second test was not helpful to Kevin.

Visual Memory

The Visual Memory tests of the WRAML2 involve short-term memory (responding immediately after the stimulus is removed) and recognition. On Picture Memory, Kevin was shown a detailed picture of a scene and then an alternate picture, and was told to mark any details that had changed from one picture to the next. Later, he was shown many picture details (e.g., fence post, dog) and marked yes or no, indicating whether he thought the detail had been in one of the pictured scenes he had seen previously. His score on both short-term and recognition memory for pictures was in the 16th percentile, the Low Average range.

The Design Memory tests involved looking at and drawing from memory designs composed of arrangements of geometric forms. After a delay, Kevin was shown cards with individual or groups of forms and marked yes or no, indicating whether he had or had not seen it on one of the original cards. The discrepancy between his short-term memory for designs and his recognition was extreme: 2nd percentile and 63rd percentile, Very Low range and the upper part of the Average range.

Attention/Concentration

The Attention/Concentration Index is an estimate of how well Kevin can learn and recall relatively nonmeaningful rote, sequential information. It is composed of two subtests, Finger Windows and Number Letter. On the Finger Windows subtest, Kevin is shown a vertically resting card containing asymmetrically located holes. The examiner demonstrates increasingly longer series of pencil pokes through different windows on the board, and Kevin was asked to immediately replicate these patterns. Kevin's performance on this task was equal to or higher than that of 5% of children his age. On the Number Letter subtest, Kevin was asked to repeat a sequence of digits and letters orally presented by the examiner. Kevin's performance on this task was equal to or higher than that of 5% of children his age. Overall, Kevin produced an Attention/Concentration Index higher than that of only 2% of children his age, placing him in the Low range.

Achievement

Kevin's achievement was measured using the WJ IV ACH Battery. Only 1% of his age peers scored the same as or lower than Kevin in overall academic achievement (very low), although there was a discrepancy of almost three standard deviations between his lowest and the highest standard scores (<0.1 to 23rd percentiles).

Reading Abilities

Because of Kevin's extremely limited reading skills, only three reading tests could be administered: Word Attack, Letter Word Identification, and Passage Comprehension. Other tests were attempted and discontinued or not attempted. On the Word Attack test, the first items require identifying letter sounds, and, subsequently, nonsense words. Kevin read the first word, probably the first word most children learn to read, and gave the sounds of a few letters. He was not able to go any further. He was much more successful reading real words (Letter-Word Identification: 14th percentile). When reading this list of both regular and irregular words, Kevin read two- to three-sound words (e.g., go, hot) but could not read words with consonant or vowel digraphs, consonant blends, or any but the most common irregular words (i.e., words that have elements that are not decodable, such as who). When Kevin attempted to sound out longer words, he often sounded out the beginning of the word correctly but guessed at the remainder. For example, when sounding out them he said thowl.

For reading comprehension, Kevin had to read a sentence and provide the missing word. Because he was unable to read some of even the simplest sentences, fewer than one child in 1,000 would score the same as or lower than he did. For the same reason, Kevin was unable to complete the Sentence Reading Fluency test, which was not administered. On the first trial item, Kevin could only read the words is and an. Overall, less than 1% of children at his age level had a Broad Reading ability score the same as or lower than Kevin, placing him in the Very Low range.

Writing Abilities

Kevin's achievement in written language, including spelling, writing fluency, and the quality of written responses, was assessed using three tests. Kevin's responses on a traditional spelling test reflected his limited reading skills. He wrote single letters, and two 2-letter phonetically regular words; however, he was unable to spell words containing three or more letters. Kevin was able to write the first letter of words accurately but lacked the phonemic awareness skills necessary to sound out and spell more difficult words. For example he spelled “winter” as “wer” and “kicked” as “cak.” Kevin's spelling achievement was in the 3rd percentile.

The Writing Samples test involved completing a sentence with one written word and writing a whole sentence according to various instructions and a picture. His score reflects his response to requests for a “good sentence” with one word. For example, when asked to write a sentence describing a picture of a girl running, he wrote, “runin.” Only four 8-year-olds in 1,000 would obtain a score the same as or lower than Kevin's.

Kevin was unable to complete Writing Fluency test, which involved using a picture and three words (printed on the page) to generate and produce as many sentences as possible in a limited time. On the sample items, the examiner read him the three words he needed to use, but Kevin was unable to formulate an acceptable sentence. For example, when given the words “dog” “big” and “is” he wrote the sentence “the is dog big.” The test was discontinued.

Mathematical Abilities

Kevin's math achievement, including problem solving, understanding numbers, automaticity of math facts, and reasoning, was assessed using three tests.

The Calculation test involved using pencil and paper to solve computation problems. Kevin answered only addition problems using 1-digit numbers, basic math facts. He attempted some subtraction problems but added rather than subtracted. Because he answered so few problems and answered the subtraction problems incorrectly, Kevin's performance was in the 2nd percentile.

The Math Facts Fluency test assessed Kevin's ability to complete basic math fact problems as quickly as possible, thereby testing both knowledge and automaticity of math fact retrieval. Kevin answered 40 problems in 3 minutes but made many errors. Again, most of his errors were adding on subtraction problems. Even so, this was Kevin's highest achievement score, 23rd percentile (Low Average to Average).

Applied Problems assessed Kevin's math reasoning abilities by requiring him to listen to a problem, determine the operation to use, perform relatively simple calculations, and answer orally. All of the items he answered were addition or subtraction and were accompanied by pictures. Kevin scored in the 19th percentile.

Summary

Kevin is an 8-year-old boy who is in second grade. His teachers have expressed concerns about his ability to retain information that is taught in class and have noted that he is often inattentive. Mrs. Reid also reported that Kevin often requires information to be repeated and clarified. Kevin has been receiving special education services since preschool. He was referred for testing to identify his learning profile and determine whether he has a learning disability and/or deficits in memory.

Cognitive

The results of this evaluation indicate that Kevin's general cognitive abilities are in the Low range. Only 3% of children his age in the norming sample obtained a Gf-Gc Composite score the same as or lower than Kevin's. Specifically, Kevin is extremely delayed in acquiring knowledge of his environment, his vocabulary, and his ability to use logical reasoning for solving unfamiliar problems. His ability to think of words in terms of sounds and to retrieve specific words from memory based on sounds is very low, as is his speed of recognizing letters and common letter patterns. Kevin had an intrapersonal strength in visualization, the ability to picture how structures or designs might look from a different perspective (Low to Low Average).

Oral Language

The major referral question prompting this evaluation was to address challenges with memory; however, a person is considerably less likely to remember what he does not understand in the first place. Most new learning has to be connected to something already known. Consequently, Kevin's low vocabulary and general knowledge put him at a great disadvantage. Because Kevin has already been diagnosed with a language disorder, all memory tasks that start with the need to understand language-based information are likely to be problematic. When considering Kevin's memory deficits, a major contributing factor, which was not specifically assessed in this evaluation, is likely to be a severe weakness in oral language comprehension.

Attention

The most basic cognitive function that allows all learning to proceed is attention. Only when attention is focused on a thought or piece of information is one able to work with it, understand it, and transfer it to long-term memory. As soon as attention is diverted from the target, the target dissipates. If Kevin is not actively attending to a learning opportunity, or keeps losing attention and refocusing when he is redirected, he is getting pieces of what is offered or getting none of it. It is similar to watching a video and having chunks of it cut out. Not only have you missed what has happened during that time but you then have to try to figure out what is going on when it starts again. In the classroom, and in life, that is debilitating.

Memory and Retrieval

Various types of memory were evaluated. Kevin's strongest memory system appears to be memory for details in pictures of real things, which was solidly in the Low Average range. Visual memory for pictures may be helpful in remembering the location of things on a map, remembering the arrangement of a room, and knowing where to put materials away. He did not demonstrate the same ability on other aspects of visual memory such as designs or patterns of movement, which were all in the Very Low to Low ranges. Kevin also had an intrapersonal strength, although still in the Low to Low Average range, in learning rote information that did not have a specific order. He definitely benefited from hearing and practicing the information repeatedly immediately and after a delay. His response to this task, however, indicated that his auditory discrimination is poor, so that he confuses similar-sounding words.

A major concern is Kevin's responses to the memory tasks that involved comprehension of connected, meaningful language. He was barely able to recall any information from the stories he heard. On delayed recall of the WRAML2 story, he named some key words, thus earning a score in the low average range, but he did not really understand the context, because he could not identify the right answer to the multiple-choice questions asked. As well, Kevin had great difficulty with paired associate learning, the ability to create an association between two inherently unrelated things, which is necessary for making the associations between letters and sounds. These are serious concerns, because to function daily in the world, a person must have certain knowledge, both of things that exist (the what) and procedures (the how to), the language comprehension to understand more, and more knowledge on which to build the new information.

Academic Achievement

Kevin's ability to give only the primary sounds of most letters and his recognition of some words placed him at the 14th percentile in word identification. He is unlikely to go much further, however, without learning to perceive and manipulate the speech sounds within words and more phonics skills. Kevin's limited decoding made reading comprehension impossible.

The same problems impeded Kevin's ability to demonstrate any writing skills except for the most basic spelling, in the 3rd percentile. He cannot sound out any but the simplest two-sound words. Furthermore, he was unable to make a syntactically correct sentence, even when he only needed to add a verb to the three words given to him and, in fact, did not seem to understand the concept of a sentence.

Of the academic skills assessed, Kevin's strongest was basic math facts, in the 23rd percentile. The way math facts are typically taught matches one of his strongest learning styles, much repetition and then distributed practice. He did not notice the change in operation signs, which lowered his score on two tests, because he just kept adding. On the math application test, however, he demonstrated that he had a basic understanding of the concept of subtraction as he answered some of those questions correctly, albeit accompanied by pictures that helped him to count. He did not use paper and pencil to solve any of the problems.

Conclusions

Kevin has a disorder in aspects of memory that substantially affect his learning. This is exacerbated by his weakness in language comprehension and his inconsistent attention. His ability to learn academic skills is seriously hampered by these difficulties as well as by his previously diagnosed auditory processing disorder, which impedes his ability to process and manipulate the speech sounds of the language. Whether his memory and academic deficits can qualify as a specific learning disability will depend on the eligibility criteria of the school district.

Recommendations

The following recommendations have been selected to address Kevin's needs. Additional recommendations should be developed collaboratively between Kevin's teachers and his parents.

Further Evaluation

  1. Kevin's parents should consider speaking to their developmental pediatrician about Kevin's difficulty with attention. Both a language disorder and an auditory processing disorder can mimic attention deficit/hyperactivity disorder (in this case, inattentive presentation), because when a person cannot understand what is going on around him, he is likely to tune out. However, because attention is critical to all learning and memory, if Kevin has a primary attention disorder, it will continue to interfere with all learning until it is addressed and treated.
  2. Because Kevin scored so low in almost all cognitive, memory, and academic areas, the parents and school should consider extending this evaluation into areas of adaptive skills to see if Kevin needs a more comprehensive educational program.
  3. Given that Kevin is now in a general education classroom, it is important that his academic progress be monitored on a regular basis. Keep in mind that Kevin has to achieve accuracy of content before he should be asked to go quickly (e.g., reading a list of words).

Accommodations

General Accommodations

  1. In the classroom, seat Kevin at the front of the room or closest to the teacher, because proximity may increase his attention and make it easier to redirect him when his attention wanders. As well, seat him away from possibly distractions such as the air conditioner or the window. Outlining the daily schedule and keeping it as consistent as possible will also be beneficial.
  2. Do not require Kevin to work under time pressure. Replace timed tests with alternative evaluation procedures. Place the emphasis in evaluation on accuracy rather than speed.
  3. When calling on Kevin in class, provide him with as much time as necessary to organize his thoughts and formulate a response. He may know the answer but need extra time to find the words. Privately, alert Kevin to this plan so that he does not feel pressured to come up with answers quickly. If needed, rehearse the correct response with Kevin before you call on him.
  4. Allow Kevin extended time on all mandated or standardized tests, including those that are read to him. Because of his difficulty with sustained and focused attention, his perceptual speed, and his difficulty with reading, spelling, and writing, this accommodation is critical to differentiate between lack of sufficient knowledge and lack of sufficient work time.

Homework Accommodations

  1. Make sure that all homework assigned to Kevin is at his instructional reading, writing, and math levels, because he will not learn from the level of work that the other students are assigned. He requires as much drill and practice as possible on basic skills.
  2. As much as possible, use videos for Kevin to learn, or at least become familiar with, content area information such as history and basic science concepts.
  3. Reduce the amount of work in each area assigned for homework so that Kevin can complete his assignment in approximately the same amount of time as other students are expected to spend. When grading a reduced assignment, consider the reduced amount of work to represent 100%. This emphasizes the quality of his work rather than the quantity.

Instruction

Teach Kevin some simple memory strategies and the situations in which to use them. [See Section IV: Memory Strategy Instruction, Mnemonic Strategies.]

Reading and Spelling

  1. Little information was provided regarding Kevin's reading program other than its being multisensory. The following recommendation builds on that Kevin needs an instructional program in basic reading and spelling skills that is systematic in introduction, practice, and reinforcement of phonemic awareness skills, phoneme-grapheme relationships, sight words, syllabication rules, structural analysis, and spelling rules. In a systematic program, skills are presented in graduated steps, from simple to complex, with students achieving mastery before the next skill is introduced. Practice assignments on the current skill incorporate previously learned skills, providing opportunities for the student to develop automaticity. Reading and spelling skills are taught simultaneously, so that the student learns how to spell every phonics generalization and word structure that he learns how to read. This type of program is called multisensory structure of language (MSSL). An example of such a program is the Wilson Reading System, which is currently being used in private tutoring with good results. Within this program, Kevin will need individual, intensive, daily instruction from a reading disability specialist trained to teach such a program. [See Section IV: Multisensory Structured Language Approaches to Teaching Reading.]
  2. Due to Kevin's low perceptual speed, he will need supplementary techniques to help him automatize orthographic skills such as recognizing common letter patterns and whole words. One technique offered here is reading speed drills using any word elements on which Kevin needs extra practice as they come up in his reading program (e.g., sight words, syllable structures, letter patterns, and phonic elements). [See Section IV: Speed Drills for Reading Fluency and Basic Skills.]
  3. Do not use separate lists for reading and spelling. Focus Kevin's spelling practice on the phonics patterns he is learning within the reading program. Use a whole word approach to teach spelling only for high-frequency sight words, and only as they come up in the systematic reading program. For whole-word spelling, use a multisensory method such as the Modified Fernald Technique to incorporate visual, oral/aural, and kinesthetic-tactile components. [See Section IV: Multisensory Study Method for Irregular Words.]
  4. Supplement Kevin's phonics instruction with decodable books that he can read during free reading time and at home. Decodable text is reading material comprising the phonics and sight words he has already learned. Reading decodable text provides the opportunity for application of newly learned skills, reinforcement of sight words, and transitioning skills learned in isolation to practical use. At his current skill level, reading aloud is best.
  5. If teaching reading comprehension strategies (e.g., paraphrasing the main idea, supporting details), teach them orally and at Kevin's oral language level, not his decoding level. As his word identification level increases, guide him to apply the listening comprehension skills to the reading material.

Math

  1. Before Kevin works a page of math computation problems, have him highlight the process signs before starting. He could also decide on a color for each of the signs and consistently highlight each in its own color.
  2. Start teaching addition facts and algorithms from the beginning, using a highly structured program. An excellent program for teaching math computation and the language of math is TouchMath (www.touchmath.com).
  3. Demonstrate new concepts and those that you are reteaching with manipulatives and, to the extent possible, highlighting any visual-spatial aspect. For example, one would teach regrouping in addition by using Base Ten Blocks on a place value mat to “trade up,” and then introduce the digits associated with each column.
  4. When teaching addition and subtraction facts, use strategies to help Kevin organize them mentally. Teach subtraction facts as a reformulation of addition facts and, later, division as the inverse of multiplication (but not until he has mastered addition and subtraction). Continue fact practice even when he seems to know them automatically. [See Section IV: Addition Facts: Organizational Structure for Memorization, Addition/Multiplication Facts: Instructional Sequence.]
  5. Build math fact fluency by using programs such as Great Leaps Math (www.greatleaps.com) and/or computer games that provide math fact drill. Effective computer programs will allow the teacher to program for the specific facts or algorithms that Kevin needs to practice and automatically monitor and document his success and errors.
  6. When Kevin has to memorize rote information, such as units of measurement, help him to create visual-auditory associations as memory aids. For example, 12 inches in a foot could be drawn as a footprint of a bare foot with 12 inchworms end to end crawling across it. Three feet to a yard is illustrated as a fenced lawn with three footprints across it.

Content Area Instruction

  1. Before new instruction, provide Kevin with an overview of the content and unfamiliar vocabulary words that will be introduced in a reading passage or lesson. Provide background knowledge of the subject area and encourage Kevin to connect the topic to previously learned information. When giving sequential information, tell him how many steps or events to listen or look for. Start with a few steps or events in order. Move on to more steps or events only after he is able to recall all requested information related to the most recent set of steps or events.
  2. When presenting information verbally, use accompanying visual images such as pictures, graphics (e.g., webs, charts, graphs), and videotapes. Enhancing auditory information may be as simple as drawing a sketch on the board to illustrate concepts such as contrasts, similarities, levels of importance, and cause–effect.

Self-Concept

Poor self-concept and emotional problems are common effects of learning disabilities if they are not treated appropriately and early. Although remediation is provided to alleviate these problems, school staff can help reduce the negative effects by giving frequent acknowledgment of actual improvement in skills or demonstration of knowledge. Make praise specific and meaningful. “Good job” may be meaningful, but it does not tell Kevin exactly what he did that was good. “You sharpened your pencil nicely” is specific but would not really matter to Kevin in terms of self-concept. “I noticed that you tried to sound out the words you didn't know rather than just guessing at them” is both specific and meaningful.

Reason for Referral

Michael suffers from a rare form of epilepsy that can cause significant cognitive decline. His parents initiated a reevaluation because they were concerned about the extent of his recent cognitive decline and because Michael has experienced frightening episodes of partial amnesia over the past six months. The Ambroses requested a reevaluation to determine whether he has declined in any of his cognitive abilities or academic skills and, if so, the extent. Thus, the current evaluation focused on Michael's cognitive abilities, memory, and academic skills.

Background Information

Michael's birth was a scheduled C-section after a 38-week pregnancy. There was a blood clot at the juncture of the placenta and the umbilical cord, and Michael had severe torticollis and a flattened head. However, he was healthy at birth, and there was no extended hospital stay. During infancy he had many middle ear infections, but no other major health concerns.

Developmentally, Michael had speech and language delays. His first words were not until 2½ years of age, and he did not produce short sentences until age 4. Michael received speech and language therapy until the age of 5. Prior to kindergarten entry, he was evaluated by his elementary school's speech-language pathologist, who concluded that Michael did not qualify anymore for speech-language services.

Michael was also evaluated at Northwestern University Learning Clinic. On the Wechsler Preschool and Primary Scale of Intelligence, Third Edition, Michael obtained a Full Scale IQ of 113, a Verbal IQ of 108, and a Performance IQ of 113. Based on Michael's weaknesses in early literacy and numeracy skills, the evaluators concluded that Michael had a learning disorder, not otherwise specified.

Michael's first psychoeducational evaluation by this examiner occurred 2 years ago as Michael was entering first grade. The referral was initiated by Michael's parents and his kindergarten teacher. Michael's parents were concerned about his slow progress in reading readiness skills, such as phonemic awareness, and about his poor retention of learning and his difficulty with word retrieval. The teacher reported that Michael had problems with attention, memory, emotional control, and speech and language. He also had difficulties learning colors, letters, and numbers. Michael had received support in arithmetic during his kindergarten year but remained delayed in arithmetic and other academic skills at the end of the school year.

At the time of the initial evaluation, no one was aware that Michael had a seizure disorder. It had been reported that he was often “spacy” and did not hear what was said, but there were no other signs that he was experiencing seizures. Just days after the initial evaluation, Michael experienced seven recognizable seizures in one day, culminating in a grand mal seizure that lasted 6 minutes. He was hospitalized locally and then transferred to Children's Hospital in Chicago. Children's Hospital diagnosed Michael with absence seizures and complex focal seizures. The seizures, which were not very observable, were originating in his left frontal lobe, an area involved with language production. Michael was placed on seizure control medication.

The discovery that Michael was having seizures in his left frontal lobe explained the inconsistencies in his oral language production. For example, he often confused verb tenses and would forget vocabulary that he had previously acquired. He also had significant word retrieval problems that seemed to come and go. Furthermore, Michael's epilepsy explained why he was not retaining new learning well. What he seemed to know one day would often be gone at a later time. Memory research has documented that seizures disrupt the memory consolidation process. That is, recently encoded (learned) information is not retained because seizures disrupt memory consolidation processes that create permanent memories.

After Michael went on seizure control medication, his parents began to notice that he occasionally appeared to have had a seizure. At times, he also had grand mal seizures. The incidents of unobserved seizures stood out because there would be a change in Michael's cognitive functioning and memory. He would become confused and have difficulty processing information. Even more apparent, his word retrieval problems would become severe. Also, his learning and performance would regress, and he would be unable to recall information that he had known for a long time. At times, these incidents involved partial amnesia. For instance, on one occasion he forgot how to flush a toilet. On another occasion he could not remember the names of his family members. Also, during these regressive episodes, Michael's parents reported that his writing would be “gibberish” and he would also become very distraught.

These incidents were reported to the neurologists who were monitoring Michael's epilepsy, and adjustments to his medication were made. However, it was not until approximately 18 months after his diagnosis that it was discovered that Michael was experiencing uncontrolled “spiking” of electrical activity in his brain, in addition to occasional seizures. This electrical spiking was causing ongoing memory problems for Michael because it was interfering with memory consolidation.

Finally, just before this reevaluation, Michael was taken to a different children's hospital for another neurological examination. At this point, it was discovered that Michael was suffering from a rare form of epilepsy. Michael was diagnosed with electrical status epilepticus during sleep (ESES). The onset of this disorder typically occurs at approximately 4 to 5 years of age. Approximately half of individuals with ESES experience neuropsychological regression consisting of IQ decrease, reduction of language (as in acquired aphasia), and motor impairment (such as ataxia or dyspraxia). No specific ongoing treatment for this syndrome is available. Seizure control medication may only work temporarily.

A year before the current evaluation, Michael was reevaluated with the WJ III Test of Cognitive Abilities (WJ III COG). His WJ III COG General Intellectual Ability (GIA) index was a standard score of 98. Most of his cognitive abilities were in the mid-Average range. His only below average scores were in Short-Term Memory, Working Memory, and Delayed Recall. His Short-Term Memory and Working Memory scores were also intra-individual weaknesses. Michael's above average Processing Speed score was an intra-individual strength. These scores were consistent with his WJ III COG results from two years ago as he was beginning first grade.

Michael is currently beginning third grade at Glendale Elementary School. He is receiving special education services under the category of other health impaired (OHI) because of the seizure disorder. He also has diagnoses of attention deficit/hyperactivity disorder (ADHD) and a specific learning disability in reading.

Behavior Observations

Michael did not attend school on the day of testing and was tested during morning hours. During testing, he engaged in effective test-taking behaviors. He was cooperative, attentive, strategic, and persistent, making an effort to do the best that he could. For example, he often self-corrected initial errors that he made. Breaks were taken when Michael requested them or when he appeared to be tiring. The current test results are considered to be reliable and valid estimates of Michael's current functioning levels.

Interpretation of Assessment Results

Michael was administered the WJ IV Tests of Cognitive Ability (WJ IV COG), the WJ IV Tests of Achievement (WJ IV ACH), and the WJ IV Tests of Oral Language (WJ IV OL). These new batteries have new tests and updated norms. Consequently, some scores may differ significantly from the WJ III scores that measure the same abilities and skills, but the examinee's abilities and skills may not have actually changed. The standard scores on the WJ IV and WJ III have a mean of 100 and standard deviation of 15 points. To be considered significant changes, the difference between these standard scores should be at least 10 points.

Cognitive Abilities and Memory

When Michael's current WJ IV scores are compared with his WJ III scores from a year ago, there appears to have been a significant decline in his overall cognitive abilities. The WJ IV General Intellectual Ability (GIA) score represents overall cognitive ability. A year ago, Michael's GIA was a mid-Average 98 (45th percentile), but currently is a Low Average 82 (11th percentile). Furthermore, his current cognitive ability composite of 82 is 31 points lower than the WPPSI-III Full Scale IQ of 113 that he obtained when he was 5 years old. Because of the extent of this apparent decline, it cannot be attributed solely to changes in the WJ's norms and tests.

Below is a summary of Michael's WJ IV COG performance compared with the scores he obtained on the WJ III a year prior:

  • His vocabulary knowledge is still average. However, the score has declined from a 104 on the WJ III COG Verbal Comprehension test to a 94 (currently measured by the WJ IV COG Oral Vocabulary test).
  • The WJ IV uses a new test called Number Series as the standard measure of Fluid Reasoning. To keep the task demands comparable, however, and to avoid comparison with a test involving math—a specific difficulty of Michael's—his Fluid Reasoning was again measured with the Analysis-Synthesis test. His score declined significantly from a mid-Average score of 105 to a Low range score of 78.
  • Michael has maintained his level of working memory. His current score of 83 on the new Short-Term Working Memory cluster is similar to his previous Working Memory score of 88.
  • His auditory memory span appears to have significantly improved, from a previous score of 77 on the Short-Term Memory cluster to a current Auditory Memory Span cluster score of 95. This improvement was attributable to a significantly improved Memory for Words (from a 69 to an 89) and a mid-Average score in Sentence Repetition. After the initial assessment two years ago, Michael completed a considerable amount of practice with face-to-face working memory exercises administered by his mother. Despite an overall decline in cognitive abilities, this cognitive training intervention may account for the maintenance of his overall working memory abilities and his improvement in auditory memory span.
  • His performance on a Processing Speed test declined from an Above Average level (previous score of 113 on Visual Matching) down to an Average level of 95 on Number-Pattern Matching (same test, new name). Again, although the new test used as a single measure of Cognitive Processing Speed is Letter-Pattern Matching, the test used here was Number-Pattern Matching to keep the task demands comparable. As well, Michael has particular difficulty with reading skills, to which Letter-Pattern Matching is closely associated.
  • Michael's current Auditory Processing is in the low end of Average (score of 90), in contrast with his previous high end of Average score of 108. However, the task demands on the current Auditory Processing cluster are considerably more complex than those on the WJ III Auditory Processing cluster. They involve rapid retrieval of words from long-term memory based on specific sounds (e.g., words that begin with /r/; and words that have /f/ at the end), as well as substituting sounds in one word to make another word. Although his Auditory Processing has been somewhat inconsistent in the past, the higher and more complex task demands of the current Auditory Processing tests are likely a major contributing factor to the decrease in Michael's score.
  • Similarly, Michael's performance on phonological processing measures has also been inconsistent in the past. His current Phonetic Coding score of 93 is still in the Average range but down from his prior Phonemic Awareness score of 104. A change in tests from Incomplete Words (i.e., filling in a missing sound to figure out a complete word) to Segmentation (i.e., saying all of the individual sounds in a word) might be partially responsible for this difference.
  • His current Long-Term Retrieval cluster score of 88 is down a little, but not significantly, from the last evaluation. There appears to be no change in his ability to encode information into long-term memory and maintain it over a period of several minutes. Again, this may the result of some working memory interventions that have been attempted with Michael over the past 2 years.
  • His retrieval speed from long-term memory, as measured by the new Speed of Lexical Access cluster on the WJ IV OL, is still Average, with a score of 93. However, it appears significantly slower than before. On the WJ III, when the same two tests were used to measure this ability a year ago, he had scores of 116 and 108.
  • His visual-spatial processing also appears to have declined. On the new Visualization test, which consists of Spatial Relations and Block Rotation, he had a score of 90, whereas last year he had a Visual-Spatial Thinking score of 102.

Pattern of Strengths and Weaknesses (PSW)

The WJ IV Intra-Cognitive Variations do not reveal any significant intra-individual strengths and weaknesses when the SEE value is set at 1.00 SD. This suggests that Michael's cognitive abilities do not vary significantly from each other. In fact, less variability exists than there was a year ago. While the majority of his cognitive abilities seem to have declined, his weaker areas of memory have improved, resulting in a fairly flat profile.

Because Michael's primary placement is for OHI, not a specific learning disability, it is not necessary to document a pattern of strengths and weaknesses in cognitive processes. There were also no significant discrepancies in the GIA/Achievement Discrepancy Procedure. However, the Scholastic Aptitude/Achievement Comparisons table does reveal that some of his academic skills are significantly lower than his specific cognitive aptitudes, especially in Written Language. This suggests that he is underachieving in written language, even though his cognitive abilities have declined.

Academic Skills

Reading

Overall, Michael's reading skills are from the middle to end of first-grade levels, with grade equivalent scores ranging from 1.4 to 1.8, except for his Reading Recall score of 2.2. His standard scores on the various reading tests are consistent with each other and generally in the low 80s. Curiously, his higher score of 93 on the new Reading Recall test indicates that he is able to retell what he has read. Compared with his reading scores from last fall, there appears to be no overall improvement, if not a slight decline. His previous scores were in the 90s with grade equivalents at beginning second-grade level.

Mathematics

Michael's mathematical skills are basically at the same level as his reading skills, generally at the end of first-grade level, with scores in the low 80s. The only exception is Applied Problems, where he obtained a score of 90 and a grade equivalent of 2.3. Compared with a year ago, there appears to be no improvement in math skills and perhaps even a slight decline.

Written Expression

Michael's writing skills are at the end of first grade to beginning second-grade level. Spelling remains his primary challenge, with a percentile rank of only 2. Spelling ability has dropped dramatically from a year ago, 19 standard score points. His speed of generating and writing short, simple sentences has also declined, but his ability to communicate his ideas in writing, untimed, remains about the same.

Summary

Michael's epilepsy appears to be causing a decline in his overall cognitive functioning. Most of his cognitive abilities seem to be significantly lower than a year ago. However, the exact amount of change is difficult to estimate because the WJ IV COG is quite different than its predecessor, the WJ III COG. Currently, Michael has Average abilities on measures of Comprehension Knowledge, Auditory Memory Span, Processing Speed, Auditory Processing, Phonological Processing, Visual Processing, and Speed of Lexical Access. In contrast, he has Low Average abilities on measures of Fluid Reasoning, Working Memory, and Long-Term Retrieval. Although many of his abilities are still in the Average range, many of them previously tested at the upper end of the Average range or even High Average. Michael does not display a pattern of cognitive strengths and weaknesses. Overall, his academic skills seem not to have declined as much as his overall cognitive ability has. Conversely, his academic skills have not improved over the past year. Most of his achievement scores are in the Low Average range (see the WJ IV Score Report).

Recommendations

  1. Because of the apparent decline in Michael's cognitive abilities, he should be reevaluated within the next year, especially if changes are made in his epilepsy treatment.
  2. Michael should continue with internet-based cognitive training and face-to-face working memory exercises. The online cognitive training exercises should include attention, working memory, speed training, and problem solving.
  3. Michael should be taught long-term memory strategies that will enhance encoding and recall of recently learned information.
  4. Parents and teachers should continue to teach Michael strategies that will help him compensate for and cope with his current cognitive challenges and his academic achievement delays.

Reason for Referral

Emma was referred for an evaluation by her grandmother and her mother, Ava Sanchez. Their main concern was trying to understand why Emma has had such difficulty learning in school when she seems capable in so many ways. She has struggled since kindergarten with the acquisition of basic reading, writing, and math skills. Both the mother and grandmother report that Emma has trouble concentrating and just seems to get lost in her thoughts at times. The purposes of this evaluation were to determine what factors are affecting Emma's academic progress, as well as what instructional methodologies would be most effective for addressing her educational needs.

Background Information

Emma lives at home with her mother and father and three siblings: Keith, age 2; Ava, age 4; and Lily, age 6. Her father is the director of property management for Naylor investments, and her mother is a registered nurse at Century Neurological Institute at St. Randall's Hospital. In an interview with the parents, Mr. Sanchez described his own struggles in school. He noted that he had difficulty with attention, and that his favorite part of school was “recess.” Mrs. Sanchez commented that Lily is “so different from Emma,” rather quiet and experiencing success in school. Ava, however, was described as “a spit-fire.” At parent night, her preschool teacher noted that Ava has difficulty engaging in group activities for the allotted time, albeit brief, and that she fidgets so much during circle time that other children leave extra space on each side of her. Her impression was that Ava is just a bit more immature than the others and will catch up in time.

In contrast to Emma's difficulty with academic skills, language and science have always been areas of strength and interest. Emma loves the outdoors and can spend hours observing plants, bugs, and animals. She loves to conduct experiments and engage in hands-on activities. She is very excited that the family may get a Yorkshire terrier in the near future. Emma is a very social child and loves being with other children.

Emma began kindergarten at Palmer Elementary School in Winnetka, IL. Because she was having such difficulty learning simple things, such as the names of the letters of the alphabet, her kindergarten teacher recommended that she repeat kindergarten. The teacher also said that Emma's reading and writing were more like a preschool child, and she suggested that the Sanchezes write a letter requesting retention. At first, Emma found her second year of kindergarten to be easy, but midway through the year she confided to her mother that it was a “big mistake” and she was not learning anything new. At this point, the Sanchezes requested that the school provide a comprehensive evaluation, but they were informed that Emma was too young for an evaluation, and it was likely that she would grow out of her difficulties.

The family then moved to the Armory School District, and Emma attended Elmwood Elementary for first grade and the first half of second grade. By the middle of second grade, her teacher reported that Emma was falling further and further behind on school benchmarks, but there was nothing that they could do because she still was not far enough behind to be eligible for special help. Her teacher suggested she attend a reading and math inter-session camp, which she did, but although Emma enjoyed the camp, she did not seem to make much progress. In mid-second grade, when the Sanchezes requested additional support for Emma, they were told again that she did not rank low enough to receive special help. During this period, Mr. Sanchez noted that Emma would often spend over three hours a night trying to complete her homework, and that little was accomplished unless one parent would sit with her the entire time. Frustrated by Emma's lack of progress and the lack of school support, Mrs. Sanchez decided it would be best to home-school Emma so that she could receive more intensive help geared to her present performance levels. She removed Emma from Elmwood at the end of the semester (December 2014). After several months, Ms. Sanchez felt that the home schooling was not the solution because Emma was still not making adequate progress. Additionally, although Emma enjoyed homeschooling, she missed the social aspects of school. The family has recently moved to the Table Mesa School District, and the plan is to enroll Emma in third grade in the fall.

Tests Administered

  1. Woodcock-Johnson IV Tests of Cognitive Ability (WJ IV COG)
  2. Woodcock-Johnson IV Tests of Oral Language (WJ IV OL)
  3. Woodcock-Johnson IV Tests of Achievement (WJ IV ACH)
    1. (A complete set of WJ IV scores are attached to the end of this report).
  4. Test of Orthographic Competence (TOC), Sight Spelling and Homophone Choice
  5. Test of Silent Word Reading Fluency-2 (TOSWRF-2)
  6. Behavior Assessment System for Children, Second Edition (BASC-2), Parent Rating Scales

Testing Observations

Testing was conducted in two 2-hour sessions. Emma was polite and engaged during all of the testing. She was cooperative in both sessions, but her ability to sustain attention was quite variable. The first session was conducted at a patio table outside. Emma had requested this, saying that it was easier for her to concentrate outside. This proved true, because Emma was highly engaged and had little difficulty concentrating. On occasion she commented on things she observed, such as a bird eating or the sound of the water in the pool, but she was easily redirected to the task. She commented about how much she loved the outdoors. Because of inclement weather, the second session was conducted at the dining room table in her grandmother's home. Emma had extreme difficulty concentrating and sustaining attention. Although she tried to do her best and apologized several times for her lack of focus and concentration, directions had to be repeated on several occasions, and she had to be reminded numerous times about what she was supposed to be doing.

Test Results

The WJ IV COG, WJ IV OL, and WJ IV ACH were scored according to age norms. Because these three batteries are co-normed, direct comparisons can be made among her cognitive, oral language, and achievement scores. These comparisons can help determine the presence and significance of any strengths and weaknesses among her abilities. These tests provide measures of Emma's specific cognitive and oral language abilities, as well as her academic achievement.

Emma's performance is compared with her age-peers, using standard score (SS) ranges:

SS Range <69 70–79 80–89 90–110 111–120 121–130 >130
Verbal label Very Low Low Low Average Average High Average Superior Very Superior

Her proficiency on specific tasks is described by relative proficiency index (RPI) levels:

RPI Range 0–3 3–24 24–67 67–82 82–95 95–98 98–100 100
Level of Proficiency Negligible Very limited Limited Limited to Average Average Average to Advanced Advanced Very Advanced

Percentile ranks and standard scores are provided for the TOC and TOSWRF. At-risk and clinically significant ratings from the BASC-2 are also presented.

Cognitive and Linguistic Abilities

Based on the tests of the WJ IV COG, Emma's overall intellectual ability, as measured by the GIA (Std), was Average; however, significant strengths and weaknesses were found among her abilities. In general, Emma's proficiency in language use and comprehension, as well as her general knowledge and vocabulary, were Advanced. Emma could easily define words, repeat back stories she had just heard, and answer questions involving general information and world knowledge. In fact, her oral language abilities were in the superior range. In contrast, her cognitive efficiency, a combination of processing speed and memory, was Limited. Cognitive efficiency is the ability to quickly recognize visual symbols (e.g., letters and numbers), hold them and related information in memory, and work with them to solve a problem or come up with an end product. This ability underlies much skill-learning, but especially basic reading, spelling, and math skills. In this regard, Emma had the most difficulty on tests of processing speed (requiring the rapid processing of symbols), as well as tests involving memory for noncontextual information (e.g., lists of unrelated items rather than stories) and for visual symbols, such as letters and numbers. Additionally, she was confused when directions involved pointing to the right or pointing to the left.

Academic Achievement

Emma's academic performance was assessed in reading, writing, and math, using selected tests of the WJ IV ACH. In addition, her performance was analyzed on the TOC and TOSWRF-2. Currently, Emma's present proficiency in all academic skills (reading, writing, and math) is very limited.

Reading

On the reading tasks, Emma had difficulty applying phonic skills, recognizing common sight words (e.g., there, when), and reading simple sentences quickly. As words increased in length, she tended to guess at their pronunciations by using the initial consonant (e.g., reading sentence as “science”).

WJ IV ACH Cluster/Tests RPI Proficiency SS(±1 SEM)
BROAD READING 31/90 Very Limited 77 (75–80)
Letter-Word Identification 4/90 Very Limited 79 (77–82)
Sentence Reading Fluency 78/90 Limited to Average 88 (83–93)
Passage Comprehension 36/90 Limited 82 (79–86)

Emma scored significantly Below Average on the TOSWRF-2, with a percentile rank of 10 and a standard score of 81. These findings suggest that Emma will read much more slowly than average age-peers and have difficulty recognizing common words appropriate for her age level.

Written Language

Emma's lowest performance was in basic written language skills. The Spelling test measured her ability to spell words, progressing from simple to more complex spelling patterns. Although she was able to represent the sounds in many of the words, she did not recall the correct letter sequences (e.g., she spelled are as “aer,” under as “undre,” and house as “heows.”) When spelling a phonically regular nonsense word that began with a /k/ sound, Emma began the word with the letters “ck,” a pattern that never starts an English word.

In contrast, on the Sentence Writing Fluency test, given three words and a picture, Emma was able to formulate and write simple sentences quickly. On Writing Samples, a test that measures skill in formulating and writing sentences or phrases in response to a variety of demands, Emma wrote complete sentences, but her spelling was so poor that many words were barely recognizable (e.g., “boleu” for bull, “blee” for belt). Emma has an unusual pencil grip, which makes it difficult for her to form letters easily and results in her putting too much pressure on her thumb.

WJ IV CLUSTER/Tests RPI Proficiency SS(±1 SEM)
BROAD WRITTEN LANGUAGE 23/90 Very Limited 64 (60–68)
Spelling 4/90 Very Limited 67 (62–71)
Sentence Writing Fluency 81/90 Limited to Average 92 (85–99)
Writing Samples 14/90 Very Limited 66 (61–72)

Emma was also administered two subtests from the TOC, Sight Spelling and Homophone Choice. Her percentile rank was 3, and her standard score was 73, with a descriptive term of “poor.” Emma had considerable difficulty spelling irregular words. Specifically, she had difficulty recalling the parts of the words that must be memorized visually because they are not spelled as they sound (e.g., was, said).

Mathematics

In basic math skills, Emma could only complete simple calculations. She was able to add and subtract single digits by counting on her fingers. She was unable to complete two-digit addition and subtraction problems with or without regrouping. On several problems, she failed to notice the operation sign. Her proficiency on the Math Facts Fluency test indicated that she will find tasks requiring her to solve simple addition and subtraction problems barely manageable. Emma had difficulty solving simple story problems and counting change below a dollar. She was, however, able to count accurately with pictures, add and subtract using pictures, identify the names of coins, and identify the time on a clock to the hour.

WJ IV CLUSTER/Tests RPI Proficiency SS(±1 SEM)
BROAD MATH 42/90 Limited 72 (68–76)
Calculation 43/90 Limited 77 (70–84)
Math Facts Fluency 82/90 Limited to Average/Average 83 (79–86)
Applied Problems 10/90 Very Limited 74 (69–78)

Clinical Clusters

On both the Academic Skills and the Academic Applications clusters, Emma's proficiency was Very Limited. These clusters measure basic skills and practical application of those skills across academic areas. (Basic Skills: word identification, computation, and spelling; Application: reading comprehension, math application, written expression). Emma's limited academic skills are likely the major reason for her difficulty with the higher-level academic applications. Her proficiency was Limited to Average on the Academic Fluency cluster, indicating that she has somewhat less difficulty in the speed with which she reads and writes simple sentences and solves math facts.

Emma's proficiency on each of the two tests of the Phoneme-Grapheme cluster was significantly different. On the Word Attack test, her proficiency was Very Limited (RPI 9/90), whereas on the Spelling of Sounds test, her proficiency was Limited to Average (RPI 81/90). Emma found it easier to spell phonically regular nonsense words (made-up words that conform to common English spelling patterns) than to read them.

WJ IV Variation and Discrepancy Procedures

On the WJ IV, intra-individual variations are computed to show the likelihood of a person obtaining a particular score, given the average of their other core cognitive, oral language, and achievement test scores. Large variations indicate areas of significant strength and weakness. On the WJ IV COG, Emma's verbal ability was a significant strength, whereas her cognitive efficiency was a significant weakness.

Additionally, based on her General Intellectual Ability—Standard score (GIA-Std), as well as her Oral Language scores, all of Emma's academic scores were significantly below expectations. Her overall verbal and reasoning abilities were significantly higher than her current levels of reading, writing, and math performance. When her Oral Language score is used to predict her reading, writing, and math performance, only one (or fewer) persons out of 100 would have such large discrepancies. When her Oral Language score is compared with her Broad Written Language score, only one out of 1,000 students with her predicted score would have a standard score as low. This means that Emma's verbal abilities are significantly higher than her current academic performance levels.

Basc-2 Parent Ratings

Ms. Sanchez completed a BASC-2 rating scale regarding her perceptions of Emma's temperament and behaviors. In interpreting results of the Clinical Scales, high scores indicate negative or undesirable characteristics. T-scores of 60–69 constitute the at-risk range (scoring above 85% of peers), and T-scores of 70 and above suggest clinically significant (severe) problems (scoring above 95% of peers). On the Adaptive Skills composite, the opposite is true; lower scores indicate characteristics of more concern. At-Risk scores are those that fall at 40 and below (scoring below 15% of peers); Clinically Significant problems are those falling at 30 and below (scoring below 5% of peers). On all scales, scores in the at-risk range may identify a significant problem that may not be severe enough to require formal treatment or may identify a potential problem that needs careful monitoring. Scores in the clinically significant range suggest a high level of maladjustment and usually warrant follow-up. The following scales for externalizing problems, internalizing problems, and behavioral symptoms had at-risk or clinically significant results:

At Risk

Clinical Scales T-score Percentile Rank Mother's Comments
Anger Control 62 88 Emma has a tendency to become irritable quickly, and has difficulty maintaining self-control when faced with adversity.
Negative Emotionality 69 96 Emma has a tendency to react negatively when faced with changes in everyday activities or routines.
Aggression 64 91 At times Emma can be argumentative and defiant.
Anxiety 60 85 Emma sometimes displays behaviors stemming from worry or fear.
Depression 66 93 At times Emma seems withdrawn and/or sad.
Adaptive Scales T-score Percentile Rank Mother's Comments
Adaptability 35 7 Emma has difficulty adapting to changing situations.
Daily Living 34 7 Emma has difficulty performing simple daily tasks in a safe and efficient manner.
Functional Communication 40 17 Emma has trouble seeking out and finding information on her own.

Clinically Significant

Clinical Scales T-score Percentile Rank Mother's Comments
Emotional self-control 73 97 Emma has a tendency to become easily upset, frustrated, and/or angered in response to environmental changes.
Executive Functioning 74 98 Emma has difficulty controlling and maintaining her behavior and mood.
Hyperactivity 74 97 Emma engages in many disruptive, impulsive, and uncontrolled behaviors.
Atypicality 73 96 Emma often seems disconnected from her surroundings.

In general, findings from the Parent Report of the BASC-2 indicate that Emma is easily distracted and has difficulty maintaining attention and self-control. She has difficulty working independently and completing tasks. She becomes upset and frustrated easily when faced with homework and challenging tasks, worries about making mistakes, and has sudden mood changes. She tends to be argumentative when she does not get her own way. These difficulties with behavior and maintaining attention most likely affect her academic performance at home and in school.

In contrast, Emma makes friends easily, compliments and shows interest in others, shares easily, and always encourages others to do their best. She enjoys social situations, has good communication skills, and does not engage in rule-breaking behaviors. She is kind, considerate, and enjoys social interactions with both adults and other children.

Conclusions

Emma will be entering third grade in the fall. She was referred for an evaluation by her grandmother and mother because of concerns regarding her ability to pay attention, as well as her academic progress. Emma's verbal ability (acquired knowledge and language comprehension) is advanced when compared with her age-peers. In contrast, her processing speed and memory were both significantly lower. Presently, her overall level of proficiency of achievement in reading, writing, and mathematics is very limited.

Diagnostic Impressions and Rationale

Specific Reading Disability (Orthographic Dyslexia)

Emma has extreme difficulty visualizing words and their spellings. She tends to rely on the way that words sound, rather than the way that they look. She confuses letters with similar appearance (e.g., b and d) and transposes the order of letters when writing (e.g., hre for her). She does not recognize common words quickly and easily. Given her high level of verbal competence, her supportive family, her retention in kindergarten, and her individualized home schooling for five months, one would predict much higher levels of academic performance.

Attention Deficit/Hyperactivity Disorder (ADHD)

Emma has extreme difficulty concentrating in situations demanding sustained attention, such as sitting in a classroom or trying to complete homework. Symptoms of ADHD were observed in the past by school staff, as well as by her parents and this evaluator.

Educational Recommendations

School Programming

  1. Eligibility for learning disabilities (LD) services should be explored as soon as Emma returns to school. Emma has severe discrepancies between her abilities and her achievement, and has not made sufficient academic progress in school even with additional assistance.
  2. As she enters third grade, Emma will require specific accommodations in the classroom, such as extended time on assignments and shortened homework assignments. Emma would benefit from sitting at the front of the classroom so as to maximize attention.
  3. Until her academic performance improves, Emma will need adjustments in the difficulty level of school and homework assignments. Although she is entering third grade, her performance levels are significantly below that of her age peers.
  4. When possible, break her in-class assignments into smaller, more manageable chunks. Give her one part at a time, with instructions to hand each in as it is completed and pick up the next. Each time she hands in a portion of the work, provide reinforcement for completed work and time on task. Using this technique, she is more likely to stay on task and complete assignments.
  5. Emma has very advanced language skills, as well as considerable world knowledge. Provide Emma with opportunities, such as oral reports and science projects, that will allow her to use her strengths and excel in certain school subjects.

Parents

  1. To make progress, Emma requires specialized academic instruction from a teacher with expertise in teaching children with attention and learning difficulties. Ideally, intensive private tutoring should be provided during this summer for an hour at least three times a week.
  2. Emma's parents are encouraged to meet with their pediatrician to discuss the possibility of attention deficit/hyperactivity disorder (ADHD) and consider medications that may be beneficial for increasing Emma's focus and ability to concentrate.
  3. Many of the behaviors that Emma exhibits result from her problems with attention, learning, and memory. Keep in mind that her difficulties do not stem from a lack of effort or caring. Emma wants to do well, but too often the expectations and academic demands are too high for her present skill levels.
  4. Emma will feel frustrated by her attention and learning difficulties. Try to minimize her frustration by providing short periods of instruction (e.g., 10 to 15 minutes) that are followed by rewards, or some type of engaging activity, such as a game. Alternatively, she may stay on task for longer periods if she is rewarded throughout the activity (e.g., chips, Monopoly money) such as for correct answers, for obviously good reasoning related to the task, or for a short period of good attention and cooperation.
  5. Incorporating games into reading practice will make reading practice more fun and improve her attention to the task. Ask the teacher or tutor what specific reading skills Emma is working on and request a list of words that will help Emma practice the skill. Make games out of word attack or sight word practice by using board games such as Chutes and Ladders or Candy Land. For example, if Emma is working on recognizing the closed syllable pattern (one vowel followed by one or more consonants to end the word) and remembering to use the short vowel sound, use a list of closed-syllable words using only the letters she has been taught. Make a stack of cards with one word written on each. (For phonics practice, you can include both real and nonsense words.) Before taking a turn, the player must choose the top card and read it. Then the player may spin the spinner (or roll the dice, depending on the game). As soon as Emma gains some fluency in reading the words, request a list of words at the next level of instruction and make new cards.

Initial Tutoring Goals

  1. Provide Emma with an intensive synthetic phonics program that will teach her phoneme-grapheme relationships directly. She will also require instruction that increases her recognition of common letter patterns, builds her understanding of spelling rules, and shows her how to use structural analysis. Reading and spelling instruction will be most effective if the patterns taught for reading are taught for spelling at the same time.
  2. Use a systematic spelling program, such as Scholastic Spelling (in conjunction with the phonics program), that will provide Emma with a solid foundation in common orthographic spelling patterns, as well as how to spell common high-frequency words.
  3. To build speed and accuracy in pronunciation of sight words and phonetically irregular words, use one-minute speed drills. Emma should be timed daily on reading lists of common, irregular words, as quickly as she can but with accuracy as the primary goal. Her daily performance should be recorded and displayed on some type of graph. She should also practice spelling these words.
  4. Teach Emma basic math skills, including addition and subtraction facts and the algorithms for adding and subtracting two-digit numbers without regrouping. When these are mastered, progress to problems involving regrouping. If Emma is unable memorize math facts, or while she is doing so, teach TouchMath as a bridge to learning the facts and as a well-structured procedure for learning the algorithms.
  5. Provide practice in math facts using fun video games. Many of these can be found on www.coolmath.com. When she is ready to learn multiplication facts, a good resource is “Timez Attack” which can be played on the internet: http://www.bigbrainz.com
  6. Provide instruction in learning the value of coins and how to add them together and make change.
  7. To maximize attention, alternate instruction in basic academic skills with games to reinforce and develop those skills or incorporate skill practice into games as described previously. Alternatively, incorporate earning chips or Monopoly money throughout the lesson, allowing her to exchange these for a reward at the end of each session.

Note to Reader

The Relative Proficiency Index (RPI) is a mathematical prediction, based on the norms, of the examinee's likelihood of success on a difficulty level at which an age- or grade-peer of average ability would have a 90% likelihood of success. The RPI is based on the size of the discrepancy between the W-score that the examinee obtains on a test and the Reference W for the age- or grade-peers. For each test, the Reference W represents a specific difficulty level at which students of average ability, and of the same age or grade as the examinee, are 90% likely to pass. The RPI answers the question: “When the difficulty level of a task is such that the average age- or grade-peer is 90% likely to be able to perform the task, how likely is it that the examinee will be able to manage it?” The symbol x/90 indicates this ratio. The numerator represents the examinee's likelihood of managing the task at that level of difficulty, and the 90 represents the likely success of the average person in the comparison group—90%. For example, if Jenny has an RPI of 65/90 on Word Attack, when sounding out unfamiliar words, her probability of success is 65% at the same difficulty level at which her peers are 90% successful. The reader is referred to Section I of this book, where the RPI is explained more fully, and to WJ III Assessment Service Bulletin #11, Development, Interpretation, and Application of the W Score and the Relative Proficiency Index (Jaffe, 2009). http://www.riverpub.com/products/wjIIIComplete/resources.html

In her evaluation, Dr. Yermish uses an alternative form of the RPI, the Generalized Relative Proficiency Index (GRPI), which, like the RPI, is based on the W-Difference. However, instead of centering the probability of success on 90% for the reference group, the GRPI is centered on the probability that the examinee will be 90% successful on the task. The question becomes: “If the difficulty level of the task is increased to the level that the examinee has a 90% chance of success, how likely is it that the average age- or grade-peers are going to be able to be successful?”

For example, although Jenny is having difficulty with Word Attack, she is “twice exceptional,” having exceptional intelligence and a disability, dyslexia. However, her obtained W-score on the WJ IV Quantitative Reasoning cluster was 45 W units higher than the Reference W (W-Difference = +45). Her RPI in Quantitative Reasoning is 100/90, which is as high as the RPI goes. This indicates that she is predicted to be 100% successful when working at a difficulty level at which her average grade-peers would be 90% successful. The GRPI, however, provides a better indication of her ability level, the level of difficulty she is likely to be able to manage. The +45 W-Difference is associated with a GRPI of 90/6, indicating that when doing tasks similar to those in the Quantitative Reasoning cluster, at a level of difficulty at which Jenny would be 90% successful, her average grade-peers are predicted to be only 6% successful. This gives us a more accurate perspective on the type and level of work that Jenny requires in the classroom if she is to have meaningful work.

Dr. Yermish explained her reason for using the GRPI as follows:

If we see a score like 90/75, we can say, “Okay, Jenny needs more challenge than the other kids, and the kind of work she is likely to find challenging would be difficult, but not impossible, for other kids her age.” In all likelihood, the teacher will have some ideas on how to do that and may even have some materials.

But if we see a score like 90/16, we'd say, “If we are going to give Jenny a real challenge, we have to run the difficulty level up to a point where the average student in her grade couldn't even begin to approach the task.” The likelihood of the teacher having materials in her file that will be appropriate for her is pretty slim. It would take some work to figure out how to do that on a practical, everyday level. The teacher might need to get help from the school librarian, learn how to create tiered assignments, and figure out how to let Jenny demonstrate skill mastery through curriculum compacting so she can get her time freed up to work on things that are interesting and challenging. When working with particularly bright children, an RPI of 100/90 may attenuate our understanding of their true ability level and the GRPI may be a more practical measure.

The following table provides the GRPI associated with each possible W-Difference.

W-Difference Scores Associated with the Generalized Relative Proficiency Indexes

WDiff GRPI WDiff GRPI WDiff GRPI
69 to 70 90/0 26 90/34 3 90/87
59 to 68 90/1 25 90/37 2 90/88
54 to 58 90/2 24 90/39 1 90/89
51 to 53 90/3 23 90/42 0 90/90
48 to 50 90/4 22 90/45 −1 90/91
46 to 47 90/5 21 90/47 −2 90/92
45 90/6 20 90/50 −3 to −4 90/93
43 to 44 90/7 19 90/53 −5 90/94
42 90/8 18 90/55 −6 to −7 90/95
41 90/9 17 90/58 −8 to −10 90/96
40 90/10 16 90/61 −11 to −13 90/97
39 90/11 15 90/63 −14 to −18 90/98
38 90/12 14 90/66 −19 to −22 90/99
37 90/13 13 90/68 −23 90/99.1
36 90/15 12 90/71 −24 90/99.2
35 90/16 11 90/73 −25 90/99.3
34 90/18 10 90/75 −26 to −27 90/99.4
33 90/19 9 90/77 −28 to −29 90/99.5
32 90/21 8 90/79 −30 to −31 90/99.6
31 90/23 7 90/81 −32 to −34 90/99.7
30 90/25 6 90/82 −35 to −39 90/99.8
29 90/27 5 90/84 −40 to −49 90/99.9
28 90/29 4 90/85 −50 90/100
27 90/32

Reason for Referral

Dustin is a 9-year-old child who lives with his family in Ferris, Texas. He was referred for assessment by his parents, who are primarily concerned about his difficulties in regulating attention and emotion, as well as concerns about potential learning disabilities affecting processing of visual/spatial information. Although he is clearly a very bright child, he has chronically struggled with writing and has not been developing age-appropriate self-care and social skills. His parents are interested in an in-depth look at his strengths and weaknesses and wish to receive guidance on how to help him cope more effectively in both the academic and social realms.

Background Information

Dustin is the only child of an intact family. He has many pets, including seven chickens and three puppies. English is the only language spoken in the home. The family history is positive for suspected ADHD, although there has been no formal diagnosis. He was delivered by Caesarian section 7 weeks prematurely, because his mother had severe pre-eclampsia, and stayed for several weeks in the NICU.

At age 3, his oral language was described as well beyond age expectations, including a wide vocabulary and use of complex sentences. For example, he stated that when he grew up, he wanted to be an ornithologist and part-time Zamboni driver. At present, he manifests a very large vocabulary and wide range of knowledge of topics of interest to him.

His early gross motor skills were delayed from birth; milestones were met at the end of the typical age ranges. He has been receiving physical therapy services since the age of 7 months. He participated in occupational therapy services through Early Intervention but was discharged at age 5. At that time, the occupational therapist viewed his problems as being primarily in the gross motor domain. However, his pencil grip was highly immature and his writing illegible. Shortly after this assessment was conducted, based on the current findings, he resumed weekly OT at South Ferris Rehabilitation Center to address fine and graphomotor coordination.

In the social realm, Dustin is very affectionate and is constantly seeking attention from or conversation with his mother or any other people around. He loves school, both his peers and his teachers. However, his mother, because of her own work in education, states that he seems immature socially. He is unable to regulate his emotions at an age-appropriate level, often crying or melting down when he becomes upset. Events that are mildly upsetting to peers (e.g., letting the class butterfly go) are deeply upsetting to him; however, he does not want to talk about his feelings with adults.

Dustin is persistent in trying to get his way. Additionally, he is often derailed when he perceives a situation to have been “unfair.” When things do not go as he wishes, he often complains that he is being punished. He tends to become fixated on an idea and then uses his superior language skills to debate and argue with peers and adults, often becoming insulting and raising his voice in anger. His words and actions are often disruptive to teachers and peers. It is difficult for adults to redirect him at those times.

He has substantial difficulty in regulating his attention, particularly in a large group or when there is a lot going on around him. He often seems to miss out on directions and thus fails to do what he has been asked to do. He has difficulty sitting still for classroom-appropriate periods of time and talks constantly during class, including during silent reading. He is typically sharing his enthusiasm for what he is reading or trying to engage with others, but he is not able to recognize when this behavior is inappropriate.

His teachers observe that he often fails to pull up his pants properly after using the toilet and that he does not often wash his hands thoroughly. Interventions (tabs on the pants for him to pull up, a timer for hand washing) have been helpful.

Academically, his progress has been very strong. He taught himself to read without any help at age 3. The primary limiting factor in academics is his poor handwriting, which has constrained his progress in writing and may be affecting his enjoyment of mathematics.

He has historically not been good at sports. Although he has participated in skating, soccer, and baseball in the past, he is not interested in participating in them at present. He is eager to participate in a range of other activities, including summer camp. He loves reading, music, iPad games, Legos, nature, and Pokemon cards.

No psychotropic medications have ever been recommended or prescribed.

Educational History

Dustin attended Children's Discovery Center from ages 3 to 5, and Milford Independent School from ages 5 to 7 (pre-K through first grade). He was retained during a preschool year because of his physical and behavioral immaturity.

He attended second grade in a combined second/third grade classroom at the Florence Statton School, a small private school. Because of his academic knowledge and strong oral language, he was able to complete the third-grade work and has now been advanced a year, so that he is now with his age-mates. As of fall 2015, he is nominally in fourth grade in a combined fourth-/fifth-grade classroom. Because he is in a private school, his parents have not sought services under the special education system. They are pleased with the small class size and individual attention he receives.

Behavioral Observations

Dustin was brought to the office by his mother. He was alert and appropriately oriented. He seemed emotionally and behaviorally somewhat younger than his stated age, although his vocabulary and the complexity of his speech were more typical of an older child. His eye contact was intermittent, and turn-taking in conversation was quite weak. He spoke nearly constantly and was clearly highly interested in forming a social connection with me. He used socially appropriate language, but sometimes in the wrong ways, greeting me as if we had just met for the day when he returned from a bathroom break. His verbalizations were also accompanied by a wide variety of grunts, sound effects, and silly foreign accents; he was highly resistant to redirection when making these sounds. He seemed not to recognize that grunts and noises were not actually comprehensible words.

Although he verbalized continually while working on test tasks, he did not use verbal mediation strategically or effectively for problem-solving. That is, he made constant commentary on the test items, but did not use language to help him work through the problems. He did not appear to use any nonverbal strategies for problem-solving, either. Rather, when he did not immediately know the answer to an item, he often claimed that it was impossible and guessed randomly.

He offered a great deal of information on all subjects, eager to show what he knew. His tone was often pedantic, and he often tried to “teach” me about information he knew that was peripherally related to the test stimuli. He also asked me a large number of questions, largely but not always within the typical boundaries of social convention. However, he often did not appear to have actually listened to the answers; for example, shortly after I had mentioned that my own son was similar in age to him, he asked if I had children. He also had a great deal of difficulty in managing the interpersonal task of sharing the test materials, allowing me to control the objects and stimulus books, letting me see his responses, and waiting while I wrote things down.

He was obviously eager to cooperate during the assessment and to do his best. However, it was extremely difficult to get him to listen to and sustain his focus on the instructions of the various tasks. He often interrupted or tried to begin the tasks while instructions were being given. He often misunderstood what he was supposed to do, or began a task correctly but forgot it along the way. Instructions often had to be repeated multiple times, to the extent permitted by standardization. When he gave an incomplete answer that could be queried, even after I explained why I was doing that, he continued to simply repeat his incomplete answers and to become a bit indignant, insisting that he had already given the right answers. He often asked for clarifications of the directions, but then did not appear to actually change his actions in response to the “new” information. He did not appear to have difficulty hearing, but rather to have difficulty in maintaining awareness of what he had initially heard correctly.

His self-monitoring was also quite weak; he made a very large number of errors, self-correcting many but missing many others. Similarly, he often announced that he was finished while he was still actually working on an item, or complained that an item was “easy” when he actually was doing very poorly on it.

He showed a high level of physical movement, frequently getting up out of the chair or moving restlessly within it, tipping it up on two legs, and the like. At several points, he fell off the chair completely. When he was given time to study stimulus materials in preparation for immediate recall, he did not use the time well, asking to go on before he had to. He was easily distracted by nonsalient stimuli inside and outside the testing room; at one point, his mother sneezed in the waiting room and he yelled loudly, “Gesundheit!” At various times when upset, he put his hand down on the front of his pants.

All possible efforts were made to help Dustin stay focused and maintain his awareness of the task requirements. He responded best to a warm and encouraging tone, but it was often necessary to be firm and even a bit sharp at times to get him to realize that what he was doing was not acceptable. It is quite likely that, had he been able to maintain his attentional focus, he could have done better on many tasks that were not specifically designed to test executive functioning. However, it also appears from his reported history that this level of performance is typical of his everyday behavior in the classroom and in the home, rather than simply being an idiosyncratic response to the testing environment. Thus, these test findings appear to be a valid reflection of Dustin's current functioning.

Tests Administered

Cognitive and Neuropsychological

  1. Differential Ability Scales, Second Edition (DAS-II)
  2. Woodcock-Johnson IV: Tests of Cognitive Abilities (WJ IV COG)
  3. Rey-Osterrieth Complex Figure Test (RCFT)
  4. Wisconsin Card Sorting Test (WCST)
  5. Delis-Kaplan Executive Functioning System (D-KEFS)
  6. Beery-Buktenica Developmental Test of Visual-Motor Integration, Sixth Edition (VMI-6)
  7. Tasks of Executive Control (TEC)

Academic

  1. Woodcock-Johnson IV: Tests of Achievement (WJ IV ACH)
  2. Test of Written Language, 4th Edition (TOWL-4)
  3. Informal Handwriting Samples

Behavioral, Social-Emotional, and Psychological

  1. Sensory Profile
  2. Social Language Development Test, Elementary (SLDT-E)
  3. Rorschach Inkblot Method, Exner Comprehensive System
  4. Picture Storytelling: Children's Apperception Test and Supplement, Thematic Apperception Test, Roberts Apperception Test
  5. Diagnostic Clinical Interview

Scores Used in the Assessment

Standardized tests provide scores based on a normative comparison with other individuals of the same age. For the tests used in this report, composite standard scores (SS) of 100 and subtest scaled scores (ScS) of 10 represent average performance. Measures that provided T scores were converted to their standard-score equivalents to reduce confusion and facilitate comparisons among the scores. Approximately 50% of the population receives standard scores between 90 and 110 (scaled scores of 8–12), which is considered to be the “average” range. Percentile ranks (PR) do not represent “percent correct,” but rather the percentage of the population who performed below the level that Dustin did. Standard scores and scaled scores are linear and can thus be compared with each other more meaningfully; for this reason, these scores are discussed in this report.

Some tests also provide relative proficiency indexes (RPIs). These show Dustin's predicted level of proficiency on tasks similar to those on the test in comparison with an average child his age who would demonstrate 90% proficiency. For example, an RPI of 95/90 would indicate that Dustin would be 95% proficient on the task when the average 9-year-old would be 90% proficient. For tests in which Dustin's scores were above average, Generalized RPIs (GRPI) indicate the level of proficiency that an average child Dustin's age would achieve on skills in which Dustin could demonstrate mastery but would not find them trivial (90% proficiency).

Scores in brackets [] provide supplementary information but are not used in composites. A double dagger ‡ before a cluster score indicates that it is composed of subtests with widely varying scores and should be interpreted with caution. A pound sign # indicates that Dustin's raw score was below the lower limits of the norming sample (floor effects), and thus may be an overestimate of his ability.

Cognitive and Neuropsychological Assessment

Differential Abilities Scale, Second Edition (DAS-II)

Composite/Subtest Composite Subtest SS Composite Subtest PR Composite Subtest SS Composite Subtest PR
GENERAL CONCEPTUAL ABILITY (GCA) ‡ 110 75 DIAGNOSTIC CLUSTERS/Subtests
Verbal 123 94 Working Memory ‡ 97 42
Word Definitions 118 88 Recall of Sequential Order 106 66
Verbal Similarities 122 93 Recall of Digits Backward 89 24
Nonverbal Reasoning 115 84 Processing Speed 82 12
Matrices 116 86 Speed of Information Processing 88 21
Sequential and Quantitative Reasoning 112 79 Rapid Naming 83 14
Spatial ‡ 88 21 [Recall of Digits Forward] 101 54
Recall of Designs 82 12 [Recall of Objects—Immediate] 91 27
Pattern Construction (Standard) 98 46 [Recall of Objects—Delayed] 94 34
Mean = 100, SD = 15.

To assess his basic cognitive functioning, Dustin was administered the Differential Abilities Scale, Second Edition (DAS-II). These tests provide a broad-based view of various cognitive factors. Subtest T-scores have been converted to Standard Scores for ease of comparison.

Dustin's overall functioning was in the above-average range. His General Conceptual Ability of 110 (PR 75) is based on the consciously mediated skills tapped on the verbal, nonverbal, and spatial factors. However, these composites should be interpreted with caution, because the subtest factor scores that constitute them are substantially different from each other. Dustin's performance on the Spatial factor was statistically significantly below both his Verbal and Nonverbal factors, with differences this large found in 1 to 2% and 2 to 5% of the population, respectively. The level of scatter among the individual subtest scores is found in only 5 to 10% of the population.

The verbal tests required Dustin to demonstrate his existing knowledge (crystallized intelligence), using language. Although he showed a strong base of learned information, in the superior range for his age, his use of language was largely concrete, rigid, and pedantic.

The Nonverbal Reasoning cluster focused on the use of fluid reasoning to solve novel problems, recognizing and following implied rules. Dustin scored in the above-average range on these tasks. He was largely unwilling to try difficult items when he was not certain of getting the correct answer.

On the Spatial Reasoning tests, Dustin was asked to draw or build in response to visual prompts. As noted previously, this factor is a relative weakness for him, and his overall performance was below average. His drawings were extremely messy, with many errors and directional confusions even on simple items that he claimed were “easy.” When the graphomotor requirement was removed, he continued to struggle. He did not recognize the gestalt (“big picture”) in any items, nor did he figure out strategies for combining small parts that he could use to make the whole. He often lost track of the overall dimensions of the stimulus, and in some cases failed to recognize when he actually was close to a correct response.

The supplementary Working Memory subtests measured Dustin's ability to hold and manipulate information in his mind. His performance was in the average range, but this factor score is significantly lower than his GCA score. Again, his self-monitoring was poor; he complained that items were easy when he was, in fact, performing poorly. On the Recall of Sequential Order subtest, he benefited strongly from visual cues to remind him of real-world information.

Although Dustin's fund of general knowledge is strong, when he needed to learn new information by memorizing a list of pictures of common objects, his performance was on the low end of average and remained that way after a short delay. He was distracted by irrelevant aspects of the task, wanting to make sure, for example, that I knew that the “mouse” was really a “lemming.” While these distractions did not affect his score directly, he seemed to not be able to direct his full attention to the learning task.

On the supplementary tests of Processing Speed, Dustin was required to make rapid but simple cognitive judgments and to retrieve simple, overlearned information from memory. His performance was below average, with the difference between this factor and his GCA being found in only 5 to 10% of the population. He made numerous errors, some of which he was able to self-correct, but at the cost of slowing down his overall performance. He was also distracted by some of the test items, wanting to use advanced and precise vocabulary on a rapid picture naming test rather than the simple response (e.g., eastern cottontail for rabbit). His score reflects his performance even after the test was stopped and the instructions re-explained to him.

Overall, Dustin's automatic skills are substantially below the mean of his mediated skills. This finding suggests that part of Dustin's difficulties in complex skills result from not being able to easily carry out the underlying tasks that should be simple and routine. Instead, these tasks take up far more of his limited attentional resources than they should, leaving him with little time left to do the hard things.

Woodcock-Johnson IV Tests of Cognitive Abilities

COMPOSITE/Test SS PR RPI GRPI
COMPREHENSION-KNOWLEDGE (Gc) 140 99.6 100/90 90/16
 Oral Vocabulary 138 99 100/90 90/16
 General Information 138 99 100/90 90/16
 Concept Formation 113 81 98/90 90/63
 Analysis-Synthesis 123 94 99/90 90/45
Mean = 100, SD = 15.

To further characterize Dustin's cognitive performance, he was given several tests from the Woodcock-Johnson IV Tests of Cognitive Abilities. These tests have structures that are often less problematic for children who have difficulty in remembering and complying with the rules of the DAS-II subtests.

The WJ IV tests of Comprehension-Knowledge were given to allow Dustin to show his fund of general information in fact-based tasks that did not require explanation. His performance was well into the Very Superior range for his age, and his RPI scores indicate that the information he knows is very different from the level of a typical child his age. However, even while answering simple questions, he was distracted by irrelevant ideas; the word “we” became “We the People of the United States of America” and a picture of a toga became a frat-party chant.

Concept Formation and Analysis-Synthesis are tests of fluid reasoning, the ability to solve novel problems using logical reasoning. These tests were given to explore Dustin's ability to learn new problem-solving tasks from instruction. Although he received constant feedback, he was unable to use it effectively; rather, he complained and argued about his answers. Once the feedback stopped, he completely lost track of the tasks and started to look instead for a pattern in the answers. Overall, it appears that problem-solving is a strength for Dustin, but not as much of a strength as simply knowing the answer.

Rey-Osterreith Complex Figure Test (RCFT) and Beery-Buktenika Developmental Test of Visual-Motor Integration, 6th Edition

RCFT SS PR
Time to Copy Within normal limits >16
Copy <1
Immediate Recall 68 1
Delayed Recall 68 1
Mean = 100, SD = 15.

Dustin was administered the RCFT and the VMI to assess his abilities in visual processing, including spatial relations, perception of the integration of details within a gestalt, visual-motor coordination, visual memory, and his “plan of attack” for these types of tasks. The tasks included copying abstract forms comprising geometric shapes, drawing a form from memory (immediate and delayed), tracing between the double lines of forms, and noticing fine similarities and differences in designs so as to choose the matching pair. On both the RCFT and the VMI, Dustin was detail-oriented and did not seem to see “the big picture” in any of the designs. On the RCFT, he was supposed to copy a complex design with numerous parts; he copied a single detail from the drawing and then handed the pen back, announcing that he was finished. He had honestly forgotten that he was supposed to copy the entire drawing, but was unresponsive to redirection. His initial copy was highly disorganized, and his time was very slow. Dustin demonstrated similar difficulties on both tests, whether copying or drawing from memory. He failed to pay attention to how the various details needed to fit together to make an overall whole; parts did not match up, straight lines did not intersect accurately, and his drawings demonstrated directional confusion.

On the RCFT and the VMI, his copying scores were very far below the 1st percentile and at the 5th, respectively. His ability to match designs on the VMI was his highest score, at 90, the low end of the Average range. His score on tracing between lines indicated that this ability was higher than only three in 10,000 children of his age (PR 0.03) and reflected severe difficulties in controlling his pencil.

Age equivalents are highly problematic scores and cannot be interpreted as precise mathematical measurements of “how far behind” he is. Rather, they are best thought of as qualitative information. However, for this test, it is worth noting that his performance was similar to that of a child far younger than himself (VMI copying, age 5–11; visual matching, age 7–8, tracing, 3–9). For the Motor domain in particular, this difference appears to indicate very substantial impairment.

Delis-Kaplan Executive Functioning System (D-KEFS)

To more fully explore how Dustin processes information and directs his own actions, he was given several tests from the Delis-Kaplan Executive Function System. There are a very large number of subtask and contrast scores; those relevant to the analysis are described below.

The Trails test first required him to scan a visual field; his speed of scanning was above average (ScS 12, PR 75), but he missed a number of targets (PR 6). He then had to use overlearned sequences to draw a path. Rather than stop and look for his next target, as most children would, he moved the pen rapidly and randomly around the page. This behavior continued despite redirection. His difficulty in finding specific targets and in maintaining the correct sequences in mind greatly slowed down his effective performance (Number Sequencing ScS 3, PR 1; Letter Sequencing ScS 8, PR 25; combined ScS 5, PR 5; letter sequencing errors PR 14). When he then had to shift set between letter and number sequences, he became confused, forgetting both the sequences and the task rules, earning the lowest possible score (ScS 2, PR <1; time-discontinue errors PR 1; combined errors ScS 1, PR <1).

The Color-Word Interference (Stroop) test required Dustin to retrieve overlearned information, inhibit automatic responding, and shift set. It was extremely difficult to train him on this task, because he found a way to “cheat” on it (partially covering the words with his finger). Although he was earnest and eager to cooperate, this behavior re-emerged every time he made an error, and he was not responsive to redirection. It was not possible to administer this test validly.

Dustin was then given two tests of ideational fluency. On the Verbal Fluency test, he drew on his vast fund of knowledge, producing a very large number of correct responses (Letter Fluency and Category Fluency both ScS 16, PR 98), with a highly sophisticated vocabulary. He did not use any phonological or semantic strategies. As before, when he had to shift set between two categories, he became disorganized, giving highly unusual responses that barely fit one category and never fit the other (Switching ScS 7, PR 16; switching accuracy ScS 2, percent switching accuracy ScS 1 [floor effects], PR <1). Across all conditions, he frequently forgot what he was supposed to do (set-loss errors and percent set-loss errors both ScS 1, PR <1).

The Design Fluency test was very difficult for Dustin. Despite multiple reminders and redirections, even with the printed rules visible throughout, he repeatedly lost track of what he was supposed to be doing. Although his overall performance was in the Average range and no consistent pattern emerged, he made a great many errors (set-loss errors ScS 5, PR 5; percent design accuracy ScS 3, PR 1).

On the Sorting test, Dustin was asked to recognize and switch flexibly among implied semantic and visual rules. His scores were largely in the Average range. When he had found the answers that were easy for him, he tended to give up rather than develop new strategies. In a recognition trial, although he did recognize some of the rules he had missed in the free-sort condition, he also forgot some of the rules he had found on his own.

The Tower test presented visual sequential logic puzzles of increasing difficulty. Dustin did not figure out any strategies from easy items that he could apply to more difficult items; rather, his moves were often random and required frequent backtracking. He tended to jump in quickly (mean first move time ScS 12, PR 75) and then to slow down substantially once he realized that he had made the problem worse (time per move ScS 7, PR 16). As on other tasks, he had severe difficulty in maintaining awareness of any of the rules, often repeating an error immediately after it had been corrected (total rule violations and rule violations per item, both ScSs 1, PR <1). His overall performance was Below Average (total achievement ScS 8, PR 25, move accuracy ScS 7, PR 16).

Tasks of Executive Control (TEC)

The Tasks of Executive Control (TEC) is a computer-administered measure of a person's cognitive response to increasing the load on working memory and the demand to inhibit an automatic response. Dustin had difficulty shifting his responses to task demands as they increased, but with practice, over time, his performance improved. As the load on working memory increased, his performance degraded more than would be expected for a child his age. In particular, he had difficulty sustaining attention in the face of increased working memory demands and the need to inhibit his impulses.

The variables with the strongest clinical association with ADHD are not, as most people would assume, omission (inattentive) or commission (impulsive) errors, but rather the speed of response and the variability of response time. Inconsistency is the hallmark of ADHD. Dustin's response speed was markedly slow, and his intra-individual variability was very high.

Academic Assessment

Woodcock-Johnson IV Tests of Achievement, Form A

COMPOSITE/Test SS* PR RPI GRPI
BROAD READING 119 90 99/90 90/47
BRIEF READING (without Sentence Reading Fluency) 122 93 99/90 90/29
 Passage Comprehension 123 94 99/90 90/42
 Letter-Word Identification 118 88 100/90 90/19
 Sentence Reading Fluency 109 73 95/90 90/81
BROAD WRITTEN LANGUAGE 102 56 92/90 90/88
BRIEF WRITING (without Sentence Writing Fluency) 107 69 95/90 90/82
WRITTEN EXPRESSION 96 40 86/90 90/93
 Writing Samples 102 54 91/90 90/89
 Spelling 110 74 97/90 90/73
 Sentence Writing Fluency 90 25 79/90 90/96
BROAD MATH 100 51 90/90 90/90
BRIEF MATH (without Math Facts Fluency) 100 51 90/90 90/90
MATH CALCULATION SKILLS 96 38 87/90 90/93
 Applied Problems 105 63 95/90 90/82
 Calculation 94 34 83/90 90/95
 Math Facts Fluency 101 53 90/90 90/90
ACADEMIC SKILLS ‡ 113 80 97/90 90/68
 Letter-Word Identification 118 88 100/90 90/19
 Spelling 110 74 97/90 90/73
 Calculation 94 34 83/90 90/95
ACADEMIC FLUENCY 100 49 90/90 90/90
 Sentence Reading Fluency 109 73 /90 90/81
 Sentence Writing Fluency 90 25 79/90 90/96
 Math Facts Fluency 101 53 90/90 90/90
ACADEMIC APPLICATIONS ‡ 113 81 97/90 90/75
 Passage Comprehension 123 94 99/90 90/42
 Writing Samples 102 54 91/90 90/89
 Applied Problems 105 63 95/90 90/82
*Mean = 100, SD = 15.

To characterize the present level of Dustin's academic performance, he was administered the WJ IV Broad Achievement cluster. In each of the major skill domains (reading, writing, and mathematics), he was given a test of skill development, a test of basic skill fluency, and a test of applied contextual knowledge. These tests were analyzed in various clusters, both within and across domains, to give a broad-based sense of general academic functioning.

In reading, Dustin's performance was largely Above Average to Superior. On a test of word identification, he showed little knowledge of how to use phonics strategies to sound out unfamiliar words, merely guessing at words as they became difficult. He did, however, rely on his large vocabulary to make effective guesses.

When he was asked to read short sentences with simple vocabulary about simple topics (Sentence Reading Fluency), he was largely able to do the task, but he was constantly distracted by looking for technicalities that could change the answers. Despite explicit instructions to give the easy answer, although he did consistently give the correct answers, he spoke frequently during the training phase and during the test itself about these ideas. Because this is a timed test, his comments affected his time on task and reduced his score.

By contrast, when he was asked to read passages of one or two sentences and show his comprehension by coming up with an appropriate word to fill in a blank (Passage Comprehension), he showed strong appreciation for the general meaning of what he read. When he was not sure of an answer, he did not make effective use of context or grammatical cues; rather, he seemed to simply guess a word that related to the general topic.

Dustin's performance in Written Language was less consistent. On these tests, whenever it was unclear what he had intended to write, clarification was sought orally; handwriting is not penalized on these tests unless the product is illegible.

On the Spelling test, he made errors on a number of relatively easy items but was able to get occasional words correct very far into the test. When he did not know an answer, as with decoding, he did not appear to know any phonics-based strategies for coming up with a reasonable answer. He was also frequently distracted by discussing the meanings of the words or the example sentences. When he was asked to quickly write short sentences with simple vocabulary (Sentence Writing Fluency), his performance was in the low end of the Average range.

In response to a variety of prompts to write single sentences (Writing Samples), he was enthusiastic, but his responses were often off-target from what was required by the prompt, or he missed important details that were necessary for full credit. He used his large vocabulary effectively and appeared to be trying to demonstrate his competence, but often omitted important words necessary to convey his intended meaning. When he was asked to read his sentences aloud (to clarify illegible handwriting), he did not notice that what he had written did not match what he said. Additionally, in a beyond-limits procedure, he was encouraged to make sure that he had done specifically what he was asked to do in the prompts, and he insisted that he had.

Dustin's performance in the domain of Mathematics was more consistent, although less in line with his general cognitive ability. On the Calculation test, he was able to show mastery of the basics of addition and subtraction but did not know how to add three numbers together and did not regroup correctly for subtraction. He did not multiply correctly on a consistent basis, and he was unfamiliar with division and fractions. He refused to attempt unfamiliar tasks. When he was asked to perform single-digit addition, subtraction, and multiplication problems under time constraints (Math Facts Fluency), he was accurate, and his speed was average for his age. Problems in digit formation appeared to slow him down. Unlike most bright children who have a flair for math, his performance was average when applying his computation skill to real-world problems (Applied Problems). To do well on this test, a student must understand the real-world situation, translate it into a computation problem, and solve it. Dustin seemed haphazard in his approach, not thinking through the situations or carrying out any strategies clearly from beginning to end. He did not use scratch paper. He was frequently confused between dollars and cents on problems involving money. If he did not immediately know the answer to a problem, he announced that it was a “trick question” with no answer at all, rather than trying to figure out how he might approach it productively.

Test of Written Language, Fourth Edition (TOWL-4)

CLUSTER/Subtest SS ScS PR
SPONTANEOUS WRITING 99 47
 Contextual Conventions 9 37
 Story Construction 10 50
SS Mean = 100, SD = 15; ScS Mean = 10, SD = 3.

For the TOWL-4, Dustin was asked to write a short story in response to a complex picture. Before he was asked to write, he listened to a story based on a different picture, along with an explanation of what made that story a good one. He was reminded to adhere to these story-construction and writing mechanics criteria in his own writing and encouraged to take time to plan his own story before beginning. Instead, he started to write immediately. The following is a precise transcription of the entire story (just 73 words), including line breaks as he wrote them:

Onc upon a time a bog and His pet humen, DuDe, were driving. when they hit a hydrin. “You furr-y mani Ac” the e state of the hydrian yelled “i'mtrainig him”said one of the eds on briving.“takin' the thing for Questinin' “said th poice. Later. “how oldareyou fur!” “One” “Okay, takehim away” “Lady that's a hi-ttinghyDran for hitt ing intraining” Said DuDe. They Lived happly ever after.

He read the story aloud, quite proud of it. After correcting spelling and writing mechanics, as well as inserting missing words, his intended story appears to have been:

Once upon a time, a dog and his pet human, Dude, were driving when they hit a hydrant. “You furry maniac!” the owner of the estate of the hydrant yelled. “I'm training him!” said one of the eds on driving. “Takin' the thing in for questionin',” said the police. Later…“How old are you, fur?” “One.” “Okay, take him away.” “Lady, that's a hitting hydrant for hitting in training, said Dude.” They lived happily ever after.

He was proud of the story he wrote, talking about how he had made it “humoristic.” However, even within a fantasy context (in which a dog might own a human and teach the human to drive), his story made little sense and had a very weak narrative structure. His writing mechanics were at times sophisticated, and at times quite immature. His scores in the average range largely reflect the range of his vocabulary and the fact that he was sometimes able to show various required features in his writing.

His handwriting was extremely immature and illegible. His writing grip was highly unusual, a tight backhand fist–like grip, with very heavy pressure (See Figure 2.1). He broke the pencil often and dropped it frequently, suggesting inconsistent muscle tone. Because of this grip, he had little ability to use fingertip control over the pencil. He was asked whether anyone had ever taught him a different way to hold a pencil, and he insisted that this was his way and that he liked it. He seemed unaware of the difficulty his choices were causing for him.

Figure depicting writing grip where a human hand is holding a pencil in an unusual manner, a tight backhand fist-like grip, with very heavy pressure.

Figure 2.1 Dustin's Writing Grip

All aspects of handwriting were very poor, including letter formation (too round, open, and large), sequence and direction of strokes, orientation, and placement in relation to the line, such that it was frequently illegible. His use of capital letters was random, and punctuation was missing or incorrect. Generally, his handwriting resembled that of a young child just learning to write. Lack of syllabication or poor spatial planning was evidenced by his breaking a word wherever he ran out of space at the end of a line. He was unable to write the alphabet correctly in either upper- or lowercase. Based on these observations, as well as observations of his ability to write the alphabet and to copy a sentence, his handwriting is far below that of his age peers. He is unlikely to be able to approach tasks that even younger children would complete with ease. (See Figure 2.2.)

Figure depicting Dustin's handwriting that is extremely immature and illegible.

Figure 2.2 Dustin's Handwriting

Behavioral, Social-Emotional, and Psychological Assessment

Several instruments were administered to Dustin to better understand his overall personality style, emotional health, social cognition, and meaning-making.

Sensory Profile

Dustin's mother completed the Sensory Profile, a questionnaire measuring the frequency of behaviors associated with difficulties in sensory processing. Dustin manifests a few differences from typical children in sensory processing behavior, most of which are mild. He showed probable differences in the realms of sensory modulation related to body position and movement, emotional responses, and activity level.

Additionally, there were clear issues related to motor skills, with a probable difference in the realm of Fine Motor/Perceptual processing and definite differences relating to low muscle tone and overall muscular endurance. He is somewhat more sedentary than most children his age, particularly boys, and his general presentation is “floppy” as well.

Social Language Development Test, Elementary

Subtest/Task SS PR Age Equiv. (years-months)
Total Test 99 46 8–5
Making Inferences # 67 1 <5–9
Identifying Thoughts 76 5 5–8
Identifying Visual Cues # 62 1 <5–6
Interpersonal Negotiation 103 58 10–5
Identifying Problems 112 78 >10–0
Proposing Solutions 91 27 6–8
Rationale for Solutions 103 58 10–7
Multiple Interpretations # 72 3 <6–1
Supporting Peers 118 88 >10–6
Mean = 100, SD = 15.

The Social Language Development Test presents a variety of real-world situations related to social cognition and pragmatic language. His scores on this test were mixed.

Consistent with his strengths in factual knowledge and his strong interest in being friendly with others, Dustin scored average to above-average in subtests that allowed him to draw on his large fund of declarative knowledge. In the Supporting Peers subtest (SS 118, PR 88), he was presented with a situation in which a peer was embarrassed (e.g., had food spilled on his clothing) or where his own thoughts might not be complimentary to the peer (e.g., does not want to go to the peer's party). Dustin manifested clear knowledge of socially appropriate supportive scripts (e.g., “thank you!” for an unwelcome gift). Most of his answers were in the form of providing empathic normalization of the peer's experience (“Hey, I have had that happen to me, too; we all have that problem sometimes”), although he sometimes used it mechanically in situations in which it was unlikely to be appropriate. He was also often distracted by irrelevant minutiae (e.g., “What color was the shirt?”).

On the Interpersonal Negotiation subtest, he was presented with a variety of situations in which “you” and a friend have mutually contradictory desires (e.g., both want something that there is only one of, each wants to do a different activity). Dustin was first asked to show his understanding of the situation by restating the problem, and then to propose a possible solution and to explain why that solution would be a good idea. Full credit required that he include the perspective of the other child and that he value the friendship over the advantages to be had within it. His average overall standard score of 103 (PR 58) was pulled up by the fact that he was able to mechanically repeat, “We each want to do different things” by rote on almost every item, bringing up the Identifying Problems task score to 112 (PR 78). When he was asked to actually propose workable solutions, he tended to simply say that he would give the friend what he wanted, or to deny that the problem was really a problem, rather than discussing and reaching a mutually acceptable solution. He also had substantial difficulty in decentering—pretending to be the child in the story and to want something different than his friend wants—so he would often say, “That's okay, I like what the friend wants better anyhow.” His score on this subtest was in the low-average range (SS 91, PR 27). When asked to explain why his solutions were good ones, he did appeal at times to notions of fairness, kindness, and friendship (SS 103, PR 58).

By contrast, Dustin had far more difficulty on tasks that required him to react to picture prompts, recognizing nonverbal cues to understand what other people might be thinking or feeling and to think flexibly about alternatives, with scores at the floor of the norms tables. Additionally, he had a great deal of difficulty providing his answers in the form of language; he would typically grunt or make some other wordless noise, insisting that those noises were the answers to what someone was thinking or feeling. It required many redirections to get him to answer in understandable language.

On the Making Inferences subtest (SS #67, PR 1, with floor effects), he was shown photographs of children and adults displaying both facial and bodily cues to emotions. He was first asked to take the perspective of the person in the picture and to speak in that person's voice (SS 76, PR 5), and then to identify what visual cues were present in the picture to support his conjecture (SS #62, PR 1, with floor effects). He was able to identify appropriate thoughts for some strong emotions, but was largely unable to conjecture accurately in more subtle situations, particularly when adults were pictured making warning or directive gestures. It is likely to be extremely difficult for Dustin to recognize the nonverbal cues when peers are reacting negatively to his actions or when adults are warning him to change his behavior.

The Multiple Interpretations subtest (SS #72, PR 3, with floor effects) required Dustin to suggest two different possibilities for what might be happening in a photograph. Although Dustin was able to make a few appropriate interpretations, when he was asked for a second possibility, he gave only minor wording changes that did not change the meaning. Particularly when one realizes that Dustin's initial ideas about what might be occurring in a real-life situation are highly likely to be inaccurate, this level of cognitive rigidity is of substantial concern.

Age equivalent scores should not be considered to be precise measurements. However, the fact that the age equivalents for the Making Inferences, Multiple Interpretations, and Proposing Solutions subtests are far below his current age raises serious concern. Despite growing up in a family with supportive parents and going to school with concerned teachers, all of whom have very likely been trying to teach these basic social cognition and pragmatic language skills, Dustin has learned little of what is needed even by much younger children. As the social world becomes yet more complex in the upper elementary and middle-school years, he is at serious risk for disorders in depression and/or anxiety.

Projective Testing

  1. Rorschach Inkblot Test
  2. Thematic Apperception Test (TAT)
  3. Roberts Apperception Test (Roberts)
  4. Children's Apperception Test and Supplement (CAT, CAT-S)

Multiple projective tests were administered; the following is an integrated summary. The Rorschach presents highly ambiguous stimuli; both the content of his responses and the quality of his perceptions are scored, giving both psychological and neuropsychological data. On the projective storytelling tests, Dustin was shown cards depicting people in ambiguous but emotionally evocative social situations and was asked to tell a story based on each image. The stories often reflect a person's perceptions of himself and of his social world.

Dustin does not see the world the way other people do; he demonstrates a severe impairment of his ability to accurately perceive events, people, and what their actions signify. Furthermore, he does not really understand why he should have to understand things the way others do. He enjoys being unconventional and creative. However, his ideas are often illogical and disconnected, and he is frequently distracted by peripheral details. He is able to see some of the most obvious clues about appropriate behavior, but often does so too late, after he has already locked into his initial mistaken ideas. Thus, it is difficult for him to anticipate the consequences of his own actions and adapt to social expectations.

He is especially prone to making mistakes when he is trying to figure out what other people are thinking and feeling and why they might be acting the way they are. As on the SLDT, he had great difficulty in coming up with reasonable interpretations of facial expressions and body language. His ideas about other people are not just immature, but at times frankly bizarre, showing poor understanding of social roles, relationships, and cause and effect. The social logic of his stories was consistently very weak. He was clearly trying to be “creative,” but his stories had no internal logic; they did not make sense even when considered as cartoons. His understanding of social cause and effect was poor. He assumes that if he is upset, it is because someone has intended for him to feel that way, and that children and adults control each other's actions and feelings through aggression.

He is constantly derailed by internal emotional distractions, particularly thoughts about being helpless in the face of difficult circumstance. However, he is not consciously aware of these thoughts and thus has little opportunity to practice coping strategies.

Dustin's cognition is highly rigid. Once he has come up with an initial reaction or focused on a single aspect of a situation, it can be difficult for him to shift set and consider new possibilities, even when they are directly presented. Furthermore, situational stressors tend to increase the complexity of his thought, making it even harder for people to understand him and for him to understand himself.

Dustin experiences frequent intensely negative and painful emotions, including deep sadness, anger, and anxiety. He sees his body and mind as flawed, unable to meet the world's challenges. He may not complain of feeling depressed per se, because of his low emotional literacy.

He is much less willing than most people his age to experience his own feelings or the feelings of those around him. Because he does not feel competent to handle them, he avoids engaging in emotionally charged situations. Thus, he further cuts himself off from the social world and many of the normal developmental tasks of childhood.

Although his independent streak may eventually contribute to an adaptive capacity for self-determination, at present, it is largely getting in his way. His tendencies to resent authority and to be irritated by other people are likely to interfere with his development of frustration tolerance and social skills.

He tends to be highly confused and uncertain about his own emotions. Furthermore, when Dustin is trying to respond to emotion in himself or others, he has much more trouble than he normally would in thinking logically and clearly. He tends to become overwhelmed and melt down. Because of his problems in social cognition, he is largely unaware of the effects his meltdowns have on himself and others.

His major strategy for dealing with dysphoric emotions is to try to keep them bottled up. This emotional blockage may find its expression in somatic symptoms (e.g., headaches). Keeping such a tight rein on his emotions, however, is exhausting and ultimately ineffective; like a cork pushed underwater, feelings tend to pop up unpredictably.

Another way he deals with distress is to move very quickly to gratify his wishes. Much of his apparently impulsive action is actually somewhat intentional, a strategic effort to help himself feel better in the now.

When he does allow himself to experience emotions, he tends to do so in highly intense and dramatic ways, beyond what is normal at his age. His inappropriate behaviors and emotional meltdowns tend to alienate him from adults and peers, further convincing him that he is incapable of managing on his own.

There are two primary ways people process information and solve problems. They can “think” their way through, carefully considering each option and its potential outcomes before acting. Alternatively, they can “feel” their way through, trying things out and adapting their strategies along the way, using their own and other people's reactions as feedback. As with handedness, neither one is “best”; rather, it is good to be flexible but to have a clear preference.

However, Dustin is caught in the middle, with an inefficient and unpredictable mix of both methods. His illogical thinking makes it hard for him to make accurate predictions. His lack of access to and understanding of the emotional world makes it hard for him to recognize and use feedback effectively. He has difficulty tolerating the experience of not-knowing. Thus, instead of sustaining a concerted effort in either direction, he vacillates unpredictably among strategies and gives up far too quickly. Although he has substantial resources, he does not deploy them effectively.

Dustin wants very much to do great things. As with many bright children, he has better skills than most children his age at drawing connections between various aspects of his experience. He also shows a greater amount of psychological complexity, contributing both to his wide repertoire of ideas and to his unpredictability. However, he has a difficult time focusing on what is important. He takes in far more information than he can organize effectively; every detail is potentially relevant, and it is hard for him to know what he can safely ignore or how to fit the details into a coherent whole. Thus, he may never feel that he has enough information to make good choices. Because his ideas are so often illogical, however, he cannot actually do things well enough to meet his high personal standards, leaving him painfully aware of his failures.

Dustin sees himself as not-fully-human, damaged, unable to use his bright mind well, vulnerable to being invaded and exploited by others, falling further behind his peers, and unable to improve his situation. For this reason, he tends to avoid paying attention to himself at all; when he does, he tends to become overwhelmed by self-critical introspection. It is difficult for him to even consider the possibility of change.

Dustin has a limited capacity to identify with real people in his life and to use them as role models to help him develop his own mature identity. Instead, he tends to identify with fantasy figures and to view other people primarily in terms of what they could do to him or for him.

He shows a high level of interest in other people and can imagine having close and supportive relationships with them. However, he often feels uneasy in the social world and tends to keep himself at a safe distance. He assumes that forceful competition and aggression are normal parts of social success. Fearing that other people will hurt or control him, he is constantly on the lookout, trying to understand what might be going on. His images of others were full of often bizarre and bloody aggression; even ideas that began as positive and innocuous quickly revealed hidden dangers.

Much of his argumentative and at times overbearing nature may stem from attempts to assert or regain control when he feels threatened. His weakness in maintaining accurate empathy for others only exacerbates his difficulty in managing the give-and-take of real relationship over time. This dynamic played out during the projective testing process itself. He was intensely curious about my actions as I recorded his responses; despite a great many explanations and redirections, he continually questioned, argued, experimented, and tried to take charge of my actions.

Summary and Diagnostic Impressions

Dustin is a 9-year-old boy who is obviously highly intelligent but has shown numerous signs of potential developmental disability. He was referred for assessment by his parents because of persistent concerns about attentional regulation, emotional regulation, and social skill development, both at home and in the classroom. Additionally, although his oral language is excellent, he has had ongoing difficulties in mastering the most basic tasks of writing. This evaluation was sought for diagnostic clarification and for recommendations regarding how to provide appropriate supports for Dustin.

Dustin has a highly uneven profile, showing areas both of strength and of disability. Twice-exceptional individuals with such complex profiles are often highly frustrated and highly frustrating; neither they nor those around them can figure out why they cannot reliably do what seems like it should come easily for them and what they are often able to do on some occasions. They benefit from a compassionate and supportive approach, in which other people help them figure out what strategies are most effective for them, rather than impose solutions from the outside. However, because Dustin's primary method of defense has been avoidance and repression, he will need a certain level of firmness within the compassion.

His greatest strength is as a knower. He has strong verbal intelligence and a strong fund of knowledge. His “idea factory” works very well, pouring out a constant stream of facts and associations. When he is dealing with familiar situations where he must retrieve known information, he can do extremely well. When he is not confident that he has a strategy that will work, he has difficulty in thinking flexibly about how he could approach unfamiliar problems. He tends to get stuck on his initial idea, or initiates a flurry of random activity.

He has notable weaknesses in the areas of visual-spatial processing, working memory, and rapid automatic processing of simple information. He may seem to act very quickly, but he makes a great many errors, and the time spent in recognizing and correcting those errors is substantial. The ideas that pour into his consciousness overwhelm his ability to keep track of them all, to choose thoughtfully among them, and to act with intention. His visual-spatial weaknesses are exacerbated by serious problems in fine motor and graphomotor coordination. He has some sensory differences, primarily related to muscle tone, endurance, and emotional regulation.

It is particularly difficult for Dustin to manage complex information that does not have a clearly defined structure. He experiences every detail in his internal and external world as potentially interesting and important, but he does not see how they might fit together into any coherent whole. The more complex the information is, the heavier the load on his working memory and the more difficulty he has in keeping track of it all before he forgets something important. Thus, he is frequently overwhelmed.

Dustin has severe weakness in understanding the social world, in making sense of what is going on around him, and in regulating his emotions and behavior. Although in some ways his presentation is not typical of many children on the autism spectrum, his neuropsychological and psychological test results, along with reported history and informal observations, all strongly support the diagnosis of autism spectrum disorder (ASD) of the type that was previously called Asperger's disorder. His failure to comprehend the social world, the minds of other people, and the norms and expectations of his everyday environments is severe. He sees the world primarily as a complex mass of detail rather than as containing any coherence; it is no wonder that he is frequently overwhelmed. He manifests substantial cognitive rigidity. He shows a high level of interest in other people and a strong desire to be a good friend; when he can draw upon previously learned information about how to act, he can sometimes come up with good ideas. However, he is almost completely unable to recognize even the simplest emotional content in a facial expression or to create a narrative with even a small amount of social logic. His understanding of cause and effect in the social world is sharply limited, so it is difficult for him to predict the consequences of his actions. Thus, he tries hard in the social world but often fails in his attempts. His sensory processing problems are common findings in ASD and do not represent a separate disorder.

Dustin has great difficulty modulating his actions based on external guidance and feedback. He often does not even recognize that he is being told to do something different from what he thinks he should do. He is so used to being right that he likely experiences the correction as a criticism, an attack on his sense of competence. Thus, he does not easily learn from experience; adults are likely to find working with him to be exhausting and frustrating. It is also very hard for him to understand that an adult authority has something important to teach him and that his own ideas of how to do something might not be good ideas or might not even be acceptable to others. While it is common for intelligent children to have many bright ideas and often to resist authority, the extent to which Dustin cannot take direction from adults is highly maladaptive.

A certain level of impulsivity and distractibility is common in children on the autism spectrum; it is hard for them to make sense of the mass of sensory data and to use social referencing to figure how to direct their attention and action. However, the level to which Dustin cannot keep track of what he should be paying attention to and what he should be doing is quite severe. His hyperactivity, impulsivity, and distractibility all create enough functional impairment that they warrant a separate diagnosis of attention deficit/hyperactivity disorder, combined type (ADHD).

His academic skills in reading and mathematics are largely age-appropriate but are substantially below what is typically found in a child with his level of intelligence, curiosity, and drive. If he knows how to do something, he can do it, but his ability to approach novel academic challenges and new learning with any sense of strategy is limited.

His handwriting is extremely poor and not even remotely functional at this point in his education. He does not reliably use even the most basic aspects of writing mechanics. Like a child just learning to write, he does not recognize that what he puts on the paper may not resemble the words he says aloud. Prior efforts to teach him have not been successful, in part because of his difficulty in taking feedback, but also because he has clearly not yet developed the graphomotor skills he needs. He thus meets criteria for a specific learning disability in written expression (also known as dysgraphia). Both his graphomotor and large-motor weaknesses also meet criteria for developmental coordination disorder.

Dustin is at substantial risk for developing major depressive disorder, generalized anxiety disorder, or similar disorders. He seems at some level to recognize his own difficulties and to despair of being able to manage them effectively. Furthermore, he is emotionally fragile, easily becoming derailed with even small frustrations or losses. The problems in emotional self-regulation he experiences are common findings in ASD, but the fact that they are common does not mean that they are not important. His frequent meltdowns create a great deal of functional impairment for him and should be treated as a very serious target for clinical intervention.

Dustin is a bright boy; his parents and teachers hold great hopes for his future. These hopes are not misplaced; my experience with students similar to Dustin is that, with appropriate support, they can “grow into” their cognitive and psychological profiles, learning to manage themselves effectively and to become successful in a wide variety of endeavors. As he moves through childhood into adolescence and young adulthood, the problems he will confront will increasingly be those that play to his areas of weaknesses in problem-solving, self-regulation, and social engagement. He will need to learn how to sit with his own process and to view himself with curiosity and compassion. Additionally, it will become increasingly important for Dustin to engage in the developmental tasks that take place in the social realm. He will need to explore the world of relationship to help him discover his own identity and learn to connect with others.

Diagnostic Impressions—DSM V

  1. 299.00 Autistic Spectrum Disorder
  2. 314.01 Attention Deficit/Hyperactive Disorder, Combined Type
  3. 315.2 Disorder of Written Expression (dysgraphia)
  4. 315.4 Developmental Coordination Disorder
  5. Not diagnosed at present, but bearing watchful waiting for the future:
    1. R/O 300.02 Generalized Anxiety Disorder
    2. R/O 311 Depressive Disorder NOS

Recommendations

General Approaches

  1. This report cannot by itself confer eligibility for special education and related services under the Individuals with Disabilities Education Act, or for accommodations under Section 504 of the Rehabilitation Act. Furthermore, because Dustin attends a private school, the services available through the public school system may be limited. However, the Child Study Team is strongly encouraged to consider these findings and recommendations carefully in considering both the question of eligibility and the appropriate formal or informal accommodations and interventions.
  2. The school Dustin currently attends seems to be a good fit for him. The teachers are clearly caring, thoughtful, and attentive, and they were open to guidance on how to help him more effectively. The tiny class sizes and high teacher: student ratios are helpful in managing Dustin's behavior and providing the constant redirection he needs. It is difficult to imagine him being successful in a typical mainstream classroom, and there are no therapeutic schools anywhere near Ferris.
  3. Given the constraints, it may be more appropriate for the public school district to provide services to Dustin by sending staff to his private school. Alternatively, it may be more efficient to pay for training for the staff at Dustin's current school, or to hire qualified providers from outside agencies to provide services or training. This is not an unreasonable strategy when district staff and resources are limited.
  4. Behavioral strategies with which educators are familiar are unlikely to be effective with Dustin. In fact, like many bright children, he may experience efforts to reward or punish him into compliance as attacks on his autonomy and self-determination, and may actively resist or even undermine these systems. Rather than seeing behavioral change as something that the adults in his life do to him, we should view it as something that we do with him, and ultimately as something that we encourage him to learn to do for himself, using adults as valued resources. This collaborative approach has been found to be far more effective with bright children, because it respects their desire for autonomy and competence, as well as leveraging their cognitive strengths.
  5. In some respects, Dustin's lack of awareness of the mismatch between what he is expected to be able to do at his age and what he can actually do can be seen as a useful tool. Focus him not on the intimidating long-term goals, but on immediate goals in his zone of proximal development that he can see as potential accomplishments.
  6. Dustin needs clear and unambiguous guidance as to expected academic and social behavior. He is unlikely to pick things up “by osmosis” from the environment. He needs a structured environment and explicit directions both in and out of school.
  7. When Dustin is distressed, it will be important to balance kindness with the need to maintain expectations. Provide judicious and gentle-but-inexorable pushes toward progress, although overt force is likely to be counterproductive. Limits are therapy.
  8. Overall, it is important to recognize that Dustin may be at very different levels in various academic subjects. Help him develop a broad base of skills, but do not allow the desire for breadth or the need to remediate any particular skill to stand in the way of enabling him to learn other areas in depth.
  9. Dustin is likely to need social skills coaching regardless of the level of his academic placement. Place him appropriately for his academic level, and then coach him on the skills he needs for the specific environment that he is in. Retaining him for “maturity” reasons would be detrimental, further exacerbate any academic mismatch, and increase his emotional upset and difficulty in self-regulation.
  10. The current school has a semi-structured process for managing peer conflict. This involves looking the other person in the eye, speaking respectfully, putting your feelings into words, and the like. This strategy will need some adaptation for Dustin. Direct eye contact may be overwhelming, and should not be a requirement for him. He is likely to need a cue card, with the steps explicitly delineated, to help him reliably remember the process.
  11. Focus on teaching Dustin empathic listening skills (e.g., rephrasing, reflecting) and taking another person's point of view without judging or arguing.
  12. Gifted children in general, and children with Dustin's cognitive and psychological profile in particular, are often highly resistant to the adult use of power based on social role (“I'm the teacher so you have to comply”) or rewards and punishments. They are typically much more responsive to adults who show themselves to have expertise he wants to share in and who use charisma to draw him in.
  13. Dustin has reportedly begun to recognize that the school office can be a place to calm down and get things done, rather than as a punishment for inappropriate behavior. Continue to encourage him to use this strategy. As he becomes more able to stay quiet while he is working through his challenges, consider moving him to a quiet space within the classroom.
  14. Dustin has expressed an interest in learning formal debate skills. Teaching him a form of language that is highly structured and that does require a great deal of self-discipline and perspective-taking may, in fact, be helpful. Furthermore, his interest in creating a debate club can also be potentially shaped into a way to help him learn strategies for making decisions within a group, rather than being dictatorial.

General Academic Strategies

  1. Use rubric-based and criterion-based grading and, if possible, with a specific focus on the skills Dustin is working on so that he can see precisely why he has gotten the grades he has and can learn from the experience. Collaborate with him on the project of figuring out what the teacher was telling him and how he could implement those suggestions. Guide him through the process of examining the feedback he has received without judgment, excuses, or arguments. Simply notice the feedback in a dispassionate manner and help him learn to internalize what he is being told.
  2. Do not assume that simply teaching an academic strategy is sufficient. Dustin needs to learn how to recognize situations in which each strategy is appropriate and how to cue himself to use these strategies. This constitutes an entire parallel curriculum that deserves careful attention. Use cue cards to help Dustin remember to implement the strategies he has learned. Allow him to use those during tests as well as schoolwork.
  3. Use point-of-performance reminders to help Dustin remember when something needs to be done. Reminding him hours earlier is likely to be much less effective.
  4. The most effective types of executive functioning guidance are those in which the adult cues the child to recall the structures or skills he is learning and to reorient his behavior accordingly. Rather than telling him what to do, ask him what he is supposed to be doing, orienting his attention to the environmental cues that should trigger the behavior. If he cannot remember, then use progressively more specific questions to guide his attention or to reteach what has been forgotten. When he knows what he should do, give him room to redirect his own behavior without shame. Over time, the goal is for Dustin to need less and less adult guidance.
  5. Particularly as homework becomes an increasingly important part of the curriculum, close collaboration between Dustin's parents and school personnel will be important. Dustin is likely to misunderstand what has been assigned and the due date, or how well he has done, making it hard for his parents to help him. It would be advisable for the adults to set up a regular method of communication, and to have it written into an IEP or 504 plan if developed. Typically, a weekly email from teacher to parents regarding missing assignments, recent grades, and upcoming projects is helpful.
  6. The books Smart but Scattered, by Peg Dawson and Richard Guare, and Late, Lost, and Unprepared, by Joyce Cooper-Kahn and Laurie Dietzel, provide explanations of the various aspects of executive functioning and concrete guidance on developing interventions to help children learn to self-regulate. I also maintain an occasional blog with advice for twice-exceptional learners at http://davincilearning.wordpress.com.
  7. To develop a mature self-concept, Dustin needs to develop a strong base of real experiences of self-efficacy, actual times when he was able to handle difficult tasks successfully. Involve Dustin in extracurricular activities he enjoys and that involve real challenge as well as constant improvement of his own skills without comparison with others. Activities such as Boy Scouts, 4-H, individual athletics, music, dance, and crafts can all be used in this way; the key is to find something he enjoys and then help him overcome the desire to give up when things become difficult.
  8. Help Dustin explicitly recognize the various facets of a task and establish priorities, so that he is less likely to be distracted by minor details.
  9. Similarly, he is likely to need considerable guided practice in the process of setting appropriate goals for himself. Teach him how to set goals that are “SMART”—specific, measurable, achievable given his current skill level, relevant to what is important in the overall situation, and time-bound.
  10. Vague guidance such as “do your best” is not likely to be helpful. Additionally, guidance framed in terms of what not to do is much less helpful than guidance about what to do. Focus on a small number of specific observable behaviors so that he does not become overwhelmed.
  11. Positive feedback and honest praise for whatever aspects of a task are done well will be most helpful for keeping Dustin “in the game” instead of looking for a way to flee it.

Academic Interventions

  1. Dustin is in serious need of direct interventions to help him with handwriting. Not all assignments can be word processed, particularly in mathematics, and his difficulties with handwriting are likely interfering not just with his ability to learn to compose extended writing, but also with his ability to learn a broad range of academic skills.
  2. Dustin struggles to reorient the pencil after each letter, never knowing quite where to put it down to start the next letter or the next word. He also does not know the proper sequence of strokes to use to form letters properly and to move fluently across the page. Focus on printing rather than cursive, and use a handwriting method with simplified letters, such as D'Nealian or Barchowsky (www.bfhhandwriting.com).
  3. Consider the use of paper with raised lines (www.therapro.com) or made at home by using a thin bead of white glue on lined paper) to provide proprioceptive feedback on baselines and midlines. Teaching him to “bump” the baseline and the midline with his lowercase letters will be helpful.
  4. Try Dustin with a variety of pencil grips (also available at Therapro) and the D'Nealian grip to see which he is willing to use. Pencil grips help students become more comfortable with correct placement and a relaxed grip. The D'Nealian grip is an alternative way to hold the pencil, with a normal tripod grip, using the pads of the first three fingers, but with the shaft of the pencil falling along the web space between the index and middle finger.
  5. Dustin will require a great deal of supervised handwriting practice. Without supervision, he is likely to slip into his current grip and his current letter formation. Handwriting Worksheet Maker (www.handwritingworksheets.com), an online program, allows you to make different types of worksheets for tracing and copying practice, in ball and stick printing or in D'Nealian manuscript and cursive, and in various sizes and colors.
  6. Manuscript handwriting is necessary for forms and applications in adult life, so Dustin should learn it. Later, because handwriting may always be difficult for him, teach him to become a proficient touch-typist. There are many typing programs available. One highly rated program that simulates travel around the world or through time as the child makes progress is Typing Instructor for Kids, Platinum by Herman Street. As a reward for progress, the child visits various lands and castles. A fun and free program is BBC Dance Mat Typing, http://www.bbc.co.uk/guides/z3c6tfr.
  7. Dustin will benefit from learning to think of writing as a multi-step process. Most children, particularly those with rigid ideation, think that they should be able to come up with finished writing on a first draft. However, Dustin will have much more success in generating prose that reflects the depth of his thought if he is encouraged to work through each step independently: ideation, organization of ideas, initial draft, improving sentence structure and flow, improving word choice, checking writing mechanics, and final presentation. A good overall model for teaching students to analyze and improve their own writing is the Six+1 Traits model; Vicki Spandel's Creating Writers Through 6-Trait Instruction and Assessment is a good text to learn to teach with this model.
  8. Consider having Dustin do fewer total assignments, with the focus on doing each one well and going through the revision process needed to do so.
  9. As Dustin begins to compose extended writing pieces, they are likely to have very little inherent structure. His efforts to be “creative” are more likely to read as “random.” However, both fiction and nonfiction writing actually have a great deal of structure. Teach Dustin about basic story structures (see Store the STORY), and the basic organizational structures for nonfiction (e.g., comparison/contrast, thesis/support, five-sentence paragraph).
  10. In the planning stage of writing, consider using graphic organizer software to enable Dustin to look at his ideas and move them around as needed. Software programs such as Scapple (www.literatureandlatte.com/scapple.php) or the open-source Freemind (freemind.sourceforge.net) enable users to experiment and organize in a visual environment. Once he has gotten his ideas into written form, he can organize them into an outline, filling in gaps and adding supporting details. The visual organizers can then be exported as outlines. Although Inspiration is frequently recommended for this type of work, the icons it uses are likely to distract Dustin from the task.
  11. Consider learning and using Self-Regulated Strategy Development (SRSD), which currently is the most extensively researched method of writing instruction, is extremely effective, and can be used with the whole class. (See Self-Regulated Strategy Development in Section IV.)
  12. Encourage Dustin to develop a portfolio of his best writing, to set personal goals, and to monitor his improvement over time. In particular, he will benefit from learning to recognize what aspects of his writing have been improving. Children often fail to perceive that improvement in writing is a long-term process, and thus they lose motivation for the effort.

Building Self-Regulatory Skills

  1. One of the key skills for Dustin to learn is going to be how to contain his thoughts in his mind and how to be judicious about which thoughts to allow out of his mouth. Teach him explicitly to say the words in his mind and then to ask himself whether they should be said aloud. As he becomes more proficient in handwriting, he will be better able to write or type his ideas as a way of containing them and making decisions about what to share.
  2. Dustin has a significant need to develop a broader, more socially acceptable, and more effective repertoire of self-soothing skills. Help him explore options for self-soothing strategies that are appropriate within the environment and unlikely to be off-putting, or even noticeable, to peers. Some possibilities to explore are mindfulness techniques, breathing techniques, guided visualizations, listening to music or nature sounds (e.g., on a simple MP3 player that cannot be used for games, such as an iPod Shuffle), and looking at pleasant pictures. Charlotte Reznick's recordings of guided visualizations at www.imageryforkids.com would be appropriate to try. Experiment to find what is helpful for him.
  3. Initially, Dustin needs to practice using his self-soothing strategies when he is not upset, and then when he is only slightly upset. The strategies are intended to prevent meltdown; once a meltdown is in progress, he will not be able to back off to attempt the strategy. He needs to become skilled in its effective use and thus to develop confidence in his ability to calm himself before he tries to use a tool when overwhelmed.
  4. Similarly, teach Dustin to recognize when he is starting to feel upset. He needs to develop increased awareness of how his emotions manifest in his mind and body, along with appropriate ways to clearly express his feelings and their intensity, rather than waiting until they become overwhelming.
  5. When Dustin is in a situation he finds challenging (e.g., he realizes that he has made a mistake) and begins to become upset, coach him to use the strategies he is working on; the new task becomes, “Stay calm when you are in X situation.” Once Dustin has calmed down, he should again be presented with the task that evoked the meltdown behavior. He should not learn that melting down is an effective way to get out of things. Over time, as Dustin demonstrates consistent success in small things, build up the task demands gradually.
  6. If a task is legitimately too difficult for him, or if he cannot self-regulate well enough to approach it, then it should not be presented at all. Break it down into smaller chunks so that Dustin can establish actual successes, even in small things.
  7. The Collaborative Proactive Solutions model, explained in the book The Explosive Child, by Ross Greene, and on the websites www.livesinthebalance.org and www.thinkkids.org, is an excellent and highly effective set of methods for helping children learn to self-regulate. It is particularly well-suited for gifted children.

Therapeutic Approaches

  1. It will be helpful for the adults in Dustin's life to educate themselves about autism spectrum disorder, especially the type of ASD that was previously called Asperger's syndrome. There are many books available; Tony Attwood's The Complete Guide to Asperger's Syndrome is an excellent starting place. His website, www.tonyattwood.com.au, also has very good information.
  2. Teach Dustin about ASD and how it manifests for him. Emphasize not that he is “defective,” but that certain aspects of ASD make it harder for him to learn to do certain things, and that he is a good learner and will be able to learn those things pretty well with guidance and practice.
  3. Dustin will strongly benefit from direct coaching in social pragmatics and language skills in focused, individual intervention. In addition, individual supportive and play-oriented psychotherapy may be a useful adjunct to skills-based speech and language therapy. The purpose of this relationship would be to provide him with a safe space to learn to explore, experience, express, and modulate his emotions, to develop “feelings literacy” and a greater sense of control. The long-term relationship with the therapist becomes a sandbox in which Dustin can learn to manage the give-and-take of social interaction without the risk of permanently alienating other people.
  4. Within the school milieu and in outside, private practices, social skills are often taught in a group, supplementing individual coaching with in vivo practice. Ideally, school-based specialists and private-practice therapists will collaborate so as to teach him a consistent set of skills. School-based specialists can provide in-the-moment interventions and can help Dustin understand the complexities of the system he is in, and private practitioners can take a broader view and increase his repertoire.
  5. Although Dustin is highly verbal, he is not a small adult, and talk therapy may work primarily to allow him to inappropriately intellectualize and repress his emotions. Rather, help him use art, play, and other expressive techniques to increase his tolerance for experiencing his own emotions.
  6. Prompt Dustin to use language to explore and express the world of thought and feeling. For example, when he is asked about his feelings and responds with a grunt, cue him to use language to explain what that grunt means.
  7. Educators at Dustin's school should maintain open lines of communication with his parents and the professionals who provide services for him outside of school. Children on the autism spectrum frequently struggle to recognize the connections between events that have occurred in school or at home and the social skills they are learning in therapy. Providing a brief “heads-up” to the outside professionals when something of concern happens in school can help them process the events with Dustin and help him learn to apply the skills he is learning to his daily life.
  8. All of the adults who work with Dustin should, as much as possible, use consistent language to describe and prompt him to use his skills.
  9. Many resources are available to help children with ASD learn about social interaction. In some areas of social pragmatics, the “social stories” approach developed by Carol Gray would be appropriate to provide him with instruction in the basics of “how people do things.” To foster social contact, it is particularly important for Dustin to learn how to behave in ways that are more typical of a child his age.
  10. A useful book for helping Dustin make sense of social expectations is Jennifer Cook O'Toole's The Asperkid's (Secret) Book of Social Rules. This book is aimed at tween-age and adolescent children, taking a compassionate but clear approach, similar to an anthropologist, to explaining what appropriate behavior is and why.
  11. Coaching: Recognize that merely being able to say what the correct behavior is does not mean that Dustin will reliably be able to recognize when the situation calls for it and implement the correct behavior accurately. Provide coaching specifically around these skills as well, and extensive opportunities for real-life practice.
  12. When Dustin is not sure what to make of a situation, he tends to guess rapidly. His ideas are not necessarily logical but, having come up with one, he tends to “lock in,” without thinking through alternatives or consequences. Coach Dustin, using a traffic-light metaphor (red light: recognize a problem and stop; yellow light: consider various options and predict the outcome of each; green light: choose one option and carry it out). Teach him to take the time he needs to regroup, explore, and work through a situation in all of its complexity, and teach him how to appropriately ask for that time when he needs it.
  13. Realize that Dustin's limited working memory and attention will make it very difficult for him to use this type of strategy on his own without substantial teaching and coaching. In particular, it will be helpful for him to have visual reminders—cue cards, checklists, and the like—that can be placed in the relevant work spaces so that he can refer to them right when he needs to. It would be best to develop those cards with him, so that he can experience them as reminders of things he wanted to do rather than as the imposition of unwelcome adult will.
  14. If he has forgotten to do something or has broken a rule, instead of telling him what to do, ask him to look at the cue card to find out what he should be doing. Keep your tone calm and bland; questions should not be framed as “gotcha!” If he begins to debate again, ask him to use the cue cards to find out what he should be doing.
  15. Teach Dustin to use imaginary stories for social exploration. Encourage him to create stories in which his characters recognize problems, stop, consider alternative approaches, predict the outcomes, and then choose and successfully implement adaptive solutions. Stories can be replayed with various choices and responses.
  16. Consider using board games and other activities that create short-term and low-stakes play situations. Board games developed in Europe, such as Settlers of Catan, Carcassonne, and Ticket to Ride (www.funagaingames.com is a good source of information) provide a good balance between cooperation and competition, and require players to flexibly balance various goals against each other.

As Dustin grows, help him to learn about himself as a whole person. Help him learn how each aspect of his cognitive and psychological profile plays an important role in his life and how it affects his social and emotional experiences. Focus on self-knowledge and self-acceptance. Teach him to recognize situations that play to his strengths and weaknesses, to work strategically, and to self-advocate appropriately. As he develops more self-confidence, help him start thinking about his long-term goals and help him see the big picture of how what he is learning now will help him toward those goals.

It has been a pleasure working with Dustin, and I wish him all the best. Please be in touch with any further questions.

Reason for Referral

Simon was referred for a 3-year review of his special education services. In addition, his parents have expressed continued concerns about his overall motor development and poor handwriting.

Background Information

Simon is the adopted son of Roger and Jeanette Schneider. He lives with his parents and an older brother, Jeremy, who is also adopted. The Schneiders adopted Simon at birth. His biological mother was 14 years old and smoked during the pregnancy. Although little is known about prenatal care, concerns have also been raised about possible drug and/or alcohol use during pregnancy.

Simon attended preschool for 2 years and then entered Walnut Cove Elementary for kindergarten. Based on parental and teacher concerns regarding motor development, Simon was referred for an occupational therapy evaluation in first grade. Results from the Peabody Developmental Motor Scale—2 indicated delays of up to 17 months in fine-motor development and up to 26 months in gross motor development. He demonstrated weakness in his flexor muscles, particularly abdominals and hip flexors, toe walking, and weaknesses in visual-motor planning. Simon was unable to hop on one foot without losing his balance. On the Test of Visual-Perceptual Skills-3 (non-motor), Simon obtained average scores on measures of visual discrimination and visual-spatial relations, and a below average score in visual memory. Recommendations were made for occupational therapy, with the goals of improving fine- and gross-motor skills, visual-perceptual motor planning, and muscle weaknesses. In addition, Simon began to receive resource support for writing and math.

The Schneiders report that although Simon has tried to participate in team sports (both soccer and baseball), the experiences were not positive. During baseball games, Simon would often sit down in the outfield. On the soccer field, Simon tried to stay away from the ball. His mother reported that on several occasions when she picked Simon up from practice, he was crying. Presently, Simon is enrolled in a karate program that he attends once per week.

Delays are still noted in gross motor development. At the age of 9, Simon is unable to ride a bike or tie his shoes with ease. He walks with an awkward gait and often trips. Because of continued toe walking, Simon is currently wearing casts on both legs to stretch his heel cords and position his feet flat on the ground. When the casts are removed, Simon is scheduled for physical therapy to help strengthen his legs. Recent results from a brain magnetic resonance imaging (MRI) indicated subtle cortical dysplasia involving the cerebellar hemispheres (the area of the brain involving motor development and balance). These findings are supported by clinical observations.

Simon is currently finishing the third grade. For the past 3 years, he has received special education services in a resource setting under the category of specific learning disability. He has also received a half-hour weekly of occupational therapy services. He currently uses a laptop in the classroom to assist with lengthy writing assignments. Simon's parents noted that he sometimes is a bit “uncooperative” but that he is creative and has a good vocabulary and sense of humor. When asked to write what he likes to do on the weekends, Simon wrote: “Watch TV, play video games, and fall down the stairs.”

Assessments

Simon was administered the Woodcock-Johnson IV Tests of Achievement (WJ IV ACH), Standard Battery. In addition, he was observed in the classroom. The teacher provided a writing sample. Testing was conducted over four sessions. The scores are appended to this report. In describing performance, the verbal labels for the relative proficiency index (RPI) are used to describe Simon's level of proficiency on tasks similar to those performed. The RPI predicts his success on similar tasks. The denominator of 90 indicates that an average age-peer would be successful on the task 90% of the time; the numerator represents Simon's expected success. For example, Simon's RPI of 24/90 on the Sentence Reading Fluency test indicates that when the average student of age 9–4 would be 90% successful in reading sentences quickly, Simon's expected success would be 24% which is Limited proficiency. Similarly, his RPI of 35/90 on the Spelling test, also indicates Limited proficiency. The RPI ranges are as follows:

RPI Descriptor Implications: Simon will find similar tasks
100/90 Very Advanced Extremely easy
98/90–100/90 Advanced Very easy
95/90–98/90 Average to Advanced Easy
82/90–95/90 Average Manageable
67/90–82/90 Limited to Average Difficult
24/90–67/90 Limited Very Difficult
3/90–24/90 Very Limited Impossible

Classroom Observations

Simon was observed in his classroom on April 23 at 1:00. The class was preparing to move to a room next door where they were about to have a fourth-grade “United Classrooms” meeting. Four classes, including Simon's, were together in one room to discuss the updated behavior plan. Simon was sitting in the back of the room. He looked at the floor and his shoes, glancing up only occasionally. He appeared to be listening. Fifteen minutes into the discussion, Simon began digging into the tread of his shoe with a pencil. He then removed his shoe and continued to run the tip of his pencil through the shoe treads, until a nearby teacher aide prompted him to put his shoe back on and listen.

Behavioral Observations

For the WJ IV ACH administration, Simon was tested in two sessions. During the first session, he complained of being tired, yawned, and kept his head down on the desk. He said that he did not get much sleep, then became teary-eyed and asked if he could please finish tomorrow. He completed two tests before he was dismissed to go back to class. Simon returned for testing the next morning and was once again mildly to moderately resistant. He complained that the examiner, while trying to get him engaged in the testing, was “annoying” him. When reminded that he agreed to come back to finish the test after being allowed to stop the day before, he became cooperative. He responded promptly, but carefully, to test questions and generally persisted with difficult tasks.

Academic Testing

Simon's level of proficiency on the Broad Reading cluster was Average to Advanced, whereas his proficiency in Broad Mathematics and Written Language was Limited. His math problem solving was Average to Advanced, whereas his math calculation skills were Limited. When viewed across the academic subjects of reading, writing, and math, Simon's performance was lowest on measures of Academic Fluency, involving timed tests of rate and automaticity. He had Very Limited proficiency on all timed tests. When his Academic Fluency (timed tests) was compared with his performance on Academic Skills and Applications, only three out of 100 students would have a score as low as his. His performances in written language and math calculation skills are described briefly.

Written Language

Simon's proficiency was Limited on most aspects of writing. Several responses on both the Writing Samples and Sentence Writing Fluency tests were scored as “0” because they were illegible. His writing style is characterized by a mix of upper- and lowercase letters that appear to be based on ease of formation. Analyses of handwriting indicated poor spacing between letters and between words, letter formation errors, oversized letters, and numerous spelling errors. Many of his letters were produced incorrectly. For example, he formed the letter “y” by writing the letter “v” and then adding a line at the bottom. He formed the letter “a” by making a loop. The line that should go down the side extends out from the top, so that his “a”s appear more like “o”s. The letter “l” is written as a capital “L” and is often oriented sideways so that it appears to be the letter “v.” He reversed many letters, particularly the letter “c.” The frequent reversals of this letter are most likely the result of the way Simon forms circles (from the bottom up to the right) rather than misperception of letter orientation.

The same types of difficulties were apparent in his classroom writing. Figure 2.3 presents a sample of Simon's writing.

Figure depicting Simon's writing sample that is highly illegible.

Figure 2.3 Simon's Writing Sample

Translation: Once I was lost on a dock and I fell in the water. I was scared and under a boat. The boat steered away and I was up. The end.

Despite his severe difficulties with handwriting, Simon enjoys drawing and is able to produce detailed designs and pictures, such as the one shown in Figure 2.4.

Figure illustrating a drawing of a boy in a traditional dress, drawn by Simon.

Figure 2.4 Simon's Drawing

Math Calculation Skills

Simon's proficiency on Math Calculation Skills was Limited. His lowest score was on Math Facts Fluency, a test requiring the rapid computation or retrieval of simple addition, subtraction, and multiplication facts. His RPI of 8/90 indicates that when average age-mates have 90% success on timed math facts, he will have only 8% success, Very Limited proficiency. This low performance is at least partially attributed to his difficulty writing numbers. On both the Calculation and Math Facts Fluency tests, he inconsistently reversed the number “7,” and consistently reversed the numbers “6” and “9.” The way Simon forms a “6,” it appears to be the number “2.” Although Simon was able to add and subtract simple problems, he missed several problems on the Calculation test because he did not pay attention to the signs. He did not regroup on a two-digit subtraction problem.

Summary

Simon is a third-grade student with a history of delays in fine- and gross-motor skills development. Presently, his greatest area of difficulty is written language, primarily in the production of written symbols. His difficulty in transcription results in slow, labored performance on all tasks involving writing (e.g., spelling, math facts). Simon has a relative strength in his ability to read and solve math problems. It is likely that apparent relative weaknesses in his abilities related to rapid symbol production are mainly attributable to the requirement of a motor response. Thus, his problems with written language appear to be related primarily to difficulties in motor production, rather than to problems in ideation or the expression of his ideas.

Based on severe difficulties in fine motor skills, Simon will continue to benefit from occupational therapy, as well as instruction in handwriting and basic math skills. Care should be taken to ensure that Simon's teachers understand that his difficulties with writing and math fact retrieval are not based on weaknesses in language or conceptual abilities, but rather on his poor motor skills. These assessment results will be reviewed with the multidisciplinary team to consider Simon's continued eligibility for special education services, as well as his present needs in instructional programming.

Recommendations and Accommodations

  1. Continue use of the laptop in the classroom.
  2. Continue systematic instruction in keyboarding so that Simon will become proficient in keyboarding and can complete written assignments in a timely manner.
  3. Provide Simon with extra time for all writing assignments. In many instances it will be necessary to adjust the required length of written assignments.
  4. When providing feedback on written language, place the emphasis on the ideas presented, rather than on the appearance of the writing. Praise Simon's attempts to write, regardless of the appearance.
  5. Minimize copying activities by providing the information on worksheets or handouts. Provide short answer activity sheets that will review skills and knowledge without requiring lengthy written answers.
  6. Permit Simon to take content area exams orally.
  7. Consult with the occupational therapist for recommendations regarding further activities that his classroom teacher may use to help him improve gross- and fine-motor skills.
  8. Use a structured and systematic program to teach handwriting, such as D'Nealian or Handwriting without Tears. Both incorporate multisensory procedures (tracing and saying) to help teach proper letter and number formation and reduce frequent letter and number reversals. Follow the sequence of the letter groupings.
  9. Provide instruction in basic math skills, including regrouping, multiplication, and division.
  10. Provide Simon with practice noting the mathematical signs in a problem before he attempts to solve them. As a first step, he could circle the sign and then perform the operation.
  11. Encourage Simon to put forth his best effort. When needed, set up a behavior management plan in which Simon can earn rewards for on-task behavior and completion of work .

Score Report

Name: Schneider, Simon
Date of Birth: 01/07/2006
Age: 9–4
Sex: Male
Date of Testing: 04/26/2015
School: Walnut Cove
Grade: 3.9
Teacher:

Tests Administered

Woodcock-Johnson IV Tests of Achievement Form A and Extended (Norms based on age 9–4)

Table of Scores

Woodcock-Johnson IV Tests of Achievement Form A and Extended (Norms based on age 9–4)

CLUSTER/Tests GE RPI SS (68% Band) PR (68% Band) Proficiency
READING 4.9 95/90 107 (104–111) 69 (60–77) Average to Advanced
 Letter-Word Identification 4.8 96/90 107 (103–110) 67 (58–75) Average to Advanced
 Passage Comprehension 4.9 94/90 106 (100–111) 65 (51–77) Average
BROAD READING 3.5 83/90 97 (95–100) 43 (37–49) Average
 Letter-Word Identification 4.8 96/90 107 (103–110) 67 (58–75) Average to Advanced
 Passage Comprehension 4.9 94/90 106 (100–111) 65 (51–77) Average
 Sentence Reading Fluency 2.6 24/90 89 (84–93) 22 (14–32) Very Limited
BASIC READING SKILLS 5.0 95/90 107 (104–111) 69 (60–76) Average to Advanced
 Letter-Word Identification 4.8 96/90 107 (103–110) 67 (58–75) Average to Advanced
 Word Attack 5.5 95/90 107 (101–113) 68 (53–80) Average
READING FLUENCY 3.1 70/90 93 (90–97) 32 (25–41) Limited to Average
 Oral Reading 4.8 94/90 106 (102–110) 65 (54–74) Average
 Sentence Reading Fluency 2.6 24/90 89 (84–93) 22 (14–32) Very Limited
MATHEMATICS 3.7 88/90 98 (95–101) 45 (38–53) Average
 Applied Problems 5.1 96/90 109 (104–114) 73 (61–83) Average to Advanced
 Calculation 2.9 64/90 89 (85–93) 24 (16–33) Limited
BROAD MATHEMATICS 2.8 62/90 88 (85–92) 22 (16–29) Limited
 Applied Problems 5.1 96/90 109 (104–114) 73 (61–83) Average to Advanced
 Calculation 2.9 64/90 89 (85–93) 24 (16–33) Limited
 Math Facts Fluency 1.6 8/90 76 (69–82) 5 (2–11) Very Limited
MATH CALCULATION SKILLS 2.2 28/90 80 (76–85) 10 (6–15) Limited
 Calculation 2.9 64/90 89 (85–93) 24 (16–33) Limited
 Math Facts Fluency 1.6 8/90 76 (69–82) 5 (2–11) Very Limited
WRITTEN LANGUAGE 2.3 57/90 87 (84–90) 20 (15–25) Limited
 Spelling 2.1 35/90 82 (79–86) 12 (8–18) Limited
 Writing Samples 2.7 76/90 94 (91–97) 34 (27–43) Limited to Average
BROAD WRITTEN LANGUAGE 2.2 55/90 85 (82–87) 15 (11–20) Limited
 Spelling 2.1 35/90 82 (79–86) 12 (8–18) Limited
 Writing Samples 2.7 76/90 94 (91–97) 34 (27–43) Limited to Average
 Sentence Writing Fluency 1.9 52/90 80 (73–86) 9 (4–18) Limited
WRITTEN EXPRESSION 2.3 65/90 87 (83–91) 19 (13–27) Limited
 Writing Samples 2.7 76/90 94 (91–97) 34 (27–43) Limited to Average
 Sentence Writing Fluency 1.9 52/90 80 (73–86) 9 (4–18) Limited
ACADEMIC SKILLS 3.1 74/90 92 (90–95) 31 (26–36) Limited to Average
 Letter-Word Identification 4.8 96/90 107 (103–110) 67 (58–75) Average to Advanced
 Spelling 2.1 35/90 82 (79–86) 12 (8–18) Limited
 Calculation 2.9 64/90 89 (85–93) 24 (16–33) Limited
ACADEMIC FLUENCY 2.1 24/90 82 (78–85) 11 (7–16) Very Limited
 Sentence Reading Fluency 2.6 24/90 89 (84–93) 22 (14–32) Very Limited
 Math Facts Fluency 1.6 8/90 76 (69–82) 5 (2–11) Very Limited
 Sentence Writing Fluency 1.9 52/90 80 (73–86) 9 (4–18) Limited
ACADEMIC APPLICATIONS 4.2 92/90 103 (99–106) 57 (47–67) Average
 Applied Problems 5.1 96/90 109 (104–114) 73 (61–83) Average to Advanced
 Passage Comprehension 4.9 94/90 106 (100–111) 65 (51–77) Average
 Writing Samples 2.7 76/90 94 (91–97) 34 (27–43) Limited to Average
BRIEF ACHIEVEMENT 3.7 88/90 99 (96–101) 46 (40–52) Average
 Letter-Word Identification 4.8 96/90 107 (103–110) 67 (58–75) Average to Advanced
 Applied Problems 5.1 96/90 109 (104–114) 73 (61–83) Average to Advanced
 Spelling 2.1 35/90 82 (79–86) 12 (8–18) Limited
BROAD ACHIEVEMENT 2.9 68/90 90 (88–92) 25 (22–29) Limited to Average
 Letter-Word Identification 4.8 96/90 107 (103–110) 67 (58–75) Average to Advanced
 Applied Problems 5.1 96/90 109 (104–114) 73 (61–83) Average to Advanced
 Spelling 2.1 35/90 82 (79–86) 12 (8–18) Limited
 Passage Comprehension 4.9 94/90 106 (100–111) 65 (51–77) Average
 Calculation 2.9 64/90 89 (85–93) 24 (16–33) Limited
 Writing Samples 2.7 76/90 94 (91–97) 34 (27–43) Limited to Average
 Sentence Reading Fluency 2.6 24/90 89 (84–93) 22 (14–32) Very Limited
 Math Facts Fluency 1.6 8/90 76 (69–82) 5 (2–11) Very Limited
 Sentence Writing Fluency 1.9 52/90 80 (73–86) 9 (4–18) Limited
STANDARD SCORES DISCREPANCY
VARIATIONS Actual Predicted Difference PR SD Interpretation at
±1.50 SD (SEE)
Intra-Achievement [Extended] Variations
BASIC READING SKILLS 107 96 11 91 +1.35
READING FLUENCY 93 97 −4 34 −0.40
MATH CALCULATION SKILLS 80 100 −20 3 −1.92 Weakness
WRITTEN EXPRESSION 87 99 −12 12 −1.16
Letter-Word Identification 107 96 11 90 +1.28
Applied Problems 109 96 13 89 +1.25
Spelling 82 101 −19 2 −1.99 Weakness
Passage Comprehension 106 96 10 84 +0.99
Calculation 89 100 −11 15 −1.03
Writing Samples 94 99 −5 33 −0.44
Word Attack 107 97 10 82 +0.92
Oral Reading 106 97 9 78 +0.78
Sentence Reading Fluency 89 97 −8 21 −0.81
STANDARD SCORES DISCREPANCY
VARIATIONS Actual Predicted Difference PR SD Interpretation at
±1.50 SD (SEE)
Intra-Achievement [Extended] Variations
Math Facts Fluency 76 100 −24 2 −2.10 Weakness
Sentence Writing Fluency 80 99 −19 4 −1.79 Weakness
STANDARD SCORES DISCREPANCY
VARIATIONS Actual Predicted Difference PR SD Interpretation at
±1.50 SD (SEE)
Academic Skills/Academic Fluency/Academic Applications [Extended] Variations
ACADEMIC SKILLS 92 93 −1 50 −0.01
ACADEMIC FLUENCY 82 98 −16 3 −1.87 Weakness
ACADEMIC APPLICATIONS 103 88 15 94 +1.58 Strength

Cognitive and Educational Evaluation Report

Name: Anthony Hall
Parents: Edgar and Anna Hall
Date of Birth: September 08, 2005
Age: 9–8
Dates of Testing: May 6, 10, and 12, 2015
School: Hanover Primary School
Grade: 3.8

Reason for Referral

Mrs. Anna Hall referred her son Anthony for an evaluation of his cognitive, linguistic, and academic abilities. The Halls have been concerned about Anthony's limited progress in reading since the age of 5. The purposes of this evaluation were to determine what factors are affecting Anthony's reading progress, as well as what instructional methodologies would be most effective for addressing his educational needs.

Background Information

Current Situation

Anthony currently resides with his mother, father, and four sisters on a farm in Eloy, Arizona. He has two younger sisters, ages 3 and 6, and two older sisters, ages 10 and 13. Mrs. Hall's brother and several of his children have been diagnosed with dyslexia and attention deficit hyperactivity disorder (ADHD). Mrs. Hall reports that now as adults, “they have horrible spelling and spell words just the way they sound.”

Anthony's parents reported that he loves information, enjoys figuring things out, and loves telling and hearing stories. In contrast, he is struggling with reading words, but not so much with comprehension. Once he figures out what something says, he understands the meaning. He often misreads simple words, such as the, a, if, and of and almost always asks for help when he has to pronounce longer words. If his parents sound out the word, he quickly gets it. If they try to point to syllables so that he has to figure it out, he gets frustrated and stops reading.

Anthony's grades are poor even though he studies before school, attends a math and reading reteach program after school until 5:30 p.m., and is pulled out of class to work with a reading teacher during school hours. His mother stated that his reading went from a 1.7 grade level in October to a 2.4 level in December to a 2.1 level in April. The school team advised that because he was working so hard and making no progress, they should pursue testing to find out what is hindering him. His third-grade teacher reported that just when she thinks that he has it down, he will forget what he has learned.

Mrs. Hall reported that they spend their nights trying to get his homework done amidst the tears. He is frustrated that he has to study harder than others and is not improving. The Halls are frustrated because Anthony is not making much improvement despite all of their efforts. When he is left to read on his own, he gets no more than 2 right out of 10 on his weekly comprehension worksheets. When his mother reads it to him, he can easily tell her all of the correct answers.

In spite of his struggles with reading, Anthony enjoys school. He likes to draw ice dragons, fire dragons, and Vikings with swords. He particularly enjoys classroom activities that involve projects.

Educational History and Prior Evaluations

Anthony began school at the age of 2. His mother taught a home preschool in which Anthony participated from the time he could walk. When he was 2, he would often show a 4-year-old child how to do the task on his worksheet without saying a word. Although he seemed bright in many ways, his mother observed that at the age of 4 and 5 when the other kids in the class were grasping letter sounds and symbols, he was not.

The Halls then took him to Head Start for an evaluation. He tested just under his age level and they were told that his development was just a little slow and that he would catch up. Anthony's mother had him tested again at age 5, just before he entered kindergarten. Mrs. Hall reported that this time he scored well because he knew what a “quart Mason jar” was and used all three words, which none of the other children did. When asked to tell about the picture of a penguin, he got excited and answered “icy cold penguins” and proceeded to tell how they play in the water. His scores were at 6 years in most areas and 4 years 6 months in a few others. His mother was told again that any concerns they had were just developmental, and that he would outgrow any problems.

Anthony attended Scott Elementary in Tucson, Arizona for the first semester of kindergarten. His mother developed a word wall, and they constantly practiced his sight words, but he was not able to retain most of them. Anthony's teacher described him as “delightful and smart,” but when she was asked how he was doing compared with the other children, she said she was concerned about his reading progress and referred him for an evaluation. The family moved to Nevada, and the testing was done at his new school. The results were similar to the prior assessments, and his parents were told again not to worry about his reading.

His teacher was sick the whole first semester and he had multiple substitutes. The principal tutored Anthony and other children who were struggling with reading, and Mrs. Hall was told that he was a great kid and doing just fine.

The Halls then moved back to Arizona to run the family farm. Anthony began third grade at St. Thomas Elementary. The third-grade teacher was brand new to teaching. In the first few months, her notes home changed from “he is a hard worker” to “wish he would pay more attention in class.” His reading helper's notes also changed from “Anthony is a great kid trying hard” (1st week) to “He is despondent and gets no support at home” (4th week). The Halls realized that he was not getting the help he needed in that school and that the situation was not going to change. They then moved Anthony to Sterling Primary School, down the road. He now likes learning again but is still struggling with reading.

Tests Administered

  1. Woodcock-Johnson IV Tests of Cognitive Ability (WJ IV COG) Standard and Extended Batteries
  2. Woodcock-Johnson IV Tests of Oral Language (WJ IV OL)
  3. Woodcock-Johnson IV Tests of Achievement (WJ IV ACH) Standard and Extended
  4. Interview with Mrs. Hall

Testing Observations

Testing was conducted in two 2-hour sessions and one 1-hour session. Although Anthony was cooperative, pleasant, and engaged during all of the testing, his ability to sustain attention was quite variable. On several occasions, directions had to be repeated, and Anthony had to be reminded about what he was supposed to be doing.

Test Results

The WJ IV was scored according to grade norms. Because these three batteries are co-normed, direct comparisons can be made among his cognitive, oral language, and achievement scores. These comparisons can help determine the presence and significance of any strengths and weaknesses among his abilities. These tests provide measures of Anthony's specific cognitive and oral language abilities, as well as his current levels of academic achievement. Standard score confidence intervals (CI) are 68%.

Anthony's performance is compared with that of his grade-peers using standard score (SS) ranges:

SS Range <69 70–79 80–89 90–110 111–120 121–130 >130
Verbal Label Verylow Low Lowaverage Average Highaverage Superior Very superior

His academic proficiency on specific tasks is described by the relative proficiency index (RPI) levels. The instructional implications suggest the difficulty level with which Anthony is likely to perceive the task with the associated RPI.

RPI Range 0/90 to 3/90 3/90 to 24/90 24/90 to 67/90 67/90 to 82/90 82/90 to 95/90 95/90 to 98/90 98/90 to 100/90 100/90
Level of Proficiency Extremely limited Very limited Limited Limited to average Average Average to advanced Advanced Very advanced
Instructional Implications Nearly impossible Extremely difficult Very difficult Difficult Manage-able Easy Very easy Extremely easy

Cognitive and Oral Language Abilities

Based on the tests of the WJ IV COG, Anthony's overall intellectual ability, as measured by the GIA, was in the Low Average range (SS = 84; CI 80–87). Two factors must, however, be considered in the interpretation of this score. First and foremost, Anthony's attention was variable and affected performance. For example, on the Story Recall test, he was able to recall numerous details on one story, and then could not recall anything on the next story. His lowest score was on the Memory for Words test, which required repeating back a string of unrelated words (SS = 72; CI 66–78). On several items, he was unable to recall even one word, saying, “I forgot that one.” Although Anthony has relative weaknesses recalling information that is not meaningful, his difficulties with learning appear to be more related to attention than to memory abilities per se.

Second, Anthony also has a tendency to stop trying as soon as a question or problem becomes challenging. For example, the Number Series test required identifying the missing number in a series. When he had difficulty on one problem, he stopped trying, maintaining that the others were “too hard.” These two factors affected his overall performance.

In addition, Anthony's performance varied across different types of tasks. Anthony had significant strengths on tasks that involved visual analysis, spatial skills, and reasoning. For example, his score on the Visual-Auditory Learning test, a test requiring him to memorize a word to associate with an abstract symbol and recall it later, was in the High Average to Superior range (SS = 119; CI 112–125), as was his score on the Analysis-Synthesis test, a measure of deductive reasoning (SS = 117; CI 112–123).

Anthony's proficiency in language use and comprehension, as well as his general knowledge and vocabulary, were Average. Anthony could define words, repeat back meaningful sentences, and answer questions involving general information and world knowledge. Anthony enjoyed these types of tests.

Oral Language Abilities and Academic Achievement

Anthony's oral language abilities were assessed using the WJ IV OL, and his academic performance was assessed in reading, writing, and math using the WJ IV ACH. On the WJ IV ACH, only performance on the Broad clusters are discussed, because there were no significant differences in his performance among measures of basic skills, fluency, and comprehension.

Oral Language Abilities

Overall, Anthony's oral language abilities ranged from the Average to High Average range.

He demonstrated adequate vocabulary knowledge, listening comprehension, and phonological awareness. Anthony had average ability to blend together speech sounds, which is a key skill for learning phonics and to segment or break apart speech sounds, a key skill for spelling. His lowest score, in the Low Average range, was on the Rapid Picture Naming test, which required the rapid naming of pictures of common objects (SS = 86; CI 81–92).

Reading

WJ IV ACH CLUSTER/Tests RPI Proficiency SS(±1 SEM) Descriptor
Word Attack 66/90 Limited 88 (83–92) Low average
Word Reading Fluency 4/90 Very limited 81 (76–86) Low to low average
BROAD READING 36/90 Limited 85 (82–88) Low average
 Letter-Word Identification 19/90 Very limited 81 (78–84) Low average
 Passage Comprehension 34/90 Limited 80 (75–84) Low to low average
 Sentence Reading Fluency 59/90 Limited 94 (89–98) Average

Anthony's proficiency was Very Limited to Limited on all reading tests. For example, on the Letter-Word Identification test, he had an RPI of 19/90, which indicates very limited proficiency in word recognition when compared with his grade-peers. He had difficulty applying phonic skills and recognizing common sight words (e.g., there and when). As words increased in length, he tended to guess at their pronunciations by using the initial consonant, rather than examining the entire word (e.g., reading sentence as “science”). Anthony tended to give up on more difficult words, rather than attempting to sound them out. Anthony was, however, more successful on pronouncing nonsense words (i.e., made-up words that conform to English spelling patterns) than on pronouncing real words. Anthony mispronounced almost every irregular word on the real-word list, whereas all of the words on the nonsense word list are regular. Nevertheless, Anthony's RPIs indicate that all types of reading skills, including recognizing words that should be familiar, sounding out new words, reading and understanding short passages, and reading quickly, will be very difficult to extremely difficult for him.

Written Language

WJ IV CLUSTER/Tests RPI Proficiency SS(±1 SEM) Descriptor
Spelling of Sounds 83/90 Average 94 (89–99) Average
BROAD WRITTEN LANGUAGE 80/90 Limited to average 93 (91–96) Average
 Spelling 55/90 Limited 88 (85–91) Low average
 Writing Samples 86/90 Average 98 (94–101) Average
 Sentence Writing Fluency 89/90 Average 99 (93–105) Average

Anthony's proficiency in writing was slightly more advanced than his reading proficiency. His lowest performance was on the Spelling test. Although he was able to represent the sounds in many of the words, he omitted salient sounds from several words and tended to spell words the way they sound, rather than the way they look (e.g., he spelled from as “frum”). Anthony found it easier to spell nonsense words than to spell real words that contain irregular elements (Spelling of Sounds, RPI 83/90; Spelling, RPI = 55/90).

On Writing Samples, a test that measures skill in formulating and writing sentences or phrases in response to a variety of demands, Anthony was able to write complete sentences. Although there is no penalty for misspelled words on this test, he made several spelling errors on common words (e.g., road as roud, hole as holl).

Mathematics

WJ IV CLUSTER/Tests RPI Proficiency SS(±1 SEM) Descriptor
Number Matrices 52/90 Limited 84 (79–89) Low average
BROAD MATH 58/90 Limited 87 (83–90) Low average
 Applied Problems 40/90 Limited 80 (75–85) Low to low average
 Calculation 68/90 Limited to average 90 (86–94) Low average to low
 Math Facts Fluency 67/90 Limited to average 92 (87–97) Low average to average

Anthony also had some difficulty on tests of mathematics. He was able to complete two-digit addition and subtraction problems without regrouping but did the computations by counting on his fingers. He was able to count accurately with pictures, add and subtract using pictures, identify the names of coins, and identify the correct time on a clock. He was unable to complete multiplication problems involving more than single digits, and to solve simple division problems. Anthony had difficulty counting change below a dollar. Again, he did not persist when he perceived the problems were getting too hard.

WJ IV Variation and Comparison Procedures

Variations

On the WJ IV, intra-individual variations are computed to show the likelihood of a person obtaining a particular score, given the average of their other cognitive, oral language, or achievement cluster scores. Large variations indicate areas of significant strength and weakness. On the Intra-Cognitive Variations, Anthony had strengths on two tests: Visual-Auditory Learning, a measure of the ability to recall visual and auditory associations, and Analysis-Synthesis, a measure of reasoning ability. He had a significant weakness on the Memory for Words test, a measure of memory span, which required repeating back a list of unrelated words. There were no significant variations among his oral language abilities or his achievement.

Comparisons

The WJ IV has several comparison procedures in which one composite of core tests is used to predict performance in other areas. Based on his Gf-Gc Composite score, a combination of verbal knowledge and reasoning ability, Anthony's reading and math problem solving scores were significantly lower than predicted. When his Broad Oral Language score was used to predict his achievement scores, all aspects of reading, as well as math problem solving were lower than expected. For example, when his predicted score (SS = 102) was compared with his actual score on the Reading cluster (SS = 79), only two out of 100 students would have a score as low. When his predicted score (SS = 102) was compared with his Math Problem Solving score (SS = 80), only three out of 100 students would have a standard score as low. In addition, significant discrepancies were found when Anthony's Academic Knowledge (knowledge of humanities, science, and social studies) was used to predict his overall reading achievement and math problem solving. These results indicate that Anthony's verbal abilities, reasoning, and academic knowledge are all significantly higher than his present performance in reading and math problem solving, and suggest that he has the aptitude for a higher level of performance in both areas.

Summary

Anthony will be entering fourth grade in the fall. He has a history of difficulty with reading, despite numerous interventions from both home and school. He was referred for an evaluation by his mother because of her concerns regarding his struggles with learning to read.

Anthony is cooperative and enjoys learning. He easily performs tasks and solves problems that are visual in nature. His verbal ability (acquired knowledge and language comprehension) is average when compared with his grade-peers, which suggests that both his reading and math performance should be close to grade level. Although he had some difficulty on tests involving memory, this seemed more related to difficulties with the control of attention, rather than to a weakness in memory per se. In addition to these test results, teacher reports indicate problems maintaining attention in the classroom.

These results also support the existence of a reading disability. Anthony's potential as measured by his oral language and reasoning abilities is much higher than his current reading performance. He does not, however, persist when he perceives that tasks have become too challenging. Instead, he just states that the problems are “too hard.” Although Anthony has received numerous interventions in the past, he has made very little progress in reading. His limited attention and his difficulty persisting on challenging tasks likely exacerbate his difficulty in learning and retaining the reading skills he has been taught.

Presently, Anthony needs targeted instruction that will improve his overall level of achievement in word reading, spelling, and mathematics, and increase his confidence in his learning abilities.

Recommendations

Parents

  1. Meet with Anthony's pediatrician to discuss the possibility of attention deficit/hyperactivity disorder (ADHD) and to pursue a formal evaluation of his attentional control. If Anthony is diagnosed with ADHD, consider medications to improve his focus and ability to concentrate. Many of the behaviors that Anthony exhibits result from difficulties with attention. Keep in mind that these difficulties do not stem from a lack of effort or caring. Anthony wants to do well, but too often the expectations and academic demands are too high for his present skill levels.
  2. Anthony will feel frustrated by his attention and learning difficulties. Try to minimize his frustration by providing short periods of instruction (e.g., 10 to 15 minutes) that are followed by rewards, or some type of engaging activity, such as a game, or incorporate reading, spelling, or math practice into games. Alternatively, he may stay on task for longer periods if he receives feedback and rewards throughout the activity (e.g., chips, Monopoly money).
  3. Encourage Anthony to persist even when tasks become difficult. Show him that if he sticks with a problem and attempts alternative solutions, he will often be able to answer the problem correctly. When Anthony says that he cannot do a problem, say: “Let's take a closer look at that.” Guide him in the steps to follow to solve the problem. As his confidence increases, he will need less support and encouragement.

School Programming

  1. Ask the school to explore eligibility for specific learning disabilities (SLD) services as soon as Anthony returns to school. Presently, Anthony has significant discrepancies between his oral language and reasoning abilities and his reading and math achievement; he has made insufficient academic progress in school even with additional assistance.
  2. As he enters fourth grade, Anthony will require specific accommodations in the classroom, such as extended time on assignments and shortened homework assignments. Until his reading and math performance improve, Anthony will need adjustments in the difficulty level in both school and homework assignments. Although he is entering fourth grade, his performance levels are well below his grade peers.
  3. Anthony would benefit from sitting at the front of the classroom so as to maximize his attention.
  4. When possible, break Anthony's in-class assignments into smaller, more manageable chunks. Give him one part at a time with instructions to hand each in as it is completed and pick up the next. Each time he hands in a portion of the work, provide reinforcement for completed work. Using this technique, he will be more likely to stay on task and complete assignments.
  5. Anthony has average oral language skills, as well as a good foundation of world knowledge. He also completes visually based tasks that require spatial manipulation with relative ease. Provide Anthony with opportunities, such as oral reports and science projects, so that he can demonstrate his strengths in certain school subjects.

Initial Instructional Goals

Reading and Spelling

  1. To make progress, Anthony requires specialized reading instruction. During the summer and ideally into the school year, he would benefit from enrollment in an online reading program, Virtual Reading Coach, developed by Mindplay in Tucson, AZ. The program begins with an initial reading assessment that determines exactly which skills he should use and then plans a prescriptive program based on the assessment results. He should spend 30 minutes per day, 5 days per week, with this program, ideally for the next 6 months. A home version is available at www.myreadingteam.com, and a school version is available at www.mindplay.com.
  2. Provide Anthony with instruction that increases his recognition of common letter patterns, builds his understanding of spelling rules, and shows him how to use structural analysis (breaking words apart into syllables) to make them easier to pronounce. Reading and spelling instruction will be most effective if the patterns taught for reading are taught for spelling at the same time.
  3. To build speed and accuracy in pronunciation of sight words and phonetically irregular words, use one-minute speed drills. Time Anthony daily on reading lists of common, irregular words, as quickly as he can but with accuracy as the primary goal. Record and display his daily performance on some type of graph. Also have him practice spelling these words. (See Speed Drills for Reading Fluency and Basic Skills in Section IV.)

Mathematics

  1. Provide practice in math facts and math using fun video games. Many of these can be found on www.coolmath.com.
  2. Teach Anthony basic math skills, including the concept of place value, adding and subtracting multidigit numbers with and without regrouping, two-digit multiplication, and simple division. Wait until he has mastered these before introducing fractions and decimals.
  3. Consider use of an online math program, such as ALEKS, to supplement math instruction (http://www.aleks.com). This program will design a math curriculum to address gaps in Anthony's knowledge.
  4. Provide instruction in learning the value of coins and how to add them together and make change.
  5. Provide daily review of different types of math problems. A good book to use would be Math4Today, which provides daily 10-minute worksheets with a variety of math problems geared at a fourth-grade level.
  6. To maximize attention, alternate instruction in math skills with games to reinforce and develop those skills.

Educational Evaluation

Student: Martha Downing
Date of Birth: March 3, 2004
Age: 11–0
Date of Testing: March 10, 2015
School: Green Acres Country Day
Grade: 5.6

Reason for Referral

Ms. Suzanne Dobson, a certified elementary teacher, referred Martha for an evaluation. Ms. Dobson currently tutors Martha after school on a daily basis. She wished to obtain a better understanding of Martha's educational needs, any factors that were interfering with educational development, and her present performance levels in academic subjects. She wondered about the need for Martha to attend a special school or a special program to address her unique learning needs. Ms. Dobson requested information regarding programs and instructional techniques that would be the most effective for helping Martha to improve her basic math skills.

Background Information

Martha is an 11-year-old girl who is currently being tutored daily by Ms. Suzanne Dobson, a certified teacher with a master's degree in elementary education. Instruction occurs each afternoon from 4:00 to 5:30 p.m. in Ms. Dobson's home. The primary purpose of the tutoring is to help Martha complete her nightly homework.

Martha was adopted as a baby, and all developmental milestones were reported by her mother as being within normal limits. She currently lives in Scottsdale, Arizona, with her mother and an 8-year-old brother, Thomas, who is also adopted. Early history includes ear infections (ages 4 months to 2 years), asthma (ages 2 to 3), and extreme reactions to noise or sudden movement (preschool years).

Martha attended Desert Corner preschool from ages 4 to 5. In kindergarten, she attended Logan Academy, a local charter school. She remained in this educational setting until the end of grade 4. At that time, a tutor recommended that Martha would benefit from a school that provided “more direction.” She then transferred to Green Acres Country Day, a private school known for its small class size, individualized attention, dedicated teachers, and an accelerated and challenging pace. She repeated grade 4. On her most recent report cards, Martha's teachers describe her as “polite, respectful, and cooperative.” There are no indications of behavioral or social difficulties in the school setting or in the afternoon tutorials. Several teachers also commented on how hard Martha has been working and that her schoolwork shows continued improvement. Her lowest grade was in math (C−). Her math teacher reported that Martha continues to work hard in math, but “needs to keep working on mastery of her basic facts.”

Prior Evaluations

Last summer, Martha received individualized tutoring in reading through the SCORE program in California. On the California Achievement Test, Reading Program CAT level 13, her Total Reading Score was at the 5th percentile rank with a grade equivalency of 2.4. Her Vocabulary was at the 11th percentile, and her Reading Comprehension was at the 2nd percentile. At the end of last summer's tutoring, Dr. Pamela Reese, Reading Specialist, conducted an informal diagnostic session with Martha. She concluded that Martha was reading at about a third-grade level for instructional material, and a mid-second-grade level for easy material.

Two years ago, Dr. D. Owens, a neurologist, saw Martha for a Quantitative Neurological Examination that included blood work, an electroencephalogram, neurophysiological measures, parent rating scales, and academic testing. Results on the Parent Rating Scales and parent interview suggested mild evidence of inattentiveness, rage episodes, aggression, problems in socialization, and mild obsessiveness. Tests of attention showed unequivocal evidence of inattentiveness; speed was below expectancy on a Rapid Automatized Naming test. In academic skills, minor problems were noted in decoding and fluency with adequate performance in reading comprehension, spelling, and handwriting. Math skills were described as weak. The dismissal diagnoses were as follows:

  1. Inattention unassociated with hyperactivity, 304.0
  2. Math calculation disorder, 315.1
  3. Reading disorder, partially compensated, 315.0
  4. Enuresis, 307.6
  5. Parasomnia, 780.56
  6. Oppositional defiant behavior, 313.82
  7. Anxious obsessive personality, 300.02/300.3

A variety of possible medical therapies were discussed to address inattention, anxiety, low school performance, and depression. Dr. Owens wrote, however, that: “None of the symptoms we have seen demand the use of a pharmaceutical agent.”

Behavioral Observations

Martha was cooperative throughout the two testing sessions, and generally appeared at ease, comfortable, and attentive. She was willing to attempt all tasks and maintained a positive attitude even as the tasks increased in difficulty. On several occasions, she was briefly distracted by noises in the environment, but was quickly redirected to the task. Martha commented that she liked school and her teachers, and did not mind spending so much time each day on her homework.

Tests Administered

  1. Woodcock-Johnson IV: Tests of Achievement (WJ IV ACH)
  2. Key-Math 3

Test Results

Because Martha has repeated fourth grade, the WJ IV Tests of Achievement were scored by both grade and age norms. Results were similar for both sets of norms, so only a complete set of obtained scores for age norms is appended to this report. On the attached score summary, Martha's abilities measured by the WJ IV ACH are described as percentile rank ranges and Relative Proficiency Indexes.

Percentile Rank Descriptor
98 or higher Very Superior
91 to 97 Superior
85 to 90 High Average
76 to 84 Average
9 to 23 Low Average
2 to 8 Low
Below 8 Very Low
RPI Range Level of Proficiency Instructional Implications
100/90 Very Advanced Extremely easy
100/90 to 98/90 Advanced Very easy
98/90 to 95/90 Average to Advanced Easy
95/90 to 82/90 Average Manageable
82/90 to 67/90 Limited to Average Difficult
67/90 to 24/90 Limited Very difficult
24/90 to 3/90 Very Limited Extremely difficult
3/90 to 0/90 Extremely Limited Nearly impossible

The percentile rank (PR) indicates where Martha's score would fall within the scores of 100 or 1,000 students of the same age and month. For example, a PR of 75 would indicate that her score was the same as or greater than 75 of 100 age-peers in the norm sample, whereas a percentile rank of 0.1 would indicate that her score was the same as or higher than only one out of 1,000 age-peers. The grade equivalent (GE) indicates the median raw score of students in that month and year of school. The Relative Proficiency Index (RPI) is a qualitative score indicating Martha's expected level of proficiency on similar tasks if her average age-peers had 90% success. RPIs above 96/90 suggest that Martha will find the task to be easy, whereas RPIs below 75/90 suggest that she will find that type of task to be difficult.

Academic Assessment Results

Present academic testing indicates that Martha has made considerable progress in both reading and writing. Her academic knowledge is commensurate with that of age-peers. In contrast, she is still struggling with the mastery of basic math concepts and skills, as well as a compromised reading rate.

Reading and Written Language

In general, Martha's word identification and word attack strategies were adequate, but her speed of reading was slow. Her lowest reading scores were on the two timed measures that constitute the Reading Rate cluster: the Sentence Reading Fluency test in which she had to read simple sentences (e.g., A car rides on water) and decide whether the answer is true or false and the Word Reading Fluency test, in which she had to circle the two words that go together in a row of four words. On both tests, her score exceeded only 13% of her age peers. Although Martha's oral reading accuracy has improved greatly, her reading rate is still slow. Her RPI of 8/90 on the Reading Rate cluster indicates that when average age-peers have 90% success, Martha will have only 8% success, which is very limited proficiency.

Martha obtained scores within the Average range on most all of the writing tests, with a Broad Written Language score equal to or better than that of 45% of her age peers. Her RPI of 88/90 indicates that when average age-peers have 90% success, she will have 88% success, which is also average proficiency. Martha seemed to enjoy writing and wrote sentences with ease and confidence. She was able to write short, simple sentences quickly and easily (Sentence Writing Fluency) and to express ideas in sentences (Writing Samples). Although her scores were within the Average range, when attempting to write more descriptive sentences, she had difficulty organizing her thoughts, and commented that “I know what I want to write but it's hard getting it down right.” Her lowest score was on the Editing test, in which her score was equal to or better than those of only 14% of her age-peers. On several of the items, she could identify the mistake but was uncertain of how to correct the error.

Mathematics

Martha's greatest difficulties were in the acquisition and mastery of basic math skills. Her score on the Math Calculation Skills cluster was equal to or better than that of only 2% of her age-peers. Her RPI on Math Calculation Skills was 6/90, indicating that when average age-mates have 90% success, Martha will have only 6% success, or very limited proficiency. Presently, she can add and subtract single-digit combinations, knows several multiplication facts, and can perform simple division (e.g., 10/2). She does not, however, have a firm foundation of fundamental math operations.

Martha did not attend to operation signs consistently and has not memorized all of the multiplication facts. Because she did not attend to the operation signs, she made mistakes on simple addition and subtraction problems (e.g., 3 + 3 = 0). She was unable to solve simple computations, because she does not understand place value or regrouping (e.g., 89–18 = 103, 17–8 = 20). Martha does not understand how to multiply a double-digit number by a single-digit number, how to add and subtract fractions, or the steps involved in long division. She was unable to find a simple fraction of a number (e.g., ⅓ of 12) or add or subtract simple fractions with common denominators (e.g., ¼ + 2/4). In addition, Martha does not monitor her work to see if the answers make sense. Many of her answers were impossible (e.g., 140/20 = 170).

Currently, Martha has minimal understanding of concepts regarding money. When shown pictures of various coins, she was unsure of which picture was a nickel and which one was a quarter. She was not able to add together simple combinations of coins. When shown real money, she identified the coins, but did not understand their monetary value. She was not sure of how many pennies are in a nickel and could not count various groups of coins. When shown 36 cents, she said it equaled 65 cents. When shown 65 cents, she said it was equal to 11 cents.

Throughout all math tasks, Martha did not seem to have a clear understanding of number concepts and relationships. She could not discern the patterns in number sequences (e.g., 6, 8, _, 12) if the interval between numbers were more than one.

Because of her difficulties in mathematics on the WJ IV ACH, Martha was administered the KeyMath-3, a diagnostic inventory of essential mathematics. The KeyMath-3 measures three main areas: Basic Concepts, Operations, and Applications. The purpose of the evaluation was to determine appropriate instructional goals for Martha in mathematics. The results are presented in this table.

Test Results Scaled Score* Test Results Scaled Score
Basic Concepts Operations
Numeration 6 Mental Computation and Estimation 5
Algebra 5 Addition and Subtraction 4
Geometry 11 Multiplication and Division 5
Measurement 5 Applications
Data Analysis and Probability 4 Foundations of Problem Solving 7
Applied Problem Solving 8
*10 is the mean, with a standard deviation of 3.
Areas Standard Score Percentile Rank Grade Equiv.
Basic Concepts 92 30 4.8
Operations 73 4 2.7
Applications 84 14 3.8
Total Test 81 10 3.6

Martha's score on Basic Concepts was significantly higher than her score on Operations. Her greatest difficulties were on tasks involving basic math operations (addition, subtraction, multiplication, and division), as well as concepts related to money. Many of her responses indicated limited knowledge of mathematical concepts. As examples, when asked the meaning of 3.7, she responded: 3 to 7th power. She could not answer questions such as: How many dimes are in a dollar? When asked to write the number for one million, she wrote 1,000. Presently, she has minimal knowledge of how to perform all four operations. In contrast to her basic skills, she had grade-appropriate knowledge of concepts related to geometry.

WJ IV Discrepancies

On the WJ IV, two types of discrepancies can be calculated to show the likelihood of a person obtaining a particular score: intra-ability (strengths and weaknesses among abilities), and ability/achievement (an ability is used to predict achievement). On the intra-ability variations, the predicted score is based on the average of five other core tests. When her Math Calculation skills were compared with her other abilities, only two of 1,000 students would have a difference of this size between her actual and predicted scores. On the ability/achievement discrepancies, the predicted score is based on the person's Academic Knowledge score, orally administered measures of knowledge regarding science, social studies, and humanities. When her Academic Knowledge was compared with reading, writing, and math performance, Martha's Math Calculation Skills were significantly below present predictions. Only one of 100 students would have a difference that size.

Summary

Martha's reading accuracy, written expression, and academic knowledge are within the Average range for her age-peers. When viewed in light of prior evaluations, Martha has made significant progress in reading and writing. This is a tribute to the high quality of instruction from teachers and tutors, as well as Martha's continued willingness to engage in hard work. In contrast, her basic math skills, understanding of mathematical concepts, and reading rate are still significant weaknesses.

Educational Recommendations

General

  1. Because Martha works hard and wants to succeed, provide frequent praise and reinforcement commending her for her hard work and effort.
  2. Make sure that Martha continues in a supportive academic environment.
  3. Encourage Martha to take increasing responsibility for completing her homework.
  4. Help Martha to continue to gain confidence in her learning abilities. She is a capable learner, a hard worker, and will continue to make steady progress.

Placement and Educational Therapy

  1. Martha will continue to profit from instruction in a general education classroom because her knowledge, as well as her levels of performance in reading and writing, are commensurate with those of her peers.
  2. Martha will continue to profit from individualized instruction to address and resolve her difficulties in basic math skills, increase her reading rate, and improve her editing skills.
  3. Martha works hard, completes assignments, and gets along well with both teachers and peers. Her successful performance indicates that she will not require a more restrictive school setting and should continue to benefit from her present placement or a local public school, if the homework demands become too great.
  4. If Martha is enrolled in a public school, she may be eligible to receive learning disability services in the area of basic math skills. Further cognitive testing should be done to explore possible correlates of math disability, including working memory and processing speed, as well as to confirm strengths in oral language.

Further Assessment

  1. Use diagnostic teaching to continually evaluate Martha's mastery of basic math skills and concepts.
  2. Before teaching math facts, give Martha a timed test to see what multiplication facts she can complete correctly within 2 minutes, or an oral test using flashcards to see which facts she can answer within 3 seconds. Use the results to develop a program for fact learning. Have Martha chart her progress as she masters new facts.

Accommodations

  1. Because of her slow reading rate, Martha will require extended time on standardized tests, as well as on some in-class assignments.
  2. Because of her slow reading rate, Martha will need adjustments in the amount of reading expected of her. It would be preferable to assign a certain amount of time for reading, rather than a certain number of pages.

Reading Speed

Provide a program specifically designed to improve reading accuracy and fluency, such as Great Leaps Reading (www.greatleaps.com). This program increases reading speed fluency while reinforcing phonics skills. One-minute timings are done that employ three stimuli: phonics, sight phrases, and short stories. Use the program designed for Grades 3–5. Chart performance on graphs so that Martha can see her progress. This activity will take approximately 10 minutes per day.

Writing

  1. Martha needs increased practice revising and editing her writing. Encourage her to look for errors and then correct them, using a spelling checker if needed.
  2. Help Martha develop keyboarding skills so that she can write class assignments on a computer.

Mathematics

Accommodations

  1. Because Martha has not developed automatic recall of math facts and algorithms, provide her with a calculator to use in all activities focused on mathematical reasoning. This will allow her to concentrate on the reasoning process without diverting attention to the more mechanical aspects of the task.
  2. If possible, reduce Martha's homework assignments in mathematics so that she can concentrate on mastering the basic skills and concepts that are critical to future learning. In lieu of homework, she could turn in daily lessons from her after-school tutorial to document her work and progress. Martha needs to fill in the gaps before proceeding with additional skills and concepts.
  3. Because she has not memorized her math facts, for computation problems provide Martha with pocket-sized charts with addition and multiplication facts. Teach her how to use the addition chart for subtraction and the multiplication chart for division. Visualizing the location of the answer may help Martha learn to retrieve it. When a fact has become truly automatic, have Martha block it out on the chart.

Resources

  1. Have Martha work at least 10–20 minutes daily using the book. Math 4 Today by Donna Pearson. In this program, designed for a fourth-and fifth-grade math curriculum, twenty-four essential math skills and concepts are reviewed. The program is designed with a continuous spiral so that concepts are repeated weekly and a ten-problem test is provided for the fifth day to ensure mastery of skills and concepts.
  2. Provide Martha with additional practice using the book 5-Minute Math Problem of the Day by Martin Lee and Marcia Miller. This book provides seven broad categories in line with the National Council of Teachers of Mathematics (NCTM) standards: Whole numbers, decimals, fractions, measurement, geometry, percentages, and algebra.
  3. Martha would benefit from a program, such as Great Leaps Math (www.greatleaps.com), which will provide daily practice designed to make basic math skills automatic.

Basic Operations

  1. Help Martha to devise a strategy to help her pay attention to computational signs before solving a problem.
  2. Martha needs practice reading and writing numbers that are greater than 1,000.
  3. Review the meaning of place value up to 1,000,000. Using manipulatives, help Martha develop an understanding of place value. Help her understand the connection between the place of the digits constituting a number and the value of the digits.
  4. Martha needs to become automatic with her math facts. Begin a systematic instructional program in which Martha graphs or charts the facts that she has mastered. Provide practice sessions with a few flash cards so that Martha can become automatic with her math facts. Use flash cards that have the answer recorded on the reverse side. Practice only a few facts for several minutes daily. (See Addition/Multiplication Facts: Instructional Sequence, and Addition Facts: Organizational Structure for Memorization in Section IV.)
  5. Provide frequent review of the newly mastered facts. Computer software drill and practice games, such as MathBlaster, (www.mathblaster.com) would also be helpful.
  6. Reteach subtraction with two-digit then three-digit numbers without and then with regrouping.
  7. Teach Martha how to multiply a two-digit number by a one-digit number. Have her use a facts chart until she has fully memorized her multiplication facts.
  8. Teach Martha how to round numbers to the nearest 100 and then to the nearest 10.
  9. Martha has difficulty performing mental computations. Encourage her to use paper and pencil when solving math problems.
  10. Help Martha learn to determine the missing number in a sequence of numbers (e.g., 1, 3, _, 7).
  11. Teach Martha how to divide a two-digit then a three-digit number when regrouping is not required. As skill progresses, teach her how to divide when regrouping is required.

Fractions, Decimals, and Percentages

  1. Martha needs to learn the following skills and concepts:
    1. the meaning of fractions, decimals, and percentages (She does not understand the purpose or meaning of a decimal point in a number or understand how fractions, decimals, and percentages are related.)
    2. how decimals relate to place value (e.g., 0.1 = 1/10)
    3. how to depict the meaning of a fraction with pictures (e.g., pie chart)
    4. how fractions can be converted into decimals and percentages
    5. how to add and subtract fractions with like and then unlike denominators
    6. how to add and subtract fractions and mixed numbers
    7. how to multiply and divide fractions with like and then unlike denominators
    8. how to multiply and divide fractions with mixed numbers
    9. how to perform the four basic operations with decimal numbers
  2. When teaching Martha concepts of fractions and decimals, use tangible objects, such as money or food. For example, demonstrate cutting a pizza in pie-shaped pieces or use an empty egg carton to teach fractions with a denominator of 12.

Money

  1. Martha is going to need considerable practice developing concepts and skills involving money. Presently, she does not identify coins consistently and does not know how to add together even small sets of change. Use real money.
  2. Teach Martha the value of all coins and bills. Provide practice in counting various combinations of change. Begin with simple combinations (e.g., one nickel and two pennies) and gradually progress to more complex calculations (e.g., two quarters, one dime, a nickel, and four pennies). When she can add change accurately to one dollar, introduce bills, and practice counting and giving change for varied combinations.
  3. Provide opportunities for Martha to practice making change. Discuss money in relationship to things that she wishes or needs to purchase. For example, if a soft drink costs $.89 and she gave the cashier $1.00, how much change would she receive?
  4. To increase Martha's flexibility in making change, provide her with practice matching equal value of coins. For example, using real money, ask her to show you as many ways as she can think of to give someone 50 cents in change. Explain to her why the combination with the fewest coins is usually returned.
  5. Teach Martha how to solve basic math operations (addition, subtraction, multiplication, and division) involving money.

Table of Scores

Woodcock-Johnson IV Tests of Achievement Form A and Extended (Norms based on age 11–0)

CLUSTER/Tests GE SS (68% Band) PR (68% Band) RPI Proficiency
READING 5.3 99 (96–102) 46 (38–54) 88/90 Average
Letter-Word Identification 5.2 98 (95–101) 44 (36–53) 86/90 Average
Passage Comprehension 5.5 100 (95–105) 49 (36–62) 90/90 Average
BROAD READING 4.1 90 (87–93) 25 (19–32) 64/90 Limited
Letter-Word Identification 5.2 98 (95–101) 44 (36–53) 86/90 Average
Passage Comprehension 5.5 100 (95–105) 49 (36–62) 90/90 Average
Sentence Reading Fluency 3.2 83 (78–88) 13 (7–21) 9/90 Very limited
BASIC READING SKILLS 5.5 100 (97–103) 50 (42–57) 90/90 Average
Letter-Word Identification 5.2 98 (95–101) 44 (36–53) 86/90 Average
Word Attack 6.4 103 (97–109) 58 (43–73) 92/90 Average
READING FLUENCY 3.7 87 (83–91) 20 (13–27) 47/90 Limited
Oral Reading 5.4 99 (96–103) 48 (39–58) 89/90 Average
Sentence Reading Fluency 3.2 83 (78–88) 13 (7–21) 9/90 Very limited
READING RATE 3.1 83 (79–87) 13 (8–19) 8/90 Very limited
Sentence Reading Fluency 3.2 83 (78–88) 13 (7–21) 9/90 Very limited
Word Reading Fluency 3.1 83 (77–89) 13 (6–23) 8/90 Very limited
MATHEMATICS 3.1 78 (75–82) 7 (5–11) 34/90 Limited
Applied Problems 4.3 92 (88–97) 30 (20–41) 76/90 Limited to Average
Calculation 2.4 68 (64–73) 2 (1–3) 8/90 Very limited
BROAD MATHEMATICS 2.8 73 (70–77) 4 (2–6) 20/90 Very limited
Applied Problems 4.3 92 (88–97) 30 (20–41) 76/90 Limited to Average
Calculation 2.4 68 (64–73) 2 (1–3) 8/90 Very limited
Math Facts Fluency 2.3 71 (65–77) 3 (1–6) 5/90 Very limited
MATH CALCULATION SKILLS 2.3 68 (64–72) 2 (1–3) 6/90 Very limited
Calculation 2.4 68 (64–73) 2 (1–3) 8/90 Very limited
Math Facts Fluency 2.3 71 (65–77) 3 (1–6) 5/90 Very limited
MATH PROBLEM SOLVING 4.1 90 (87–94) 25 (19–34) 73/90 Limited to Average
Applied Problems 4.3 92 (88–97) 30 (20–41) 76/90 Limited to Average
Number Matrices 3.8 90 (85–94) 25 (16–35) 69/90 Limited to Average
WRITTEN LANGUAGE 5.2 98 (95–101) 45 (38–53) 88/90 Average
Spelling 5.1 97 (93–100) 42 (33–51) 85/90 Average
Writing Samples 5.6 100 (96–105) 50 (38–62) 90/90 Average
BROAD WRITTEN LANGUAGE 5.2 98 (95–101) 44 (37–51) 88/90 Average
Spelling 5.1 97 (93–100) 42 (33–51) 85/90 Average
Writing Samples 5.6 100 (96–105) 50 (38–62) 90/90 Average
Sentence Writing Fluency 5.0 97 (91–103) 42 (28–58) 87/90 Average
BASIC WRITING SKILLS 4.2 91 (88–94) 26 (20–33) 72/90 Limited to Average
Spelling 5.1 97 (93–100) 42 (33–51) 85/90 Average
Editing 3.1 84 (79–88) 14 (8–22) 54/90 Limited
WRITTEN EXPRESSION 5.3 99 (95–102) 47 (37–56) 89/90 Average
Writing Samples 5.6 100 (96–105) 50 (38–62) 90/90 Average
Sentence Writing Fluency 5.0 97 (91–103) 42 (28–58) 87/90 Average
ACADEMIC SKILLS 3.9 87 (85–89) 19 (15–24) 59/90 Limited
Letter-Word Identification 5.2 98 (95–101) 44 (36–53) 86/90 Average
Spelling 5.1 97 (93–100) 42 (33–51) 85/90 Average
Calculation 2.4 68 (64–73) 2 (1–3) 8/90 Very limited
ACADEMIC FLUENCY 3.1 80 (76–83) 9 (6–13) 25/90 Limited
Sentence Reading Fluency 3.2 83 (78–88) 13 (7–21) 9/90 Very limited
Math Facts Fluency 2.3 71 (65–77) 3 (1–6) 5/90 Very limited
Sentence Writing Fluency 5.0 97 (91–103) 42 (28–58) 87/90 Average
ACADEMIC APPLICATIONS 5.0 97 (93–100) 41 (33–49) 86/90 Average
Applied Problems 4.3 92 (88–97) 30 (20–41) 76/90 Limited to Average
Passage Comprehension 5.5 100 (95–105) 49 (36–62) 90/90 Average
Writing Samples 5.6 100 (96–105) 50 (38–62) 90/90 Average
ACADEMIC KNOWLEDGE 6.1 103 (99–107) 57 (47–67) 92/90 Average
Science 6.2 102 (96–108) 56 (40–71) 92/90 Average
Social Studies 6.0 102 (97–108) 56 (41–70) 92/90 Average
Humanities 6.1 103 (97–109) 57 (42–72) 92/90 Average
BRIEF ACHIEVEMENT 4.9 96 (93–98) 38 (32–44) 83/90 Average
Letter-Word Identification 5.2 98 (95–101) 44 (36–53) 86/90 Average
Applied Problems 4.3 92 (88–97) 30 (20–41) 76/90 Limited to Average
Spelling 5.1 97 (93–100) 42 (33–51) 85/90 Average
BROAD ACHIEVEMENT 3.8 86 (84–88) 17 (14–21) 59/90 Limited
Letter-Word Identification 5.2 98 (95–101) 44 (36–53) 86/90 Average
Applied Problems 4.3 92 (88–97) 30 (20–41) 76/90 Limited to Average
Spelling 5.1 97 (93–100) 42 (33–51) 85/90 Average
Passage Comprehension 5.5 100 (95–105) 49 (36–62) 90/90 Average
Calculation 2.4 68 (64–73) 2 (1–3) 8/90 Very limited
Writing Samples 5.6 100 (96–105) 50 (38–62) 90/90 Average
Sentence Reading Fluency 3.2 83 (78–88) 13 (7–21) 9/90 Very limited
Math Facts Fluency 2.3 71 (65–77) 3 (1–6) 5/90 Very limited
Sentence Writing Fluency 5.0 97 (91–103) 42 (28–58) 87/90 Average
STANDARD SCORES DISCREPANCY
VARIATIONS Actual Predicted Difference PR SD Interpretation at
±1.50 SD (SEE)
Intra-Achievement [Extended] Variations
BASIC READING SKILLS 100 92 8 83 +0.94
READING FLUENCY 87 92 −5 31 −0.49
READING RATE 83 93 −10 18 −0.90
MATH CALCULATION SKILLS 68 98 −30 0.2 −2.86 Weakness
MATH PROBLEM SOLVING 90 94 −4 37 −0.34
BASIC WRITING SKILLS 91 92 −1 44 −0.15
WRITTEN EXPRESSION 99 92 7 73 +0.63
Letter-Word Identification 98 91 7 77 +0.75
Applied Problems 92 93 1 46 0.11
Spelling 97 92 5 68 +0.47
Passage Comprehension 100 91 9 80 +0.83
Calculation 68 98 30 0.2 2.83 Weakness
Writing Samples 100 93 7 72 +0.58
Word Attack 103 93 10 81 +0.89
Oral Reading 99 93 6 70 +0.52
Sentence Reading Fluency 83 92 9 20 0.85
Math Facts Fluency 71 98 27 1 2.24 Weakness
Sentence Writing Fluency 97 93 4 64 +0.36
Number Matrices 90 95 5 34 0.41
Editing 84 93 9 18 0.93
Word Reading Fluency 83 94 11 18 0.93
STANDARD SCORES DISCREPANCY
VARIATIONS Actual Predicted Difference PR SD Interpretation at
±1.50 SD (SEE)
Academic Skills/Academic Fluency/Academic Applications [Extended] Variations
ACADEMIC SKILLS 87 89 −2 41 −0.22
ACADEMIC FLUENCY 80 93 −13 8 −1.41
ACADEMIC APPLICATIONS 97 85 12 89 +1.24
READING RATE 83 94 −11 15 −1.02
STANDARD SCORES DISCREPANCY
VARIATIONS Actual Predicted Difference PR SD Interpretation at
±1.50 SD (SEE)
Academic Knowledge/Achievement Comparisons
BRIEF ACHIEVEMENT 96 102 −6 30 −0.54 No
BROAD ACHIEVEMENT 86 102 −16 10 −1.27 No
READING 99 102 −3 40 −0.25 No
BROAD READING 90 102 −12 18 −0.91 No
BASIC READING SKILLS 100 102 −2 44 −0.14 No
READING FLUENCY 87 101 −14 15 −1.03 No
READING RATE 83 101 −18 10 −1.30 No
STANDARD SCORES DISCREPANCY
VARIATIONS Actual Predicted Difference PR SD Interpretation at
±1.50 SD (SEE)
Academic Knowledge/Achievement Comparisons
MATHEMATICS 78 102 −24 3 −1.95 Yes (-)
BROAD MATHEMATICS 73 101 −28 1 −2.21 Yes (-)
MATH CALCULATION SKILLS 68 101 −33 1 −2.44 Yes (-)
MATH PROBLEM SOLVING 90 102 −12 16 −1.01 No
WRITTEN LANGUAGE 98 102 −4 39 −0.29 No
BROAD WRITTEN LANGUAGE 98 102 −4 39 −0.29 No
BASIC WRITING SKILLS 91 102 −11 18 −0.92 No
WRITTEN EXPRESSION 99 102 −3 42 −0.21 No
ACADEMIC SKILLS 87 102 −15 12 −1.19 No
ACADEMIC FLUENCY 80 101 −21 6 −1.58 Yes (-)
ACADEMIC APPLICATIONS 97 102 −5 33 −0.44 No

Psychoeducational Report

Name: Fred Armstrong
Date of Birth: October 26, 2003
Age: 11 years 5 months
Date of Testing: April 1, 2015
School: Highland Heights
Grade: 6 (2014–2015)
Primary Home Language: English
Evaluator: Michael E. Gerner, Ph.D.

Reason for Referral

Fred's mother, Sylvia Armstrong, requested an independent psychoeducational evaluation. She reported that she believes her son is very intelligent, but he often seems unhappy and disinterested in school and says that his teachers do not understand him. Ms. Armstrong has wondered whether they should find a different school for Fred. She requested information concerning Fred's abilities for ongoing educational planning. This evaluation was directed to address this referral question by investigating Fred's learning and behavioral profiles, cognitive abilities, and academic levels.

Measures Used

  1. Wechsler Intelligence Scale for Children, Fifth Edition (WISC-V), Verbal Comprehension Index
  2. Woodcock-Johnson IV Tests of Cognitive Ability (WJ IV COG)
  3. Woodcock-Johnson IV Tests of Achievement (WJ IV ACH)
  4. Behavior Assessment Scale for Children, Structured Developmental History (BASC-2, SDH)
  5. Behavior Assessment Scale for Children, Second Edition: Self-Report of Personality (BASC-2, SRP-C)
  6. Behavior Assessment Scale for Children, Second Edition: Parent Rating Scale—Mother (BASC-2, PRS-C)
  7. Behavioral Observations
  8. Clinical Interview
  9. Record Review

Evaluation Design and Test Considerations

The Woodcock-Johnson IV Tests of Cognitive Ability (WJ IV COG) and the WJ IV Tests of Achievement (WJ IV ACH) were selected because they operationalize recent advancements in cognitive science for the assessment of intelligence. Considerable factor-analytic evidence has been amassed that human cognitive performance comprises at least 9–10 broad cognitive abilities and more than 70 narrow abilities. Many of these abilities can be reliably assessed with psychometric instruments. Narrow abilities are assessed by tests or subtests (depending on the test battery) and are grouped into clusters or composites to assess the broader abilities. The broad abilities include aspects of fluid reasoning, comprehension-knowledge (crystallized/verbal intelligence), short-term working memory, visual-spatial thinking, auditory processing, long-term retrieval, processing speed, quantitative knowledge, and reading and writing. The Wechsler Intelligence Scale for Children, Fifth Edition (WISC-V), Verbal Comprehension Index, is a robust measure of verbal intelligence for the school-aged population, allowing for additional verbal elaboration and clarification of lexical knowledge, general information, and verbal-conceptual reasoning.

Understanding the Scores in This Report

The following table describes the standard score and percentile rank ranges used in this report, along with their verbal descriptors, and the percentage of individuals who score within each category. All scores are in comparison with students of Fred's age (year and month) in the norming sample, representative of the population of the United States.

Standard Score Range Descriptor Percentage of People Percentile Rank Range
>130 Very Superior 2 98th & above
121–130 Superior 7 91st to 97th
111–120 High Average 16 75th to 90th
90–110 Average 50 25th to 74th
80–89 Low Average 16 9th to 24th
70–79 Low 7 3rd to 8th
<69 Very Low 2 2nd & below

Conclusions and Recommendations

Fred is an academically capable individual with verbal ability and verbal-conceptual reasoning in the Superior range of intelligence. His cognitive abilities and reading, math, and written expression skills fall within the High Average to Superior range. Fred surpassed 94% to 96% of students his age in the United States in the areas of crystallized intelligence (acquired information), fluid reasoning, short-term working memory, and visual processing, all of which were uniformly in the Superior to Very Superior ranges. Fred is also proficient in mathematics problem solving (reasoning), likewise surpassing 95% of his age group. His ability to comprehend verbal instructions, create and think with verbal concepts, reason inductively, and problem solve with quantitative information are outstanding learning assets.

In fact, there were no deviations or significant weaknesses that could be expected to have a negative educational impact in an otherwise High Average to Superior learning and cognitive ability profile. The most advantageous IQ/Ability estimates are Fred's WJ IV Gf-Gc Composite and his WISC-V Verbal Comprehension composite. Both of these composite scores surpassed 96% of students Fred's age and fell in the Superior to the Very Superior range. Fred's Broad Reading, Mathematics, and Written Language performance exceeded 85%, 93%, and 86% of students his age, respectively. No composites or discrete skills fell below the Average range

Fred is a student with outstanding scholastic aptitude who shows no evidence of a significant weakness in academic achievement or in cognitive processing. He has the ability, skills, and behavioral strengths to succeed in a wide range of educational settings at the highest level. Fred and his mother report that he has a number of other talents and special abilities, including music, acting/theater, visual and graphic arts, and performance arts in general.

In this evaluation, Fred was engaged and attempted to complete all tasks. In addition, behavioral and emotional ratings scales completed by Fred and his mother did not indicate any significant problems in the majority of the domains measured. Fred's responses suggested elevated concerns in only two areas, Attitude toward Teachers and Interpersonal Relationships, associated with less than positive experiences in his current school. Fred does not identify with his classmates, whom he feels are sometimes critical of him. Also, he feels that some of his teachers misunderstand him, misjudge his motivations, and are overly rigid and unfair. His mother's only area of concern is that he “sometimes” has physical complaints, such as headaches or other pains. There were a large number of individual items rated “sometimes,” such that the overall score in this category just met the “at-risk” level of problems. This, however, was the only area to be so rated. All other areas actually reflected behavioral and emotional strength. Based on background information, current levels of performance, and the results of this evaluation, the following recommendations are made:

  1. In terms of capabilities, Fred has the intelligence, academic skills, and behavioral assets to succeed in a wide range of educational settings at the highest level; the challenge is finding a school or educational program that is most suitable for him in terms of scholastic rigor, school climate, and diversified program offerings in the expressive and performing arts.
  2. Opportunities for academic excellence and performing arts development are viewed as equally important. Fred needs to express and develop his other considerable strengths and interests in expressive and performance arts, such as acting, music, and graphic/visual arts. If these types of classes are unavailable at his current school, an alternative school setting should be considered.
  3. If Fred changes schools, he would benefit from one that has a gifted and talented program or accelerated classes in which he could enroll.
  4. Fred and his mother have described a number of concerns regarding the flexibility and responsiveness of his current school program. While Fred's academic needs can be met in a variety of educational settings given his sophisticated abilities, the current match between Fred and his current school environment needs to be carefully evaluated on an ongoing basis. Possible alternative school settings for the following year will be discussed at the postevaluation meeting with Fred and his mother.

These conclusions and recommendations are based on the information that follows.

Background Information

Developmental/Medical Information

The following information is based on a Structured Developmental History completed by Fred's mother. At birth, Fred's Apgar score was 10 and birth weight 6 pounds 8 ounces. His condition was described as “healthy, normal, beautiful” by his mother. All developmental milestones were within normal limits.

Performance in the General Curriculum

Fred attends Highland Heights, a charter school with a dedicated college preparatory curriculum. Ms. Armstrong reported that Fred works hard to maintain an “A” average but earns lower grades in some classes. She also said, “Fred is mostly successful at most all he achieves. He is fully capable of appropriate expression when afforded an opportunity and encouraged to do so. He is talented musically, spatially (imagination), and with performing, yet I fear he may have to leave his group of friends because of some problems at his school…problems that have easy solutions if the school would accommodate. If he could have a way to avoid bullies and distractions and [have] the full support and understanding from his teachers, he could more easily thrive.”

Standardized Group Test Results

The two sixth-grade Fall progress reports showed 10 As and 6 Bs in academic classes. A fifth-grade End of Year Report showed a grade point average of 94 (out of 100) across eight classes for the academic year. The fifth-grade AIMS report (Arizona's Instrument to Measure Standards) noted that Fred “Exceeds the Standard” in Reading, “Meets the Standard” in Writing, and “Exceeds the Standard” in Mathematics.

Vision and Hearing

Results of screenings within the past year were normal.

Effects of Culture, Educational Disadvantage, Limited English Proficiency, Assistive Technology

Fred has not experienced educational, economic, or environmental disadvantages. His English is well beyond proficient. Fred has not required assistive technology devices other than what is typically used for students in his grade, such as recorders, calculators, tablets, media devices, and computers.

Behavioral Observations

Fred was cooperative and motivated throughout the evaluation. His level of effort and concentration in the one-to-one testing environment suggests that the results are a valid and reliable estimate of his current abilities.

Test Results

Cognitive Abilities

Fred's estimates of overall intelligence are his WISC-V Verbal Comprehension composite standard score of 127 and his WJ IV Gf-Gc composite standard score of 126, which surpass 96% of children his age. In reviewing these results, it is important to remember that an intelligence test does not measure global or innate intelligence. An intelligence test is an estimate of scholastic aptitude, or the kind of ability and reasoning that is required in traditional academic work. An intelligence test does not measure many important abilities such as creativity, mechanical aptitude, divergent thinking, personal adaptability, social intelligence, interpersonal understanding, self-knowledge, and common sense.

In the course of the evaluation, it was evident that Fred was proficient in verbal comprehension and excelled when he could analyze and verbally elaborate on information. Verbal intelligence is also called “crystallized intelligence,” and it encompasses a number of narrower abilities, such as language development, vocabulary knowledge, and a range of general knowledge. On the WISC-V Verbal Comprehension composite and the WJ IV COG Comprehension-Knowledge cluster, Fred exceeded 96% and 89% of his peers, respectively. Although both performances are well above average, the WISC-V is considered the more accurate estimate of Fred's verbal intelligence. In terms of performance on subtests constituting this composite, Fred exceeded 75% of his age-peers in vocabulary knowledge. Most significantly, he surpassed 99% of his peers in conceptualizing the similarities among apparently dissimilar verbal concepts.

On the WJ IV COG Fluid Reasoning cluster (i.e., the ability to solve novel problems without having prior knowledge and related experience), which includes measures of both quantitative and inductive reasoning, Fred scored in the Superior range, exceeding 95% of his age group. His quantitative reasoning on the Number Series test surpassed 93% of his peers. His inductive reasoning on the Concept Formation test exceeded 95% of children his age.

Fred demonstrated another strong cognitive ability on the Short-Term Working Memory cluster, on which he surpassed 94% of students his age nationally. In addition, Fred's performance on the Long-Term Retrieval cluster surpassed 73% of students his age overall. This performance, somewhat lower than Fred's other cognitive abilities, is not suggestive of a weakness, because it is still substantially above average. On the Visual-Auditory Learning test, which assesses associative memory, the ability to link pairs of inherently unrelated elements (e.g., words to visual symbols) and retrieve them over time, Fred exceeded 81% of his age group.

Fred's performance on the WJ IV COG Visual Processing cluster was in the Superior range, incorporating the ability to integrate and visualize abstract designs and patterns (96th percentile) and visual memory for pictorial information (85th percentile). Visual processing ability is useful in higher-level mathematics, such as geometry; sciences, such as physics and anatomy; and visual arts and design, such as architecture.

Achievement

On the WJ IV ACH, Fred demonstrated academic achievement in the High Average to Superior range in basic reading, writing, and math skills; their application in reading comprehension, written expression, and math problem solving; and speed of application. Fred scored beyond 85% of students his age on the Broad Reading cluster, which incorporates the skills of word recognition, reading speed, and comprehension of brief passages. Reading comprehension was in the Average to High Average range, surpassing 66% of his age peers. His ability to read both phonetically regular and irregular words exceeded 82% of his age group. Fred's reading speed also fell into the High Average range, surpassing 78% of age peers.

On the WJ IV ACH writing tests, Fred scored beyond 86% of students his age on the Broad Written Language cluster, which incorporates written content, spelling, and writing speed. For writing content, Fred wrote individual sentences in response to varied prompts. His score was in the High Average range, surpassing 85% of his peers. Fred's spelling was Average, exceeding 62% of age peers. On a test of writing speed, writing short, simple sentences as fast as possible, Fred exceeded 93% of students his age.

On the Broad Mathematics cluster, Fred surpassed 93% of students his age. This cluster includes calculation, speed of math fact retrieval, and problem solving. In relation to students his age, his work on computation problems exceeded 94%, his rapid recall of basic math facts exceeded 79%, and his application of these skills to practical (word) problems exceeded 96%. His score on a test of math reasoning, requiring him to complete increasingly complex number patterns, surpassed 95% of his peer group.

In general, Fred surpassed 85% of age peers on overall academic skills (word identification, spelling, computation), 95% of age peers on the application of those skills (comprehension of brief passages, writing individual sentences, solving practical math problems), and 85% on the speed with which he could read and comprehend brief sentences, write simple sentences, and recall basic math facts.

Behavior and Emotional Rating Scales

In general, Fred's self-ratings on the BASC-2 Self-Report of Personality did not indicate “at-risk” or “clinically significant” concerns in most areas, including hyperactivity, attitude towards school, attention problems, anxiety, atypicality, depression, aggression, sense of inadequacy, and overall adaptive behavior (relations with parents, self-esteem, and self-reliance). The same pattern was observed in his mother's BASC-2 ratings, with one exception. The results are detailed for Fred and his mother in the Psychometric Summary.

Behavioral and emotional ratings scales completed by Fred and his mother indicated a general pattern of no significant problems in the majority of the domains measured. For example, Fred indicated that he “often” or “almost always” is good at making decisions; he is proud of his parent (mother); he is a dependable friend; and he listens to people when they talk to him. Fred also indicated that it is “true” he is a good person; he feels good about himself; and if he has a problem he feels he can usually work it out.

Fred's self-ratings (Figure 2.5) indicated concern in two areas, however, and these were attitude towards teachers and interpersonal relationships. This appeared to be associated with less than positive interpersonal experiences with peers and classmates in his current school environment. For example, Fred indicated that his classmates can be somewhat critical of him, and they do not tend to like or understand him. It also was the case that Fred feels a number of his teachers can misjudge his motivations and behavior. In general, Fred indicated that teachers do not generally understand him, they can be overly rigid, and in terms of fairness and “even-handed” treatment he perceives them as “unfair.”

Figure illustrating T scores on the BASC-2 self-report. T score across various attributes such as attitude, social stress, depression, self-esteem, and so on is plotted, where Fred rated his attitude toward teacher at a T score of 80 and interpersonal relations at a score of 29.

Figure 2.5 T Scores on the BASC-2 Self Report

Fred's mother's ratings (Figure 2.6) supported the impression of an intelligent, creative, and talented child who does not show significant emotional and behavioral problems at home. The only area of concern for Fred's mother was that he “sometimes” has physical complaints, headaches, or pains and sometimes he expresses fear of getting sick. In fact, most the items of this subscale were rated “sometimes,” and this resulted in the overall score rising just above the level to indicate “at-risk” level problems. This was the only area to be somewhat elevated. All other behavioral-emotional domains were nonsignificant and reflected areas of behavior-emotional strength on the Parent Rating Scale. For example, Fred “often” or “almost always” is creative, adjusts well to changes in family plans, makes friends easily, gives good suggestions for solving problems, and tries to bring out the best in others.

Figure illustrating T scores for BASC-2 parent rating. T score across various attributes such as hyperactivity, aggression, anxiety, withdrawal, leadership, and so on is plotted, where Fred's mother rated most of the attributes as “sometimes”, resulting in the overall score rising just above the level to indicate “at-risk” level problems.

Figure 2.6 T Scores for BASC-2 Parent Rating

This concludes the Interpretation section of Fred's psychometric test results. Please note that the Conclusions and Recommendations of this evaluation appeared in the beginning of this report.

Psychometric Summary

Wechsler Intelligence Scale for Children, Fifth Edition, Verbal Comprehension Index

Subtest Scaled Score Percentile Standard Score
Similarities Subtest 18 99
Vocabulary Subtest 12 75
VERBAL COMPREHENSION COMPOSITE 96 127

Woodcock-Johnson IV Tests of Cognitive Abilities

Table of Scores

CLUSTER/Tests W GE SS PR RPI Proficiency
Gf-Gc COMPOSITE 523 13.2 126 96 99/90 Advanced
Oral Vocabulary 520 12.5 121 92 98/90 Advanced
Number Series 528 >14.0 122 93 99/90 Advanced
General Information 519 11.4 115 85 98/90 Advanced
Concept Formation 525 >14.7 120 91 99/90 Advanced
COMP-KNOWLEDGE (Gc) 520 11.8 119 89 98/90 Advanced
Oral Vocabulary 520 12.5 121 92 98/90 Advanced
General Information 519 11.4 115 85 98/90 Advanced
FLUID REASONING (Gf) 526 >14.4 125 95 99/90 Advanced
Number Series 528 >14.0 122 93 99/90 Advanced
Concept Formation 525 >14.7 120 91 99/90 Advanced
S-TERM WORK MEM (Gwm) 524 14.9 124 94 99/90 Advanced
Verbal Attention 527 >17.9 126 96 99/90 Advanced
Numbers Reversed 521 13.0 115 85 98/90 Advanced
L-TERM RETRIEVAL (Glr) 503 13.0 109 73 94/90 Average
Story Recall 497 5.7 99 48 89/90 Average
Visual-Auditory Learning 510 >13.7 113 81 96/90 Average to Advanced
VISUAL PROCESSING (Gv) 516 >17.9 125 95 98/90 Advanced
Visualization 518 >17.9 127 96 99/90 Advanced
Picture Recognition 513 >17.9 115 85 97/90 Average to Advanced
NUMBER FACILITY 522 9.6 116 86 98/90 Advanced
Numbers Reversed 521 13.0 115 85 98/90 Advanced
Number-Pattern Matching 524 8.0 112 79 98/90 Advanced
PERCEPTUAL SPEED 529 8.0 112 79 98/90 Advanced
Letter-Pattern Matching 534 8.0 110 74 98/90 Advanced
Number-Pattern Matching 524 8.0 112 79 98/90 Advanced
COGNITIVE EFFICIENCY 527 9.5 116 85 98/90 Advanced
Letter-Pattern Matching 534 8.0 110 74 98/90 Advanced
Numbers Reversed 521 13.0 115 85 98/90 Advanced
COG EFFICIENCY (Ext) 526 10.4 120 91 98/90 Advanced
Verbal Attention 527 >17.9 126 96 99/90 Advanced
Letter-Pattern Matching 534 8.0 110 74 98/90 Advanced
Numbers Reversed 521 13.0 115 85 98/90 Advanced
Number-Pattern Matching 524 8.0 112 79 98/90 Advanced
Note: Proficiency Ranges (e.g., Limited, Average, Advanced) are associated with Relative Proficiency Indexes (RPI), not with standard scores. The RPI is a mathematical prediction, based on the individual student's score and those of the comparison group (age or grade) that indicates the probability of the individual's success on a task when his or her age-/grade-peers have a 90% chance of success.

Woodcock-Johnson IV Tests of Achievement Table of Scores

CLUSTER/Tests W GE SS PR RPI Proficiency
READING 522 9.7 116 86 98/90 Advanced
Letter-Word Identification 526 9.6 114 82 98/90 Advanced
Passage Comprehension 518 9.7 114 82 98/90 Average to Advanced
BROAD READING 527 9.5 115 85 98/90 Advanced
Letter-Word Identification 526 9.6 114 82 98/90 Advanced
Passage Comprehension 518 9.7 114 82 98/90 Average to Advanced
Sentence Reading Fluency 537 9.3 112 78 99/90 Advanced
READING COMPREHENSION 507 7.4 106 66 94/90 Average
Passage Comprehension 518 9.7 114 82 98/90 Average to Advanced
Reading Recall 496 4.4 93 33 84/90 Average
READING FLUENCY 520 7.6 107 69 96/90 Average to Advanced
Oral Reading 502 5.3 97 42 86/90 Average
Sentence Reading Fluency 537 9.3 112 78 99/90 Advanced
READING RATE 528 8.2 110 75 98/90 Advanced
Sentence Reading Fluency 537 9.3 112 78 99/90 Advanced
Word Reading Fluency 518 7.5 107 68 97/90 Average to Advanced
MATHEMATICS 534 13.0 126 96 99/90 Advanced
Applied Problems 534 >17.9 125 96 99/90 Advanced
Calculation 534 13.0 123 94 99/90 Advanced
BROAD MATHEMATICS 533 13.0 122 93 99/90 Advanced
Applied Problems 534 >17.9 125 96 99/90 Advanced
Calculation 534 13.0 123 94 99/90 Advanced
Math Facts Fluency 532 8.6 112 79 98/90 Advanced
MATH CALCULATION SKILLS 533 9.8 119 89 99/90 Advanced
Calculation 534 13.0 123 94 99/90 Advanced
Math Facts Fluency 532 8.6 112 79 98/90 Advanced
MATH PROBLEM SOLVING 529 >17.9 125 95 99/90 Advanced
Applied Problems 534 >17.9 125 96 99/90 Advanced
Number Matrices 524 >16.4 118 88 99/90 Advanced
WRITTEN LANGUAGE 517 8.5 111 78 97/90 Average to Advanced
Spelling 516 6.9 104 62 94/90 Average
Writing Samples 518 13.0 116 85 98/90 Advanced
BROAD WRITTEN LANGUAGE 519 10.1 116 86 97/90 Average to Advanced
Spelling 516 6.9 104 62 94/90 Average
Writing Samples 518 13.0 116 85 98/90 Advanced
Sentence Writing Fluency 522 >17.9 122 93 98/90 Advanced
WRITTEN EXPRESSION 520 17.0 123 93 98/90 Advanced
Writing Samples 518 13.0 116 85 98/90 Advanced
Sentence Writing Fluency 522 >17.9 122 93 98/90 Advanced
ACADEMIC SKILLS 525 9.1 116 85 98/90 Advanced
Letter-Word Identification 526 9.6 114 82 98/90 Advanced
Spelling 516 6.9 104 62 94/90 Average
Calculation 534 13.0 123 94 99/90 Advanced
ACADEMIC FLUENCY 530 9.9 115 85 99/90 Advanced
Sentence Reading Fluency 537 9.3 112 78 99/90 Advanced
Math Facts Fluency 532 8.6 112 79 98/90 Advanced
Sentence Writing Fluency 522 >17.9 122 93 98/90 Advanced
ACADEMIC APPLICATIONS 523 13.0 124 95 99/90 Advanced
Applied Problems 534 >17.9 125 96 99/90 Advanced
Passage Comprehension 518 9.7 114 82 98/90 Average to Advanced
Writing Samples 518 13.0 116 85 98/90 Advanced

Behavior Assessment Scale for Children, Second Edition

Interpretation of BASC-2 T-scores:

  1. 60–69 constitute the “elevated” or “at-risk” range
  2. 70 and above suggest severe or “clinically significant” problems

Adaptive Skills:

  1. Below 40 indicates “at risk”
  2. 30 and below: severe or clinically significant problems

Self-Report of Personality (FRED)

Parent Rating Scale (Mother)

Note to Reader

Skylar illustrates a classic profile of a student with severe dyslexia and dysgraphia, as well as ADHD. In this case, the Woodcock-Johnson IV Tests of Cognitive Abilities (WJ IV COG) were effective for demonstrating patterns of strengths and weaknesses (intra-individual variations) that are often seen in individuals with dyslexia. Despite years of intensive interventions in reading and writing, these skills are still considerably lower than his oral language and reasoning abilities, represented by the Gf-Gc Composite. Because of the interventions, he has advanced sound blending and segmentation abilities. In contrast, he has delayed automaticity and slow processing on all tasks requiring a rapid response to visual symbols (e.g., processing speed, rapid automatized naming). In addition, Skylar demonstrates a pattern typical of individuals with reading disabilities: a low score on the WJ IV Cognitive Efficiency cluster (a combination of short-term memory and processing speed), and a discrepancy between his knowledge and reasoning and his processing efficiency. With individuals with dyslexia, what starts out as a problem in the acquisition and accuracy of basic skills often evolves into a problem in the ease and rate of application of these skills.

Furthermore, when interpreting scores for specific learning disability (SLD) classification, thoughtful consideration of the discrepancies may make the difference as to whether a student who needs services actually receives services. Again, Skylar's case is illustrative. When scored according to grade norms, Skylar does not exhibit any ability–achievement discrepancies; however, if age norms are used, he does—between the general intellectual ability (GIA) score and Broad Written Language. Although many of Skylar's abilities are above his levels of achievement, one set of scores shows a discrepancy and the other does not. Accurate identification of SLD cannot be simplified to the mere existence (or lack thereof) of an ability–achievement discrepancy. One also has to consider why his GIA is so low. Skylar's processing deficits and slow performance on all time-limited tasks are included in, and thus have lowered, his GIA score. Consequently, an ability–achievement discrepancy is less likely to be evident. In contrast, when his Gf-Gc Composite is compared with his reading and writing performance, significant discrepancies exist.

This case also illustrates that even with the very best of help, individuals with dyslexia and dysgraphia rarely catch up with peers. Although some of Skylar's reading scores might be described by some as being “within normal limits” (not below the 16th percentile), one must consider the amount of time and effort that went into achieving these scores. Does a score that is just “within normal limits” indicate that a child does not have a disability when it has taken years of constant, intensive, and specialized tutoring to obtain them? Now, even with all of this tutoring, Skylar's reading and writing are still slow and effortful, bringing him to a difficult decision: Should he quit soccer, in which he excels, so that he can have enough time to finish his homework? Unlike his friends, he cannot do both, because it takes him too long to complete his assignments.

Skylar is receiving numerous accommodations in school. Although these accommodations have allowed him to earn good grades, they are not helping him to improve his reading and writing skills. A balance needs to exist so that a student is not being “over-accommodated.” Skylar needs to spend time doing authentic reading and writing activities, or these skills will not improve. At the same time, he needs accommodations so he can demonstrate what he really knows and understands.

One last, important consideration is that in the past, several of Skylar's teachers have perceived him as lazy because he is so capable in many other ways. A common remark has been, “You could do it, if you would just try.” The reality is that, when tasks involve reading and writing, Skylar is trying—and he is expending far more effort than others to accomplish the same or a poorer result. The following analogy applies: “It was as if he were driving in a NASCAR race in first gear while everyone else was cruising along in fifth gear” (Lindstedt & Zaccariello, 2008). Thus, both teachers and parents need to understand that students with dyslexia or dysgraphia cannot work at the rate of normally achieving peers or produce the same quantity of work, nor should they be expected to do so.

A decision was made to use relative proficiency indexes (RPI) and percentile ranks within the body of the report, because they are easier for teachers and parents to understand than standard scores. Moreover, RPIs often give a more realistic representation of a student's proficiency in a skill than do the standard scores and percentile ranks. Because the RPI does not vary with the standard score, it shows just how far behind his peers Skylar is regarding the difficulty level of tasks that he can handle. In addition, an effort was made to minimize a discussion of all of the scores to make the information more understandable for his parents and teachers and to explain the general findings and conclusions.

Diagnostic Evaluation Report

Name: Skylar Swanson
Date of Birth: 5/27/2003
Age: 11–11
Date of Testing: April 14–16, 2015
School: Paul E. Lonigan Middle School
Grade: 6.8

Reason for Referral

Skylar is currently a sixth-grade student at Paul E. Lonigan Middle School in Huntsville, TX. He was referred for an evaluation by his mother, Ms. Arlene Swanson, who is a history teacher at a local high school. Skylar was last evaluated in first grade, and Ms. Swanson wanted information regarding his present academic needs, as well as recommendations to help Skylar continue to be successful in school. His parents also want to find ways to ensure that Skylar maintains a positive attitude toward school. Skylar started saying in kindergarten that he hated school and continued comments similar to these until the fifth grade.

Background Information

Skylar lives at home with his biological mother and father, who recognized difficulties in his literacy development early. Ms. Swanson reported that her father had dyslexia, and despite being very successful, he could not spell even simple words. At 18 months, Skylar enjoyed listening to books and looking at the pictures, but he averted his gaze whenever printed text was presented. As a young child, he did not enjoy playing with chalk or crayons.

Skylar attended a private Christian school for preschool and part of kindergarten. During his early educational development, Skylar's parents and teachers were concerned about his ability to learn letters and sounds, write uppercase and lowercase letters, copy and trace shapes, and write his name. Concerns regarding fine- and gross-motor development were also noted. Skylar had trouble catching and throwing balls, and learning to tie his shoes. In addition, he was terrified of his kindergarten teacher and afraid to go to school. The teacher told his parents that if Skylar remained at the school, he would be retained.

Mid-kindergarten, due to concerns regarding attention, Skylar was tested by Dr. Ramon Flynn and was diagnosed with attention deficit/hyperactivity disorder (ADHD). Skylar began medical treatment with Concerta (18 mg), with good results. Dr. Flynn recommended that Skylar receive neurofeedback treatment based on the results of a quantitative electroencephalogram (an excess of delta waves in the prefrontal and occipital areas). Skylar received neurofeedback for the remainder of the year and during the summer before entering first grade. Dr. Flynn also recommended that Skylar's school placement be changed immediately because of his anxiety and fear. Skylar was transferred to public school and completed kindergarten at Ricardo Elementary School.

In first grade, despite the continued use of Concerta, the use of flash cards for sight-word study each evening at home, and supplemental academic assistance, Skylar continued to struggle with reading words, writing legibly, organization, and completing tasks in a timely matter. A recommendation was made for a comprehensive evaluation at the Ashmore Child Development Center. The results of this evaluation (2010) indicated that Skylar had average intellectual and oral language abilities with a weakness in processing speed (WISC-IV: Verbal Comprehension Index, 104; Perceptual Reasoning Index, 106; Working Memory Index, 102; and Processing Speed Index, 88; CELF-4 Screening, meets age level criterion). On academic testing, he was unable to recite the alphabet, write the alphabet in sequence, or form letters correctly. Despite average performance in mathematics (WJ III ACH Calculation standard score of 108), his oral reading quotient on the GORT-4 was a standard score of 70. He was diagnosed with developmental dyslexia, developmental coordination disorder, ADHD, and as being at risk for a disorder in written expression. After this evaluation, he was immediately enrolled in a resource reading program at his school two to three times per week. The program was specifically designed for children with dyslexia and used a multisensory teaching approach.

Skylar has received some type of reading and writing support throughout elementary school. In second grade he began a weekend intensive tutoring program (2–3 hours per day) with Dr. Richman from the Huntsville Reading Institute. His parents followed up with the tutoring each day for 45 minutes after school and saw that he was making progress with his reading and spelling skills. In the summer between second and third grade, the family moved temporarily to Houston so that Dr. Richman could work with Skylar for two hours daily. During this period, Ms. Swanson noticed that his fluency improved. In fourth grade, weekend work with Dr. Richman continued along with the supplementary daily home program. In fifth grade, Skylar was enrolled in the district's Scottish Rite reading program, which provided daily intervention. Although Skylar has clearly made progress with all of the additional support and specialized tutoring, his parents and teachers have expressed concerns about his slow reading rate, poor organizational skills, written expression that does not reflect his level of knowledge, extremely poor spelling, inability to complete tasks within time limits, and lack of automaticity with math facts.

Last August, Skylar transferred to Paul E. Lonigan Middle School for sixth grade. So far, he has been successful in most of his classes with the provision of accommodations, such as listening to books on CD and dictating his writing assignments. He has continued to take Concerta, which has been increased to 54 mg. He is currently enrolled in the Scottish Rite Literacy Program and meets his Accelerated Reader goal each six weeks. On recent testing on the Gates-MacGinite Reading Test, Level 7/9, he obtained grade equivalencies of 6.3 on the vocabulary section and 7.4 on the comprehension section. His language arts teacher reported that Skylar works hard, completes his assignments, and turns in his work. He works well with other students and is a pleasure to have in class. His science teacher described Skylar as a great student who turns in quality work but needs extra time to complete his work during class. Because he often loses his place, she also transfers his test answers from the test record to a bubble sheet. Although the resource teacher has only seen Skylar a few times, she reported that she is extremely concerned about his poor organization of the materials in his binder. Skylar continues to fail his weekly “mad minute” math test, composed of 40 simple math facts, and he takes two days to complete his tests in science and language arts. His teachers prefer that he finishes the tests in their classrooms, rather than taking the tests to complete in a resource setting. In describing his current school setting, Skylar told his mother that his previous school was like a “rock,” whereas his present school is like a piece of “gold.”

Despite a good year, Skylar still becomes frustrated from time to time regarding his difficulties in reading and spelling and completing his school work. Ms. Swanson expressed concern regarding the fact that Skylar spends hours and hours each evening and most weekends on homework. He continues to struggle with reading and spelling, and lately, he has started having difficulty with math concepts. In class, he is frequently the last one to finish his math assignments, which often makes him miss or be late for his physical education class. Ms. Swanson further noted that Skylar has always had difficulty demonstrating his knowledge on timed measures and completing tasks within a time frame. As long as time constraints are removed, Skylar does much better in all academic areas, as well as in completing chores at home. Ms. Swanson was concerned that in the past, a few of Skylar's teachers have attributed his slow or inadequate performance to laziness or lack of motivation. Ms. Swanson reported that Skylar has recently elected to no longer play soccer, because with games and practice, he is not able to complete his homework. For the past year, Skylar has been learning to do his written work on a computer. Although his writing has improved in both expression and legibility, his keyboarding is still inefficient.

Skylar does not receive special education services, but he currently has a 504 Accommodation plan that provides extra assistance with high-stakes testing and oral administration of classroom tests. He also receives library books on CD via Recordings for the Blind and Dyslexic and listens to books for his Accelerated Reading (AR) program, rather than reading them. Recently, it was recommended that he be discontinued from the AR program, because his comprehension scores are high after listening to lengthy books.

Skylar has taken and passed the Texas Assessment of Knowledge and Skills (TAKS) reading with bundled accommodations (specific procedures including extended time, testing over two days, reading of proper nouns, and small group administration designed for the evaluation of students with dyslexia), TAKS math with oral administration, and TAKS writing with transcription. Because he frequently loses his place when transferring his gaze from the test booklet to the Scantron answer sheet, the answers were bubbled in for him. In addition, he has used a digital voice recorder for classroom writing tasks.

Skylar enjoys traveling, nature, and drawing. He is creative with drawing projects despite fine-motor issues. He creates elaborate ship designs to replicate famous ships. He also enjoys going to the library to find books of interest. Skylar was currently reading The Lost Ships of Robert Ballard. He commented that the book was really hard to read, but even so, it was “really worth it.”

Tests Administered

  1. Woodcock-Johnson IV Tests of Cognitive Ability (WJ IV COG)
  2. Woodcock-Johnson IV Tests of Oral Language (WJ IV OL)
  3. Woodcock-Johnson IV Tests of Achievement (WJ IV ACH)
  4. Rapid Automatized Naming and Rapid Alternating Stimulus Tests (RAN/RAS) (numbers and letters)
  5. Test of Written Language—4 (TOWL-4) (Story Only)
  6. Spelling list of irregular words

Testing Observations

Throughout testing, Skylar was extremely cooperative and motivated to do well. He attempted all items and persisted as items increased in difficulty. On many occasions, he had trouble processing lengthy directions and would ask for the instructions to be repeated. In addition, on several writing tests he required additional time to complete his written responses.

Tests Results

Skylar's performance is compared with that of his grade-peers using percentile ranks.

Percentile Rank Range Percentage of People Verbal Label
98th & above 2 Very Superior
91st to 97th 7 Superior
75th to 90th 16 High Average
25th to 74th 50 Average
9th to 24th 16 Low Average
3rd to 8th 7 Low
2nd & below 2 Very Low

His proficiency on specific tasks is described by relative proficiency index (RPI) levels. The RPI indicates the difficulty level a student would be expected to be able to manage in tasks similar to those used in the test. The numerator indicates the expected level of success on a task when a typical age-peer would be 90% proficient. For example, Skylar's RPI of 11/90 on the Spelling test predicts that when the average student of age 11–11 would be 90% successful in spelling a list of age-appropriate words, Skylar would be only 11% successful, indicating very limited proficiency. Descriptions of Skylar's levels of proficiency are based on his RPI scores.

RPI Range Level of Proficiency Instructional Implications
100/90 Very Advanced Extremely easy
100/90 to 98/90 Advanced Very easy
98/90 to 95/90 Average to Advanced Easy
95/90 to 82/90 Average Manageable
82/90 to 67/90 Limited to Average Difficult
67/90 to 24/90 Limited Very difficult
24/90 to 3/90 Very Limited Extremely difficult
3/90 to 0/90 Extremely Limited Nearly impossible

Percentile ranks and descriptive ratings are provided for both the RAN tests and TOWL-4 within this report.

Cognitive Abilities

Based on the tests of the WJ IV COG, Skylar's General Intellectual Ability (GIA) score suggested average proficiency (PR = 34; RPI = 85/90). Because of the wide variation in Skylar's scores on the tests that make up the GIA, this score does not provide a good estimate of Skylar's overall intelligence or learning potential. In contrast on the Gf-Gc Composite, a measure of knowledge and reasoning abilities, Skylar's illustrated average to advanced proficiency (PR = 87; RPI = 97/90). A brief summary of cognitive strengths and weaknesses is presented in the following sections.

Strengths

As noted, Skylar demonstrated advanced proficiency on measures of comprehension-knowledge (acquired knowledge, language comprehension, and vocabulary) (Gc), and measures of fluid reasoning (inductive and deductive reasoning) (Gf). On the WJ IV OL, he also displayed advanced proficiency on the Phonetic Coding cluster, which includes two measures of phonological awareness (Sound Blending, RPI = 100/90; PR = >99) and Segmentation, RPI = 98/90; PR = 97), which suggests good ability to blend and synthesize sounds when using phonics to pronounce words. These high scores are most likely attributable to the extensive phonics instruction that he has received in school, as well as in the summer tutoring.

Weaknesses

Skylar had limited or very limited proficiency on all tasks requiring speeded responses. His lowest score was on the WJ IV COG Perceptual Speed cluster, RPI = 8/90; PR = 5, which measures the ability to rapidly identify similarities and differences among visual symbols, such as letters or numbers. Skylar's slow response times suggest that he will find it very difficult to manage age-level tasks requiring rapid recognition and comparison of symbols, such as letter and number patterns. Skylar also had limited proficiency on the Short-Term Working Memory cluster (RPI = 57/90). A memory weakness of this type can affect all tasks involving following verbal directions. His proficiency on the Cognitive Efficiency cluster, which is a combination of perceptual speed and working memory tests, was also very limited (RPI = 14/90; PR = 6).

In addition, his performance on all timed measures was considerably lower than his performance on untimed measures. For example, Skylar had advanced proficiency on the WJ IV COG Oral Vocabulary test, which measures word knowledge and linguistic reasoning, RPI = 98/90; PR = 90, but limited proficiency on Rapid Picture Naming, which requires the quick naming of (pictured) common objects, RPI = 76/90, PR = 32.

Skylar also demonstrated weaknesses on RAN/RAS letter and number naming, both strong predictors of reading failure. These rapid naming tasks required Skylar to name a series of visually presented digits, and then a set of letters as accurately and quickly as possible. Both of Skylar's scores were in the Low range, placing him within the lowest 6% of his age peers.

Test Age Equivalent Grade Equivalent Percentile Rank Standard Score Descriptive Rating
Numbers 7–0 2.0 6 77 Poor
Letters 6–6 1.4 6 77 Poor

Overall, these results suggest that Skylar has a specific learning disability in perceptual speed that affects his performance on all timed tasks, as well as his ability to acquire fluency in his application of academic skills. Weaknesses were also noted in working memory, his ability to hold and manipulate information in memory.

Academic Achievement

Skylar's academic skills were assessed in reading, writing, and math, using selected tests of the Woodcock-Johnson IV Tests of Achievement (WJ IV ACH) and the story-writing subtest of the Test of Written Language, Fourth Edition (TOWL-4). In addition, performance was analyzed on his ability to spell a list of words with irregular spelling patterns.

Reading and Spelling

The WJ IV ACH was used to assess four reading skills: word identification, word attack, reading rate and fluency, and reading comprehension. Skylar's proficiency on the reading tests ranged from Extremely Limited to Limited. His lowest scores were on Sentence Reading Fluency test (RPI= 1/90; PR = 5, and the Word Reading Fluency test (RPI = 4/90; PR = 12).

Skylar also had difficulty applying phonic skills. His score on the Word Attack test was RPI = 55/90; PR = 14, indicating that his proficiency in reading nonsense words that follow regular phonics and orthographic patterns was limited. He made errors on medial vowel sounds (e.g., ip for ep), a skill typically mastered in first grade. When a person encounters a word he has never seen, it is a nonsense word until he pronounces and, hopefully, matches it to a word in his oral vocabulary. Consequently, if Skylar has not already memorized a word, he has limited ability to sound it out.

Although Skylar was able to pronounce real words with some accuracy on the Letter-Word Identification test (RPI = 14/90; PR = 9), he lacked confidence in his answers, and his responses were often slow and nonautomatic. He had difficulty pronouncing words that were longer than three syllables (e.g., elevator). Although it is clear that Skylar has had a great deal of help in learning to read, he still does not pronounce words accurately or quickly.

As is typical of individuals with dyslexia, Skylar obtained a low score on the Spelling test, with an RPI = 14/90; PR = 9. Qualitative analysis of his responses indicated that his knowledge of spelling rules and orthographic patterns is very limited. Skylar had the most difficulty spelling irregular words that do not conform to English spelling rules. Generally, he spelled them the way the word sounds rather than the way it looks (e.g., ses for says, enney for any, wher for where, becose for because). These types of errors are present in all of his writings, including class papers and stories. Figure 2.7 shows Skylar's spelling on a list of irregular words. Of interest is that he spelled “island” and “ocean” correctly, most likely because of his interest in ships and the sea. In another writing sample, he spelled the word “see” (meaning “to look”) as “sea.”

Figure depicting a list of words written by Skylar.

Figure 2.7 Skylar's Spelling

Written Language

Skylar's story (Figure 2.8) and scores on this component of the TOWL-4 follow.

Test Percentile Rank Scaled Score/SS Descriptive Rating
Contextual Conventions 24 8 Average
Story Composition 37 9 Average
Spontaneous Writing 32 93 Average
Figure depicting a page of Skylar's story.

Figure 2.8 Skylar's Story

Although the content of Skylar's story was age appropriate and the ideas were cohesive, he only produced four complete sentences within a 15-minute period. His writing was slow and laborious. He used correct punctuation but made a few errors in syntax and sentence structure. Further examination and discussion of classroom writing samples indicates that Skylar can organize and structure his ideas when writing, but the actual production requires a great deal of effort, as does deciding how to spell words.

Mathematics

Skylar's mathematics skills were assessed in three areas: math computation, automaticity of math facts, and practical application of math knowledge. Both RPIs and percentile ranks indicated that math computation, math fluency, and practical application were all typical for his grade (Broad Mathematics RPI = 92/90; PR = 54). Skylar did, however, have difficulty with problems involving adding and subtracting with money.

Gf-Gc Composite Comparison

When Skylar's Gf-Gc Composite is compared with his other cognitive abilities, he has significant weaknesses in Short-term Working Memory, Perceptual Speed, and Cognitive Efficiency. When the Gf-Gc Composite is compared with his academic performance, he has significant weaknesses in all aspects of reading and writing performance.

Diagnostic Impressions

Based on past diagnoses, as well as the current test results, Skylar displays a pattern of strengths and weaknesses that is consistent with the diagnosis of a specific learning disability. His verbal comprehension and reasoning abilities are far more advanced than his current performance levels in reading and writing. Skylar has significant weaknesses in several aspects of memory: working memory, his ability to recall and rearrange verbal information, and his ability to visualize and recall orthographic patterns (letter combinations) needed for fluent reading and spelling. Thus, even though he has seen a word numerous times (e.g., where), he does not recall the correct spelling pattern. These weaknesses in memory make it difficult for him to listen to and follow lengthy, complex directions and to recall and apply previously learned phonics rules and orthographic patterns during reading and spelling tasks; they can also contribute to possible errors that appear to be careless on math problems. Moreover, his significant weakness in speed of processing symbols has further affected his automaticity with reading and spelling. The combination of these areas of weakness has severely impaired his ability to acquire age-expected literacy despite significant amounts of specialized instruction.

The results of this evaluation indicate that Skylar has specific reading and writing disabilities (dyslexia and dysgraphia) and will require accommodations and interventions to promote skill development and academic success. Dyslexia is apparent in his slow word perception and poor spelling, and dysgraphia is evident in his poor spelling and fine-motor control, as well as his slow speed of written production. In addition, his ADHD exacerbates these difficulties. Care must be taken to ensure that teachers do not initially perceive Skylar as being “lazy” or “unmotivated.” The reality is that he is a hard-working and motivated student who expends considerably more time and effort to accomplish what other students can do quickly with relative ease.

Educational Recommendations

Technology and Accommodations

  1. Provide Skylar with direct training in keyboarding skills and use of a word processing program. One excellent program is Mavis Beacon Teaches Typing.
  2. For lengthy reading assignments, provide Skylar with recorded versions. When he is listening to books on a CD, however, encourage Skylar to follow along with a printed copy. When possible, select books that are written at Skylar's independent reading level, so that the process of tracking the print as he listens will help him to increase his rate of word perception and reading speed.
  3. Do not penalize Skylar for misspelled words on any writing assignments. Instead, he should be encouraged to try and correct any errors when editing by using a hand-held or computer spell-checker.
  4. Skylar is likely to have difficulty following complex classroom and test directions. Ensure that he understands what he is supposed to do before he begins the task or test.
  5. Encourage Skylar to ask for clarification of directions whenever he is unsure of what to do.
  6. Although appropriate accommodations are being made for Skylar, it is equally important that he continue to spend time engaged in reading and writing activities. Although his performance will improve when he listens to information that others are reading and dictates his responses or ideas instead of writing them, these activities will not increase his reading and writing skills.

Reading and Spelling

  1. Skylar needs continued instruction in an instructional program in basic reading and spelling skills that is systematic in introduction, practice, and reinforcement of phoneme-grapheme relationships, sight words, syllabication rules, structural analysis, and spelling rules. In a systematic program, skills are presented in graduated steps, from simple to complex, with students achieving mastery before the next skill is introduced. Reading and spelling skills should be taught simultaneously, so that Skylar learns how to spell every phonics generalization and word structure that he learns how to read. An example of such a program is the Wilson Reading System http://www.WilsonLanguage.com. Skylar will need individual, intensive, daily instruction from a learning disability or reading specialist with training in this program.
  2. Skylar should not be exited from Accelerated Reader support program; instead, he should be encouraged to read shorter books, rather than listen to books on CDs. Although his listening comprehension is average to advanced, this is not an accurate reflection of his present level of reading skill. Skylar has been receiving many accommodations (e.g., text read orally, use of a scribe) that have allowed him to be successful within his classes and demonstrate his knowledge, but when these accommodations are used, the results are not accurate reflections of his current reading and writing abilities, which are still far below both ability and grade level. Although Skylar is likely to need all of these accommodations in the future to fully demonstrate his knowledge, for now it is important to help Skylar improve his reading and writing skills.
  3. Skylar would benefit from further instruction in spelling. Ideally, spelling should be taught along with reading in the same program. If reading tutoring is not available, however, an appropriate program to help Skylar with the development of his spelling skills would be Spellography (www.sopriswest.com).
  4. Provide Skylar with a personalized, alphabetized list of high-frequency words (e.g., would, they, any, said) that he has not mastered for spelling. Encourage Skylar to refer to this list at any time he is writing.
  5. Because of Skylar's low perceptual speed, he will need supplementary techniques to help him automatize orthographic skills such as quickly recognizing common letter patterns and whole words. One technique that would be effective is the use of reading speed drills using any word elements on which Skylar needs extra practice as they come up in his reading program (e.g., sight words, syllable structures, letter patterns, and phonic elements). (See Speed Drills for Reading Fluency and Basic Skills in Section IV.)
  6. Do not use separate lists for reading and spelling. Focus Skylar's spelling practice on the phonics patterns he is learning within the reading program. Use a whole-word approach to teach spelling only for high-frequency sight words, and only as they come up in the systematic reading program. For whole-word spelling, use a multisensory method to incorporate visual, oral/aural, and kinesthetic-tactile components. (See Multisensory Study Method for Irregular Words in Section IV.)

Mathematics

  1. Skylar requires further instruction in adding and subtracting various amounts of money, which can be reinforced in both the school and home environment. Encourage Skylar to practice making change and estimating the amount of money required to complete transactions.
  2. Although Skylar can continue to practice and increase his automaticity with math facts, the more important goal is that he continues to develop his conceptual knowledge. Place greater instructional emphasis on building problem-solving skills, rather than drilling on math facts. Skylar also may use various computer software or websites to practice math facts. Numerous flash games can be found on the website www.coolmath-games.com and, for multiplication, www.multiplication.com.

Homework and In-class Assignments

  1. Skylar will require extra time to complete most in-class assignments and tests. If extra time cannot be made available, reduce the amount of work assigned.
  2. For homework, specify the amount of time Skylar should spend on the assignment rather than the amount of work that he should complete. What a typical student can accomplish in half an hour is likely to take Skylar three times longer. Set a realistic time to be spent on an assignment and permit Skylar to turn in what he could complete during that time frame. Another option would be to shorten the assignment, such as having him complete only the even-numbered problems of a math worksheet or write a one-page rather than a three-page report.
  3. Do not require Skylar to miss or be late to his physical education class or any other special classes (e.g., music, art) because he has not finished his assignments. Shortening his assignments should provide him with the necessary time to complete them.

Psychoeducational Evaluation

Student: Jennifer Challis
Parents: Fran and Theodore Challis
Date of Birth: April 7, 2003
Age: 12–3
Date of Testing: June 24, 26, 31, 2015
School: Eklund Middle School
Grade: 6.9

Reason for Referral

Jennifer has just completed her first year in the self-contained GATE program at Eklund Middle School. The school questions whether she should have an IEP because she does not want to be pulled from class for special help and because she is doing well in all of her classes except for math. The purpose of this evaluation was to update her cognitive, language, and academic strengths and weaknesses to ascertain her current need and eligibility for special education.

Background Information

Jennifer lives with her mother, her older biological sister, Astrid, and one older adopted brother, William. Ms. Challis and Jennifer's father are divorced.

Familial History of Learning Abilities/Disabilities

Jennifer's sister, Astrid, has long been classified as twice exceptional—gifted with a specific learning disability. She has also been diagnosed with attention deficit/hyperactivity disorder (ADHD). Her brother, William, is also classified as gifted. William has significant psychological challenges secondary to his experiences before his adoption into the Challis family and likely genetically. Although this has no bearing in terms of biological predictors for Jennifer, it has impacted the environment of the home. According to Ms. Challis, the girls' father has characteristics of both learning disabilities and ADHD.

Developmental and Educational History

All of Jennifer's developmental milestones were achieved at the normal ages. Jennifer attended preschool as a peer model at the Scottish Rite Early Language Program and kindergarten at Cosmos, a private school for gifted children. For first grade, Jennifer tested into the Southland Unified School District (SUSD) self-contained program for gifted and talented education (GATE) but instead was enrolled in Reichman Primary Magnet School and the GATE resource program. At mid-year, Jennifer's general education teacher reported that Jennifer's attention was so poor and her behavior “so out of control” that she was unable to assess her achievement. At this point, Jennifer, previously diagnosed with ADHD, started medication. Her teacher reported that her positive behavior change was dramatic. Jennifer has continued to take medication, and her teachers have had no complaints about her behavior. Being well behaved is important to Jennifer. Jennifer has always been thin and is a picky eater. About a year ago, she was diagnosed with an anxiety disorder and was prescribed mirtazapine. This medication is particularly helpful in stimulating Jennifer's appetite, and she has gained some much-needed weight.

Jennifer has just completed sixth grade in the self-contained GATE program at Eklund Middle School in SUSD. She had been in charter schools for the past few years, none of which had programs that were appropriate for her. She likes the GATE program but feels the classes are too crowded (34 students per class). She said that she loves her Social Studies class because of the high level of thought-provoking questions and discussion. She enjoys having friends in her classes and feels accepted.

Jennifer has refused to be pulled from class for specialized help because she becomes anxious at the thought of missing information that the teacher is presenting. Also, although she is distracted by noise during tests, she does not want to take her tests outside her classroom, which has become familiar and safe. Moving to an unfamiliar place intensifies her anxiety. Jennifer has a severe fear of dogs and is uncomfortable with cats. She is active in a theater group and likes to sing.

Previous Evaluations

I first evaluated Jennifer at her mother's request when she was age 9–4 (July 2012). Jennifer was articulate about her difficulties with both writing and attention in class and at home. She stated that although she loved to read, when she tried to write, she could not remember what many of the letters looked like and had to mentally visualize the alphabet strip on the wall of the classroom. Once she had the image, she could not “remember how to tell [her] hand” to produce it. She had the same difficulty with numbers. Her handwriting was almost illegible and her spelling nonphonetic. She also reported that while talking or writing, she often could not recall the word she meant to use.

The results of that evaluation indicated that Jennifer's general intelligence was in the Very Superior range with highly significant discrepancies among her cognitive abilities. Her performance was extremely high in Fluid Reasoning, Visual-Spatial Thinking, Auditory Processing, and Phonemic Awareness, with most of her other cognitive abilities in the High Average range. In comparison with her intellectual ability, she had significant weaknesses in speed of word retrieval, spelling, writing fluency, and academic knowledge. She was not able to demonstrate knowledge of phonics either in spelling or in reading unfamiliar words, but when tested on an informal reading inventory, her oral reading accuracy was 100% on a fifth grade passage with fluency appropriate for end of grade 3. She met the criteria for twice exceptional—gifted with developmental dysgraphia, exacerbated by a weakness in word retrieval when under time pressure. Also of concern were her basic math skills and knowledge of math facts. She did not use the normal algorithms for computation problems and figured them out with a logical, albeit inefficient method.

Subsequent to this evaluation, I started working with Jennifer in educational therapy, focusing on handwriting, reading decoding, spelling, and written expression. We met twice per week for three years. Toward the end of therapy, we focused on word processing, so that the mechanics of handwriting would not interfere with expression of her thoughts. For the first few months, Jennifer had meltdowns at the thought of writing. Finally, using a story structure, she wrote a couple of stories and then, on her own, began to write poetry. Eventually, again on her own, she wrote complete, imaginative, detailed stories.

At the beginning of Jennifer's sixth-grade year, I administered the Gray Oral Reading Test—5 (GORT-5) and the KeyMath Diagnostic Test—3. On the GORT-5, her overall score was a percentile rank of 81. Her subtest percentiles were: Rate: PR 50; Accuracy: PR 75; Fluency: PR 63; Comprehension: PR 91. In the major areas assessed by the KeyMath-3, Jennifer's percentile scores were: Basic Concepts: PR 95; Operations: PR 95; and Applications: PR 91.

On the 2013–2014 AIMS (Arizona's Instrument to Measure Standards), Jennifer exceeded the standard in both Reading and Math, and met the standard in Writing. She explained to me that when a topic is assigned, she thinks of so many different perspectives that she does not know how to start. Regarding the extended writing section of the AIMS, she interpreted the question in two different ways but did not want to ask which was the correct one. That area was her lowest score, at the 50th percentile.

Behavioral Observations

I tested Jennifer in three sessions in my office, where we typically met for educational therapy. For all sessions, Jennifer had full benefit of her medication. We stopped for periodic breaks and snacks. Jennifer was cooperative and appeared to put forth her best effort throughout all of the testing. She listened attentively to test items and responded articulately. On untimed math, fluid reasoning, and quantitative reasoning tests, she often spent an inordinately long time working out a problem, which extended the testing time considerably. For example, on a test of number patterns, it took her 38 minutes to answer 31 problems. On most of the tests, she generally recognized when she could not answer an item correctly and accepted her inability to do so. For example, as she was taking an increasingly long time on the higher items of the Spelling test, trying out different spellings, I reminded her that she was not expected to know all of the items. She replied, “I'm at that point now—but at least I know what the words mean.” On many tests, she spontaneously explained any difficulty she was having. On timed tests in which she had to come up with verbal responses quickly, she explained that either she could not recall the word she wanted or her mind became flooded with so many responses that she could not get just one out. Additionally, I observed that if Jennifer is interrupted in the middle of a process, she may forget what she was doing and have to start over.

Sources of Information

Woodcock-Johnson IV Tests of Cognitive Abilities (WJ IV COG)

Woodcock-Johnson IV Tests of Oral Language (WJ IV OL)

Woodcock-Johnson IV Tests of Achievement (WJ IV ACH)

Interview with Jennifer

Analysis of classwork

Validity of Test Results

All of the tests administered were appropriate for Jennifer's ethnicity and language. The following were considered and ruled out as primary contributing factors to the referral concerns: visual, hearing, or motor impairments; cultural factors; environmental or economic disadvantage; and limited English proficiency. Jennifer's performance on these tests should be considered a valid representation of her current functioning when she has taken her medication and she is motivated.

Explanation of Scores

Jennifer's scores on the tests of the Woodcock-Johnson IV (WJ IV) are reported as percentile ranks (PR), standard scores (SS), verbal descriptors for SS ranges, and Relative Proficiency Indexes (RPI).

Percentile ranks indicate the percentage of children in Jennifer's grade in the norming sample whose score on a test was the same as or lower than Jennifer's.

The RPI is a mathematical prediction of a person's success on a task similar to the one used in the test, when the average student of her age would be 90% successful. RPIs are norm-based and are particularly useful in showing a weakness in performance that may not be evident from the standard scores. For example, although Jennifer obtained a standard score of 94 on the Math Facts Fluency test, which is considered average according to standard scores, her RPI of 77/90 indicates that when a typical grade-peer is 90% successful in rapidly recalling math facts, Jennifer's expected success would be 77%.

For the purposes of this report, descriptors for standard score, percentile rank, and RPI ranges are:

Descriptor Standard Score Percentile Rank RPI Proficiency Instructional Implications
Very Superior 131 or above 98 or above 100/90 Very advanced Extremely easy
Superior 121–130 92–97 98/90–100/90 Advanced Very easy
High Average 111–120 77–91 95/90–98/90 Average to advanced Easy
Average 90–110 25–76 82/90–95/90 Average Manageable
Low Average 80–89 9–24 67/90–82/90 Limited to average Difficult
Low 70–79 3–8 24/90–67/90 Limited Very difficult
Very Low 69 or below 2 or below 3/90–24/90 Very limited Extremely difficult

A complete set of scores is appended to this report. Included are two tables, one based on Jennifer's RPIs and the other based on her standard scores. The table illustrates her performances, from low to high, on the tests or clusters administered.

Test Results

Note to the reader: Throughout the report, Jennifer's abilities and scores are described in comparison with her “grade-peers.” This refers to the students in the national norming sample, not to Jennifer's classmates in the self-contained GATE program or with the other sixth-grade students in her school.

Cognitive Abilities

General Intelligence

General intelligence may be considered as a system of relatively independent cognitive abilities interacting to deal with a specific cognitive task. The WJ IV COG has two measures of general intelligence, the General Intellectual Ability (GIA) composite and the Gf-Gc composite. Gf stands for fluid intelligence (on-the-spot problem-solving) and Gc for crystallized intelligence (acquired knowledge). The Gf-Gc composite reflects a person's performance in the higher-level cognitive abilities of reasoning, language development, and knowledge of one's environment and culture. It differs from the GIA in that it does not incorporate scores from automatic, or lower-level, cognitive processes that are most often the major contributors to learning disabilities.

Jennifer's Gf-Gc composite score was in the Very Superior range, equal to or above 98% of others in her grade (PR 98). She demonstrated similar strengths in Fluid Reasoning, Quantitative Reasoning, and Visual Processing.

Fluid Reasoning

Jennifer demonstrated Very Superior ability in Fluid Reasoning, as high as or higher than 99% of her grade-peers (PR 99, RPI 100/90). A person uses fluid reasoning when the information and procedures she knows may be useful but are not sufficient to solve the problem; consequently, she must apply logical reasoning to any available information.

Quantitative Reasoning

Jennifer also scored in the Very Superior range in the tests of Quantitative Reasoning, equal to or above 98% of her grade-peers (PR 98, RPI 99/90). Quantitative reasoning is the ability to reason with numbers, with the relations among numbers, and use different operations to solve problems. The content of the reasoning process is quantitative knowledge (e.g., facts, algorithms, the number of eggs in a dozen). A person has to use quantitative reasoning with problems that come up in daily life and that do not fit neatly into mathematical routines and procedures that she already knows. Examples include figuring out the amount of carpet needed for three rooms of varying sizes, or how to apply IRS rules to a company's taxes. On a separate and more complex test of math reasoning (not within this cluster) that required Jennifer to figure out the mathematical rule underlying a matrix of two different number patterns, she scored far into the Very Superior range (Number Matrices, PR >99.9). I heard the director of a university psychology department state that he and his colleagues were not able to solve all of the problems. Jennifer missed only one, and that was far from the most difficult. She was extremely persistent, and at item 20, without her knowledge, I started timing her. Her shortest time was 36 seconds; her longest was 4 minutes.

Visual Processing

Visual processing (PR 97, RPI 98/90) is the ability to use mental imagery for a variety of purposes. Among the many aspects of visual processing are visualization (i.e., the ability to grasp the spatial relationships among objects and to imagine what an object, design, or mental image would look like from a different perspective or if the components were rearranged), visual memory (i.e., the ability to hold a visual image in mental awareness for up to approximately 30 seconds), and imagery (i.e., the ability to mentally construct vivid images). Each of these aspects of visual processing has important applications. For example, Jennifer uses imagery to support reading comprehension. She once commented that she knows she is not understanding what she is reading if she cannot visualize it.

Auditory Memory Span

Jennifer's auditory memory span (PR 86), as measured, was in the High Average range. Jennifer's auditory memory span is the amount of auditory (and in this case, verbal) information she can hold in immediate awareness before using it. This indicates that the information is available to be used for verbal rehearsal (i.e., repeating it until the task is completed) or to work with it so as to transfer it to long-term memory.

Short-Term Working Memory

Short-term working memory describes a person's ability to direct attention to the information held in conscious awareness, and maintain it, whereas other cognitive abilities operate on the information. Short-term working memory is a dynamic process, an operation performed on information, rather than a stable ability. Some tasks that depend on short-term working memory are

  1. Working the steps of a long division problem while simultaneously retrieving the needed math facts from long-term memory
  2. Writing an essay in which previously learned information is used to explain a point of view while simultaneously formulating ideas, figuring out the best way to organize them, and selecting the words to use
  3. Driving to a new location while trying to revisualize the map you left at home

Test results indicate that Jennifer's ability to work with information held in immediate awareness is stronger when she does not have to mentally reorganize the information. Of three tests of short-term working memory, she performed significantly better on the test in which she could repeat the string of items to herself and say the one requested aloud (Verbal Attention: PR 76; Numbers Reversed: PR 42; Object-Number Sequencing: PR 39).

Long-Term Retrieval

Long-term retrieval is the ability to transfer information from long-term memory to short-term working memory so it can be used. A key characteristic of long-term retrieval is fluency—the speed, accuracy, and completeness with which a person can retrieve the required information. Fast retrieval of information allows a person immediate access to it for use in solving a problem or performing a task. Jennifer scored in the higher end of the Average range in tasks that required her to show that she remembered and could use meaningful information that she had just heard or learned (PR 73, RPI 94/90).

Cognitive Processing Speed/Perceptual Speed

Test results indicate that Jennifer's speed in doing relatively simple tasks and in noting similarities and differences among visual symbols is similar to that of her average grade-peers. Cognitive processing speed allows us to develop automaticity in lower-level skills so that cognitive attention and energy are available to learn and work with more complex and higher-level tasks. Perceptual speed is important in developing automatic identification of visual symbols for use in higher-level tasks, such as reading, writing, and math (Cognitive Processing Speed: PR 56, RPI 92/90; Perceptual Speed: PR 56, RPI 93/90).

General Information

Jennifer has a good understanding of objects in the world around her, where they would generally be found, and what they are used for. The ability to do well on this test also indicates a strong vocabulary, in that she had to recognize the object by its name, and higher items are uncommon (e.g., Where would you find a sepulcher?) (PR 81, RPI 98/90).

Oral Language

Auditory Processing

Auditory processing and phonetic coding skills are foundational skills for learning to sound out words for reading and spelling. Jennifer's performance on these tests was in the higher end of the Average range. She can identify individual speech sounds within words, the position of a specific sound within a word, change sounds to make other words, and repeat long multisyllable words that she had never heard before. On one test, she had one minute to say all the words she could think of that started with a specific sound. She came up with only six, afterwards saying, “There were too many in my head.” On her second trial (different sound), she only got 12. These examples were typical of her difficulty in generating words when the emphasis was on speed. She did well on the other, untimed, tests within this cluster. Auditory processing is not a problem for Jennifer, but speed and pressure are (Auditory Processing: PR 66, RPI 94/90).

Phonetic Coding

Phonetic coding is the ability to blend individual sounds together to make words and to pull whole words or syllables apart into individual sounds. These are foundational skills for beginning reading and spelling. Jennifer's performance was in the Superior range. Her relatively lower score on blending was unrelated to the task. Many of the higher-level words included the schwa sound, which sounds like /uh/; so when the recorded voice said the sounds one at a time, some of them were /uh/. At the end of the test, Jennifer said, “By the time he got to ‘uh’ every time, I got distracted because I couldn't remember the first part. ‘Uh’ was kind of in the middle of the word and I'd get stuck on that because I knew I'd end up saying ‘uh.’” Jennifer sometimes says “uh” when she's thinking or does not know an answer (PR 91, RPI 98/90).

Listening Comprehension and Oral Expression

Almost all of Jennifer's other language skills were in the High Average to Superior range. She has a wide and flexible vocabulary, both receptively and expressively. She can repeat long and complex sentences and follow lengthy instructions in syntactically complex sentences. (For this last test, she pointed to objects in a picture.) She demonstrated comprehension of discourse of one to two sentences as the sentences increased in length, complexity, and conceptual level (Broad Oral Language: PR 85, RPI 97/90; Oral Expression: PR 89, RPI 98/90; Listening Comprehension: PR 88, RPI 97/90).

Vocabulary

Jennifer performed in the Superior range on Oral Vocabulary, indicating that she not only knows the meaning of many words but has the vocabulary to generate synonyms and antonyms (PR 95). Her score was lower but still in the higher end of Average when naming pictures (Picture Vocabulary: PR 73).

Speed of Lexical Access

In relation to her strength in her other language abilities, and especially to her superior cognitive abilities, Jennifer demonstrated a significant weakness in the time it takes her to retrieve a specific word from long-term memory when under pressure. The two tests in this cluster had time limits and required Jennifer to respond as fast as possible. The first was to name pictures of common objects (e.g., door, box); the second was to come up with as many words as possible within a given category (e.g., articles of clothing). Although Jennifer was slow on both tasks, she had more difficulty with the picture naming task. Sounding exasperated, she said, “I can't process it fast enough [visually]. Then when I do process, I can't think of the word fast enough and when I do, I can't get it out of my mouth. And I see all the other objects and I recognize them and I know what they are and I'm trying to think of the one in front of me.” On this test, Jennifer out-performed only 13% of her grade-peers. Of the word retrieval task, she said, “Staying within one category helped because without that too many things are popping into my head.” On a story retelling task, she said “stop pedals” because she couldn't remember the word “brakes” although she had just heard it in the story.

Jennifer reports that when she feels pressured, she often cannot find the word that she wants to use. The pressure could be from an external source, as in working against a time limit, or internal, as when urgently attempting to state a point. Her RPI indicates that when under pressure, Jennifer's chance of success in quickly coming up with the word she wants is 70% compared with typical grade-peers who have a 90% likelihood of success (Speed of Lexical Access: PR 16, RPI 70/90). When comparing her Speed of Lexical Access score with the average of her other oral language tests, only 4% of her grade-peers would have such a wide discrepancy. In comparison with her Gf-Gc composite, only 2% would have such a wide discrepancy.

Orthographic Processing, Recognition, and Retrieval

Orthography refers to the visual features of our printed language, including letter form and orientation, common letter combinations, syllable patterns, and whole words. Orthographic processing is the rapid, accurate formation of these features in memory. Orthographic recognition is the ability to recognize a match between what one currently sees and the image that is stored in long-term memory (e.g., multiples choice spelling). Orthographic retrieval is the ability to retrieve from memory a visual image of these features (e.g., visualizing the ch and u for the word chorus, rather than spelling it corus or koris). Good orthographic processing is necessary for acquiring sight words and for spelling; more so for those features of words that are not phonetic.

Three years ago, Jennifer had such limited orthographic abilities that she could not remember the form of many letters or their orientation. For this evaluation, Jennifer was not given a test specifically addressing orthographic abilities. Instead, these abilities were assessed qualitatively from the WJ IV reading and spelling tests and her spelling in an essay from her Social Studies class. Jennifer's orthographic recognition is stronger than her orthographic retrieval so, when writing, if she cannot remember the spelling of a word, and if she has time, she writes the word multiple ways and tries to see which looks right. On the Spelling test, she did this on 12 of the 34 words she was given. Of these, she got one right; all, however, were phonetically correct. As she made repeated changes on one of the words, she said, “I wish I hadn't erased the first one I did ‘cause it was closer than the one I have now,” but she could not remember her initial spelling. After spelling a different word multiple times, she looked at it and said, “I'm missing a letter…but I know there's a qu.” The average reader learns much of his or her spelling from seeing a word repeatedly along with, hopefully, spelling rules taught in school. Jennifer is a voracious reader but only learned to recognize common letter patterns after three years of structured and systematic educational therapy based on the Wilson Reading System. Currently, her reading of regular and irregular words and her spelling are much improved, but an ongoing problem in orthography is evident in her misspelling of words that should be automatic by now, such as morman (her religion), cristmas, belive, and diffrent.

Academic Achievement

Reading

On the WJ IV ACH reading tests, Jennifer demonstrated skills in the high average range in knowledge of phonics and orthographic conventions (the spelling patterns of words in English), word attack, identification of irregular words, comprehension of one- to two-sentence passages, and vocabulary.

Basic Reading Skills

Basic Reading Skills refers to the ability to identify words on sight and apply phonics and knowledge of orthography to read unfamiliar words (PR 86, RPI 98/90). Jennifer's decoding errors when reading real words out of context were in misreading the short vowel sounds of I and E and in pronouncing foreign words according to English phonics (e.g., gauche, cologne). In her oral reading in context, of nine errors, only one word was mispronounced; the other errors (for which she lost credit) were repetitions of words or word parts, or syntactically and semantically acceptable omissions or substitutions of words (e.g., in for on) (Oral Reading: PR 31, RPI 79/90). Although she had already reached the cutoff point, I had her read the last paragraph. Her errors here and in reading nonsense words indicate that she is having difficulty reading words of four or more syllables that are probably not in her oral vocabulary, and she would benefit from more instruction in structural analysis and some systematic instruction in morphology.

Reading Comprehension

Jennifer's Passage Comprehension and Reading Vocabulary scores were in the High Average range. Jennifer's score was lower (mid-average) in retelling brief stories (80 to 90 words) immediately after reading them silently. She read them too quickly, between 247 and 387 words per minute, and left out many key details. In contrast, on the Qualitative Reading Inventory-5 within the past year and after reading passages of approximately 350 to 400 words, her retellings were excellent and included most key details with good organization. Subsequently, she answered inferential comprehension questions correctly. Her Reading Recall score is an underestimate of her true ability (Passage Comprehension: PR 90, RPI 99/90; Reading Vocabulary: PR 83, RPI 97/90; Reading Recall: PR 55, RPI 91/90).

Reading Rate

When reading short, simple sentences and rows of words silently, with a response indicating comprehension, Jennifer's reading rate was in the higher end of the Average range. This is a reasonable estimate of her silent reading speed when reading simple text. As with many bright students, she considered a few of the sentences in more depth than intended, which may have slowed her down. For example, she questioned a statement for which the answer was intended to be “true,” explaining, correctly, a situation in which it would be false (PR 64, RPI 96/90).

Writing

Grade Equiv. RPI Standard Score Percentile Rank
2011 2014 2011 2014 2011 2014 2011 2014
BASIC WRITING SKILLS 4.0 10.0 91/90 97/90 101 110 (107–114) 53 75
Editing 5.8 10.2 98/90 97/90 111 111 (106–117) 78 77
Spelling 2.9 9.8 72/90 97/90 92 109 (105–113) 30 73
[Spelling of Sounds] 3.5 10.8 93/90 93/90 97 106 (99–113) 43 65
WRITTEN EXPRESSION 4.5 15.6 93/90 97/90 106 117 (113–121) 65 87
Writing Samples 5.5 >17.9 95/90 98/90 108 116 (112–121) 70 86
Sentence Writing Fluency 3.9 13.0 90/90 96/90 100 112 (106–118) 50 79

This table shows Jennifer's strong progress in writing from the August 2011 evaluation to the current one. A caveat is that the WJ III (2011) and the WJ IV do not have the same norming sample; consequently, the comparability of the scores is somewhat limited. Jennifer's major problem at that time was dysgraphia. Her handwriting was almost illegible, she could not mentally visualize letters; when she did visualize a letter or try to copy it, she could not remember the strokes or sequence of strokes to do so. Jennifer could not revisualize the spellings of words she could easily read and did not know how to sound the words out to spell them phonetically. Her written expression was restricted by the tremendous effort it took to get her thoughts on paper.

Handwriting

Jennifer worked hard and was motivated to learn during her three years of educational therapy. Currently, even when rushed, her handwriting (print) is quite legible. It is neat if she focuses on it, but at the cost of diverting attention from her flow of thoughts and stating them articulately. She can now, however, use word processing, which facilitates putting thoughts on paper.

Spelling

On all of the WJ IV writing tests, with the exception of spelling, Jennifer scored in the High Average range. Her spelling of both real and nonsense words was in the higher end of the Average range.

Her RPIs indicate that previously, when grade-peers were 90% successful in spelling words in lists, she was likely to be 73% successful; currently, she would be 97% successful. Jennifer has benefited from learning spelling rules and having sufficient practice to integrate them. On her 2011 writing sample (Figure 2.9), 19% of her words had spelling errors; on her 2014 Social Studies essay (Figure 2.10), only 8% of her words had spelling errors (only twelve words were spelled incorrectly but some were repeated multiple times). On the WJ IV Spelling test, when writing a word that contained an irregular element, Jennifer wrote the word multiple ways, trying to see which one looked correct. She could not always tell, but it was a good strategy. All of the words were phonetically accurate. When writing in context, Jennifer is still not a good speller, but using a word processing program with a spell checker will help, especially if she can tell the difference between homophones.

Figure depicting a sample of Jennifer's writing with multiple spelling mistakes (grade 3.9).

Figure 2.9 Jennifer's Writing Sample, Grade 3.9

Figure depicting a sample of Jennifer's writing with less spelling mistakes (grade 6.9).

Figure 2.10 Jennifer's Writing Sample, Grade 6.9

Figure 2.11 Jennifer's Writing Sample, Grade 7, “Harmony”

Pain. Searing pain. Unimaginable heat, to the highest degree, scorches my form. Flame envelopes me with blinding light. The fire dances; reds, yellows, and oranges blur, breaking my vision into little black spots. The flame burns my form and drowned my screams in agony. It roars with intense power and the shades of red disperse. A burst of blue flame appears, with more intense temperature that causes my skin to peel away. I suffer and gasp for air but intake the fire, scorching my throat. I slowly disintegrate until I am nothing more than a flaming pile of ashes. The flame, with one last roar, diminishes.

Sapphire is revealed, now enveloped in blue flame, with eyes of silver. She glows with intense magical focus. She lifts me with a flick of her wing. I am transported to my grove. She places me on an empty patch of grass. Then, with grace, she dances.

As she swoops, dives, and flickers like a roaring fire, the grove is filled with silver light. Slowly, I take the form of a tree. With a swoop and a twirl a pure-white trunk is formed; a wisp of her wings and branches form. She sings and one by one large light-green leaves bloom. She creates the sweetest sap good for sugar. With one last note, she spreads her wings and the ashes have formed a work of art, a single maple tree. She takes flight and I stand tall, branches reaching toward the sky, and laugh. For I have my wish at last.

Written Expression

On the Writing Samples test, Jennifer complied with the instructions but did not engage in the task as she does when writing a story. Nevertheless, her RPI was 97/90, indicating that she is ahead of her grade-peers in using written language to respond to brief and diverse writing tasks. Her longer essays, such as school assignments or tests, are more representative of her written expression, but her creative writing is exceptional. Her stories are imaginative, vividly detailed, with good use of dialogue and inclusion of the necessary story elements. Figure 2.11 shows a page of a short “book” she wrote using a computer.

In contrast, based on my previous work with Jennifer, she has difficulty doing the work required in writing a research paper, including narrowing her topic so as to make it workable, selecting the most useful and related information from her sources, organizing the information, and putting it in her own words.

The writing fluency task Jennifer was asked to do was to write short, simple sentences as fast as possible. Because she was given a picture and three words for each item, the task of sentence formulation was greatly simplified. She wrote the sentences quickly and easily.

Mathematics

As noted in the section on Cognitive Abilities, Jennifer's quantitative reasoning ability is in the top 2% of her grade-peers in the norming sample. Her quantitative knowledge (e.g., facts, specific algorithms, units of measure) is not at that level. She was successful on many problems because she is so persistent, but she frequently does not see the simplest way to solve a problem.

An area of persistent difficulty for Jennifer is her ability to retrieve simple addition, subtraction, and multiplication facts quickly. On the Math Facts Fluency test, her RPI was 77/90. This limitation necessarily diverts her attention from the higher level of the problem she is working on, making it more difficult to monitor her progress through a problem and maintain an overview. For example, in multiplying a negative by a positive number, she got the sign correct, carried from the units to the tens column, then added 4 and 7 incorrectly.

Jennifer did well on the tests of Calculation and Applied Problems (Mathematics: PR 89, RPI 98/90). Usually, she was slow and inefficient but correct; occasionally, she was quick and prone to simple errors.

Jennifer would do well to learn how to keep her work more organized within the work space. Although she started by leaving space around her work on each item, she started in the middle of the page and soon ran out of space, with some areas crowded and others fairly empty. This did not cause her to make any errors but it does make it more difficult to keep track of where she is within a process.

Knowledge

On questions regarding knowledge of science, social studies, and humanities, Jennifer scored in the upper half of the Average range (Academic Knowledge: PR 63, RPI 93/90). Her knowledge of her environment, such as what different objects are used for and where they would typically be found (e.g., What is a kiln used for? Where would you find a whisk?) was in the High Average range (General Knowledge: PR 81, RPI 98/90).

Summary and Diagnostic Impressions

Attitude and Effort

Jennifer put in her best effort and reflected on her own mental process. She often recognized when she had made a mistake and accepted it if she could not correct it. As well, when she had difficulty with a task or item, she often described how her mental process interfered with her performance. Because of her persistence in trying to work to the best of her ability, areas of cognitive and academic weakness notwithstanding, she is likely to take longer than her classmates to complete the same amount of work.

Cognitive Strengths

Jennifer's Gf-Gc composite places her overall intellectual ability in the Very Superior range, equal to or higher than 98% of her grade-peers nationally. Additionally, she has superior or very superior cognitive and language strengths in fluid reasoning (the ability to use reasoning to solve new or unfamiliar problems), quantitative reasoning (the ability to apply fluid reasoning to numbers and numerical relationships), visual processing (incorporating visualization, visual memory, and imagery), and oral vocabulary. Jennifer's other cognitive strengths included auditory memory span (the amount of information she can hold in immediate memory without having to work with it) and general information about objects in the environment.

Cognitive Weaknesses

Jennifer displayed a significant weakness in the speed with which she can retrieve specific words from memory, especially under pressure. No matter what predictor it was compared with, this ability was significantly low. This was also her lowest cognitive/language ability in her 2012 evaluation.

Jennifer also demonstrated significant difficulty on those tests of short-term working memory that required her to mentally reorder seemingly unrelated pieces of information. Only 6% of grade-peers would have such a wide discrepancy between her Gf-Gc and her Short-term Working Memory score. This may be the reason that if Jennifer is interrupted in the middle of a process, she forgets what she was doing and has to start over.

Despite tremendous progress, Jennifer's spelling still evidences an orthographic weakness. She profits from learning the rules for spelling accompanied by repeated practice in applying them.

Effect on Academics

Three years ago, Jennifer was diagnosed with developmental dysgraphia, the characteristics of which are (a) a deficit in handwriting automaticity—including the ability to revisualize and produce legible letters rapidly and without conscious effort, (b) poor spelling, (c) problem in motor planning finger movement, and (d) difficulty in self-regulating the composing process (Berninger & O'Malley May, 2011). Jennifer has made good progress in the characteristic skill deficits, to the point where her standard scores on tests will no longer document this as a disability. Still, it must be recognized that what is automatic for other students is not automatic for Jennifer. The mechanics of writing and spelling will still require cognitive attention and energy, diverting these to some extent from ongoing generation of thoughts, where they should be focused. Although Jennifer is gifted at writing creative stories, she has considerable difficulty with planning, researching, and producing expository writing.

Even though Jennifer has a broad and flexible vocabulary, she also has difficulty retrieving the specific word she wants to use from memory. This affects the speed with which she can express her thoughts in writing and her fluency in speaking, especially when she feels under pressure (e.g., time, a sense of urgency to state her ideas). When she is writing, this difficulty exacerbates the problems caused by dysgraphia.

Jennifer's quantitative reasoning is in the Very Superior range; her quantitative knowledge, reflected in her actual math skills, is in the High Average range. She has the capability but still lacks some of the math concepts, algorithms, and facts necessary to keep up with her class. A serious impediment is her extremely slow retrieval of math facts. Furthermore, working almost any kind of math problem requires the mental reordering of the elements of the problem, an area of weakness for Jennifer. Last year, when she was administered the KeyMath-3, which did not allow paper or pencil for most of the tests, she wrote on the table with her finger for the problems that required computation.

Eligibility for Special Education Services

The criteria in Jennifer's school district for eligibility for special education services, and accordingly, an IEP, stipulate that the academic area of disability must be below average. The district will permit a 504 accommodation plan for a twice-exceptional student such as Jennifer, but it does not recognize any cognitive, language, or academic area at or above average as a disability. Jennifer's only below-average ability was in word retrieval (Speed of Lexical Access). For that, Jennifer and her mother prefer accommodations rather than school-based therapy, which, in any case, is not available. Consequently, Jennifer does not qualify for special educational services.

Recommendations

Within the recommendations are suggestions for accommodations. Please discuss these with Jennifer. She does not want to look any different from the other students, and some of the accommodations may embarrass her. Omit those but recognize the extra pressure she will experience.

General

Because of Jennifer's difficulty with short-term working memory, revisualizing words for spelling, and effortful handwriting, all math and writing homework will be more difficult and take her longer than would be expected for most of her classmates. Additionally, although she gets good benefit from her ADHD medication, she is still easily distracted by noise when she is trying to concentrate. Furthermore, by the time Jennifer gets home from school, has a snack, and is ready to start homework, the medication that allows her to focus and maintain concentration is no longer working, making homework that much more taxing. Consequently, the following accommodations are suggested:

  1. Seat Jennifer in the front row of the class, near the teacher, and/or surrounded by quiet students so that she can see the teacher and visual information easily and so she is less likely to be distracted.
  2. Shorten Jennifer's in-class assignments and tests (discreetly) while still providing the same variety of problems or work. Alternatively, allow Jennifer as much time as she needs to complete the work. This may need to extend to after school, but do not require her to take incomplete work home unless she has no other homework.
  3. During tests, make sure the classroom stays quiet until Jennifer is finished or the end of the period. (No paper crinkling, backpack zipping, putting materials away in anticipation of leaving, talking.)
  4. When providing extra instruction outside of regular class time, maintain a quiet and nondistracting environment.
  5. If possible, schedule Jennifer for a study period at the end of the day to allow her to start her homework and have the support of a teacher while her medication is still active in her system. If this is not possible in middle school, it should be done in high school.

Self-Advocacy

Teach Jennifer how to advocate for herself when she realizes that she has a problem or to ask questions for clarification when she does not understand an assignment or an instruction on a test. Next year, she should begin to learn how to discuss her disabilities and need for accommodations with a teacher, starting, perhaps, with a school counselor.

Writing

  1. To compensate for her slowness in speed of word retrieval and spelling, allow Jennifer to use an electronic spell checker and thesaurus during writing assignments in class. If she is willing, allow her to use a laptop.
  2. Do not mark down Jennifer's grade for errors in spelling, capitalization, or handwriting.
  3. Provide Jennifer with training in keyboarding skills and the use of a word processing program on a computer or tablet (whichever is easier for her), with the goal of having her type accurately and fast enough to express her thoughts fluently.
  4. For different types of expository writing, teach Jennifer pre-writing strategies for organization of ideas and details. Include taking expected length and time constraints into consideration.
  5. Provide Jennifer with a copy of the teacher's lecture notes or of the PowerPoint slides (in handout form) so that she does not have to take notes while listening.
  6. Even if she is receiving the teachers’ notes or slide handouts, Jennifer should learn how to take notes during lectures unless it requires so much attention that it interferes with her understanding of the lecture. Being given a copy of another student's notes or the teacher's notes is an accommodation that can follow her into college.

Math

  1. Consider re-administering the KeyMath-3 to see what concepts and skills Jennifer is missing that her classmates are expected to know. Use the test qualitatively, rather than for scores. Allow Jennifer to use a pencil and paper for all subtests, and observe her process in solving the problems.
  2. Because Jennifer has a weakness in short-term working memory tasks that require her to reorder information mentally, always provide her with pencil and paper for her math work. Do not require or expect her to do mental math.
  3. Jennifer had difficulty with the following types of math problems. Reteach any that the class has already covered or that the students were expected to know coming into this math class.
    1. Retrieving basic facts automatically—Provide drill and practice with all four operations until retrieval from memory is automatic every time. Help Jennifer to see the reciprocal relationships among addition and subtraction, and multiplication and division, and to create strategies that can act as hooks to retrieve the answer to particularly difficult facts.
    2. Finding the common denominator in a set of fractions with unlike denominators
    3. Figuring elapsed time
    4. Using percentages to figure discounts, tax, interest, including keeping track of the principal and adding or subtracting the percentage
    5. Finding the square root of a number
    6. Finding the hypotenuse of a right triangle when two sides are given
    7. Using fractions: Understanding whether to use multiplication or division to solve the problem, solving multiple step problems with unlike denominators, finding the least common denominator
  4. When Jennifer learns a new math skill, it is important for her to understand the concept, which will support her ability to recognize the most efficient ways of solving the problems. Furthermore, she needs repeated practice (massed, then distributed) in working a wide variety of actual problems, systematically increasing in complexity. Make sure she masters different types of computation problems incorporating the concept or skill you are teaching, then introduce word problems. You might have her review a particular level or set of problems on the Kahn Academy website, but make sure she can answer questions in the classroom at the level of the other students. Intermittent practice subsequent to mastery is critical for Jennifer.
img
img

Table of Scores

Woodcock-Johnson IV Tests of Cognitive Abilities (Norms based on grade 6.9)

CLUSTER/Tests GE RPI Proficiency SS (68% Band) PR
GEN INTELLECTUAL ABIL 13.0 97/90 Average to advanced 123 (119–128) 94
Oral Vocabulary 15.1 98/90 Advanced 124 (118–130) 95
Number Series >14.0 100/90 Very advanced 133 (127–138) 98
Verbal Attention 11.4 96/90 Average to advanced 110 (104–117) 76
Letter-Pattern Matching 7.0 91/90 Average 100 (90–111) 51
Phonological Processing 8.2 92/90 Average 104 (97–111) 60
Story Recall >15.5 95/90 Average 111 (107–116) 78
Visualization >17.9 98/90 Advanced 121 (115–127) 92
Gf-Gc COMPOSITE >17.9 99/90 Advanced 132 (128–137) 98
Oral Vocabulary 15.1 98/90 Advanced 124 (118–130) 95
Number Series >14.0 100/90 Very advanced 133 (127–138) 98
General Information 12.2 98/90 Advanced 113 (108–118) 81
Concept Formation >14.7 100/90 Advanced 128 (119–137) 97
COMP-KNOWLEDGE (Gc) 13.0 98/90 Advanced 119 (115–123) 90
Oral Vocabulary 15.1 98/90 Advanced 124 (118–130) 95
General Information 12.2 98/90 Advanced 113 (108–118) 81
COMP-KNOWLEDGE (Ext) 13.0 98/90 Average to advanced 117 (113–120) 87
Oral Vocabulary 15.1 98/90 Advanced 124 (118–130) 95
General Information 12.2 98/90 Advanced 113 (108–118) 81
Picture Vocabulary 10.6 96/90 Average to advanced 109 (103–115) 73
FLUID REASONING (Gf) >14.4 100/90 Very advanced 136 (130–143) 99
Number Series >14.0 100/90 Very advanced 133 (127–138) 98
Concept Formation >14.7 100/90 Advanced 128 (119–137) 97
Analysis-Synthesis >13.8 99/90 Advanced 121 (115–127) 92
S-TERM WORK MEM (Gwm) 7.8 93/90 Average 103 (98–109) 59
Verbal Attention 11.4 96/90 Average to advanced 110 (104–117) 76
Numbers Reversed 6.0 86/90 Average 97 (91–103) 42
S-TERM WORK MEM (Ext) 7.0 91/90 Average 101 (96–105) 52
Verbal Attention 11.4 96/90 Average to advanced 110 (104–117) 76
Numbers Reversed 6.0 86/90 Average 97 (91–103) 42
Object-Number Sequencing 6.0 85/90 Average 96 (90–102) 39
COG PROCESS SPEED (Gs) 7.3 92/90 Average 102 (95–109) 56
Letter-Pattern Matching 7.0 91/90 Average 100 (90–111) 51
Pair Cancellation 7.6 94/90 Average 103 (96–110) 58
PERCEPTUAL SPEED 7.3 93/90 Average 102 (95–110) 56
Letter-Pattern Matching 7.0 91/90 Average 100 (90–111) 51
Number-Pattern Matching 7.6 94/90 Average 104 (96–112) 61
AUDITORY PROCESS (Ga) 9.6 94/90 Average 106 (102–111) 66
Phonological Processing 8.2 92/90 Average 104 (97–111) 60
Nonword Repetition 11.4 95/90 Average to advanced 106 (102–111) 67
L-TERM RETRIEVAL (Glr) 13.0 94/90 Average 109 (105–113) 73
Story Recall >15.5 95/90 Average 111 (107–116) 78
Visual-Auditory Learning 13.0 93/90 Average 104 (100–109) 61
VISUAL PROCESSING (Gv) >17.9 98/90 Advanced 128 (121–134) 97
Visualization >17.9 98/90 Advanced 121 (115–127) 92
Picture Recognition >17.9 99/90 Advanced 125 (116–134) 95
QUANT REASONING >13.8 99/90 Advanced 131 (126–136) 98
Number Series >14.0 100/90 Very advanced 133 (127–138) 98
Analysis-Synthesis >13.8 99/90 Advanced 121 (115–127) 92
AUD MEMORY SPAN >17.9 99/90 Advanced 116 (111–122) 86
Memory for Words 13.0 97/90 Average to advanced 109 (103–115) 72
Sentence Repetition >17.9 99/90 Advanced 120 (113–126) 91
NUMBER FACILITY 7.1 91/90 Average 101 (94–108) 53
Numbers Reversed 6.0 86/90 Average 97 (91–103) 42
Number-Pattern Matching 7.6 94/90 Average 104 (96–112) 61
VOCABULARY 13.0 97/90 Average to advanced 117 (113–122) 88
Oral Vocabulary 15.1 98/90 Advanced 124 (118–130) 95
Picture Vocabulary 10.6 96/90 Average to advanced 109 (103–115) 73
COGNITIVE EFFICIENCY 6.6 89/90 Average 99 (92–105) 47
Letter-Pattern Matching 7.0 91/90 Average 100 (90–111) 51
Numbers Reversed 6.0 86/90 Average 97 (91–103) 42
COGNITIVE EFFICIENCY (Ext) 7.5 93/90 Average 103 (98–109) 59
Verbal Attention 11.4 96/90 Average to advanced 110 (104–117) 76
Letter-Pattern Matching 7.0 91/90 Average 100 (90–111) 51
Numbers Reversed 6.0 86/90 Average 97 (91–103) 42
Number-Pattern Matching 7.6 94/90 Average 104 (96–112) 61
Note: Bolded clusters indicate that the confidence intervals of the component tests overlap.
Tests GE RPI Proficiency SS (68% Band) PR
Oral Vocabulary 15.1 98/90 Advanced 124 (118–130) 95
Story Recall >15.5 95/90 Average 111 (107–116) 78
Visual-Auditory Learning 13.0 93/90 Average 104 (100–109) 61
Memory for Words 13.0 97/90 Average to advanced 109 (103–115) 72
Number Series >14.0 100/90 Very advanced 133 (127–138) 98
Verbal Attention 11.4 96/90 Average to advanced 110 (104–117) 76
Numbers Reversed 6.0 86/90 Average 97 (91–103) 42
Object-Number Sequencing 6.0 85/90 Average 96 (90–102) 39
Letter-Pattern Matching 7.0 91/90 Average 100 (90–111) 51
Number-Pattern Matching 7.6 94/90 Average 104 (96–112) 61
Pair Cancellation 7.6 94/90 Average 103 (96–110) 58
Phonological Processing 8.2 92/90 Average 104 (97–111) 60
Nonword Repetition 11.4 95/90 Average to advanced 106 (102–111) 67
Visualization >17.9 98/90 Advanced 121 (115–127) 92
Picture Recognition >17.9 99/90 Advanced 125 (116–134) 95
General Information 12.2 98/90 Advanced 113 (108–118) 81
Concept Formation >14.7 100/90 Advanced 128 (119–137) 97
Analysis-Synthesis >13.8 99/90 Advanced 121 (115–127) 92

Woodcock-Johnson IV Tests of Oral Language (Norms based on grade 6.9)

GE RPI Proficiency SS (68% Band) PR
ORAL LANGUAGE 11.6 96/90 Average to advanced 112 (107–118) 80
Picture Vocabulary 10.6 96/90 Average to advanced 109 (103–115) 73
Oral Comprehension 13.0 96/90 Average to advanced 113 (106–120) 80
BROAD ORAL LANGUAGE 13.0 97/90 Average to advanced 116 (110–121) 85
Picture Vocabulary 10.6 96/90 Average to advanced 109 (103–115) 73
Oral Comprehension 13.0 96/90 Average to advanced 113 (106–120) 80
Understanding Directions >16.8 97/90 Average to advanced 116 (107–124) 85
ORAL EXPRESSION 13.0 98/90 Advanced 118 (113–124) 89
Picture Vocabulary 10.6 96/90 Average to advanced 109 (103–115) 73
Sentence Repetition >17.9 99/90 Advanced 120 (113–126) 91
LISTENING COMP 13.7 97/90 Average to advanced 118 (111–125) 88
Oral Comprehension 13.0 96/90 Average to advanced 113 (106–120) 80
Understanding Directions >16.8 97/90 Average to advanced 116 (107–124) 85
PHONETIC CODING >17.9 98/90 Advanced 120 (115–125) 91
Segmentation >17.9 99/90 Advanced 121 (116–126) 92
Sound Blending 13.0 97/90 Average to advanced 111 (105–117) 76
SPEED of LEXICAL ACCESS 3.6 70/90 Limited to average 85 (80–90) 16
Rapid Picture Naming 2.9 47/90 Limited 83 (78–89) 13
Retrieval Fluency 5.4 86/90 Average 95 (88–101) 36
VOCABULARY 13.0 97/90 Average to advanced 117 (113–122) 88
Picture Vocabulary 10.6 96/90 Average to advanced 109 (103–115) 73
Oral Vocabulary 15.1 98/90 Advanced 124 (118–130) 95
AUDITORY MEMORY SPAN >17.9 99/90 Advanced 116 (111–122) 86
Sentence Repetition >17.9 99/90 Advanced 120 (113–126) 91
Memory for Words 13.0 97/90 Average to advanced 109 (103–115) 72
Tests GE RPI Proficiency SS (68%) Band) PR
Picture Vocabulary 10.6 96/90 Average to advanced 109 (103–115) 73
Oral Comprehension 13.0 96/90 Average to advanced 113 (106–120) 80
Segmentation >17.9 99/90 Advanced 121 (116–126) 92
Rapid Picture Naming 2.9 47/90 Limited 83 (78–89) 13
Sentence Repetition >17.9 99/90 Advanced 120 (113–126) 91
Understanding Directions >16.8 97/90 Average to advanced 116 (107–124) 85
Sound Blending 13.0 97/90 Average to advanced 111 (105–117) 76
Retrieval Fluency 5.4 86/90 Average 95 (88–101) 36
Sound Awareness >17.9 96/90 Average to advanced 117 (104–130) 87

Woodcock-Johnson IV Tests of Achievement Form A and Extended (Norms based on grade 6.9)

CLUSTER/Tests GE RPI Proficiency SS (68%) Band) PR
READING 13.0 99/90 Advanced 120 (116–124) 91
Letter-Word Identification 13.0 99/90 Advanced 116 (112–120) 86
Passage Comprehension 13.0 99/90 Advanced 119 (113–124) 90
BROAD READING 10.5 98/90 Advanced 113 (109–117) 81
Letter-Word Identification 13.0 99/90 Advanced 116 (112–120) 86
Passage Comprehension 13.0 99/90 Advanced 119 (113–124) 90
Sentence Reading Fluency 7.6 94/90 Average 103 (97–108) 57
BASIC READING SKILLS 13.0 98/90 Advanced 116 (112–120) 86
Letter-Word Identification 13.0 99/90 Advanced 116 (112–120) 86
Word Attack 13.0 97/90 Average to advanced 113 (107–120) 81
READING COMPREHENSION 12.8 96/90 Average to advanced 114 (110–118) 83
Passage Comprehension 13.0 99/90 Advanced 119 (113–124) 90
Reading Recall 7.6 91/90 Average 102 (98–106) 55
READING COMP (Ext) 12.8 97/90 Average to advanced 116 (112–119) 85
Passage Comprehension 13.0 99/90 Advanced 119 (113–124) 90
Reading Recall 7.6 91/90 Average 102 (98–106) 55
Reading Vocabulary 12.7 97/90 Average to advanced 114 (109–120) 83
READING FLUENCY 6.6 88/90 Average 99 (96–103) 48
Oral Reading 5.1 79/90 Limited to average 93 (89–97) 31
Sentence Reading Fluency 7.6 94/90 Average 103 (97–108) 57
READING RATE 8.2 96/90 Average to advanced 105 (101–110) 64
Sentence Reading Fluency 7.6 94/90 Average 103 (97–108) 57
Word Reading Fluency 8.7 97/90 Average to advanced 107 (101–113) 68
MATHEMATICS 13.0 98/90 Advanced 118 (114–123) 89
Applied Problems 17.8 99/90 Advanced 119 (114–125) 90
Calculation 10.4 98/90 Advanced 113 (108–118) 81
BROAD MATHEMATICS 8.7 96/90 Average to advanced 108 (105–112) 71
Applied Problems 17.8 99/90 Advanced 119 (114–125) 90
Calculation 10.4 98/90 Advanced 113 (108–118) 81
Math Facts Fluency 5.8 77/90 Limited to average 94 (90–99) 35
MATH CALCULATION SKILLS 7.3 93/90 Average 103 (99–107) 57
Calculation 10.4 98/90 Advanced 113 (108–118) 81
Math Facts Fluency 5.8 77/90 Limited to average 94 (90–99) 35
MATH PROBLEM SOLVING >17.9 100/90 Very advanced 142 (135–149) 99.7
Applied Problems 17.8 99/90 Advanced 119 (114–125) 90
Number Matrices >16.4 100/90 Very advanced 150 (141–160) >99.9
WRITTEN LANGUAGE 13.0 98/90 Advanced 115 (112–119) 85
Spelling 9.8 97/90 Average to advanced 109 (105–113) 73
Writing Samples >17.9 98/90 Advanced 116 (112–121) 86
BROAD WRITTEN LANG. 13.0 97/90 Average to advanced 116 (112–119) 86
Spelling 9.8 97/90 Average to advanced 109 (105–113) 73
Writing Samples >17.9 98/90 Advanced 116 (112–121) 86
Sentence Writing Fluency 13.0 96/90 Average to advanced 112 (106–118) 79
BASIC WRITING SKILLS 10.0 97/90 Average to advanced 110 (107–114) 75
Spelling 9.8 97/90 Average to advanced 109 (105–113) 73
Editing 10.2 97/90 Average to advanced 111 (106–117) 77
WRITTEN EXPRESSION 15.6 97/90 Average to advanced 117 (113–121) 87
Writing Samples >17.9 98/90 Advanced 116 (112–121) 86
Sentence Writing Fluency 13.0 96/90 Average to advanced 112 (106–118) 79
ACADEMIC SKILLS 11.3 98/90 Advanced 115 (112–119) 85
Letter-Word Identification 13.0 99/90 Advanced 116 (112–120) 86
Spelling 9.8 97/90 Average to advanced 109 (105–113) 73
Calculation 10.4 98/90 Advanced 113 (108–118) 81
ACADEMIC FLUENCY 7.2 91/90 Average 101 (98–105) 54
Sentence Reading Fluency 7.6 94/90 Average 103 (97–108) 57
Math Facts Fluency 5.8 77/90 Limited to average 94 (90–99) 35
Sentence Writing Fluency 13.0 96/90 Average to advanced 112 (106–118) 79
ACAD. APPLICATIONS 13.2 99/90 Advanced 122 (118–125) 93
Applied Problems 17.8 99/90 Advanced 119 (114–125) 90
Passage Comprehension 13.0 99/90 Advanced 119 (113–124) 90
Writing Samples >17.9 98/90 Advanced 116 (112–121) 86
ACAD. KNOWLEDGE 8.2 93/90 Average 105 (101–109) 63
Science 9.1 94/90 Average 105 (100–111) 64
Social Studies 6.0 85/90 Average 96 (91–102) 40
Humanities 11.4 97/90 Average to advanced 111 (105–117) 77
PHON-GRAPH KNOWL 13.0 96/90 Average to advanced 111 (106–117) 78
Word Attack 13.0 97/90 Average to advanced 113 (107–120) 81
Spelling of Sounds 10.8 93/90 Average 106 (99–113) 65
BRIEF ACHIEVEMENT 13.0 98/90 Advanced 118 (114–121) 88
Letter-Word Identification 13.0 99/90 Advanced 116 (112–120) 86
Applied Problems 17.8 99/90 Advanced 119 (114–125) 90
Spelling 9.8 97/90 Average to advanced 109 (105–113) 73
BROAD ACHIEVEMENT 10.1 97/90 Average to advanced 114 (111–116) 82
Letter-Word Identification 13.0 99/90 Advanced 116 (112–120) 86
Applied Problems 17.8 99/90 Advanced 119 (114–125) 90
Spelling 9.8 97/90 Average to advanced 109 (105–113) 73
Tests GE RPI Proficiency SS (68% Band) PR
Word Attack 13.0 97/90 Average to advanced 113 (107–120) 81
Letter-Word Identification 13.0 99/90 Advanced 116 (112–120) 86
Passage Comprehension 13.0 99/90 Advanced 119 (113–124) 90
Reading Vocabulary 12.7 97/90 Average to advanced 114 (109–120) 83
Oral Reading 5.1 79/90 Limited to average 93 (89–97) 31
Sentence Reading Fluency 7.6 94/90 Average 103 (97–108) 57
Word Reading Fluency 8.7 97/90 Average to advanced 107 (101–113) 68
Reading Recall 7.6 91/90 Average 102 (98–106) 55
Math Facts Fluency 5.8 77/90 Limited to average 94 (90–99) 35
Calculation 10.4 98/90 Advanced 113 (108–118) 81
Applied Problems 17.8 99/90 Advanced 119 (114–125) 90
Number Matrices >16.4 100/90 Very advanced 150 (141–160) >99.9
Spelling 9.8 97/90 Average to advanced 109 (105–113) 73
Spelling of Sounds 10.8 93/90 Average 106 (99–113) 65
Editing 10.2 97/90 Average to advanced 111 (106–117) 77
Writing Samples >17.9 98/90 Advanced 116 (112–121) 86
Sentence Writing Fluency 13.0 96/90 Average to advanced 112 (106–118) 79
Science 9.1 94/90 Average 105 (100–111) 64
Social Studies 6.0 85/90 Average 96 (91–102) 40
Humanities 11.4 97/90 Average to advanced 111 (105–117) 77
STANDARD SCORES DISCREPANCY
VARIATIONS
Intra-Cognitive [Extended]
Actual Predicted Difference PR SD Interpretation at
±1.50 SD (SEE)
COMP-KNOWLEDGE (Gc) 119 111 8 73 +0.61
COMP-KNOWLEDGE (Ext) 117 112 5 66 +0.41
FLUID REASONING (Gf) 136 111 25 99 +2.22 Strength
FLUID REASONING (Ext) 134 112 22 98 +1.99 Strength
S-TERM WORK MEM (Gwm) 103 114 −11 19 −0.88
S-TERM WORK MEM (Ext) 101 115 −14 10 −1.29
COG PROCESS SPEED (Gs) 102 111 −9 26 −0.63
AUDITORY PROCESS (Ga) 106 115 −9 22 −0.77
L-TERM RETRIEVAL (Glr) 109 111 −2 46 −0.11
VISUAL PROCESSING (Gv) 128 109 19 92 +1.37
QUANTITATIVE REASONING 131 112 19 96 +1.76 Strength
AUDITORY MEMORY SPAN 116 112 4 64 +0.36
PERCEPTUAL SPEED 102 112 −10 24 −0.70
VOCABULARY 117 112 5 68 +0.46
ORAL LANGUAGE 112 112 0 53 +0.07
PHONETIC CODING 120 112 8 72 +0.58
SPEED of LEXICAL ACCESS 85 111 −26 3 −1.85 Weakness
Oral Vocabulary 124 113 11 84 +0.99
Number Series 133 110 23 97 +1.85 Strength
Verbal Attention 110 112 −2 45 −0.12
Letter-Pattern Matching 100 110 −10 24 −0.72
Phonological Processing 104 115 −11 18 −0.92
Story Recall 111 109 2 56 +0.14
Visualization 121 108 13 83 +0.95
General Information 113 110 3 60 +0.26
Concept Formation 128 109 19 92 +1.44
Numbers Reversed 97 113 −16 11 −1.25
Number-Pattern Matching 104 111 −7 32 −0.48
Nonword Repetition 106 111 −5 36 −0.35
Visual-Auditory Learning 104 109 −5 37 −0.32
Picture Recognition 125 107 18 89 +1.25
Analysis-Synthesis 121 110 11 80 +0.84
Object-Number Sequencing 96 113 −17 7 −1.47
Pair Cancellation 103 109 −6 34 −0.41
Memory for Words 109 111 −2 44 −0.15
Picture Vocabulary 109 110 −1 48 −0.05
Oral Comprehension 113 110 3 58 +0.21
Segmentation 121 112 9 74 +0.63
Rapid Picture Naming 83 108 −25 4 −1.74 Weakness
Sentence Repetition 120 110 10 76 +0.72
Understanding Directions 116 112 4 62 +0.31
Sound Blending 111 109 2 55 +0.13
Retrieval Fluency 95 111 −16 12 −1.17
Number Matrices 150 109 41 99.9 +3.06 Strength
STANDARD SCORES DISCREPANCY
VARIATIONS
Intra-Oral Language [Extended]
Actual Predicted Difference PR SD Interpretation at
±1.50 SD (SEE)
ORAL EXPRESSION 118 105 13 87 +1.14
LISTENING COMP 118 104 14 88 +1.18
PHONETIC CODING 120 101 19 91 +1.33
SPEED of LEXICAL ACCESS 85 109 −24 4 −1.76 Weakness
VOCABULARY 117 105 12 86 +1.08
AUDITORY PROCESS (Ga) 106 101 5 65 +0.40
Picture Vocabulary 109 105 4 65 +0.38
Oral Comprehension 113 104 9 77 +0.75
Segmentation 121 101 20 92 +1.40
Rapid Picture Naming 83 107 −24 4 −1.71 Weakness
Sentence Repetition 120 104 16 88 +1.17
Understanding Directions 116 103 13 82 +0.92
Sound Blending 111 101 10 76 +0.70
Retrieval Fluency 95 108 −13 16 −0.98
Oral Vocabulary 124 105 19 94 +1.58 Strength
Phonological Processing 104 101 3 58 +0.19
Nonword Repetition 106 101 5 66 +0.41
STANDARD SCORES DISCREPANCY
VARIATIONS Intra-Achievement [Extended] Actual Predicted Difference PR SD Interpretation at
±1.50 SD (SEE)
BASIC READING SKILLS 116 115 1 56 +0.14
READING COMPREHENSION 114 114 0 50 0.00
READING COMP (Ext) 116 115 1 53 +0.08
READING FLUENCY 99 114 −15 9 −1.34
READING RATE 105 112 −7 30 −0.54
MATH CALCULATION SKILLS 103 114 −11 16 −1.01
MATH PROBLEM SOLVING 142 114 28 99 +2.42 Strength
BASIC WRITING SKILLS 110 116 −6 25 −0.68
WRITTEN EXPRESSION 117 111 6 71 +0.56
Letter-Word Identification 116 115 1 55 +0.12
Applied Problems 119 114 5 69 +0.50
Spelling 109 115 −6 27 −0.62
Passage Comprehension 119 114 5 69 +0.50
Calculation 113 113 0 49 −0.03
Writing Samples 116 109 7 73 +0.60
Word Attack 113 112 1 54 +0.11
Oral Reading 93 112 −19 6 −1.56 Weakness
Sentence Reading Fluency 103 112 −9 21 −0.82
Math Facts Fluency 94 111 −17 8 −1.40
Sentence Writing Fluency 112 111 1 56 +0.14
Reading Recall 102 111 −9 25 −0.68
Number Matrices 150 109 41 99.9 +3.02 Strength
Editing 111 115 −4 37 −0.34
Spelling of Sounds 106 113 −7 29 −0.55
Reading Vocabulary 114 113 1 54 +0.10
STANDARD SCORES DISCREPANCY
VARIATIONS Actual Predicted Difference PR SD Interpretation at
±1.50 SD (SEE)
Academic Skills/Academic Fluency/Academic Applications [Extended] Variations
ACADEMIC SKILLS 115 111 4 71 +0.56
ACADEMIC FLUENCY 101 114 −13 11 −1.22
ACADEMIC APPLICATIONS 122 107 15 94 +1.58 Strength
COG PROCESS SPEED (Gs) 102 109 −7 31 −0.49
PERCEPTUAL SPEED 102 111 −9 26 −0.63
READING RATE 105 113 −8 26 −0.63
STANDARD SCORES DISCREPANCY
COMPARISONS Actual Predicted Difference PR SD Interpretation at
±1.50 SD (SEE)
Gf-Gc Composite/Other Ability Comparisons
S-TERM WORK MEM (Gwm) 103 119 −16 12 −1.20
S-TERM WORK MEM (Ext) 101 119 −18 6 −1.56 Weakness
COG PROCESS SPEED (Gs) 102 113 −11 22 −0.79
PERCEPTUAL SPEED 102 114 −12 21 −0.81
SPEED of LEXICAL ACCESS 85 113 −28 2 −1.97 Weakness
AUDITORY PROCESS (Ga) 106 118 −12 17 −0.97
PHONETIC CODING 120 116 4 63 +0.34
L-TERM RETRIEVAL (Glr) 109 116 −7 29 −0.56
VISUAL PROCESSING (Gv) 128 115 13 82 +0.91
AUDITORY MEMORY SPAN 116 114 2 57 +0.18
NUMBER FACILITY 101 117 −16 12 −1.19
COGNITIVE EFFICIENCY 99 117 −18 8 −1.38
COG EFFICIENCY (Ext) 103 118 −15 12 −1.18
BRIEF ACHIEVEMENT 118 124 −6 26 −0.64
BROAD ACHIEVEMENT 114 124 −10 15 −1.04
READING 120 123 −3 39 −0.28
BROAD READING 113 122 −9 19 −0.89
BASIC READING SKILLS 116 120 −4 38 −0.31
READING COMPREHENSION 114 122 −8 23 −0.73
READING COMP (Ext) 116 124 −8 21 −0.81
READING FLUENCY 99 119 −20 5 −1.69 Weakness
READING RATE 105 117 −12 17 −0.94
MATHEMATICS 118 122 −4 34 −0.41
BROAD MATHEMATICS 108 122 −14 10 −1.30
MATH CALCULATION SKILLS 103 120 −17 7 −1.45
MATH PROBLEM SOLVING 142 124 18 97 +1.87 Strength
WRITTEN LANGUAGE 115 121 −6 34 −0.42
BROAD WRITTEN LANGUAGE 116 120 −4 36 −0.37
BASIC WRITING SKILLS 110 121 −11 18 −0.90
WRITTEN EXPRESSION 117 116 1 55 +0.12
ACADEMIC SKILLS 115 123 −8 24 −0.70
ACADEMIC FLUENCY 101 119 −18 7 −1.46
ACADEMIC APPLICATIONS 122 123 −1 43 −0.17
PHONEME-GRAPHEME KNOWLEDGE 111 117 −6 33 −0.45
STANDARD SCORES DISCREPANCY
COMPARISONS Actual Predicted Difference PR SD Significant at
±1.50 SD (SEE)
GIA/Achievement Discrepancy Procedure
BRIEF ACHIEVEMENT 118 118 0 50 −0.01 No
BROAD ACHIEVEMENT 114 119 −5 28 −0.58 No
READING 120 117 3 62 +0.29 No
BROAD READING 113 117 −4 36 −0.36 No
BASIC READING SKILLS 116 116 0 53 +0.07 No
READING COMPREHENSION 114 116 −2 42 −0.19 No
READING COMP (Ext) 116 117 −1 43 −0.17 No
READING FLUENCY 99 114 −15 9 −1.32 No
READING RATE 105 114 −9 24 −0.72 No
MATHEMATICS 118 117 1 56 +0.16 No
BROAD MATHEMATICS 108 118 −10 17 −0.93 No
MATH CALCULATION SKILLS 103 117 −14 10 −1.29 No
MATH PROBLEM SOLVING 142 117 25 99 +2.43 Yes (+)
WRITTEN LANGUAGE 115 117 −2 46 −0.11 No
BROAD WRITTEN LANGUAGE 116 117 −1 46 −0.10 No
BASIC WRITING SKILLS 110 117 −7 27 −0.62 No
WRITTEN EXPRESSION 117 115 2 58 +0.21 No
ACADEMIC SKILLS 115 118 −3 41 −0.23 No
ACADEMIC FLUENCY 101 115 −14 10 −1.29 No
ACADEMIC APPLICATIONS 122 118 4 68 +0.46 No
ACADEMIC KNOWLEDGE 105 116 −11 16 −0.99 No
PHONEME-GRAPHEME LEDGE 111 115 −4 38 −0.31 No
ORAL LANGUAGE 112 115 −3 43 −0.18 No
BROAD ORAL LANGUAGE 116 115 1 52 +0.06 No
ORAL EXPRESSION 118 113 5 66 +0.42 No
LISTENING COMP 118 115 3 60 +0.26 No
STANDARD SCORES DISCREPANCY
COMPARISONS Actual Predicted Difference PR SD Significant at
±1.50 SD (SEE)
Oral Language/Achievement Comparisons
READING 120 110 10 79 +0.82 No
BROAD READING 113 109 4 62 +0.29 No
BASIC READING SKILLS 116 109 7 71 +0.56 No
READING COMPREHENSION 114 110 4 62 +0.31 No
READING COMP (Ext) 116 111 5 64 +0.36 No
READING FLUENCY 99 107 −8 26 −0.63 No
READING RATE 105 107 −2 44 −0.15 No
MATHEMATICS 118 109 9 76 +0.72 No
BROAD MATHEMATICS 108 109 −1 49 −0.02 No
MATH CALCULATION SKILLS 103 108 −5 36 −0.35 No
MATH PROBLEM SOLVING 142 109 33 99.5 +2.56 Yes (+)
WRITTEN LANGUAGE 115 108 7 71 +0.55 No
BROAD WRITTEN LANGUAGE 116 108 8 72 +0.58 No
BASIC WRITING SKILLS 110 109 1 53 +0.08 No
WRITTEN EXPRESSION 117 106 11 78 +0.77 No
ACADEMIC SKILLS 115 109 6 69 +0.50 No
ACADEMIC FLUENCY 101 107 −6 34 −0.42 No
ACADEMIC APPLICATIONS 122 111 11 82 +0.92 No
ACADEMIC KNOWLEDGE 105 112 −7 24 −0.72 No
PHONEME-GRAPHEME KNOWLEDGE 111 108 3 61 +0.28 No
PHONETIC CODING 120 107 13 82 +0.91 No
SPEED of LEXICAL ACCESS 85 108 −23 4 −1.77 Yes (−)
STANDARD SCORES DISCREPANCY
COMPARISONS Actual Predicted Difference PR SD Significant at
±1.50 SD (SEE)
Academic Knowledge/Achievement Comparisons
BRIEF ACHIEVEMENT 118 103 15 88 +1.19 No
BROAD ACHIEVEMENT 114 103 11 79 +0.82 No
READING 120 103 17 90 +1.31 No
BROAD READING 113 103 10 78 +0.77 No
BASIC READING SKILLS 116 103 13 85 +1.05 No
READING COMPREHENSION 114 103 11 80 +0.84 No
READING COMP (Ext) 116 103 13 84 +0.99 No
READING FLUENCY 99 102 −3 40 −0.25 No
READING RATE 105 102 3 59 +0.23 No
MATHEMATICS 118 103 15 89 +1.23 No
BROAD MATHEMATICS 108 103 5 67 +0.43 No
MATH CALCULATION SKILLS 103 103 0 50 −0.01 No
MATH PROBLEM SOLVING 142 103 39 >99.9 +3.21 Yes (+)
STANDARD SCORES DISCREPANCY
COMPARISONS Actual Predicted Difference PR SD Significant at
±1.50 SD (SEE)
Academic Knowledge/Achievement Comparisons
WRITTEN LANGUAGE 115 103 12 82 +0.93 No
BROAD WRITTEN LANGUAGE 116 103 13 83 +0.96 No
BASIC WRITING SKILLS 110 103 7 71 +0.55 No
WRITTEN EXPRESSION 117 103 14 84 +1.00 No
ACADEMIC SKILLS 115 103 12 84 +0.98 No
ACADEMIC FLUENCY 101 102 −1 47 −0.08 No
ACADEMIC APPLICATIONS 122 103 19 94 +1.56 Yes (+)
PHONETIC CODING 120 102 18 91 +1.33 No
SPEED of LEXICAL ACCESS 85 102 −17 11 −1.24 No
STANDARD SCORES DISCREPANCY
COMPARISONS Actual SAPT Predicted Difference PR SD Significant at
±1.50 SD (SEE)
Scholastic Aptitude/Achievement Comparisons
READING 120 122 115 5 67 +0.43 No
BROAD READING 113 122 116 −3 40 −0.25 No
BASIC READING SKILLS 116 114 109 7 74 +0.64 No
READING COMPREHENSION 114 122 114 0 49 −0.01 No
READING COMP (Ext) 116 122 115 1 52 +0.06 No
READING FLUENCY 99 122 114 −15 9 −1.31 No
READING RATE 105 122 115 −10 20 −0.84 No
MATHEMATICS 118 127 119 −1 47 −0.09 No
BROAD MATHEMATICS 108 127 120 −12 13 −1.14 No
MATH CALCULATION SKILLS 103 127 119 −16 7 −1.46 No
MATH PROBLEM SOLVING 142 120 113 29 99 +2.49 Yes (+)
WRITTEN LANGUAGE 115 115 110 5 68 +0.48 No
BROAD WRITTEN LANGUAGE 116 115 110 6 70 +0.51 No
BASIC WRITING SKILLS 110 114 110 0 52 +0.04 No
WRITTEN EXPRESSION 117 115 109 8 75 +0.67 No

Woodcock-Johnson IV Tests of Cognitive Abilities Test Session Observations

  1. Level of conversational proficiency: Typical for age/grade
  2. Level of cooperation: Exceptionally cooperative throughout the examination
  3. Level of activity: Typical for age/grade
  4. Attention and concentration: Unusually absorbed by the tasks
  5. Self-confidence: Appeared at ease and comfortable (typical for age/grade)
  6. Care in responding: Slow and careful in responding
  7. Response to difficult tasks: Noticeably increased level of effort for difficult tasks

Woodcock-Johnson IV Tests of Oral Language Test Session Observations

  1. Level of conversational proficiency: Advanced
  2. Level of cooperation: Exceptionally cooperative throughout the examination
  3. Level of activity: Typical for age/grade
  4. Attention and concentration: Unusually absorbed by the tasks
  5. Self-confidence: Appeared at ease and comfortable (typical for age/grade)
  6. Care in responding: Slow and careful in responding
  7. Response to difficult tasks: Noticeably increased level of effort for difficult tasks

Woodcock-Johnson IV Tests of Achievement Form A and Extended Test Session Observations

  1. Level of conversational proficiency: Typical for age/grade
  2. Level of cooperation: Exceptionally cooperative throughout the examination
  3. Level of activity: Typical for age/grade
  4. Attention and concentration: Unusually absorbed by the tasks
  5. Self-confidence: Appeared at ease and comfortable (typical for age/grade)
  6. Response to difficult tasks: Noticeably increased level of effort for difficult tasks

Woodcock-Johnson IV Tests of Achievement Form Aand Extended Qualitative Observations

  1. Letter-Word Identification: Identified initial items rapidly and accurately but had difficulty applying phoneme-grapheme relationships to later items
  2. Applied Problems: Solved initial problems with no observed difficulty but demonstrated increasing difficulties solving the later items (typical)
  3. Spelling: Spelled initial items easily and accurately; spelling of later items reflected a need for further skill development (typical)
  4. Passage Comprehension: Appeared to read initial passages easily but appeared to struggle as the reading increased in difficulty (typical)
  5. Calculation: Solved initial problems quickly with no observed difficulties but demonstrated less automaticity with the later items (typical)
  6. Writing Samples: None of the above, not observed, or does not apply
  7. Word Attack: Identified initial items rapidly and accurately and identified more difficult items through increased application of phoneme-grapheme relationships (typical)
  8. Oral Reading: Errors involving mispronunciation (1), omission (1), insertion (1), substitution (3), and repetition (3) were observed
  9. Math Facts Fluency: Solved problems slowly
  10. Sentence Writing Fluency: Wrote sentences with remarkable ease and accuracy

Psychoeducational Evaluation

Student: Brandon Goldberg
Parents: Clayton and Lillian Goldberg
Date of Birth: August 14, 2002
Age: 12–9
Date of Testing: May 28–31, 2015
School: White Mountain Middle School
Grade: 6.9

Reason for Referral

Brandon moved from his elementary school to middle school and had a particularly difficult sixth-grade year. His parents requested a comprehensive evaluation before seventh grade so as to better understand his cognitive, language, and academic strengths and weaknesses. They wanted specific information to share with teachers and to help in revising his IEP, as well as provide recommendations for specific accommodations and modifications that would help Brandon be successful in 7th grade. Brandon has previous, longstanding diagnoses of dyslexia, attention deficit hyperactivity disorder (ADHD), autism (diagnosed as Asperger's disorder), and a speech disorder.

Background Information

Brandon lives at home with his mother, father, and older brother, Joshua, age 17. Brandon's mother is a dentist, and his father is a manager at Santa Cruz Savings Bank. Dr. Goldberg supplied the following information regarding the family: As a young child, Joshua had a mild speech articulation disorder, which was easily remediated. He has no other disabilities and is attending the UC Santa Cruz on a full scholarship. Dr. Goldberg said that between her family and her husband's, there is likely a long genetic history of ADHD and autism. She commented, “I can't think how Joshua escaped it.”

Brandon was the product of a normal, full-term pregnancy and an uncomplicated birth. He weighed 9 pounds, 2 ounces at birth. His physical development was delayed. He started speaking on time but regressed because of hearing problems secondary to otitis media. He had tympanostomy tubes and a tonsillectomy. At age 3 he began speech therapy through Brava Marina School District for an articulation disorder.

Brandon attended preschool from ages 3 to 5 and then kindergarten at Del Norte Elementary School. Early in Brandon's first-grade year (age 6), Dr. David Kelly, a neuropsychologist, diagnosed him with a reading disorder and ADHD—Combined type. Brandon had negative reactions to Focalin and Concerta, which were discontinued. In his fifth-grade year, he started taking Vyvanse, starting at half the lowest dose and very gradually increasing. Currently, Brandon gets good benefit from the medication, and it is obvious when he has missed it.

In March of Brandon's first-grade year, Dr. Morris Tremaine, a pediatric neurologist, diagnosed him with low muscle tone, hyperextensibility, and coordination difficulties. That year Brandon also had surgery on his Achilles tendons to correct toe-walking. Brandon received school-based occupational therapy through fifth grade and speech therapy through sixth grade. Brandon is right-handed.

From kindergarten through grade 5, Brandon worked with Claire Hoffman, learning disabilities specialist at Del Norte School, four to five times per week in a small group. For reading and spelling intervention, she used Fundations, a highly structured, systematic, phonics-based program for younger students. She stated that Brandon's learning disabilities were the most severe she has encountered and that his poor attention and resistance to attempting difficult tasks interfered significantly with his learning. She noted that he had “a very long processing time for responding to questions or performing tasks after instructions were given,” but she could not always tell whether the problem was processing time or wandering attention. Ms. Hoffman also noted that Brandon had great difficulty with handwriting but was resistant to intervention.

At the beginning of fifth grade, at age 10, Brandon was diagnosed with Asperger's disorder by Dr. Corey Walsh, clinical psychologist. Because of Brandon's history of reading difficulty, Dr. Walsh referred him to me for consultation regarding reading instruction and accommodations. I started seeing Brandon for reading and spelling intervention, usually twice per week, in October of 2013. We use the Wilson Reading Program, and Dynamic Roots for morphology. Early on we worked on handwriting (manuscript). Brandon was and still is able to form his letters correctly when he focuses on it, but he has not practiced sufficiently to automatize good letter formation.

Because Brandon's speech disorder was impeding his word pronunciation and fluency in oral reading, I recommended that he be evaluated by a speech-language pathologist. In January 2014, at age 11–5, Brandon was evaluated by Daphne Planchard, CCC-SLP. She diagnosed Brandon with “mildly reduced oral motor skills, mild to moderate speech articulation disorder, and mild to moderate speech dysfluencies.” Brandon began speech therapy with Ms. Planchard, but scheduling conflicts prevented him from continuing.

For sixth grade, Brandon moved to White Mountain Middle School. The transition was difficult, and his assignment to his special education advocate and two of his teachers was a poor fit. In addition, he was either not doing his homework or not turning it in. Consequently, his parents hired Sam Colter, a special education teacher, as an academic coach to help him stay organized and complete his homework. This was quite successful. Last March, subsequent to a meeting requested by Dr. Goldberg, Brandon was assigned to a different special education advocate and English teacher. His mother homeschooled him for science.

Brandon is returning to White Mountain for seventh grade. Mr. Colter has transferred to White Mountain and will be his special education advocate and teacher of his study skills class. Dr. Goldberg, Mr. Colter, and I will meet with Brandon's teachers during the first two weeks of school to share the results of this evaluation and talk with the teachers about accommodations and modifications.

The results of hearing and vision screenings done in September 2014 were normal. Brandon plays soccer on a community youth team.

Personality Characteristics

Brandon is social, gets along with other students, and wants to be liked. He has a strong sense of self, does not easily take another person's perspective, and will argue for what he “knows” or believes on a topic, or his reasons for doing something differently than instructed. He is fairly inflexible about what he is willing and not willing to do. Frequently, discussions and explanation to try and change his mind are fruitless. He is highly sensitive to perceived disrespect or unfair treatment and generally responds by shutting down regarding that person or situation. These qualities have been problems at home and school, especially this past year, exacerbated by two of his teachers who, incorrectly, questioned the authenticity of his work. If Brandon trusts and respects a teacher, however, he will make every effort to impress him or her.

In school, Brandon does not advocate for himself and rarely asks for help. He does participate in discussions and volunteer answers to teacher questions. Possibly secondary to his ADHD, he is highly disorganized regarding belongings and school materials, is likely to ignore homework in a class he does not like, and frequently forgets to turn in homework he has completed.

When discussing attentional difficulties with Brandon, he explained that he thinks of full control of his attention as “having all of my pieces.” He describes levels of difficulty by the number of pieces he has lost. For example, when finishing one test, he commented, “I lost three pieces.” He said that if he gets worn out and then does something interesting, “I seem to find the ability to keep my pieces together. But when I lose a lot of pieces, I lose control; I get silly, tired, irritable.” In school, the number of pieces he has also depends on his interest in the class, teacher, or topic, or the behavior of the other students.

Test Behavior

Brandon was tested in four sessions of one to three hours each, with breaks as needed. Testing took place in my office. Because I have been providing Brandon educational therapy for well over a year, he was comfortable with me and in my office. Testing sessions were scheduled so that Brandon had full benefit from his medication. He put good effort into all of the tests and told me when he was having difficulty concentrating by referring to “losing pieces.” We took breaks then or switched to a test that I thought he would find interesting. Throughout testing, Brandon often requested encouragement regarding his performance, asking, “How badly am I doing?”

The personality characteristics described influenced Brandon's performance on some of the tests. Despite reminders that he had could use his paper and pencil, Brandon worked all of the problems on three of four math tests mentally. On one test, he used the paper to multiply 36 × 2 and then erased it. Later, when I asked him to work some of the problems on paper to show me how he solved them, he flatly refused, explaining his reason as, “I don't like to and I don't need to.” Considering the items on which he made errors, likely working on paper would have improved his scores.

Similarly, on the ten tests that had time limits, Brandon seemed to ignore my instruction to “work as fast as you can without making mistakes.” Although I emphasized “work as fast as you can” and the time limits, he worked methodically, possibly going somewhat faster than usual but seemingly unconcerned with the time limit. On four of the time-limited tests, he had to be reminded to cross out unwanted responses rather than erasing them, and on one test, he actually stopped working to explain why he erased. I stopped the timer, got him back to work, and started it again. Later, I asked him whether he had really worked as fast as he could on the time-limited tests. He said, “somewhat,” or something similar. When I asked the reason, he said that he wanted to be sure he did not make mistakes.

Consequently, his scores on time-limited tests may be spuriously low. The consistency of low scores in these tests, however, and his higher score in a numbers-only test, indicates that speed is, indeed, a problem. When helping me to write the recommendations for my meeting with his teachers, Brandon also noted that he cannot “hurry up” and that asking him to do so is likely to slow him down.

Tests Administered and Description of Scores

  1. Woodcock-Johnson IV Tests of Cognitive Ability (WJ IV COG): Tests 1–18
  2. Woodcock-Johnson IV Tests of Oral Language (WJ IV OL): Tests 1–12
  3. Woodcock-Johnson IV Tests of Achievement (WJ IV ACH): Tests 1–13, 15–20

All of the tests administered were appropriate for Brandon's ethnicity and primary language. The following were considered and ruled out as contributing factors to the referral concerns: educational disadvantage, environmental deprivation, economic factors, and primary language of the home.

The WJ IV tests were scored according to grade norms. Accordingly, Brandon's abilities and achievement are compared with those of other students who have completed sixth grade. His test performance is described as percentile ranks (PR) and their associated ranges.

Percentile Rank Range Descriptor Percentile Rank Range Descriptor
>98 Very Superior 9–24 Low Average
92–98 Superior 2–8 Low
76–91 High Average <2 Very Low
25–75 Average

Brandon's performance is sometimes described by the relative proficiency index (RPI). The RPI indicates the level of success a person is predicted to have on tasks similar to those used in the test. The comparison is always to average grade-peers who have achieved mastery, represented by 90% proficiency. For example, Brandon's RPI of 98/90 on the Calculation test indicates that he is likely to be 98% proficient on similar math tasks when a typical grade-peer would be 90% proficient. Verbal descriptors for each RPI range indicate proficiency levels and the level of difficulty with which Brandon is likely to perceive the task.

RPI Range Proficiency Instructional Implications RPI Range Proficiency Instructional Implications
100/90 Very Advanced Extremely easy 67/90 to 82/90 Limited to Average Difficult
98/90 to 100/90 Advanced Very easy 24/90 to 67/90 Limited Very difficult
95/90 to 98/90 Average to Advanced Easy 3/90 to 24/90 Very Limited Extremely difficult
82/90 to 95/90 Average Manageable 0/90 to 3/90 Extremely Limited Nearly impossible

A full set of scores is appended to this report.

Executive Summary and Conclusions

For the convenience of the reader, the Executive Summary is placed first in this report, followed by Recommendations, and then the Detailed Evaluation Results on which the previous sections were based. The most comprehensive understanding of the results of this evaluation will be gained by reading the entire report.

Brandon has well-established diagnoses of dyslexia, ADHD, autism (diagnosed as Asperger's disorder), and a speech disorder specific to articulation and dysfluency. Brandon gets good benefit from Vyvanse. Brandon's idiosyncrasies, especially his resistance to working fast under time limits, affected his work on a number of tests. This was taken into consideration in the interpretation of his performance.

Cognitive and Language Strengths

Brandon's Gf-Gc composite indicates that his general intelligence is in the Superior range when the influences of his disabilities are removed from the equation. Commensurate with his level of intelligence are his abilities to (a) use logical reasoning to solve problems in novel situations (Fluid Reasoning), (b) apply fluid reasoning to mathematics and numerical patterns (Quantitative Reasoning), (c) hold auditory information in memory for a short time (Auditory Memory Span), and (d) work with information while holding it in memory (Short-Term Working Memory). When compared with grade peers nationally, Brandon demonstrated significant strengths in Fluid Reasoning, Quantitative Reasoning, Vocabulary, and Short-Term Working Memory.

Cognitive and Language Weaknesses

Brandon demonstrated extremely slow cognitive processing, especially regarding visual symbols. All of Brandon's work with visual symbols was slow—with pictures, letters, and numbers when recalling math facts. Although Brandon was resistant to being hurried by the external pressure of a time limit, his scores are just too low for that to be the sole explanation. Accordingly, Brandon's test results indicate significant weaknesses in Cognitive Processing Speed (the ability to quickly perform relatively easy or overlearned tasks) and/or Perceptual Speed (the ability to rapidly and accurately recognize visual symbols), both of which are important in developing the lower-level skills of reading, writing, and math to an automatic level. Brandon also demonstrates a weakness in orthographic processing and retrieval, a key ability in recognizing letter patterns and whole words for reading and in retrieving those images from memory for spelling. Brandon also displayed a weakness in Speed of Lexical Access, the speed with which he can retrieve specific words from memory, especially under pressure.

When compared with Brandon's overall intellectual ability (Gf-Gc), his cognitive processing speed, perceptual speed, and the speed of retrieval for specific words are severely impaired. The discrepancy between his expected scores and his obtained scores in Cognitive Processing Speed, Perceptual Speed, and Speed of Lexical Access would be found in only three of 1,000 students, five of 100 students, and two of 1,000 students, respectively. These deficits interfere with his accuracy and ease of reading, writing, and retrieval of math facts. Likely, because of Brandon's resistance to work fast even under a time limit, his weaknesses in these abilities are not as extreme as the scores indicate. This does not, however, obviate a weakness in perceptual speed; rather, it might be the origin of Brandon's antipathy to working quickly at the perceived cost of accuracy.

Academic Achievement

When Brandon's academic skills are compared with his overall intellectual ability, his skills in computation and math problem solving are within the expected range, as is his facility with phonetic coding (which has been trained). In contrast, all of his reading skills and his speed of math fact retrieval are significantly below expectations.

At this point, Brandon's difficulties with reading and spelling are related both to perceptual speed and a weakness in orthographic processing and retrieval. Lack of automaticity in these foundational skills interfere with reading comprehension and written expression. To his credit, Brandon has made considerable progress over the past two years in his use of phonics skills and syllabication for reading and spelling. Another factor affecting Brandon's reading comprehension is a lack of strategic reading with an intention to remember what he reads.

Recommendations

Regardless of whether Brandon continues private educational therapy, the following recommendations pertain to his academic needs.

Accommodations

General

  1. Reduce the amount of work that Brandon is assigned while maintaining the intention of the assignment so that he does not have to spend more time than is expected of other students.
  2. Brandon is not able to be flexible in his work pace, and he does not work fast. Consequently, asking him to hurry may actually slow him down. If he has shown continual effort and he is not on track to complete an assignment in class, consider accepting, without penalty, as much as he has finished or allow him to complete the assignment in study skills class.
  3. Although Brandon has an exceptional vocabulary, when he feels pressured, he often cannot retrieve the specific words he wants to express his thoughts. If you are going to ask Brandon to speak about a topic in class, provide him some warning and tell him how much time he has to think about what he will say.
  4. To motivate Brandon, when having him research information or practice skills, use technology as much as possible. If he is using technology to practice skills, however, make sure that he learns to transfer them into their intended application.

Reading and Writing

  1. For textbooks and trade books, provide Brandon with recordings that he can listen to along with the book so that he can keep up with the class and participate in discussions. If possible, provide the audio file in the least obvious format, such as a tablet or cellphone with ear buds or a Bluetooth earphone. He should be able to handle worksheets and handouts if they are clear, if the lines are well spaced, and if he has enough time. It will be helpful if you review them before asking the students to work with them.
  2. Provide Brandon with a set of the teacher's notes of the class lecture/information, the handout of the slides from a PowerPoint (or similar) lecture, or a copy of the notes taken by an instructional aide or skilled classmate. Please make sure to be discreet about this.
  3. Be aware that when Brandon reads aloud, he will not slow down to figure out a word he does not recognize on sight, and his dysfluency may interfere, resulting in guessing at words he would normally be able to read, stumbling over words, and losing the context. Do not have Brandon read aloud unless he volunteers or you are evaluating his decoding.

Attention and Organization

  1. Use a simple behavioral rating chart to have the teachers monitor and document Brandon's attention and organization in every class for at least one full week. At the end of the week(s), send the chart electronically to Dr. Goldberg so that she and Brandon's doctor can decide whether a change in medication dosage is due. A sample behavior rating chart is attached.
  2. Provide a study skills class, preferably at the end of the day, for the rest of Brandon's middle and high school career. Use this class to help him make sure his planner is filled out, organize homework, complete in-class or homework assignments, make sure all completed homework has been turned in, and to have him check the district website for his grades and possible missing assignments. It is helpful to have a teacher in this role who has knowledge and a true understanding of students with disabilities, especially autism, and who can tell the difference between the emotional ramifications of a longstanding disability and noncompliance.
  3. Seat Brandon near the front of the classroom so that he is less likely to be distracted, or near where the teacher usually stands so that it is easier for the teacher to redirect him or check in with him.
  4. If you think that Brandon may not be paying attention, quietly and privately ask him if he has all of his “pieces,” and if not, what he or you can do to help him pull them together.
  5. If Brandon is speaking or answering a question, you may have to help him stay on topic or redirect him. Use guiding questions to do so.
  6. Provide Brandon with intermediate deadlines for long-term projects to help him stay on track and to provide guidance along the way. Try to set up individual meetings at each due date to review his progress and support him in planning the next step.
  7. Make sure that Brandon writes down all assignments and appointments in his planner.

Maintaining a Positive Attitude toward School

  1. Try to find ways to have Brandon help other students with work in which he is skilled and confident.
  2. Brandon responds well to sincere and meaningful praise. Tell him specifically what he has done well and why it is useful, helpful, or important.
  3. Be careful to act toward Brandon in a fair and respectful manner. Brandon can take offense at an offhand or casual remark or question, where another student might not, and he will feel alienated. For example, if you ask whether someone helped him with a piece of writing, he might assume that you are questioning the authenticity of his work. At another time, and unrelated to any assignment, you might ask him whether he gets some help with schoolwork at home and, if so, the nature of the help.
  4. Avoid arguing with Brandon. He is often unable to accept that he needs help in or reteaching of a skill, or to accept that another way of doing something might be more efficient or effective (e.g., showing his math work to avoid errors). If you can give a good explanation of what you are suggesting, do so, or, if he will pay attention, show him an example. If he is still resistant, let it go. He will have to experience the consequences to believe them.
  5. Avoid using sarcasm, even in humor, and criticizing Brandon, especially in front of other students. Brandon shuts down when he feels he is being criticized or ridiculed, and he may not be able to respond in a normal manner to the teacher after that.
  6. Try to keep a consistent routine and schedule in the classroom and at school in general. Knowing what is going to happen, and when, is important to Brandon's sense of comfort. If you have said an activity or lesson will start or end at a certain time, try to keep to the schedule. If a change is unavoidable, let Brandon know what the change will be and the reason. Brandon is not likely to be resistant, but he may not be able to engage himself in the alternate plan.

Intervention

Reading

  1. Provide Brandon with an intensive program in morphology so that he learns the meanings, spellings, and origins of affixes, Latin roots, Greek combining forms, and odd words that come from the Anglo-Saxon or Danish as each of these peoples invaded England. Brandon is interested in morphology and the history related to the sequence of influences on the English language. Provide this program in a structured and systematic manner, because that will make it easier for Brandon to learn it; however, this does not need to make it less dynamic. One resource is Unlocking Literacy: Effective Decoding and Spelling Instruction, 2nd Edition, by Marcia Henry. A good program to use is WORDS, 2nd Edition, also by Marcia Henry. If you use WORDS, start with a review of chapter : Syllable Patterns, but Brandon will be most interested in chapter 3: Anglo-Saxon, Latin, and Greek Layers of Language. Make sure he transfers the knowledge he gains into his reading decoding in context.
  2. If a Wilson Reading Program class is offered at school by a certified Wilson teacher, schedule Brandon into the class. If the class is learning reading and spelling rules that Brandon already knows, ask him to model the procedures and/or help other students.
  3. Check Brandon's oral reading fluency only as an indication of his automaticity with word identification and decoding new words. The goal is speed and accuracy in silent reading to facilitate comprehension.
  4. Teach Brandon different types of text organizational structures (e.g., comparison/contrast, opinion and support, cause and effect) and how to associate the information he reads with the appropriate framework. In this regard, he may enjoy creating and filling in these frameworks using Inspiration (www.inspiration.com). The goal is for him to work with the information and organize it for the purpose of transferring it to long-term memory and recalling it efficiently.

Writing

  1. Teach Brandon to use the same type of structures to organize his thoughts for writing. Consider using Self-Regulated Strategy Development (SRSD) for this purpose. SRSD is a well-researched and effective method for teaching organizational structures for writing. When Brandon was in fifth grade, he had some brief training in using SRSD for writing opinion papers. His subsequent work was not very detailed, but it was well organized and well stated. A brief explanation of SRSD is attached.
  2. Teach Brandon to touch-type and to use a word processing program skillfully. He has good ideas, but the physical act of writing prevents him from getting them down on paper. Regular, supervised practice will have to be incorporated into the program or either he will not do it or he will do it incorrectly.

Math

  1. Investigate whether Brandon is able to translate word problems into computation on paper and solve them, showing all of the steps. Working them mentally is fine if he is always correct, but if he is not, he will not be able to find his errors.
  2. Reteach the following math skills and provide ample practice in applying them: calculating percentages, figuring interest on money, dividing fractions, and multiplying negative integers.

Detailed Evaluation Results

Cognitive Abilities

General Intelligence

The WJ IV provides two measures of “intelligence.” The General Intelligence Ability (GIA) composite includes one measure from each of seven major cognitive abilities, including those that are representative, or are directly influenced by, a person's areas of disability. In Brandon's case, these include cognitive processing speed and recognition of letter patterns. The GIA places Brandon's intelligence in the high end of the Average range (PR 75), but the standard scores that constitute Brandon's GIA have a scatter of over 3.8 standard deviations. Consequently, the GIA is not the best representation of Brandon's intelligence.

The Gf-Gc composite represents a combination of fluid intelligence (Gf) and crystallized intelligence (Gc). The Gf-Gc composite represents the cognitive abilities of logical reasoning, language development, and information about one's environment. Brandon's scores within this composite also have a significant scatter of 1.7 standard deviations. The Gf-Gc composite places Brandon's intelligence in the Superior range, equal to or surpassing 96% of his grade peers.

The factors that matter, however, are not an overall score, but Brandon's cognitive, language, and academic areas of strengths and weakness, so that an educational plan can be developed that will give him the best opportunity for success in school and life.

Cognitive Strengths

Fluid Reasoning

Brandon has particular strengths in the ability to use inductive and deductive reasoning to form new concepts and solve problems that often involve unfamiliar information or procedures, when what one knows already is not sufficient to solve the problem (Fluid Reasoning: PR 92, RPI 98/90). Academically, these abilities are strongly related to mathematical reasoning, understanding scientific concepts and procedures, and higher-level reading comprehension. One aspect of fluid reasoning is quantitative reasoning, the ability to reason with numbers, the mathematical relations among numbers, and different operations to solve problems (Quantitative Reasoning: PR 89, RPI 98/90). This is different from quantitative knowledge—the content of math (e.g., facts, vocabulary). Quantitative reasoning is required when problems come up in daily life that do not fit neatly into mathematical routines and procedures that a person already knows. Test results indicate that Brandon's abilities in these areas exceeded 89% to 91% of his grade peers, nationally.

Memory

Brandon also has strengths in his ability to hold unrelated words (e.g., bones in the body, names of planets) and sentences in mental awareness for a few seconds (Auditory Memory Span: PR 66, RPI 95/90), useful for repeating them to himself until he is ready to do something with them. He is even more proficient with short-term working memory tasks that require him to hold information in mind and work with it by reordering it or by combining it with known information, thus transforming it (Short-Term Working Memory: PR 83, RPI 97/90). Short-term working memory is the ability that allows us to take new information and procedures and store them in long-term memory so that they are available for later use. A real-life example of a task that requires short-term working memory is combining information from the owner's manual in your car and your existing knowledge of how car radios work to learn how to program your new radio.

Brandon is also ahead of most of his grade peers in his knowledge of information regarding locations and uses of objects in his environment (General Information: PR 77, RPI 97/90).

Average Cognitive Abilities

Although Brandon performed in the Average range on the measures of Auditory Processing (PR 43, RPI 88/90), he had difficulty retrieving words from long-term memory. Two of three tasks involved coming up with words incorporating a specific sound. At the end of the first test, he commented, “I'm only good for short bursts. I wear out quickly.” The second test involved generating words starting with a specific sound. Several times, he paused for over 10 seconds, even after a prompt to continue, trying to think of another word. His dysfluencies on these tests were minimal and did not affect his score. Brandon had no difficulty with the third task: Substituting one sound for another in a given word to make a new word (e.g., Change the /ō/ in groan to /ā/.) This test did not require the same type of retrieval from long-term memory.

The tests of Long-Term Retrieval (PR 39, RPI 88/90), require the student to learn information and then recall or apply it over a few minutes. Brandon's performance places his ability to recall recently learned information in the Average range compared with his grade peers but compared with his overall intelligence (Gf-Gc), it is a relative weakness. Only eight of 100 grade peers would have a discrepancy of this size between overall intellectual ability and Long-Term Retrieval. Furthermore, when retelling brief stories (presented on a recording), the number of details he gave varied greatly from story to story. The total, however, was sufficient for his score to fall in the lower end of the Average range.

Cognitive Weaknesses

Brandon demonstrated a significant weakness in the area of Cognitive Processing Speed, the ability to quickly and fluently perform relatively easy or over-learned tasks, especially when focused concentration is required. He scored as well as or better than only 3% of grade peers in the norming sample. The two tests required him to scan rows of common letter combinations (e.g., sh, or), or pictures, and circle like pairs (Letter-Pattern Matching: PR 9, RPI 22/90; Pair Cancellation, PR 2, RPI 5/90). What is interesting is that he performed significantly better, well into the Average range, when doing the same type of scanning and matching task with numbers (Number-Pattern Matching: PR 45, RPI 86/90). He was given the number matching test sometime after the letter matching test. Looking at the trial items, he commented, “Numbers will be a lot easier.” And so they were. He appeared to work faster, and because he made no errors, he did not erase, saving those few seconds of time. It's likely that Brandon's strong quantitative reasoning and math skills (see Academic Achievement: Math), his apparent comfort with numbers, and his apparent motivation to work fast on this test compensated for his slow processing.

Oral Language

Oral Language Strengths

Brandon demonstrated specific strengths in the language abilities of phonetic coding and vocabulary knowledge. Phonetic coding is the ability to blend sounds together to make whole words and to segment words into their individual sounds. On these tests, Brandon scored as well as or better than 88% of his grade-peers. In accordance with the Wilson reading and spelling programs, Brandon has been trained in phonetic coding since kindergarten, with a new focus since starting private tutoring. Consequently, one would expect strong results.

Brandon's ability to give synonyms and antonyms for words of increasing infrequency (Oral Vocabulary) was in the Superior range, demonstrating both breadth and flexibility in his knowledge of word meanings and expressive vocabulary. Brandon's performance equaled or surpassed 99% of his grade-peers.

Brandon's other language abilities, except for speed of word retrieval (see next discussion) were in the upper end of the Average range.

Oral Language Weaknesses

Again, Brandon had difficulty in the speed with which he retrieved words from long-term memory (Speed of Lexical Access: PR 2, RPI 39/90). He was considerably slower when naming pictures of common objects (e.g., fork, cat) than when allowed the flexibility to come up with any word within a given category (e.g., things you can wear). On these tests, Brandon performed as well as or better than only 1% and 23% of his grade peers, respectively. Brandon did have some dysfluency during these tests but also had many long pauses as he tried to think of the next word. Comparison of some of his language-related test scores suggested that his ability to access the words he wants to use is improved when he is not restricted to finding one specific word. (Oral Vocabulary: PR 99 vs. Picture Vocabulary: PR 64; Retrieval Fluency: PR 23 vs. Rapid Picture Naming: PR 1).

Academic Achievement

Math

Calculation and Problem Solving

Test results indicate that math calculation and problem solving are Brandon's strongest academic skills, at or above 91% of his grade peers. Notably, he did all of this work, except on the Calculation test, mentally. He is logical in thinking about numbers and seeing numerical patterns. Despite his scores, Brandon needs review regarding how to calculate percentages, divide fractions, and multiply negative integers. He also needs practice in applying these skills. He commented that he did not know what “interest” meant when referring to money (Mathematics: PR 91, RPI 99/90; Math Problem Solving: PR 91, RPI 98/90; Number Matrices: PR 77, RPI 97/90; Number Series: PR 90, RPI 99/90).

Math Facts Fluency

Brandon did not seem to be having difficulty but was but very slow in answering simple, one-digit addition and subtraction facts. He did not get far enough for multiplication. His two errors were adding instead of subtracting. His speed in answering simple math facts would equal or surpass that of only 10% of his grade peers (Math Facts Fluency: PR 10, RPI 24/90).

Reading

Basic Reading Skills

Based on the results of the WJ IV ACH, Brandon's Basic Reading Skills, the ability to sound out both real words and words he has never seen (nonsense words that follow English spelling rules), outside of a meaningful context, is in the lower end of the Average range, or higher than 38% of grade peers nationally. The word identification test (real words), which contained irregular words, gave Brandon more trouble. Brandon's RPI indicates that when a typical grade peer would be 90% successful on a similar task, Brandon would be 78% successful. Because I did not hear a word, I had him reread that page. He got that word right, in addition to two others (not credited) that he had previously mispronounced. Most of his mispronunciations were close to accurate and were on words probably unfamiliar to him. Most of his syllable divisions were accurate. His pronunciation of nonsense words was in the mid-average range. He made errors on some word elements that he has pronounced correctly in the past but he likely benefited from the fact that nonsense words can only have regular pronunciations.

Reading Comprehension

The Passage Comprehension test (PR 26, RPI 76/90) has one- to two-sentence passages with a missing word that the reader needs to fill in. Brandon's responses, even those that were incorrect, indicated that he read the passages accurately. The words he provided fit the syntax of the sentence and were semantically related but indicated insufficient thought regarding the content. I considered a possible difficulty with word retrieval but Brandon did not demonstrate behaviors typical of a word finding problem, and he did not pick up on specific clues within the passages that would have guided him to the right response.

Brandon's recall of details in brief passages that he read silently was similar to his recall of short stories that he heard (Reading Recall, PR 35, RPI 85/90; Story Recall, PR 37, RPI 87/90). Both were in the lower end of Average. Before starting the test, I asked about his attention, and he said it was fine. His recall of the first four passages was variable, but on the fifth, he recalled only 3 of 15 details. He commented that the story “didn't leave an impression.” On the sixth (and last) story, he recalled only 4 of 17 details.

On Brandon's last informal reading inventory, in March 2014, his instructional and independent oral reading levels for expository material varied between fifth and sixth grades, depending on his familiarity with the material and his attention. On the same passages, however, his responses to questions were at the instructional to frustration levels, even on the sixth-grade passage in which the content was familiar, and his oral reading accuracy was 98%. Brandon would benefit from reading comprehension strategies to increase both his comprehension and memory.

Reading Rate

Brandon reads slowly. His reading rate was assessed on a variety of tasks, all of which required him to understand what he was reading. The results indicate that Brandon's reading rate is very limited, higher than only 12% of his grade-peers. His RPI of 9/90 indicates that when a typical student at his grade level is 90% proficient in reading at a rate appropriate for his grade level, Brandon will be only 9% proficient.

Writing

Written Expression

Brandon was asked to write individual sentences, each one according to different instructions. He perceived this test as very difficult, although most of his sentences were of good quality, equal to or better than 78% of those of his grade-peers. He needed firm “encouragement” to begin after being told that he could not dictate his answers. Part of the instructions were to “write neatly,” which takes considerable focus for Brandon. He printed slowly and uncharacteristically neatly, periodically stopping to fix perfectly legible letters. Fortunately, this is not a time-limited test. Although the student does not know this, spelling and handwriting do not count toward or against the score.

The WJ IV does not assess longer pieces of writing. In our work together, however, Brandon has consistently demonstrated difficulty organizing his thoughts and information for writing assignments. He benefits from thorough training in organizational strategies and support in their application.

Writing Rate

The score on Sentence Writing Fluency is invalid and so will not be reported. My experience with Brandon is that his handwriting is either neat, slow, and labored, or fast and sloppy. In this case, Brandon completely ignored the time limit and instructions to work fast. He wrote slowly, erased once, fixed letters that were fine as is, went back to previous sentences to add punctuation, and took time to fold his booklet just right after turning the page. Brandon's need to be precise compromised the intent of the test.

Spelling

In general, Brandon's spelling is poor, but he was more proficient in spelling nonsense words (PR 27, RPI 80/90) than real words (PR 18, RPI 54/90). On both lists, his error words were phonetically correct but did not follow spelling conventions (e.g., obsirv/observe). His spelling of real words may have been lower because two of the words he missed were irregular. His errors included both spelling rules that he has learned in tutoring and spelling rules that have not yet been introduced but that should be automatic to a student in middle school.

Academic Knowledge

Brandon demonstrated good knowledge of academic content, answering questions in the areas of science, social studies, and humanities. His performance equaled or surpassed 76% of others at his grade level. His RPI indicates that his knowledge is average to advanced.

Brief Summary

Brandon is a young man with reasoning and knowledge in the Superior range. When compared with grade peers in the norming sample, he has significant strengths in fluid reasoning, quantitative reasoning, auditory memory span, short-term working memory, phonetic coding, math computation and problem solving, vocabulary, and general knowledge. He has significant weaknesses in cognitive processing speed and/or perceptual speed, speed of lexical access, and orthographic processing and retrieval. These deficits are major contributing factors to his weak proficiency in all measures of word identification, reading comprehension, reading speed, spelling, and speed of math fact recall. Based on history and qualitative analysis of Brandon's class assignments, he also has severe difficulty organizing his thoughts for writing and in the mechanics of handwriting.

Working with Brandon is a pleasure and always interesting. Please do not hesitate to call or email me with any questions regarding this report.

Table of Scores

Woodcock-Johnson IV Tests of Cognitive Abilities (Norms based on grade 6.9)

CLUSTER/Tests GE SS (68% Band) PR RPI Proficiency
GEN INTELLECTUAL ABIL 8.8 109 (105–113) 94/90 72 Average
Oral Vocabulary >17.9 137 (132–143) 99/90 99 Advanced
Number Series >14.0 119 (113–124) 99/90 90 Advanced
Verbal Attention 7.6 102 (96–109) 92/90 56 Average
Letter-Pattern Matching 3.4 80 (72–87) 22/90 9 Very Limited
Phonological Processing 6.7 99 (92–106) 90/90 48 Average
Story Recall 5.2 95 (90–100) 87/90 37 Average
Visualization 6.6 99 (94–105) 90/90 48 Average
Gf-Gc COMPOSITE >17.9 126 (123–130) 99/90 96 Advanced
Oral Vocabulary >17.9 137 (132–143) 99/90 99 Advanced
Number Series >14.0 119 (113–124) 99/90 90 Advanced
General Information 11.1 111 (106–116) 97/90 77 Average to Advanced
Concept Formation >14.7 117 (112–123) 99/90 88 Advanced
COMP-KNOWLEDGE (Gc) 14.6 123 (119–127) 99/90 94 Advanced
Oral Vocabulary >17.9 137 (132–143) 99/90 99 Advanced
General Information 11.1 111 (106–116) 97/90 77 Average to Advanced
COMP-KNOWLEDGE (Ext) 13.0 118 (115–122) 98/90 89 Advanced
Oral Vocabulary >17.9 137 (132–143) 99/90 99 Advanced
General Information 11.1 111 (106–116) 97/90 77 Average to Advanced
Picture Vocabulary 8.9 106 (100–112) 94/90 64 Average
FLUID REASONING (Gf) >14.4 122 (117–126) 99/90 93 Advanced
Number Series >14.0 119 (113–124) 99/90 90 Advanced
Concept Formation >14.7 117 (112–123) 99/90 88 Advanced
FLUID REASONING (Ext) >14.2 121 (117–125) 98/90 92 Advanced
Number Series >14.0 119 (113–124) 99/90 90 Advanced
Concept Formation >14.7 117 (112–123) 99/90 88 Advanced
Analysis-Synthesis 13.0 113 (107–119) 98/90 81 Advanced
S-TERM WORK MEM (Gwm) 11.3 111 (107–116) 96/90 78 Average to Advanced
Verbal Attention 7.6 102 (96–109) 92/90 56 Average
Numbers Reversed 13.0 116 (111–121) 98/90 85 Advanced
Object-Number Sequencing 15.0 114 (109–119) 98/90 98/90 Advanced
COG PROCESS SPEED (Gs) 2.9 72 (66–78) 10/90 10/90 Very Limited
Letter-Pattern Matching 3.4 80 (72–87) 22/90 22/90 Very Limited
Pair Cancellation 2.4 70 (63–77) 5/90 5/90 Very Limited
AUDITORY PROCESS (Ga) 6.1 97 (93–102) 88/90 88/90 Average
Phonological Processing 6.7 99 (92–106) 90/90 90/90 Average
Nonword Repetition 5.6 97 (92–101) 86/90 86/90 Average
L-TERM RETRIEVAL (Glr) 5.2 96 (92–99) 87/90 87/90 Average
Story Recall 5.2 95 (90–100) 87/90 87/90 Average
Visual-Auditory Learning 5.3 97 (93–101) 88/90 88/90 Average
VISUAL PROCESSING (Gv) 6.9 100 (95–105) 90/90 90/90 Average
Visualization 6.6 99 (94–105) 90/90 90/90 Average
Picture Recognition 7.2 100 (93–108) 90/90 90/90 Average
QUANTITATIVE REASONING >13.8 119 (114–123) 98/90 98/90 Advanced
Number Series >14.0 119 (113–124) 99/90 99/90 Advanced
Analysis-Synthesis 13.0 113 (107–119) 98/90 98/90 Advanced
AUDITORY MEMORY SPAN 9.5 106 (101–111) 95/90 95/90 Average to Advanced
Memory for Words 8.9 104 (98–110) 94/90 94/90 Average
Sentence Repetition 10.0 107 (100–113) 96/90 96/90 Average to Advanced
NUMBER FACILITY 8.4 108 (101–114) 95/90 95/90 Average to Advanced
Numbers Reversed 13.0 116 (111–121) 98/90 98/90 Advanced
Number-Pattern Matching 6.5 98 (91–105) 86/90 86/90 Average
PERCEPTUAL SPEED 5.0 87 (81–93) 57/90 57/90 Limited
Letter-Pattern Matching 3.4 80 (72–87) 22/90 22/90 Very Limited
Number-Pattern Matching 6.5 98 (91–105) 86/90 86/90 Average
VOCABULARY 13.8 121 (117–126) 98/90 98/90 Advanced
Oral Vocabulary >17.9 137 (132–143) 99/90 99/90 Advanced
Picture Vocabulary 8.9 106 (100–112) 94/90 94/90 Average
COGNITIVE EFFICIENCY 5.7 94 (88–100) 81/90 81/90 Limited to Average
Letter-Pattern Matching 3.4 80 (72–87) 22/90 22/90 Very Limited
Numbers Reversed 13.0 116 (111–121) 98/90 98/90 Advanced
COG EFFICIENCY (Ext) 6.3 96 (91–101) 86/90 86/90 Average
Verbal Attention 7.6 102 (96–109) 92/90 92/90 Average
Letter-Pattern Matching 3.4 80 (72–87) 22/90 22/90 Very Limited
Numbers Reversed 13.0 116 (111–121) 98/90 98/90 Advanced
Number-Pattern Matching 6.5 98 (91–105) 86/90 86/90 Average

Woodcock-Johnson IV Tests of Oral Language (Norms based on grade 6.9)

CLUSTER/Tests GE SS (68% Band) PR RPI Proficiency
ORAL LANGUAGE 9.5 108 (103–113) 70 94/90 Average
Picture Vocabulary 8.9 106 (100–112) 64 94/90 Average
Oral Comprehension 10.4 109 (102–116) 72 95/90 Average to Advanced
BROAD ORAL LANGUAGE 9.0 107 (102–112) 68 94/90 Average
Picture Vocabulary 8.9 106 (100–112) 64 94/90 Average
Oral Comprehension 10.4 109 (102–116) 72 95/90 Average to Advanced
Understanding Directions 7.8 102 (96–108) 56 92/90 Average
ORAL EXPRESSION 9.4 108 (102–113) 69 95/90 Average to Advanced
Picture Vocabulary 8.9 106 (100–112) 64 94/90 Average
Sentence Repetition 10.0 107 (100–113) 68 96/90 Average to Advanced
LISTENING COMP 9.1 107 (101–112) 67 93/90 Average
Oral Comprehension 10.4 109 (102–116) 72 95/90 Average to Advanced
Understanding Directions 7.8 102 (96–108) 56 92/90 Average
PHONETIC CODING >17.9 117 (113–122) 88 98/90 Advanced
Segmentation >17.9 111 (106–116) 76 97/90 Average to Advanced
Sound Blending >17.9 118 (112–124) 89 99/90 Advanced
SPEED of LEXICAL ACCESS 1.8 70 (65–76) 2 39/90 Limited
Rapid Picture Naming K.8 67 (61–74) 1 9/90 Very Limited
Retrieval Fluency 4.1 89 (83–95) 23 80/90 Limited to Average
VOCABULARY 13.8 121 (117–126) 92 98/90 Advanced
Picture Vocabulary 8.9 106 (100–112) 64 94/90 Average
Oral Vocabulary >17.9 137 (132–143) 99 99/90 Advanced
AUDITORY MEMORY SPAN 9.5 106 (101–111) 66 95/90 Average to Advanced
Sentence Repetition 10.0 107 (100–113) 68 96/90 Average to Advanced
Memory for Words 8.9 104 (98–110) 59 94/90 Average

Woodcock-Johnson IV Tests of Achievement Form A and Extended (Norms based on grade 6.9)

CLUSTER/Tests GE SS (68% Band) PR RPI Proficiency
READING 5.3 92 (89–95) 30 77/90 Limited to Average
Letter-Word Identification 5.5 94 (90–98) 33 78/90 Limited to Average
Passage Comprehension 4.9 90 (85–96) 26 76/90 Limited to Average
BROAD READING 4.1 83 (80–87) 13 42/90 Limited
Letter-Word Identification 5.5 94 (90–98) 33 78/90 Limited to Average
Passage Comprehension 4.9 90 (85–96) 26 76/90 Limited to Average
Sentence Reading Fluency 3.3 77 (72–83) 6 3/90 Extremely Limited
BASIC READING SKILLS 5.8 96 (92–99) 38 84/90 Average
Letter-Word Identification 5.5 94 (90–98) 33 78/90 Limited to Average
Word Attack 6.4 99 (94–104) 47 89/90 Average
READING COMPREHENSION 5.0 91 (87–95) 28 81/90 Limited to Average
Passage Comprehension 4.9 90 (85–96) 26 76/90 Limited to Average
Reading Recall 5.2 94 (91–98) 35 85/90 Average
READING COMP (Ext) 6.5 98 (95–102) 45 89/90 Average
Passage Comprehension 4.9 90 (85–96) 26 76/90 Limited to Average
Reading Recall 5.2 94 (91–98) 35 85/90 Average
Reading Vocabulary 10.7 111 (106–117) 77 96/90 Average to Advanced
READING FLUENCY 3.7 81 (77–85) 10 26/90 Limited
Oral Reading 5.1 93 (89–97) 31 79/90 Limited to Average
Sentence Reading Fluency 3.3 77 (72–83) 6 3/90 Extremely Limited
READING RATE 3.8 82 (78–87) 12 9/90 Very Limited
Sentence Reading Fluency 3.3 77 (72–83) 6 3/90 Extremely Limited
Word Reading Fluency 4.4 87 (80–94) 20 23/90 Very Limited
MATHEMATICS 13.0 120 (116–124) 91 99/90 Advanced
Applied Problems >17.9 123 (117–128) 93 99/90 Advanced
Calculation 10.4 113 (108–118) 81 98/90 Advanced
BROAD MATHEMATICS 7.4 103 (99–106) 57 92/90 Average
Applied Problems >17.9 123 (117–128) 93 99/90 Advanced
Calculation 10.4 113 (108–118) 81 98/90 Advanced
Math Facts Fluency 3.9 81 (76–86) 10 24/90 Very Limited
MATH CALC SKILLS 5.9 94 (90–98) 34 79/90 Limited to Average
Calculation 10.4 113 (108–118) 81 98/90 Advanced
Math Facts Fluency 3.9 81 (76–86) 10 24/90 Very Limited
MATH PROBLEM SOLVING 16.6 120 (115–124) 91 98/90 Advanced
Applied Problems >17.9 123 (117–128) 93 99/90 Advanced
Number Matrices 13.0 111 (106–116) 77 97/90 Average to Advanced
WRITTEN LANGUAGE 6.3 97 (94–100) 42 87/90 Average
Spelling 4.6 86 (83–90) 18 54/90 Limited
Writing Samples 13.0 112 (107–116) 78 97/90 Average to Advanced
BROAD WRITTEN LANG 4.9 88 (85–92) 22 74/90 Limited to Average
Spelling 4.6 86 (83–90) 18 54/90 Limited
Writing Samples 13.0 112 (107–116) 78 97/90 Average to Advanced
Sentence Writing Fluency 2.5 70 (63–77) 2 36/90 Limited
WRITTEN EXPRESSION 5.2 92 (88–97) 30 82/90 Average
Writing Samples 13.0 112 (107–116) 78 97/90 Average to Advanced
Sentence Writing Fluency 2.5 70 (63–77) 2 36/90 Limited
ACADEMIC SKILLS 6.3 96 (94–99) 41 85/90 Average
Letter-Word Identification 5.5 94 (90–98) 33 78/90 Limited to Average
Spelling 4.6 86 (83–90) 18 54/90 Limited
Calculation 10.4 113 (108–118) 81 98/90 Advanced
ACADEMIC FLUENCY 3,4 76 (72–79) 5 15/90 Very Limited
Sentence Reading Fluency 3.3 77 (72–83) 6 3/90 Extremely Limited
Math Facts Fluency 3.9 81 (76–86) 10 24/90 Very Limited
Sentence Writing Fluency 2.5 70 (63–77) 2 36/90 Limited
ACADEMIC APPLICATIONS 10.0 110 (107–114) 75 96/90 Average to Advanced
Applied Problems >17.9 123 (117–128) 93 99/90 Advanced
Passage Comprehension 4.9 90 (85–96) 26 76/90 Limited to Average
Writing Samples 13.0 112 (107–116) 78 97/90 Average to Advanced
ACADEMIC KNOWLEDGE 10.2 110 (107–114) 76 96/90 Average to Advanced
Science 9.1 105 (100–111) 64 94/90 Average
Social Studies 12.3 114 (108–119) 82 98/90 Advanced
Humanities 9.3 107 (102–113) 69 95/90 Average to Advanced
PHONEME-GRAPH KNOW 5.2 95 (91–99) 36 85/90 Average
Word Attack 6.4 99 (94–104) 47 89/90 Average
Spelling of Sounds 4.1 91 (85–96) 27 80/90 Limited to Average
STANDARD SCORES DISCREPANCY
VARIATIONS Actual Predicted Difference PR SD Interpretation at
±1.50 SD (SEE)
Intra-Cognitive [Extended] Variations
COMP-KNOWLEDGE (Gc) 123 99 24 98 +1.98 Strength
COMP-KNOWLEDGE (Ext) 118 99 19 94 +1.59 Strength
FLUID REASONING (Gf) 122 102 20 96 +1.73 Strength
FLUID REASONING (Ext) 121 102 19 95 +1.65 Strength
S-TERM WORK MEM (Gwm) 111 105 6 72 +0.57
S-TERM WORK MEM (Ext) 114 105 9 80 +0.84
COG PROCESS SPEED (Gs) 72 105 −33 1 −2.44 Weakness
AUDITORY PROCESS (Ga) 97 105 −8 25 −0.67
L-TERM RETRIEVAL (Glr) 96 104 −8 24 −0.69
VISUAL PROCESSING (Gv) 100 104 −4 39 −0.27
QUANTITATIVE REASONING 119 102 17 93 +1.48
AUDITORY MEMORY SPAN 106 104 2 56 +0.16
PERCEPTUAL SPEED 87 106 −19 8 −1.38
VOCABULARY 121 99 22 97 +1.91 Strength
ORAL LANGUAGE 108 99 9 76 +0.70
PHONETIC CODING 117 104 13 84 +0.98
SPEED of LEXICAL ACCESS 70 105 −35 1 −2.52 Weakness
Oral Vocabulary 137 99 38 >99.9 +3.23 Strength
Number Series 119 102 17 92 +1.39
Verbal Attention 102 104 −2 44 −0.14
Letter-Pattern Matching 80 105 −25 4 −1.80 Weakness
Phonological Processing 99 105 −6 33 −0.45
Story Recall 95 104 −9 25 −0.66
Visualization 99 103 −4 39 −0.29
General Information 111 99 12 81 +0.88
Concept Formation 117 102 15 88 +1.17
Numbers Reversed 116 104 12 82 +0.93
Number-Pattern Matching 98 105 −7 30 −0.53
Nonword Repetition 97 104 −7 30 −0.53
Visual-Auditory Learning 97 103 −6 33 −0.44
Picture Recognition 100 103 −3 44 −0.15
STANDARD SCORES DISCREPANCY
VARIATIONS Actual Predicted Difference PR SD Interpretation at
±1.50 SD (SEE)
Intra-Cognitive [Extended] Variations
Analysis-Synthesis 113 102 11 81 +0.88
Object-Number Sequencing 114 104 10 79 +0.81
Pair Cancellation 70 104 −34 1 −2.43 Weakness
Memory for Words 104 104 0 50 0.00
Picture Vocabulary 106 99 7 69 +0.50
Oral Comprehension 109 99 10 77 +0.74
Segmentation 111 104 7 69 +0.50
Rapid Picture Naming 67 105 −38 0.5 −2.59 Weakness
Sentence Repetition 107 104 3 60 +0.25
Understanding Directions 102 104 −2 44 −0.15
Sound Blending 118 103 15 86 +1.06
Retrieval Fluency 89 106 −17 12 −1.19
Number Matrices 111 102 9 75 +0.68
STANDARD SCORES DISCREPANCY
VARIATIONS Actual Predicted Difference PR SD Interpretation at
±1.50 SD (SEE)
Intra-Oral Language [Extended] Variations
ORAL EXPRESSION 108 96 12 83 +0.96
LISTENING COMP 107 95 12 85 +1.04
PHONETIC CODING 117 97 20 92 +1.42
SPEED of LEXICAL ACCESS 70 105 −35 0.5 −2.56 Weakness
VOCABULARY 121 96 25 99 +2.23 Strength
AUDITORY PROCESS (Ga) 97 96 1 54 +0.09
Picture Vocabulary 106 96 10 77 +0.75
Oral Comprehension 109 95 14 87 +1.15
Segmentation 111 98 13 82 +0.90
Rapid Picture Naming 67 104 −37 0.4 −2.64 Weakness
Sentence Repetition 107 97 10 77 +0.74
Understanding Directions 102 96 6 69 +0.49
Sound Blending 118 97 21 93 +1.46
Retrieval Fluency 89 105 −16 13 −1.14
Oral Vocabulary 137 96 41 >99.9 +3.35 Strength
Phonological Processing 99 96 3 59 +0.22
Nonword Repetition 97 97 0 49 −0.03
STANDARD SCORES DISCREPANCY
VARIATIONS Actual Predicted Difference PR SD Interpretation at
±1.50 SD (SEE)
Intra-Achievement [Extended] Variations
READING COMP (Ext) 98 106 −8 22 −0.79
READING FLUENCY 81 104 −23 2 −2.13 Weakness
READING RATE 82 104 −22 3 −1.89 Weakness
MATH CALCULATION SKILLS 94 101 −7 27 −0.62
MATH PROBLEM SOLVING 120 99 21 96 +1.80 Strength
WRITTEN EXPRESSION 92 101 −9 21 −0.82
Letter-Word Identification 94 105 −11 11 −1.21
Applied Problems 123 99 24 98 +2.13 Strength
Spelling 86 106 −20 3 −1.88 Weakness
Passage Comprehension 90 105 −15 8 −1.43
Calculation 113 101 12 89 +1.22
Writing Samples 112 101 11 81 +0.89
Word Attack 99 104 −5 33 −0.44
Oral Reading 93 104 −11 19 −0.89
Sentence Reading Fluency 77 104 −27 1 −2.46 Weakness
Math Facts Fluency 81 101 −20 5 −1.64 Weakness
Sentence Writing Fluency 70 101 −31 0.4 −2.64 Weakness
Reading Recall 94 104 −10 23 −0.73
Number Matrices 111 99 12 80 +0.85
Word Reading Fluency 87 104 −17 10 −1.26
Spelling of Sounds 91 105 −14 13 −1.13
Reading Vocabulary 111 105 6 73 +0.60
STANDARD SCORES DISCREPANCY
VARIATIONS Actual Predicted Difference PR SD Interpretation at
±1.50 SD (SEE)
Academic Skills/Academic Fluency/Academic Applications [Extended] Variations
ACADEMIC SKILLS 96 93 3 65 +0.39
ACADEMIC FLUENCY 76 102 −26 0.4 −2.63 Weakness
ACADEMIC APPLICATIONS 110 88 22 99 +2.35 Strength
COG PROCESS SPEED (Gs) 72 102 −30 1 −2.18 Weakness
PERCEPTUAL SPEED 87 102 −15 13 −1.13
READING RATE 82 102 −20 4 −1.76 Weakness
STANDARD SCORES DISCREPANCY
VARIATIONS Actual Predicted Difference PR SD Interpretation at
±1.50 SD (SEE)
Gf-Gc Composite/Other Ability Comparisons
S-TERM WORK MEM (Gwm) 111 115 −4 39 −0.29
S-TERM WORK MEM (Ext) 114 116 −2 46 −0.11
COG PROCESS SPEED (Gs) 72 111 −39 0.3 −2.77 Weakness
PERCEPTUAL SPEED 87 111 −24 5 −1.69 Weakness
SPEED of LEXICAL ACCESS 70 110 −40 0.2 −2.83 Weakness
AUDITORY PROCESS (Ga) 97 115 −18 8 −1.41
PHONETIC CODING 117 113 4 64 +0.35
L-TERM RETRIEVAL (Glr) 96 113 −17 8 −1.38
VISUAL PROCESSING (Gv) 100 112 −12 19 −0.86
AUDITORY MEMORY SPAN 106 111 −5 35 −0.39
NUMBER FACILITY 108 113 −5 33 −0.45
COGNITIVE EFFICIENCY 94 114 −20 7 −1.51 Weakness
COGNITIVE EFFICIENCY (Ext) 96 115 −19 7 −1.50 Weakness
BRIEF ACHIEVEMENT 99 120 −21 2 −2.01 Weakness
READING 92 119 −27 1 −2.40 Weakness
BROAD READING 83 118 −35 <0.1 −3.30 Weakness
BASIC READING SKILLS 96 116 −20 5 −1.69 Weakness
READING COMPREHENSION 91 118 −27 1 −2.37 Weakness
READING COMP (Ext) 98 120 −22 2 −2.02 Weakness
READING FLUENCY 81 116 −35 0.2 −2.90 Weakness
READING RATE 82 114 −32 1 −2.52 Weakness
MATHEMATICS 120 118 2 57 +0.18
BROAD MATHEMATICS 103 118 −15 7 −1.46
MATH CALCULATION SKILLS 94 116 −22 3 −1.85 Weakness
MATH PROBLEM SOLVING 120 119 1 52 +0.05
WRITTEN LANGUAGE 97 117 −20 5 −1.60 Weakness
BROAD WRITTEN LANGUAGE 88 117 −29 1 −2.33 Weakness
WRITTEN EXPRESSION 92 113 −21 6 −1.54 Weakness
ACADEMIC SKILLS 96 119 −23 2 −2.01 Weakness
ACADEMIC FLUENCY 76 115 −39 <0.1 −3.33 Weakness
ACADEMIC APPLICATIONS 110 119 −9 18 −0.91
PHONEME-GRAPHEME KNOWLEDGE 95 114 −19 7 −1.50 Weakness
STANDARD SCORES DISCREPANCY
VARIATIONS Actual Predicted Difference PR SD Interpretation at
±1.50 SD (SEE)
Oral Language/Achievement Comparisons
READING 92 104 −12 15 −1.02 No
BROAD READING 83 104 −21 4 −1.70 Yes (−)
BASIC READING SKILLS 96 104 −8 25 −0.68 No
READING COMPREHENSION 91 105 −14 14 −1.06 No
READING COMP (Ext) 98 105 −7 28 −0.58 No
READING FLUENCY 81 103 −22 5 −1.68 Yes (−)
READING RATE 82 103 −21 6 −1.54 Yes (−)
MATHEMATICS 120 104 16 89 +1.23 No
BROAD MATHEMATICS 103 104 −1 45 −0.12 No
MATH CALCULATION SKILLS 94 104 −10 25 −0.69 No
MATH PROBLEM SOLVING 120 104 16 89 +1.24 No
WRITTEN LANGUAGE 97 104 −7 32 −0.48 No
BROAD WRITTEN LANG 88 104 −16 13 −1.10 No
WRITTEN EXPRESSION 92 103 −11 23 −0.73 No
ACADEMIC SKILLS 96 104 −8 28 −0.58 No
ACADEMIC FLUENCY 76 103 −27 2 −2.05 Yes (−)
ACADEMIC APPLICATIONS 110 105 5 67 +0.44 No
ACADEMIC KNOWLEDGE 110 105 5 70 +0.54 No
PHONEME-GRAPHEME KNOWLEDGE 95 103 −8 26 −0.64 No
PHONETIC CODING 117 103 14 84 +1.01 No
SPEED of LEXICAL ACCESS 70 104 −34 0.5 −2.55 Yes (−)

Woodcock-Johnson IV Tests of Achievement Form Aand Extended Test Session Observations

  1. Level of conversational proficiency: Advanced
  2. Level of cooperation: Cooperative (typical for age/grade)
  3. Level of activity: Typical for age/grade
  4. Attention and concentration: Attentive to the tasks (typical for age/grade)
  5. Self-confidence: Appeared tense or worried at times
  6. Care in responding: Prompt but careful in responding (typical for age/grade)
  7. Response to difficult tasks: Generally persisted Form A with difficult tasks (typical for age/grade)

Woodcock-Johnson IV Tests of Achievement Form Aand Extended Qualitative Observations

  1. Oral Reading: Errors involving mispronunciation (9), omission (2), insertion (1), and ignoring punctuation (1) were observed.

Reading Assessment

Name: Glenn Van Houten
Parents: Hans and Steve Van Houten
Date of Birth: November 2, 2001
Age: 13–2
Dates of Testing: January 2, 4, 5, and 9, 2015
School: Woodlawn Preparatory School
Grade: 7.4
Date of Report: January 15, 2015

Reason for Referral

Dr. Frances Firek, neuropsychologist, evaluated Glenn for possible learning problems in November of 2014. Based on her findings, she recommended that the possibility of a reading disability should be investigated further. This assessment is a follow-up to Dr. Firek's recommendation.

Background Information

Glenn lives with his two fathers, Hans and Steve Van Houten. The Van Houtens adopted Glenn as a 3-year-old and do not have information regarding his biological history. Glenn's fathers have pursued multiple psychological and psychiatric assessments, frequently receiving different diagnoses and recommendations for treatment. To date, Glenn has been diagnosed with anxiety disorder, dysthymia, Tourette syndrome, bipolar disorder, and attention deficit/hyperactivity disorder (ADHD). The reader is referred to Dr. Firek's report for further details. According to Hans Van Houten, Glenn's current psychiatric treatment is attempting to address severe ADHD. Glenn took Vyvanse, the only medication to have a beneficial effect, throughout sixth grade. Early in seventh grade, the benefit faded and, on a higher dose, his tics increased to a level that the Vyvanse was discontinued. Last September, all medications were discontinued, and in late November, he started using a low-dose Daytrana patch. Currently, this is his only psychiatric medication.

Steve Van Houten stated that Glenn does not understand what he reads and, when reading literature, loses the gist of the story. Even when he has chosen the book, he often loses interest and does not finish it. Mr. Van Houten also reported that until November of first grade, when Glenn first began to display signs of serious separation anxiety and depression, he was curious and loved to learn all kinds of information and try out new words that he heard. “He seemed far ahead of other kids his age.” He identified the onset of these disorders as the point at which Glenn lost interest in learning.

When I asked Glenn about his reading difficulties, he focused on Huckleberry Finn: “I didn't get anything out of it.” He did not understand the dialect, the social climate of the deep South in the 1800s, and was lost in the details. He said that when he really likes a book, such as those in the Percy Jackson series, he reads for recreation and “won't stop.” He did not like Eragon, which he read in fifth grade, because “it had too many details, went too slow, and [he] didn't know all the vocabulary.” When he is reading a book that does not grab his interest, his mind wanders, and after he has read a few pages, he realizes that he does not know what he has read. He said that he loves learning about history, particularly wars, and is having no problems in math. He thinks that the Daytrana helps his mood somewhat but is not sure whether it helps with attention, because, as it is summer, he does not have any schoolwork.

Dr. Firek's conclusions, based on the results of the Wechsler Intelligence Scale for Children V (WISC-V) and selected tests of the Wechsler Individual Achievement Tests III (WIAT-III), were that Glenn's cognitive abilities are in the mid-Average range, and he does not demonstrate any learning disabilities other than the possible reading disorder. She interpreted Glenn's scores on the WIAT-III as indicating a weakness in “phonological coding,” resulting in a weakness in reading comprehension. Other than this, she concluded that the difficulties Glenn was having in school were a result of his psychiatric disorders. Although Dr. Firek stated in her report that Glenn's current medications included Intuniv, Hans Van Houten stated definitely that Glenn was not on any medication during the evaluation and that he had never taken Intuniv.

Assessment Measures and Strategy

Woodcock-Johnson IV (WJ IV)

  1. Tests of Cognitive Abilities
  2. Selected tests: Phonological Processing, Nonword Repetition, Oral Vocabulary
  3. Tests of Oral Language
  4. Selected tests: Oral Comprehension, Sound Blending, Segmentation, Rapid Picture Naming, Retrieval Fluency
  5. Tests of Achievement—Form B
  6. Selected tests: Letter-Word Identification, Word Attack, Passage Comprehension, Sentence Reading Fluency, Oral Reading, Reading Recall, Spelling

Qualitative Reading Inventory, Fifth Edition (QRI-5)

The QRI-5 is an informal reading inventory that provides narrative and expository passages that advance in vocabulary, sentence structure, and content according to grade level. The higher levels are Grade 6, upper middle school (UMS), and high school (HS). At each grade, achievement is assessed in terms of performance levels:

  • Independent: The level at which a student can read and comprehend without assistance
  • Instructional: The level at which a student can be instructed profitably
  • Frustration: The level at which a student is completely unable to read with adequate word identification or comprehension

Details regarding the functioning levels are provided in the Appendix.

Diagnostic Teaching of Reading Comprehension Strategies

I administered the QRI-5 first so that I could use the analysis to decide which WJ IV tests were necessary. Diagnostic teaching was conducted after the WJ IV was administered so that my instruction would not affect Glenn's test performance. The tests of the WJ IV were scored according to age norms and grade norms; the largest standard score difference was two points. Age scores are reported here to facilitate comparison with Dr. Firek's results.

Behavioral Observations

Throughout our sessions, Glenn was focused, courteous, and cooperative. He engaged in conversation easily, although he sometimes had difficulty finding the words to state exactly what he wanted to say. His physical activity was typical for his age, and he appeared at ease with me and in my office. He attempted all of the items that were presented to him on the WJ IV and was prompt but not impulsive in his responses.

Results

Qualitative Reading Inventory–5

Level Words in Lists Title & Type Words in Context WCPM** Prior Knowledge Comprehension Retelling
6 Lifeline of the Nile Expository 131 Familiar Frust. Good—some key details omitted
6 Building Pyramids Expository Familiar Instr. Good—some key details omitted
UMS* Indep. Malcolm X Narrative Unfamiliar Frust. Very poor
UMS Biddy Mason Narrative Indep. 142 Somewhat familiar Indep. Good—some key details omitted
UMS Immigration-Part 1 Expository Indep. Unfamiliar Frust. Poor—Got main idea but not subtopics & supporting details
UMS Immigration-Part 2 Expository Unfamiliar Frust.
HS Instr.
*UMS = Upper Middle School; HS = High School; Indep. = Independent Level; Instr. = Instructional Level; Frust = Frustration Level.
**WCPM = Words correct per minute

Oral Reading Accuracy

When reading words out of context (in a list), Glenn's Independent level was upper middle school (20/20), and his Instructional level was high school (16/20). At the higher level, the only word he had to stop and sound out was immunodeficiency.

Glenn's accuracy when reading words in context was at the Independent level for both the Grade 6 and upper middle school passages.

Reading Rate

Glenn read the upper middle school passage at 142 words correct per minute (WCPM). His WCPM is typical for the students assessed in the 4th quarter of grade 7 (150 = 50th percentile) (Hasbrouck, & Tindal, 2005). He read with expression and observed punctuation accurately.

Reading Comprehension

Glenn's comprehension was Instructional on a sixth-grade expository passage about building pyramids and independent on an upper middle school narrative about a former slave. To a large extent, his comprehension varied according to his prior knowledge of the topic. For decades, reading research has shown that prior knowledge has a strong influence on comprehension of the material read. Accordingly, Glenn did not understand the passages for which he had no background knowledge (e.g., Malcolm X, location of European countries, immigration to the United States in the 1800s). In contrast, he did have some knowledge about slavery in the United States. Although narratives are often easier for students to understand than expository text, Glenn's retellings were reasonably good on both.

Dr. Firek stated that Glenn's performance on the WIAT-III indicated “relatively poor basic phonemic decoding and passage comprehension.” Given Glenn's standard score of 110 on the WIAT-III Spelling test, however, and his accuracy and fluency in reading the passages of the QRI-5 (my testing), a problem in phonemic awareness or phonetic coding seemed unlikely. Consequently, I administered selected tests from the WJ IV COG, WJ IV OL, and WJ IV ACH.

Woodcock-Johnson IV: Tests of Cognitive Abilities and Oral Language

CLUSTER/Test W AE GE RPI Proficiency SS (68% Band) PR
AUDITORY PROCESS (Ga) 512 16–10 11.4 94/90 Average 107 (102–112) 68
Phonological Processing 506 12–9 7.4 89/90 Average 99 (92–106) 47
Nonword Repetition 517 >30 >17.9 97/90 Average to advanced 110 (105–116) 76
PHONETIC CODING 517 >30 14.4 97/90 Average to advanced 113 (109–118) 81
SPEED of LEXICAL ACCESS 509 12–2 6.8 87/90 Average 97 (93–102) 42
Oral Vocabulary 524 27 13.8 98/90 Advanced 119 (113–124) 89
Oral Comprehension 519 24 13.2 97/90 Average to advanced 114 (107–121) 83

Glenn's performance on the WJ IV Phonetic Coding test was in the High Average range. His RPI indicates that when other students of his age would be 90% proficient on blending individual sounds into words and segmenting words into sounds, basic auditory skills for reading and spelling, Glenn would be 97% proficient. Furthermore, he performed in the High Average range in tasks requiring him to perceive and manipulate the sounds within words and retrieve sounds from long-term memory. On a timed subtest within the Phonological Processing test, he seemed a bit slow on generating words starting with a specific sound; nevertheless, his score was in the mid-Average range. Because he did appear to have some hesitancy in finding words in normal conversation, I administered the tests of the Speed of Lexical Access cluster. His performance, although not quite as high as some of his other oral language tests, was still well into the Average range. Word-finding does not appear to be a problem. Checks on a couple of the other areas of oral language indicated High Average to Superior performance in understanding passages of one to two sentences and in receptive and expressive vocabulary.

Woodcock-Johnson IV: Tests of Achievement (Form A)

CLUSTER/Test AE GE RPI Proficiency SS (68% Band) PR Descriptor
BASIC READING SKILLS 27 13.4 99/90 Advanced 119 (114–125) 90 High average to superior
Letter-Word Identification 16–9 11.3 99/90 Advanced 115 (111–119) 84 High average
Word Attack >30 >18.0 99/90 Advanced 120 (111–129) 91 High average to superior
READING RATE 14–1 8.5 90/90 Average 102 (97–106) 55 Average
Sentence Reading Fluency 13–3 7.8 90/90 Average 100 (96–104) 49 Average
Word Reading Fluency 13–8 8.2 94/90 Average 106 (101–11) 65 Average
READING COMPREHENSION 14–0 8.4 91/90 Average 103 (98–108) 55 Average
Passage Comprehension 13–0 7.6 90/90 Average 99 (94–104) 48 Average
Reading Recall 13–9 8.3 95/90 Average to advanced 107 (102–112) 68 Average
Spelling 12–10 7.4 90/90 Average 99 (95–102) 46 Average

Glenn's performance on tests of basic reading skills were somewhat higher than on similar tests of the WIAT-III, in the High Average to Superior range. He read lists of real words with no difficulty. His only error on reading nonsense words was pronouncing ce in a word with the /k/ sound rather than /s/. Further, although Glenn's Spelling score was one of his lowest (but still average), all of his errors were phonetically correct (e.g., calander for calendar). Of the many reasons that could account for Glenn's previous poor performance on Word Attack and Passage Comprehension, weaknesses in phonemic awareness and phonetic coding are not among them.

Glenn's Passage Comprehension score on the current test was in the mid-Average range, relatively higher than his WIAT-III score. On some of the items on the Passage Comprehension test, however, he did respond without sufficient consideration of the wording of the item, and some of his errors did not make sense. In a subsequent session, I readministered a page of the test that included three incorrect items and two correct items. His response accuracy was exactly opposite—wrong on the items that were previously right and right on those that were wrong. Without indicating whether his answers were right or wrong, I asked Glenn to explain his reasoning on all five items. On four of them, he had just given the first response that came to mind; he had not understood the fifth item at all, and his initial answer, which had been correct, was a good guess. Glenn's Sentence Reading Fluency, Word Reading Fluency, and Reading Recall tests were also in the Average range, indicating a typical reading rate and the ability to immediately recall information he has read, at least on short passages (up to 90 words).

Analysis of Glenn's responses to comprehension questions, retellings, and interview revealed the lack of certain behaviors and strategies that, if used, would improve his reading comprehension:

  1. Glenn's mind wanders when he does not have a strong interest in the material and/or when he perceives it as too hard. His attentional difficulties are likely a strong factor contributing to this problem. He needs to learn how to monitor his comprehension to make sure that he is comprehending.
  2. Glenn does not deliberately make a mental picture of what he is reading, although he sometimes does so spontaneously when reading a book he enjoys. When retelling the passages, he attempted to revisualize the text itself, paragraph by paragraph, rather than the meaning.
  3. Glenn does not identify unknown vocabulary and try to infer the meaning from context. When asked to underline all the words in a passage for which he did not know the meaning, he found five. With guidance, he found five more and could not figure out their meanings.
  4. Glenn does not read actively; thus, he does not notice when what he is reading does not make sense. He does not try to organize related details and fit them into a unifying concept. As an example, one of the passages stated that it was impossible for the people to sail their boats from north to south because the river flowed from south to north and there were waterfalls in the way. Glenn read “impossible” as “possible” and did not notice that the resultant meaning clashed with the logic of the other details.
  5. Glenn did not use the graphics provided with the passages to help him understand the text.

During the time we had for diagnostic teaching, I introduced Glenn to the following strategies to see whether he would be able to use them. All would be successful if he is able to both remember to use them and to sustain attention to the strategies and to his reading.

  1. For expository and narrative material: How to visualize the material he is reading
  2. For expository material: How to (a) read a paragraph and then paraphrase the information, and (b) study a graphic before reading the related material
  3. For narratives: How to identify key elements of short narratives, often referred to as story grammar

Summary and Conclusions

Results of tests and diagnostic teaching indicate that Glenn does not have a reading disorder. His oral reading accuracy and fluency are strong; however, similar to many students of his age, he is lacking strategies that would be of significant benefit to him in comprehending all types of reading material. His attention difficulties and other psychiatric disorders are likely to be complicating factors in his use of the comprehension strategies that he learns. These difficulties could be reduced by effective medication, behavioral incentives, and reminders to use the appropriate strategy for the reading task.

Recommendations

  1. Teach Glenn to visualize what he is reading (narrative and expository) and to be aware that when he cannot visualize a part of the text, he is likely not understanding it. He needs to stop and either reread or use a different strategy.
  2. Teach Glenn how to figure out the meaning of words from the context. Several good instructional programs are available for teaching the specific clues to look for in the text. A list of different types of context clues is attached. [See Appendix.]
  3. Teach Glenn strategies for reading expository text, such as Survey, Question, Read, Recite, Review (SQ3R). A similar strategy that is best done with a group is Collaborative Strategic Reading. [See Appendix.]
  4. Teach Glenn a strategy for identifying the key elements in any narrative and how to ask himself questions about them. Key elements include: the setting (characters, time, place), the plot or major problem, the sequence of events, the resolution of the problem, and the ending. A useful strategy for short stories is STORE the Story, although there are many story frameworks. [See Appendix.]

Appendix

Reading Performance Levels

Independent Level

  1. The level at which a student can read and comprehend without assistance.
  2. Word Identification in Isolation: 90% or higher (obtained from word lists)
  3. Word Identification in Context: 98% or higher (obtained from oral reading of passages)
  4. Comprehension: 90% or higher

Instructional Level

  1. The level at which a student can be instructed profitably.
  2. Word Identification in Isolation: 70% to 89% (obtained from word lists)
  3. Word Identification in Context (obtained from oral reading of passages)
    • 90% to 97%: Total accuracy
    • 95% to 97%: Total acceptability
  4. Comprehension: 70% to 89%

Frustration Level

  1. The level at which a student is completely unable to read with adequate word identification or comprehension.
  2. Word Identification in Isolation: Less than 70% (obtained from word lists)
  3. Word Identification in Context (obtained from oral reading of passages)
    • Less than 90%: Total accuracy
    • Less than 95%: Total acceptability
  4. Comprehension: Less than 70%

Source: Leslie, L., & Caldwell, J. S. (2011). Qualitative Reading Inventory—5. Boston, MA: Pearson Education/Allyn & Bacon.

Psychoeducational Evaluation

Name: Gilbert Welch
Parents: Dr. Marian Schwartz and Dr. Bernard Welch
Date of Birth: November 5, 2000
Age: 14–6
Dates of Testing: May 3, 5, and 9, 2015
Date of Report: May 16, 2015
School: Chatham Academy
Grade: 8.7

Reason for Referral

Gilbert was referred for evaluation by his mother, Dr. Marian Schwartz. She stated that although Gilbert consistently earns good grades, his test scores, especially those on standardized tests, are lower than expected, especially compared with the scores of some of his classmates. Because of this, she is concerned that certain opportunities will be closed to him because his test results do not represent his true capabilities. She was also concerned that he seems to spend an excessive amount of time on his schoolwork. She would like a profile of his learning strengths and weaknesses, answers to the questions about test scores and homework, suggestions for appropriate high school and college programs, and recommendations for supports that would be useful.

Background Information

Gilbert is the oldest child in an intact family. He has a younger brother and sister, 11 and 9. The brother has no learning problems. The sister does not like to read but her school has not noted any problems. Gilbert's mother is a professor of economics at Indiana State University, and his father is a cardiologist. Dr. Schwartz reported that she has dyslexia and that many of her family members were said to have dyslexia as well. Dr. Welch has no family history of disabilities. Dr. Schwartz's pregnancy, labor, and delivery of Gilbert were normal. Gilbert has had no major childhood illnesses or injuries. Results of vision and hearing tests done this month were normal. He sleeps approximately 9 hours per night.

Gilbert plays soccer on a city league. He also plays the cello, plays clarinet in the school marching band, and schedules his own private lessons. Dr. Schwartz stated that Gilbert has no behavioral problems other than the usual sibling irritations. In fact, Gilbert was chosen “Student of the Year” at Chatham Academy for his kindness to other students; she noted that other students look to him as a leader. Dr. Schwartz feels confident that Gilbert has no intrinsic attention problems. He works on his homework independently and for extended periods, but often takes longer than expected to complete it. She estimated that it takes him 15 hours to complete a week's assignments that the teachers intended to take 10 hours. Dr. Schwartz noted that Gilbert reads slowly and often complains that he does not have time to finish his work in class. Although he does well in English class, he works hard at it and sometimes cannot finish his homework because it takes too long. His spelling of simple words is incorrect but phonetic. Periodically, he becomes stressed about his homework, but the stress always resolves.

Educational History

Gilbert attended preschool through fifth grade at Greenwood Montessori School; he transferred to Chatham Academy for sixth grade and currently is completing eighth grade there.

In the primary grades, some teachers made comments about Gilbert being distracted and distracting others, but no similar concerns have been noted on his report cards since third grade. In fourth grade, some teachers noted difficulty finishing classwork on time, and although no such concerns were noted in fifth grade, Gilbert's self-assessments stated his desire to improve his on-task focus and completion of his work plan.

Dr. Schwartz and the Greenwood report cards indicated that Gilbert had strong oral language skills but difficulty learning basic reading skills. Because he was a young first-grader, his mother had him retained. By the end of third grade, his oral reading level was reported as beginning fourth grade. Math has been a continuous strength.

Gilbert's grades at Chatham Academy have been As, interspersed with very few Bs. Comments indicate that he is a diligent student with a strong work ethic. Teacher comments in sixth and seventh grades noted concerns that Gilbert's strong focus on producing quality work caused him stress and a lack of balance in his life. No such comments were on his first three quarter reports in eighth grade. This report is being written before the fourth quarter report.

Previous Test Results

When Gilbert was in fifth grade, Dr. Schwartz requested intelligence testing for the purpose of educational planning. Dr. Julie Wilson, psychologist, administered the Wechsler Intelligence Scales for Children, Fourth Edition (WISC-IV), and the Raven's Standard Progressive Matrices (Raven), a test of nonverbal reasoning. His WISC-IV and standard scores (SS) were: Verbal Comprehension: SS 112; Perceptual Reasoning: SS 117; Working Memory: SS 102; Processing Speed: SS 83. Although both Processing Speed subtests had a scaled score of 7, Dr. Wilson noted that the extra time Gilbert took to correct several items on the Coding subtest may have invalidated that score. Of all the tests given, only the Processing Speed tests have time limits. In contrast, Gilbert's Raven score was SS 116.

In December of 2014, Gilbert took the time-limited Cognitive Abilities Test (CogAT) and the Raven. His CogAT subtest scores were: Verbal: SS 109; Nonverbal: SS 109; Quantitative: SS 103. His Raven score was SS 121.

Interview with Gilbert

I asked Gilbert what information he would like from the testing. He said that he would like class-specific study strategies, strategies to help him take tests more efficiently, and a better understanding of himself as a learner. He said that he does not have any current concerns but does want to do well on standardized tests that have importance for college admission. Although currently most of his teachers allow him sufficient time on exams, he feels that he is slower than other students. He said he was always the last one to finish his history exams, but the teacher always lets him finish, even if he has to stay after class. In math, he explained that he understands the concepts but is slow in “doing the simple stuff,” such as basic computations. Students use calculators in his advanced math class. Last year, the students were not allowed to use calculators for the first semester, and he stated that this restriction helped him to develop faster retrieval of math facts. I gave him a 2-by-4-digit multiplication problem to do. He worked through the problem correctly almost to the end, but when adding the two products, he correctly added four consecutive sets of digits but then multiplied the last set.

Gilbert agreed with his mother's statement that he is a slow reader but that he has no difficulty pronouncing words. Gilbert thinks he is a good writer except for poor spelling. He explained to me how he does his homework and studies for each class. His methods may be more strategic than those of many other middle school students but are not as efficient as they could be.

I asked Gilbert whether he visualizes what he reads. He explained that he knows he understands better if he has a picture in his mind but “pictures come to [me]” only for narratives, such as in literature or anecdotes used in the history text. He does not visualize the concepts he reads in straightforward expository text, such as history and science.

When asked, Gilbert also said that he is aware of having some word-finding difficulties when talking and that it might happen more often when he is tired or under pressure. He gave a recent example of being at an interactive play in which the audience was asked for their ideas. He said, “I had a really good idea but when I tried, I couldn't figure out how to say it.” He thought that this is more of a problem when speaking than when writing because when writing, he has the advantage of time to think and resources, such as the thesaurus and the computer.

Gilbert thinks that he might want to become an engineer and build levees in cities that are 10 feet or less above sea level, because of the threat of the oceans rising as a result of global warming.

Current Evaluation

The information provided in this report was obtained from selected tests from:

  1. Woodcock-Johnson IV
  2. Tests of Cognitive Abilities (WJ IV COG)
  3. Tests of Oral Language (WJ IV OL)
  4. Tests of Achievement (WJ IV ACH)
  5. Rapid Automatic Naming and Rapid Alternating Stimulus Tests (RAN/RAS)
  6. Qualitative Reading Inventory, Fifth Edition (QRI-5): Upper middle school passage
  7. Interview with Marian Schwartz and Bernard Welch, parents
  8. Analysis of classwork

All of the tests administered were appropriate for Gilbert's ethnicity and language. The following were considered and ruled out as primary contributing factors to the referral concerns: visual, hearing, or motor disability; cultural factors; environmental or economic disadvantage; and limited English proficiency.

Behavioral Observations During the Evaluation

Throughout the report, examples of test items and Gilbert's responses are similar to those actually given but have been altered to maintain the security of standardized tests.

Gilbert's tests were administered in one 4-hour session, two 3-hour sessions, and one 1-hour session. Midway through the longer ones, he rode his bike home for lunch and returned. Throughout, Gilbert was cooperative and courteous, smiled easily, and seemed at ease. He asked for repetition and clarification when needed, noticeably increased his effort when he encountered difficult items, and made self-corrections. His attention and concentration were excellent. As he worked through the tests, I noticed his use of a strategy only twice. When repeating increasingly lengthy series of unrelated words, he tapped a finger for each word in the sequence, apparently keeping track of the number of words before repeating. During the RAN/RAS tests, he began to use a quick but steady pace of naming rather than trying to rush through. Gilbert's performance on these tests should be considered to be a valid representation of his current functioning.

Explanation of Scores Used in this Report

Gilbert was administered tests assessing a wide variety of cognitive factors, language abilities, and academic skills to ascertain his strengths and weaknesses compared with others of his age. Because Gilbert was retained in first grade, the use of age norms rather than grade norms were used; thus, his academic achievement was compared with that of students in ninth grade. To ensure that this did not disadvantage him, I ran the scores using both age and grade norms and compared them. The difference was 2 or fewer standard score points on all tests.

Gilbert's performance on the abilities measured by the Woodcock-Johnson IV (WJ IV) and the Rapid Automatized Naming/Rapid Alternating Stimuli (RAN/RAS) test are described as percentile ranks (PR), standard scores (SS), standard score ranges created by 68% confidence bands, and, occasionally, relative proficiency indexes (RPI).

The RPI provides different information from peer-comparison scores (e.g., standard score, percentile rank) regarding a person's ability in a task. The SS and PR indicate the position of a score within the continuum of scores earned by others of Gilbert's age (a rank ordering), whereas the RPI is a criterion-referenced score, based on the norms, that predicts his success on tasks similar to that used on the test. The RPI is represented as a fraction. The denominator of 90 indicates that an average age-peer would be successful on the task 90% of the time; the numerator represents Gilbert's expected success. For example, Gilbert's RPI of 82/90 on the Math Facts Fluency test indicates that when the average student of age 14–6 would be 90% successful in speed of retrieving math facts from memory, Gilbert's expected success would be 82%. RPIs are particularly useful in showing a weakness in actual ability that may not be evident from standard scores.

In this report, descriptors for standard scores and RPI ranges are as follows:

Standard Score Descriptor Scaled Score RPI Proficiency Implications: Gilbert will find similar tasks
131 or above Very superior 17 to 19 100/90 Very advanced Extremely easy
121–130 Superior 14 to 16 98/90–100/90 Advanced Very easy
111–120 High average 13 95/90–98/90 Average to advanced Easy
90–110 Average 8 to 12 82/90–95/90 Average Manageable
80–89 Low average 7 67/90–82/90 Limited to average Difficult
70–79 Low 4 to 6 24/90–67/90 Limited Very difficult
69 or below Very low 1 to 3 3/90–24/90 Very limited Impossible

Clusters are made up of two or three tests. The cluster assesses a broad ability such as visual processing, whereas the component tests assess narrower abilities, such as spatial relations and memory for visual details, which contribute to the broad ability. When the confidence bands (CB) of the component tests overlap, only the broad ability, represented by the cluster score, is discussed. When the confidence bands of the component tests do not overlap, the narrow abilities are discussed.

Test Results Cognitive and Oral Language Abilities

Gilbert's General Intellectual Ability (GIA) standard score (SS) is 119, bordering the High Average and Superior ranges, as high as or higher than 90% of his age-peers. The scores constituting the GIA spanned 2.7 standard deviations (SD), indicating that the GIA is not a unitary factor but encompasses many different levels of abilities.

Strengths

Reasoning

Gilbert performed in the Superior range in Fluid Reasoning (PR 96, SS 127), the ability to use inductive and deductive reasoning to form concepts and solve problems without depending on previously learned information or procedures, to learn from feedback regarding errors. Academically, this ability is strongly related to mathematical reasoning, understanding scientific concepts and procedures, and higher-level reading comprehension.

One aspect of fluid reasoning is quantitative reasoning, the ability to use logical reasoning with concepts involving mathematical relations and properties. Gilbert performed in the Superior range in the Quantitative Reasoning cluster (PR 93, SS 122). The tests required Gilbert to complete number patterns of increasing complexity and a task that did not use numbers but that involved reasoning with algebraic properties.

Auditory Processing and Phonetic Coding

Auditory Processing as assessed by the WJ IV is the ability to discriminate and mentally manipulate the individual speech sounds within words, including under time pressure. Phonetic Coding involves the ability to break down spoken words and syllables into individual sounds (segmentation) and blend individual sounds into words. Blending and segmentation provide a critical foundation for learning initial reading and spelling skills. Gilbert performed in the Superior range on both the Auditory Processing (PR 96, SS 126) and the Phonetic Coding clusters (PR 90, SS 119). His phonetic coding ability is evident in his phonetic spelling, representing all of the sounds in a word, even when the word is not orthographically correct (e.g., such as spelling “shun” for tion).

Rapid Automatized Naming (RAN)

Rapid naming indicates the speed and efficiency with which a person can name a visual prompt. Over six tests, Gilbert named (a) rows of pictured objects, (b) colors, (c) numbers, (d) letters, (e) alternating letters and numbers, and (f) alternating letters, numbers, and colors. His performance was interesting. He scored in the Superior range on the first test, dropped to Average on the second, and then improved test by test back to Superior on the most complex (PR 93, SS 122). Gilbert appeared to learn the task and develop a strategy; on the last four tests, he maintained a quick, steady rhythm despite the increasing complexity. His results are presented under Speed of Lexical Access.

Visual Processing

Gilbert's performance on the WJ IV Visual Processing cluster was in the High Average range (PR 81, SS 113). Visual processing is the ability to perceive and work with visual representations of objects and designs. Examples are envisioning how a design or object might look when seen from another perspective and perceiving how parts of a structure combine to make the whole. Another aspect is having a good eye and memory for visual detail. In middle school and beyond, these abilities are necessary for courses such as geometry and physics, and for acquiring technical skills.

General Knowledge

Gilbert performed in the High Average range (PR 86, SS 116) on a test of general knowledge, sampling information typically learned through daily experience (e.g., where to find a bannister, what a bank is used for) as well as on tests of academic knowledge (i.e., humanities, social studies, science) (Academic Knowledge; PR 90, SS 119).

Average Abilities for Age Level

Gilbert demonstrated a variety of memory abilities in the upper part of the Average range (PR 59–75, SS 104–110). His performance on all of the following tests indicates strong and consistent memory abilities.

Auditory Memory Span

Auditory Memory Span (PR 70, SS 108) represents the amount of auditory verbal information a person can hold in mind for a few seconds, without having to do any work with it or transform it in any way. Once a person is no longer attending to the information, it dissipates. These tests assessed Gilbert's ability to repeat sentences and sequences of unrelated words presented orally.

Short-Term Working Memory

Short-Term Working Memory (PR 59, SS 104) is a capacity that integrates multiple aspects of memory and facilitates the more complex mental work related to learning and to the activities of daily living. We use it to process, reconfigure, or transform information and procedures that come into our awareness from outside (e.g., classroom instruction) or that we retrieve from long-term memory. For example, a person working through a multiple-step math word problem must read and consider what the problem is asking, what information can be ignored, what operations are required, and the order in which these must be done—while simultaneously recalling the related math facts and the steps of the math operations from long-term memory. In reading, working memory comes into play as a person reads successive sentences, recalls what he has read previously, and integrates the new concepts into the information he has acquired from past experience.

The tasks of the Short-Term Working Memory cluster were recalling specific information from a series of numbers and animals, reordering strings of numbers of increasing length (e.g., Say “2–9–8–5–1” backwards), and reordering combinations of words and numbers. Another test required Gilbert to listen to oral instructions of increasing length and syntactic complexity, hold them in memory, and then follow them by pointing to specified objects in a detailed picture (e.g., Don't point to the man on the bicycle unless a person is crossing the street next to a park) (Understanding Directions: PR 44, SS 98).

Long-Term Retrieval

Generally, Gilbert's long-term retrieval, the ability to retrieve information from long-term memory once it is completely removed from immediate awareness, is in the upper part of the Average range (PR 71, SS 108). One task involved learning words paired with abstract symbols and then “reading” a story written with the symbols; the other involved retelling the details of brief narratives he had just heard. When retelling the narratives, Gilbert earned points for using specific words but omitted some key details. His retelling of an expository passage was similar. (See Reading Comprehension.)

Weaknesses

Perceptual Speed

Gilbert demonstrated a significant weakness relative to his other cognitive abilities in perceptual speed (Perceptual Speed: PR 22, SS 89), the ability to rapidly scan visual symbols, such as letters and numbers, and identify similarities and differences based on their visual appearance. His RPI of 80/90 indicates that in tasks requiring rapid processing of visual symbols, such as reading, Gilbert would be 80% successful compared with typical age-peers who were 90% successful. Of 14-year-olds with the same predicted score, only 4% would score as low as or lower than Gilbert. An important point here is that Gilbert does not demonstrate difficulty with accuracy of visual processing, only in the amount of time it takes.

Speed of Lexical Access

Two of the WJ IV OL tests assess lexical access, or the ability to quickly retrieve words from long-term memory. Gilbert's performance on Retrieval Fluency was Average (PR 56, SS 102). He was given a category (e.g., articles of clothing) and had to name as many examples as he could as fast as possible. In contrast, his performance on the Rapid Picture Naming test was in the Low Average range (PR 15, SS 85). This test involved naming pictures of familiar objects (e.g., cup, scissors) as quickly as possible. Oddly, when Gilbert came to a picture of a bottle, he paused, said “I don't know,” and moved on. Gilbert's performance on this test reflects his reported difficulty in retrieving a specific word when speaking or, when writing, the extra time and resources he needs.

The difference between his proficiency on these two tasks is more clearly expressed by the relative proficiency index (RPI). Gilbert's RPI for Retrieval Fluency was 90/90 (average), whereas his RPI on Rapid Picture Naming was 64/90 (limited). The prediction is that on a similar task, when the average age-peer would be 90% successful in quickly finding a needed word, Gilbert would be 64% successful. The difference in Gilbert's performance on these tests may be attributable to the difference in task demands. Rapid Picture Naming requires a specific word in response, whereas Retrieval Fluency allows a wide range of words as long as they fit within the broad category.

Academic Achievement

Reading

On the WJ IV ACH reading tests, Gilbert's performance was generally in the Average range. These tests indicated that he has sufficient skills in phonics, orthography, and syllabication to sound out unknown words (Word Attack: PR 74, SS 110). Additionally, he read real words accurately (Letter-Word Identification: PR 66, SS 106), understood one- to two-sentence passages well enough to supply the missing word (Passage Comprehension: PR 58, SS 103), and read simple sentences rapidly and with comprehension (Sentence Reading Fluency: PR 40, SS 96). He also read sentences accurately with expression (Oral Reading: PR 59, SS 104), and was able to recall information from passages (Reading Recall: PR 48, SS 99).

To obtain a better idea of Gilbert's reading in simulated school text, I asked him to read an upper middle school passage (423 words) from the Qualitative Reading Inventory—Fifth Edition (QRI-5). He read “1800s” as “1980s.” Other than that, his accuracy was 99%, and his reading rate was 159 correct words per minute, a little better than average for a student in the spring of eighth grade.

To get a better sense of his ability to understand and remember information on a lengthier passage than provided on the WJ IV, I asked Gilbert to read the passage again, silently, for meaning. The passage explained the reasons for the large influx of immigrants in the late nineteenth to early twentieth centuries, separating them into “push factors” (negative conditions in one's own country) and “pull factors” (attractions of moving to the United States). Before his initial reading of the passage, Gilbert's answers to questions indicated that he was not familiar with this topic, but his prediction of what it would be about was correct. Gilbert was told that he would retell the passage after reading it and then I would ask him questions. His retelling of the passage was accurate and organized but sparse. He divided his retelling into pull factors and push factors, and gave an introductory sentence for each, followed by details. Whereas the passage discussed five examples of push factors, and six of pull factors, Gilbert mentioned only two of each. Consistent with his explanation of the type of information he visualizes, the one factor he recalled in detail was an anecdote about the pogroms in Russia. He answered seven of ten questions correctly, placing this passage at his instructional level for comprehension.

Writing

Spelling. Gilbert performed in the mid-Average range on both spelling tests of the WJ IV—one with real words and the other with pseudo-words that follow the spelling rules of English (Spelling PR 51, SS 100; Spelling of Sounds, PR 60, SS 104). Gilbert's spelling of real words was phonetically accurate. His errors were orthographic, indicating that he had an incomplete mental image of the words. Analysis of some of his school writing assignments indicated a similar pattern of errors (e.g., fluxuate, hieght), including limited knowledge of how to add a suffix to a baseword (e.g., usualy).

Sentence Writing Fluency. The Sentence Writing Fluency test assessed Gilbert's speed in formulating and writing simple sentences from a prompt. His score bordered the Average/High Average ranges (PR 77, SS 111). Before starting, he asked the number of pages and then wrote very fast in an obvious, albeit unsuccessful, effort to finish them all.

Written Expression. The Writing Samples test (PR 76, SS 110) assessed Gilbert's ability to write individual sentences according to varied demands (e.g., Write a sentence about this picture that has “although” in the middle”). Again, his score bordered the Average/High Average ranges.

An analysis of Gilbert's school essay on censorship found that most of his sentences were syntactically correct, with a variety of sentence structures and good flow from one to the other. More than occasionally, however, his intended meaning was unclear (e.g., “Not allowing the press to damaging information about national security is also valuable to withhold”). As this was word processed, he found most of his spelling errors but errors remained in capitalization and punctuation. A descriptive paragraph about his experience while snorkeling was vivid in imagery and well written.

Math

Math Calculation Skills. The WJ IV Calculation test presents a variety of math computation problems sampling knowledge of math computations at progressive levels. Gilbert's performance on this test bordered the Superior/Very Superior ranges (PR 97, SS 129). The problems he solved included algebraic equations, rational numbers involving square roots, and straight-line equations.

In sharp contrast, his performance on the Math Facts Fluency test was his weakest academic skill, in the Low Average range (PR 23, SS 89). This test assesses rapid retrieval of simple addition, subtraction, and multiplication facts from long-term memory. Gilbert was accurate but slow. His performance was consistent with his self-report that he understands the concepts in math but is slow in “doing the simple stuff.” The use of a calculator makes complex math easier and more efficient but eliminates the practice needed to develop and maintain automaticity of math facts. Nevertheless, automaticity of these facts should have been established in elementary school.

Math Application. Gilbert's performance on Number Matrices, a measure of quantitative reasoning, was in the Superior range (PR 92, SS 120). He scored in the High Average range on the Applied Problems test (PR 84, SS 115), assessing the ability to translate practical math problems into computation and solve them. Gilbert correctly solved problems that involved rate, time, and distance; elapsed time; positive and negative numbers, and most questions regarding the area of two- and three-dimensional objects. His success in solving some problems that required similar procedures was inconsistent, as though he did not see the similarities. As an example, he had two problems that required either adding or subtracting a percentage from a given number (e.g., discount vs. interest). He easily solved the first, but in the second, could not figure out how to translate the percentage into a dollar amount. Also, on a question regarding the perimeter of a circle, he knew the equation and the number represented by pi but could not figure out how to get from 125π to the final answer. His answers were sometimes surprising; on a question regarding length, he answered in centimeters (and, when asked, translated it to inches). When answering a question about area, he answered “6 inches squared” rather than “6 square inches.” The number was correct, but the answer as stated was incorrect.

Answers to the Referral Questions

The purpose of this evaluation was to answer the following three questions:

1. Why are Gilbert's test scores, especially on standardized tests, unexpectedly lower than his school grades?

Gilbert's test scores do not appear to be lower than his school grades would predict. His scores on the most recent Stanford 10, when compared with a national sample of grade peers, are in the High Average to Superior range. This is consistent with the findings of this evaluation. These scores may be lower than those of some of his grade peers at Chatham Academy because, due to the nature of the student body, the entire class was performing at a higher level than the national sample of seventh-grade students. One should expect the same to happen this year. Chatham Academy is academically rigorous and generally serves students from educated families; consequently, the mean score of any grade level is likely to be higher than that of a national sample. Given his school-based scores on the Stanford 10, Chatham Academy appears to have been a good fit for Gilbert.

A second reason that Gilbert may not score as well as he expects on standardized tests is that his school grades are based on classwork, homework assignments, and tests. It is apparent that he puts a great deal of effort and time into his classwork and homework, and his tests assess specifically what he is expected to learn. This is not true of standardized tests, which sample, rather than test in depth, a much broader range of skills. Gilbert's hard work is reflected in his school grades but not to the same extent as on standardized assessments. His school grades are the more pertinent measure.

2. Why does it take Gilbert longer than his peers to complete tests and homework?

Some important findings of this evaluation are that Gilbert has superior reasoning abilities, average to above average memory and oral language abilities, and a significant weakness in perceptual speed. The extra time Gilbert needs to process visual symbols will have an effect on almost any reading or math task, and any task that requires quick recognition of or discrimination between symbols. If he hurries because of time limits or perceived time limits, he is liable to make errors. Gilbert stated that on tests, when most people have finished and left the room, he feels that he must hurry to finish, even if there are no time limits. He will need to learn to ignore this impulse. Furthermore, when writing, Gilbert's difficulty with rapid word retrieval when searching for a specific word might cause him to take more time with the task than another student would need. Consequently, when given a task or test with a time limit that is sufficient for most of his classmates, he will not have the time to demonstrate his excellent reasoning skills and the knowledge he has worked hard to obtain. These findings echo Dr. Wilson's statement that “Gilbert has strong higher order reasoning skills but he may not be able to demonstrate his skills as effectively under tight time constraints.”

These findings might help Gilbert to understand a possible reason that his scores on the CogAT were relatively low. His scores on Dr. Wilson's evaluation are surprisingly consistent with his scores on similar tests/clusters on the WJ IV. Based on these, one would think that his scores on the CogAT would have been higher in Quantitative and Nonverbal Reasoning. An important difference, however, is that the tests of the CogAT have time limits. Similar tests on the WISC-IV and the WJ IV do not have time limits. Had Gilbert been able to take the test without time limits, his scores would likely have been higher.

3. What type of educational programming do the evaluation results suggest for high school and college?

Gilbert has aptitude for many areas, particularly those involving mathematics. In high school, he should definitely consider taking Honors or Advanced Placement math classes and, if offered, a class in logic. He would probably be most comfortable in advanced English and History classes as well. A caution, however, is that given his drive to excel in his classes and his need for extra time to do assignments, it is important that he does not take an academic load that will prevent him from doing extracurricular activities in which he is interested—or even just curious about. Spending time this summer learning writing conventions, and strategies for studying, reading comprehension, and test taking will not only help compensate for his slow processing speed, but will make him more efficient in all of his courses.

Regarding college, Gilbert demonstrated so many strengths on this evaluation that he should be able to go into any field he finds interesting. Aside from his strong cognitive abilities, his willingness to work hard and his ability to focus his attention will help him achieve in almost any endeavor. The particular areas in which Gilbert excelled were logical reasoning, visualization of objects in space as well as part–whole relationships, quantitative reasoning, and math. Engineering and architecture are two disciplines that come to mind as requiring these strengths. When he is ready to start considering which colleges to apply to, it would be helpful for him to take an interest and aptitude inventory so that he can include the results in selecting a college. That way, he will not inadvertently end up at a school without a program he might want to pursue later.

Recommendations

For the School

  1. For tests that require reading, writing, math, or that incorporate visual symbols or figures, provide Gilbert with enough time to complete all questions on the exam without time pressure. Because of Gilbert's difficulty with speed of processing visual symbols, it is critical to differentiate between lack of sufficient knowledge and lack of sufficient work time.
  2. For tests given with Scantron response sheets: To accommodate for Gilbert's weakness in the rapid processing of symbols, and if Gilbert thinks it will be helpful, allow him to mark his answers directly in the booklet rather than on the Scantron sheet.

For Gilbert

  1. Learn how to explain the information in this report so that you can speak with your teachers to advocate for more time on tests and to ask for help when you need it. Make an appointment with me if you want help with this.
  2. Make an effort to make a mental picture of everything you read, especially history and science. You may need help to learn this but until that happens, try to develop this skill on your own.
  3. After you finish writing a rough draft of a paper, read it out loud, sentence by sentence, to see whether each expresses the idea you intended and will be clear to your reader. Underline any words or sentences that need changing so you can go back and revise them. Read each new draft aloud.
  4. Use computer programs and hard copy math fact speed drills to practice rapid retrieval of math facts. Hard copy may be more helpful because it will incorporate a written response similar to the action required on tests.
  5. For all high-stakes tests, such as the Stanford 10, PSAT, and SAT, plan time to make use of practice tests, available online. Use the practice tests to become familiar with how the overall test is organized, the type of content, the format of the questions, and how long it takes you to complete different parts of the test. Familiarity with the format of the test and answering specific types of questions will improve your efficiency, and thus, decrease your time. Many of these programs also provide good suggestions on how to think about the questions or how to approach the exam.
  6. When writing answers to essay questions on tests, do not write “off the top of your head.” Decide on specific points that will answer the question, write a couple of notes to remind you of each point, then write directly to those points.
  7. For essay questions, when listing elements, ask the teacher ahead of time whether it is permissible to use bullet points rather than writing out complete sentences for each.

For the Parents

  1. Gilbert does not show an academic deficit that would qualify him as a student with a learning disability. Fortunately, he is at a school that is willing to provide time extensions to all students.
  2. Strongly consider hiring an educational therapist to work with Gilbert for the summer to teach him the skills listed below. If Gilbert learns these skills, he will find the work of high school and college considerably easier. Furthermore, efficient study strategies and test-taking strategies will help him compensate for his slow perceptual skills.

For the Educational Therapist

  1. Teach Gilbert reading comprehension strategies for different types of disciplines, especially the sciences (biology, chemistry) and the social sciences (e.g., history).
  2. To improve Gilbert's understanding and retention of all types of information, teach him to create mental images for all types of academic material, rather than just narratives.
  3. Teach Gilbert how to use a multiple-step process for writing, including prewriting, writing the rough draft, revising, and editing.
  4. Review with Gilbert the rules of capitalization and punctuation, and how to edit especially with those in mind.
  5. Look for patterns in Gilbert's spelling errors and teach him the rules he is missing rather than having him try to memorize the spellings of individual words, unless they are irregular words.
  6. Teach Gilbert specific test-taking strategies, including:
    • general guidelines (see Appendix for examples)
    • how to answer different types of question formats (e.g., multiple-choice, true-false, short answer, short essay, lengthy essay) (see Appendix for examples)
    • the use of test preparation programs for high-stakes tests

Appendix

Examples of Test-Taking Strategies

  1. General procedures: (a) Read the test instructions before starting; (b) Ask for clarification of any instructions that you do not understand clearly; (c) Look through the test to get an idea of how much you need to do in the time allowed; (d) Decide how much time you should use to answer each item; (e) Answer easy or known items first and then go back to the hard ones; and (f) Answer all items unless there is a penalty for guessing. If Gilbert skips a question, he should circle its number and write the number down after the last question to remind himself to go back to answer it.
  2. Multiple-choice questions: (a) Make sure you know the meaning for “All of the above” and “None of the above”; (b) Read the question stem; (c) Underline any key words (e.g., not, unless, except, incorrect, false, never, always); (d) Read the stem of the question along with each answer choice; (e) Cross out each incorrect answer choice. If more than one choice is left when you are done, read the stem with each of the answer choices remaining. Choose the best; (f) Do not change an answer unless you are sure it is incorrect. Generally, your first answer is correct.
  3. True-False questions: (a) Assume an answer is true unless the statement can be proved false; (b) Be sure that all parts of the statement are correct before marking it true; (c) Watch for negatives such as not or prefixes such as in (e.g., infrequently), because these can completely change the meaning of a statement; (d) Simplify statements that contain a double negative (e.g., “Birds that are not black cannot hunt at night” becomes “Only birds that are black can hunt at night.”); (e) Assume that absolute statements are false, and that qualified statements are true.

Diagnostic Evaluation

Name: Annette Hernandez
Date of Birth: June 4, 1999
Age: 15–11
Dates of Testing: May 16, 2015; May 27, 2015
School: Kino Learning Center
Grade: 9.9

Reason for Referral

Annette's mother referred her for an evaluation because she wished to gain a better understanding of her daughter's present levels of academic performance and her current educational needs. Annette has had long-standing difficulties in reading and spelling, and it has been six years since she has had a comprehensive evaluation. In addition, the family will be moving to Kirkland, Washington, at the end of June and they would like updated information to bring to the new school.

Background Information

Annette had no notable birth complications, and all developmental milestones were achieved at the appropriate ages. Throughout her school years, the family has moved five times because of changes in her father's employment. Although Annette has adjusted to each school setting, she is looking forward to being near Seattle for several years. Annette attended St. Mary's in Vancouver, WA, for kindergarten and first grade. Because of concerns regarding slow academic development, Annette was retained in kindergarten. At the of 6, she began receiving tutoring with a Language Skills Therapy tutor who used a multisensory, structured phonics approach to teach Annette basic reading and writing skills. In second grade and third grade, Annette attended Elder Ridge Elementary in Denver, CO. For fourth and fifth grades, Annette attended Harper Elementary School in Bannockburn, IL. Despite her family's moves, Annette continued to receive private tutoring through fifth grade.

For the past 4 years, from sixth through ninth grade, Annette has attended Mesa Learning Center, a private school in Tucson, Arizona. The philosophy of this school is that instruction is based on what is best for each child and that each child is unique. Annette did not have an IEP but in this school setting the teachers work with students individually and set appropriate goals for each student. Annette's teachers describe her as a hard-working student with a good attitude. She does not get frustrated easily and is willing to persist to complete tasks. She is sometimes quiet in class, but asks questions when she is engaged in tasks. Her English teacher noted that Annette's reading comprehension has improved considerably.

Annette currently lives with her parents and her older brother. Her brother, who has had similar difficulties with reading and spelling development, has just finished high school and will start the University of Arizona in the fall. He will receive support services from the Strategic Alternative Learning (SALT) program. Annette also has an older sister, who has just graduated from Arizona State University with a degree in marketing and sales.

In the fall of 2015, Annette will start at Douglas Preparatory School in Kirkland, WA, and she may receive special education services through the public school district. In describing her current academic performance, Annette explained that she still has a hard time pronouncing and spelling certain words and that math is not her “strong suit.” She enjoys art projects and is currently interested in painting tropical scenes.

Prior Evaluations

Annette has had several evaluations in her school years. In first grade (2006), Annette was evaluated by Angela Baker, M.S., an Educational Specialist in Portland, OR, because of problems with reading and spelling. The test results showed that she had difficulty memorizing words for reading and spelling, had trouble learning the sounds of letters, and often reversed letters. She also had trouble with mathematics; she did not know the quantity indicated by some numbers, and added by counting on her fingers. Furthermore, problems with attention and concentration affected her performance on all tasks. In contrast, she had strengths in nonverbal abilities such as art, music, and performing arts. Annette was diagnosed with a language-based learning disability.

This same year, Annette's tutor suggested that Dr. Randolph Simmons evaluate her to get a better understanding of her cognitive strengths and weaknesses. Dr. Simmons' results indicated that Annette had good logical reasoning when the problem did not involve language and that she had a good eye for visual details. He stated that she had weak attention and concentration, difficulty understanding some aspects of oral language, and problems in reading, written language, and mathematics. He recommended multisensory reading and writing instruction, such as an Orton-Gillingham approach (which she was already receiving), TouchMath, and year-round tutoring. He also said that learning would be easier for her if her instruction included visual information, such as pictures and simple charts.

In third grade (2008), Annette was evaluated by Dr. Jeff Rett, an educational consultant from the Dyslexia Institute, in Denver, CO. Dr. Rett diagnosed her with moderate “dysphoneidetic” dyslexia, meaning that she had problems working with the sounds within words and remembering what words look like.

In fourth grade (2009), the IEP team of Harper Elementary School referred Annette to Rosemary Arnold, Ph.D., psychologist, for a psychoeducational evaluation, and to Josephine Barker, SLP-CCC, for a speech-language evaluation. Dr. Arnold reported that Annette's general intelligence was in the lower end of the average for her age and that her reading, written language, and math skills were one to two years below grade level. Dr. Arnold diagnosed Annette with specific learning disabilities in all three academic areas and recommended special education. She also reported that Annette felt embarrassed about her problems with learning and thought she was “dumb”; she had difficulty paying attention, and was having problems with some friendships. All of these made learning even more difficult for her. The results of the speech-language evaluation were that Annette's understanding of and use of vocabulary were somewhat weak but that her sentence structure and grammar, and her understanding of how to speak with people in a social setting, were similar to those of others of her age. She recommended that Annette receive speech and language services to increase her vocabulary and her ability to listen more attentively in the classroom.

Throughout the fourth and fifth grades at Harper Elementary, Annette's teachers used curriculum-based measurements (CBMs) to monitor her progress. Annette made steady but slow progress. From the fall of fourth grade to the spring of fifth grade, Annette's reading rate increased from 47 to 87 correct words per minute. The average reading rate for spring of fifth grade is 139 (Hasbrouck & Tindal, 2005).

Tests Administered

  1. Woodcock-Johnson IV Tests of Cognitive Abilities (WJ IV COG)
  2. Woodcock-Johnson IV Tests of Oral Language (WJ IV OL)
  3. Woodcock-Johnson IV Tests of Achievement (WJ IV ACH)
  4. (Scores attached at the end of this report).
  5. Test of Silent Word Reading Fluency-2 (TOSWRF-2)
  6. CBM passages, sixth and eighth grades

Standard Scores and Percentile Ranks

The following table shows the standard score (SS) and percentile rank (PR) ranges and the verbal labels associated with them. The table also states the percentage of individuals who score within each range. The terms “cluster” and “composite” refer to a group of two or more tests that assess a similar ability. The cluster or composite score represents the person's performance on the combination of tests. Almost all of the scores in this test are given as percentile ranks. These scores show the percentage of students of Annette's age (year and month) whose score was the same as or below hers. For example, PR 61 means that Annette's age peers obtained either the same score or a lower score than she did.

Standard Score Range Descriptor Percentage of People Percentile Rank Range
>130 Very Superior 2 98th & above
121–130 Superior 7 91st to 97th
111–120 High Average 16 75th to 90th
90–110 Average 50 25th to 74th
80–89 Low Average 16 9th to 24th
70–79 Low 7 3rd to 8th
<69 Very Low 2 2nd & below

Relative Proficiency Index (RPI)

The Relative Proficiency Index and descriptions of a person's proficiency based on the RPI ranges is another method of describing findings on the various tests of the WJ IV. The RPI is an index of the student's predicted proficiency or success on tasks similar to the ones tested. An RPI of 71/90 means that the student is predicted to be 71% proficient in tasks similar to the ones on the test when an average age-peer would be 90% proficient (i.e., would have achieved mastery). A person with an RPI of 71/90 has limited to average proficiency and would find the task difficult.

To achieve a full understanding of a student's performance on the tests of the WJ IV, it is best to consider the standard scores or percentile ranks, and the RPIs.

Test Behavior

Annette was extremely cooperative and attentive during the three 2-hour testing sessions, despite the fact that testing was conducted in the first session after a full school day. Annette worked diligently, but she required a considerable amount of time on both the reading comprehension and mathematics tests. On several occasions, Annette asked for instructions to be repeated or she noted that she had not been listening.

RPI Range Level of Proficiency Instructional Implications
100/90 Very Advanced Extremely easy
100/90 to 98/90 Advanced Very easy
98/90 to 95/90 Average to Advanced Easy
95/90 to 82/90 Average Manageable
82/90 to 67/90 Limited to Average Difficult
67/90 to 24/90 Limited Very difficult
24/90 to 3/90 Very Limited Extremely difficult
3/90 to 0/90 Extremely Limited Nearly impossible

WJ IV Cognitive Battery (WJ IV COG)

According to the results of this evaluation, Annette's overall intelligence is in the lower end of the Average range (general intellectual ability: SS 91, PR 27). However, those scores that make up the general intellectual ability (GIA) varied over 1.5 standard deviations, so a GIA score does not provide a viable summary of Annette's cognitive abilities. Although many of her test scores were within the Average range, specific strengths and weaknesses were noted across different types of abilities.

Annette's abilities were slightly below, similar to, or better than others of her age in the following cognitive areas:

  • Short-term working memory (PR 41): Annette is able to integrate many aspects of memory that allow her to do the complex mental work related to learning and to the activities of daily living. She can hold information in mental awareness while working with it (possibly adding to it or changing it), associate it with information already in her long-term memory, and place it in long-term memory.
  • Long-term retrieval (PR 27): Based on these tests, Visual-Auditory Learning, and Story Recall, Annette's Long-Term Retrieval cluster score was in the lower end of the Average range. Her RPI is likely a better measure of her response on this cluster, suggesting that when her average age-peers are 90% successful in recalling previously learned information from long-term memory, she will be 62% successful. She recalled many of the key details of the brief stories she listened to and was able to learn the words associated with abstract symbols well enough to “read” a story written in the symbols. She used strategies to help her learn the word-symbol associations and had trouble on those symbols that were more difficult to associate with their meanings. Annette's ability to store and retrieve information from her memory is adequate for narratives and learning word-symbol association if she is able to create a “hook” for them. A positive note is that she spontaneously created a strategy to help her learn the word–symbol associations. In contrast, she may not be as strong with information that seems arbitrary on its face, such as letter sounds when presented in different syllable configurations, words and their meanings, math facts and algorithms, unless she can create or is directly taught the underlying logical concept.
  • Fluid reasoning (PR 36): Annette is able to use inductive reasoning to work out generalizations or rules that govern a problem, and so solve it. In this test, the problems were presented as visual designs, and the instructions were verbal and somewhat complex. This was a controlled learning test, so Annette was given corrective feedback each time she made an error; she often used that feedback to work out subsequent problems.
  • Visual processing (PR 77): Annette demonstrated a strength in visual processing, the ability to perceive visual representations (e.g., pictures, maps, charts, structures) as whole entities and, simultaneously, note their parts. She can envision how something would look if seen from a different perspective or if it were taken apart and reassembled. She is also good at recalling the details of pictures she has seen previously. In school, visual processing is particularly important in courses such as geometry, physics, and art, as well as in finding one's way from one part of the building or campus to another.

Annette's most significant weaknesses were in the following areas:

  • Processing speed (PR 6) and the related area of perceptual speed (PR 5): Processing speed is the ability to do simple tasks quickly; perceptual speed is the ability to look quickly at visual symbols, such as letters and numbers, and instantly see their similarities and differences. These abilities are important because if a person can do the simple parts of a task or skill quickly and easily, attention is available to focus on the more complex or meaningful parts of the task. For example, if a person can recall math facts automatically, she has attention available to focus on the multiple steps in a division problem, while estimating whether the final answer makes sense, or to combine that answer with the other operations necessary in a multiple-step word problem. In reading, if a person has learned to sound out or recognize words to an automatic level, she has more attention available to focus on the meaning. The same is true for every other task or activity; the more you can do automatically, the more attention you have available for the more complex and/or meaningful levels. On the speed tasks, Annette was accurate but slow.
  • Vocabulary (PR 10): Annette had difficulty giving antonyms and synonyms for words. Sometimes she did not know what the target word meant, and sometimes she could not think of an antonym or synonym. She had similar difficulty naming pictures of objects, some of which are usually learned in school (e.g., calculator, sieve).
  • General information (PR 15): Annette had difficulty giving the uses for or typical locations of different objects in the environment (e.g., What are blinds used for? Where would you find a whisk?). This test also requires vocabulary knowledge, because it is not presented with pictures. Annette might know what a whisk is if she saw it but not know its name.

WJ IV Oral Language

Annette's ability to understand spoken information, including understanding and remembering instructions involving complex syntax, was similar to that of her age peers, equaling or exceeding 45%. Because her vocabulary is low, however, she may not understand some of the words in a lecture or in reading material. She has more difficulty expressing herself orally, partially because she may not know the specific words necessary to articulate her ideas and partially because she sometimes cannot find the words she wants to use. In testing, she had great difficulty coming up with antonyms and synonyms for given words, a difficulty noted previously in her speech and language evaluation, and was considerably slower than age peers in naming pictures of common objects that she does know (e.g., spoon, door). Again, her performance met or exceeded that of only 5% of age peers.

Surprisingly, given the amount of tutoring that Annette has had, her knowledge of phonetic coding was quite weak. Although her percentile rank indicated that she exceeded 25% of her age peers, her RPI of 75/90 indicated that she has not mastered the most basic subskills for reading and spelling –—blending individual sounds into whole words and pulling words apart into individual sounds. On similar sound blending and segmenting tasks, she is likely to be 75% successful when her typical age peers are 90% successful.

WJ IV Achievement

On the Woodcock-Johnson Tests of Achievement (WJ IV ACH), most of Annette's scores on the reading, written language, and mathematics tests were in the Low to Low Average range. Annette had the most difficulty on measures of mathematics and reading comprehension.

Reading Accuracy and Comprehension

Annette had weaknesses in basic reading skills, identifying words in lists, many of which were irregular, and reading nonsense words that were spelled according to English spelling patterns. Her performance in basic reading skills met or surpassed that of only 17% of others her age, placing her performance in the Low Average range. Her comprehension of brief passages and her ability to tell the details of what she had read were considerably lower, higher than that of only 2% of age peers. Although she was instructed to read the passages to herself, she subvocalized as she read. She had difficulty breaking up the longer words into syllables as an aid in sounding them out. On the items closer to her age level, she used a questioning tone after reading a word correctly, as though she did not know what it meant. Despite her difficulty, Annette commented that she enjoys reading when she likes the book.

Reading Rate

Annette's performance on measures of reading speed was Below Average. On the TOSWRF-2, Annette's performance fell in the Low Average range, with a percentile rank of 14, and grade equivalent of 6.2, indicating that the speed at which she recognized individual words was similar to that of sixth-grade students. On the two WJ IV tests of reading rate (Sentence Reading Fluency, Word Reading Fluency), both of which required Annette to understand what she was reading, her scores were in the Low range, higher than only 3% and 5% of other 15-year-olds, respectively. When asked whether she required more time in school when taking tests, Annette responded, “my teachers don't believe in tests.”

Annette read several CBM passages at the sixth- and eighth-grade levels. At the sixth-grade level, her median reading rate was 138 words correct per minute (WCPM) with no reading errors. On the eighth-grade passages, Annette's median reading rate was 100 WCPM, with an average of six errors. The 50th percentile on eighth-grade passages is approximately 150 WCPM. These findings indicate that Annette will need an adjustment in the difficulty of the reading material, as well as shortened reading assignments or extended time.

Mathematics

Annette had the most difficulty with the measures of mathematics. Her overall math performance exceeded only 2% of her age-peers. Annette's lowest score was on a speed test of recalling addition and subtraction math facts, in which her performance exceeded that of only 1% of her age-peers. The time ran out before she reached the multiplication facts. Her work on math computation problems indicated that, when compared with others at her age or grade level, Annette has forgotten many basic math skills. She missed problems involving subtraction with regrouping; multiplication of double-digit numbers; long division; and adding, subtracting, and multiplying with fractions. She was unable to solve word problems involving fractions and had difficulty counting coins quickly and accurately. Several of Annette's responses showed that she had difficulty with estimation, because her answers were not reasonable. For example, when given a problem such as: Sally walks 2½ miles to the store from her house. How many miles does she walk to the store and back? Annette responded after thinking for a minute, “30 and 10/40s.”

Written Language

Annette's ability to formulate and write sentences was solidly in the Average range. On the Sentence Writing Fluency test, when writing brief, simple sentences against a time limit, she exceeded 36% of her age peers. Without a time limit and asked to write more complex sentences according to a variety of prompts, she scored higher than 45% of her age peers. Annette had difficulty with spelling, exceeding only 10% of her age peers. Although incorrect, many of her misspellings were close approximations of the real word (e.g., spelling mention as menshion). She occasionally confused homonyms (e.g., their for there) and vowel sounds (e.g., most for must). Thus, Annette's ability to express her thoughts and ideas in writing surpasses her spelling ability.

Discrepancies

Although no longer required by federal law, Annette's school district still requires an aptitude-achievement discrepancy for a student to qualify for services as a person with learning disabilities. Annette has significant discrepancies between her intellectual ability and her reading comprehension and mathematics achievement. The size of the discrepancies between her GIA and her scores on the Reading Comprehension and Basic Math clusters are such that they would be found in only 6% and 3%, respectively, of her age peers.

More extensive discrepancies were apparent when her oral language ability was used to predict her reading comprehension and basic math skills. In fact, when her oral language was compared with her reading comprehension, only 5 in 1,000 people would have a discrepancy as large; when her oral language was compared with her math achievement, only 2 out of 100 would have a discrepancy as large.

Conclusions

Annette is a student with an established history of specific learning disabilities. Each evaluation of Annette across the school years has demonstrated consistent findings; she has average to above average nonverbal reasoning, attention to and memory for visual detail, and oral language comprehension. This evaluation found relative strengths in short-term working memory and long-term retrieval for narratives and other types of information, depending on the strategies she uses to process it.

Annette demonstrated significant weaknesses in processing speed, perceptual speed, and vocabulary. Despite years of tutoring in reading, Annette still has a relative weakness in phonetic coding. These weaknesses have contributed to Annette's difficulty in learning basic reading skills and basic math skills. Her limited reading has likely affected her acquisition of new vocabulary. Presently, both math and reading comprehension measures are significantly below what would be predicted based on her intelligence and her oral language abilities. Given her cognitive and language strengths, if Annette increased her vocabulary and learned basic reading and math skills to an automatic level, she would likely get good benefit from learning reading strategies and a focus on the concepts underlying math algorithms and applications.

Annette is motivated, responsible, and willing to work hard to accomplish her goals. With the proper supports and interventions, adjustment of the difficulty levels of the academic tasks, and the establishment of realistic goals, she is capable of being successful in an academic environment.

Recommendations

Strengths

  1. Ensure that Annette has time for electives that capitalize on her strengths (e.g., art classes, design classes, theater).
  2. Annette is likely to benefit from using and reviewing graphic organizers and visual displays to help her with vocabulary development and reading comprehension.
  3. Use pictures as much as possible when teaching vocabulary and incorporate strategies such as the Key Word technique. (See Mnemonic Strategies in Section IV.)
  4. In content area classes, visual content (e.g., pictures, videos, graphs, charts, semantic webs) associated with lectures will help Annette to understand and retain the information.

Accommodations and Modifications

  1. Annette will require extended time on any testing involving reading and mathematics. This will include high-stakes tests, such as the SATs. She will need double time for the reading and mathematics sections. She is unlikely to need extended time on written essays.
  2. Within the classroom, Annette will need adjustments to the difficulty level of reading, writing, and mathematics assignments.
  3. Annette will need specific guidelines regarding what she is expected to learn in content area classes. She should be provided with structured study guides for in-class exams. These guides should state explicitly what she is expected to learn.
  4. Encourage Annette to ask for clarifications of directions and assignments, when needed.
  5. Because it takes Annette longer to complete tasks than most of her peers, make sure that the assigned homework does not require an inordinate amount of time to complete. If possible, specify the amount of time that she should spend working on an assignment, rather than the number of problems to complete.

Reading and Written Language

  1. Annette would benefit from direct instruction to improve her performance in pronouncing multisyllabic words. She should learn to identify prefixes, suffixes, and root words. She would benefit from a program, such as REWARDS (Sopris West) that teaches a strategy for pronouncing longer words.
  2. Annette should be encouraged to write her papers on a computer so that she that she can revise with ease and have immediate access to spell check. On initial drafts of papers, provide Annette with feedback and editorial assistance before grading her written products.
  3. Annette should continue to engage in meaningful reading and writing activities that will promote growth in vocabulary and general knowledge.
  4. Annette would benefit from listening to audio books as she follows along with a printed copy. Select books that are of high interest to Annette, such as mysteries.

Mathematics

  1. At home, encourage and provide opportunities for Annette to improve her practical math skills involving time, money, and measurement. For example, have Annette count the change after a purchase, use a measuring cup when baking, or discuss the concept of fractions using a pizza.
  2. Annette would benefit from structured math tutoring from a highly qualified math teacher. For math instruction, she needs to review and practice the following skills:
    • drill and practice on math facts (Many good online programs are available for this purpose.)
    • remembering to pay attention to the mathematical signs before completing computations
    • counting coins accurately and quickly with sums up to $25.00
    • determining future time (e.g., if it is 9:00 a.m. now, what time would it be one hour and fifteen minutes from now?)
    • reviewing basic measurement concepts (e.g., how many inches in a foot, feet in a yard)
    • remembering to regroup when adding and subtracting
    • performing all operations involving fractions (addition, subtraction, multiplication, division)
    • performing all operations with negative numbers
    • performing all operations with numbers with decimal points
    • converting fractions to mixed numbers
    • expressing a mixed number in lowest terms
    • finding the least common denominator
    • converting fractions to decimals and percentages
    • reviewing the relationships among fractions, decimals, and percentages
    • determining a certain percentage of a number (e.g., 25% of 80)
    • estimating answers before solving problems
    • solving basic algebraic equations

Note

The following case study, done by a private licensed psychologist, started a controversy. Anita's parents were unhappy with a diagnosis of intellectual disability and did not feel that it was an accurate representation of their daughter's strengths and weaknesses. They also questioned why, in all the time Anita had been in school, and with all of her other evaluations, no evaluator had ever diagnosed her previously as having an intellectual disability. They took her records and this report to another private psychologist for a second opinion. That psychologist's opinion was that the current evaluation results and Anita's educational history did not meet the criteria for a diagnosis of intellectual disability. Based on all of the information presented, the psychologist could not come up with a clear diagnosis of any specific disability. His opinion was that Anita could have multiple cognitive, language, and attentional weaknesses, but not meet the Diagnostic and Statistical Manual of Mental Disorders, Fifth Edition, or IDEA 2004 criteria for any specific disability, and still require special education. This case report is included because the authors think that many children, adolescents, and adults who are evaluated for learning problems do not fit cleanly into one or even multiple diagnoses. This issue is an important one for discussion.

Psychological Evaluation

Name: Anita Velasquez
Date of Birth: June 1, 1997
Age: 18 years 1 month
Date of Evaluation: June 30, 2015
School: Campbell High School
Grade: 11.9

Reason for Referral

Anita's parents requested a psychological evaluation to determine Anita's present level of cognitive functioning. Over the years, there has been much confusion over Anita's actual cognitive and adaptive abilities, as well as the determination of an appropriate plan for her education.

Background Information

Anita is an 18-year-old adolescent completing her junior year of high school. She currently has an IEP and receives special education services under the category of specific learning disability (SLD). Her mother stated that her hearing and vision have always tested as normal. Her last hearing and vision evaluations were approximately one-and-a-half years ago.

Developmental, Social, and Educational History

According to an Osceola County Psychoeducational Report (2006), the following developmental history was obtained from Anita's mother (via a translator). Anita was the product of a normal pregnancy and delivery. Developmental milestones were reported to have been attained within normal limits. Anita had hearing problems at age 3 and high fevers at age 5, prompting her parents to wait a year before enrolling her in kindergarten. She began wearing glasses in 2005 at age 8. Her father speaks Spanish and English, and her mother speaks only Spanish. They immigrated from Puerto Rico just before Anita was born, so she is an American citizen. She began speaking English when she enrolled in pre-kindergarten and began receiving instruction in English as a second language in kindergarten.

School History

Anita attends Campbell High School in Campbell City, Florida. Previously, she attended school in the Fort Lauderdale School District (FLSD). In Spring 2014, in tenth grade, Anita participated in the Florida Comprehensive Assessment Test (FCAT) at her former school, Loma Vista High School. The results indicated that she “did not meet the standard” in the areas of Biology, Mathematics, or Literature and Composition.

The following is a summary of Anita's previous evaluation results and information in her Individualized Education Plans (IEP), earliest information presented first.

Anita's April 2014 IEP cited information from a special education report written in 2011, when Anita was 14. The report stated that Anita received services under the category of SLD and speech/language impairment (SLI). Her areas of weakness were attention, nonverbal reasoning, working memory, and visual-motor integration, with significant processing deficits in verbal reasoning, verbal comprehension, and auditory processing. Academic weaknesses included math calculation, math reasoning, reading comprehension, and spelling. Concerns were also noted regarding her low self-esteem. The 2014 IEP did not include the scores on which the 2011 diagnoses of SLD and SLI were based.

In November 2011, Dr. Michael Madrone, Ph.D., a psychologist in private practice, evaluated Anita and reported the following results (cited in a FLSD Psychoeducational Report, 3/18/2012):

Wechsler Intelligence Scale for Children, Fourth Edition (WISC-IV)

Standard Score
Full Scale IQ 74
Verbal Comprehension Index 83
Perceptual Reasoning Index 71
Working Memory Index 77
Processing Speed Index 88
*Standard scores: Mean = 100, SD = 15.

An FLSD evaluation conducted in March 2012 reported the results for the following test batteries:

Reynolds Intellectual Assessment Scales (RIAS)*—Standard Score

Standard Score
Composite Intelligence Index 70
Composite Memory Index 89
Verbal Intelligence Index 83
Nonverbal Intelligence Index 83
*Standard scores: Mean = 100, SD = 15.

For this evaluation, Anita's parents and teacher completed the Adaptive Behavior Assessment System, Second Edition (ABAS-II), a rating scale of adaptive behaviors. A significant discrepancy existed between the school and home reports, with the parents expressing considerable concerns.

Adaptive Behavior Assessment System, Second Edition (ABAS-2)*

Parents Teacher
General Adaptive Composite 56 101
Conceptual Composite 51 93
Social Composite 58 99
Practical Composite 70 108
*Standard scores: Mean = 100, SD = 15.

Anita's 2013 IEP reported that she needed speech-language therapy to improve her ability “to answer questions, express thoughts and ideas, and participate in oral discussion.” Test results showed significant weaknesses in syntax, semantics, and comprehension.

The 2014 IEP stated that Anita's parents continued to be concerned about her understanding of time-related and money-related problems. They requested, and the team agreed, that Anita should transfer into Essentials English (special education), because she could not keep up in her current English class.

Observations and Impressions

Anita was seen in the morning for one formal testing session. She was friendly, engaged easily, and cooperated fully. Throughout testing, she followed directions and consistently attended to and persisted on tasks, but she was passive in her approach. She demonstrated adequate comprehension regarding test instructions and appeared to put forth her best effort. Considering that Anita was tested in a one-to-one environment, with breaks as needed, likely her functioning on the tests indicate her optimal ability.

Tests Administered

  1. Adaptive Behavior Assessment System, Second Edition (ABAS-II) Parent Form (Ages 5–21)
  2. Woodcock-Johnson IV Tests of Cognitive Abilities (WJ IV COG)
  3. Woodcock-Johnson IV Tests of Achievement (WJ IV ACH) Form A
  4. Parent Interview
  5. Review of Records

Explanation of Scores

The descriptors used refer to ranges of percentile ranks (PR). The PR shows the percentage of 18-year-olds whose score is equal to or lower than Anita's.

Percentile Rank Range Descriptor Percentile Rank Range Descriptor
98 or higher Very superior 16–24 Low average
91–97 Superior 9–15 Below average
85–90 Above average 2–8 Low
76–84 High average 0.7–1.9 Very low
25–75 Average 0.6 or lower Extremely low

The relative proficiency index (RPI) indicates Anita's likely proficiency in a skill when the average 18-year-old would be 90% proficient (indicating mastery of the skill). The top number is Anita's proficiency; the 90 on the bottom represents the comparison with mastery.

RPI Proficiency Instructional Implications
100/90 Very advanced Extremely easy
98/90 to 100/90 Advanced Very easy
95/90 to 98/90 Average to advanced Easy
82/90 to 95/90 Average Manageable
67/90 to 82/90 Limited to average Difficult
24/90 to 67/90 Limited Very difficult
23/90 to 24/90 Very limited Extremely difficult
0/90 to 3/90 Extremely limited Nearly impossible

Except for the section on adaptive skills, all of the tests and scores referred to in this report are from the WJ IV. The scores are based on comparison with other people of Anita's age in the norming sample.

Test Results and Interpretation

Cognitive Abilities

Intellectual Functioning

“Intelligence” can be thought of as the strength and efficiency of a combination of brain functions working together to allow us to learn new information (what) and procedures (how to) and, thus, function in the world. In turn, the strength and efficiency of the individual brain functions (usually called cognitive abilities and cognitive processes) as well as their ability to work in combination determine the ease with which we learn new information and procedures. Cognitive abilities and processes can often be increased through education and explicit training, more so if started when a child is young.

The WJ IV has two measures of general intelligence, the General Intellectual Ability composite (GIA), which is based on a combination of important cognitive abilities, and the Gf-Gc Composite, which represents a person's ability to use logical reasoning and the amount of knowledge she has acquired. The Gf-Gc Composite does not include the cognitive abilities that typically cause learning disabilities. Anita's overall intelligence, as measured by both the GIA and the Gf-Gc Composite, was in the 1st percentile. A caveat, however, is that both scores include a test that requires some knowledge of math, which is an area of significant weakness for Anita, so that reduced her score. Nevertheless, a rough estimate of her GIA or Gf-Gc Composite scores without including this test would be within the 3rd to 7th percentile rank (PR)—as high as or higher than only 3% to 7% of people of her age. The Fluid Reasoning cluster, which was intended to assess her ability to use logical reasoning to solve problems in unfamiliar situations, also contained the math-related test. If just the other test is considered, her fluid reasoning ability is in the Low range, equal to or above that of only 5% of her age-peers.

Comprehension-Knowledge

Anita's comprehension-knowledge was in the Low range. This is a general concept that represents the knowledge she has acquired deliberately or incidentally throughout her lifetime and her ability to communicate it verbally to someone else (Mather & Wendling, 2014, p. 33). Her RPI indicates that when average age-peers would be 90% proficient in this type of knowledge and communication, Anita would be only 50% proficient.

Cognitive Processing Speed

Anita's test results indicate that her ability to perform relatively easy or over-learned tasks, especially when focused concentration is required, is significantly slower than that of average 18-year-olds. Consequently, it is more difficult for her, and takes her longer, to learn simple skills to the point at which they are automatic, freeing her cognitive attention to focus on more complex tasks. Anita scored in the Low range (PR 4).

Anita's perceptual speed was equally slow (PR 7), indicating that she has difficulty rapidly and accurately comparing visual symbols (e.g., letters, numbers, math symbols) and recognizing similarities and differences. Perceptual speed is important in developing automatic identification of these symbols for use in higher-level tasks, such as reading, writing, and math.

Short-Term Working Memory

Short-term working memory can be thought of as the agent that holds information in mental awareness while we work with it and transform it in some way. It is the gateway through which new or transformed information is placed in long-term memory. For example, working memory is activated when a child is trying to sound out a new word—sequentially retrieving the sound for each letter from long-term memory, holding on to them in sequence, and blending the sounds while trying to match the result to a word in his or her vocabulary. Working memory has a limited capacity in terms of the length of time that information can be held unless the person is continuously focusing attention on it and working with it. Anita's short-term working memory was in the Low range (PR 5). When compared with other 18-year-olds who would be successful 90% of the time in using short-term working memory in age-appropriate tasks, Anita is predicted to be 39% successful.

Long-Term Retrieval

Anita demonstrated a personal strength in her ability to retell the details of brief stories immediately after listening to them. She scored in the Below Average range (Story Recall, PR 14) and would likely be 76% successful in similar tasks compared with her age-peers' 90%. Although she retold the details of the story immediately after hearing them, long-term retrieval is considered the retrieval of information from long-term memory after the information has been removed from immediate awareness. It is likely that the meaningful context and the familiar structure of a story were benefits to Anita in understanding and storing the information.

Visualization

The Visualization test on the WJ IV measures the ability to perceive and mentally work with the spatial aspect of objects, visualizing how an object would look if it were rotated or seen from a different perspective. Anita performed in the Low range on this test (Visualization, PR 7).

Vocabulary and Language Comprehension

Subjectively, Anita's language seemed age-appropriate in casual conversation. She was, however, unable to express higher-level ideas and did not understand idioms and analogies. In testing, Anita demonstrated limited vocabulary, with little ability to provide synonyms and antonyms for given words. Sometimes she said, “I know what the word means but I can't think of another one.” She scored in the Low range (PR 7, RPI 56/90).

The other oral language test administered was Story Recall, discussed previously. Because Anita was able to retell the brief stories, one may reasonably assume that she understood them. Accordingly, it appears that she understands brief discourse, at least in a narrative structure, although the number of details she was able to retell accurately was at the level of an average 9-year-old.

Phonological Processing

Anita performed in the Below Average range on a test of various skills that assess an understanding of words as composed of individual sounds, including the ability to retrieve words from memory, with and without a time limit, based on the composition or position of a particular sound in the word (e.g., “Tell me a word that begins with /f/”). These skills are necessary for learning basic reading and spelling skills and are either taught before or along with those skills. Some children develop them without instruction. Anita demonstrated Below Average phonological processing skills (PR 10). Based on her RPI of 64/90, tasks similar to these will likely be very difficult for Anita.

Academic Achievement

CLUSTER/Test GE AE SS (68% CI) PR RPI Proficiency
BASIC READING SKILLS 4.2 9–7 75 (72–78) 5 28/90 Limited
Word Attack 4.0 9–5 84 (79–89) 14 61/90 Limited
Letter-Word Identification 4.7 10–1 70 (66–74) 2 9/90 Very Limited
Sentence Reading Fluency 4.8 10–3 77 (71–72) 6 5/90 Very limited
READING COMP 4.4 9–10 74 (71–77) 4 49/90 Limited
Passage Comprehension 5.5 10–11 81 (77–86) 11 43/90 Limited
Reading Recall 3.2 8–7 76 (71–82) 6 61/90 Limited
Reading Vocabulary 4.4 9–10 75 (70–80) 5 41/90 Limited

Reading

Anita demonstrated decoding skills in the Below Average range when decoding phonetically regular nonsense words. She was able to sound out one-syllable and some two-syllable words (Word Attack, PR 14, RPI 61/90). Her reading of real words was in the Very Low to Low range, equal to or higher than only 2% of her age-peers. One of the reasons she was more successful in reading the nonsense words rather than the real words is that only the real words contained irregular elements—word parts that cannot be sounded out but must be memorized (e.g., enough, friend). Additionally, she did not know how to divide words into syllables.

In all of the tests of reading comprehension, including reading one- to two-sentence passages for comprehension, reading vocabulary, and retelling the details of brief stories she read, her RPIs indicated that tasks similar to these, which are required daily in the classroom and in daily life, will be very difficult for her. All of these skills were significantly lower than those of average 18-year-olds.

CLUSTER/Test GE AE SS (68% CI) PR RPI Proficiency
BASIC WRITING SKILLS 5.0 10–5 76 (72–79) 5 30/90 Limited
Editing 4.6 11–2 82 (78–87) 12 47/90 Limited
Spelling 5.8 10–0 73 (69–77) 4 17/90 Very limited
WRITTEN EXPRESSION 1.5 6–11 35 (23–47) <0.1 3/90 Extremely limited
Writing Samples 4.6 10–0 80 (75–85) 9 49/90 Limited
Sentence Writing Fluency <K.5 <5–11 1(1–14) <0.1 0/90 Extremely limited

Basic Writing Skills

Anita's editing abilities are basic. She knew that the end of a sentence needs punctuation and that the beginning needs a capital letter, and she was able to catch some basic spelling errors. She did not identify grammatical errors; occasionally, she recognized that a sentence sounded wrong but could not correct it. Her RPI of 47/90 indicates that editing anything she writes will be very difficult for her.

Spelling was more difficult. Similar to her reading, she was able to spell words of one syllable only if they were spelled the way they sound, such as pet. Her RPI of 17/90 suggests that spelling is extremely difficult for her.

Written Expression

The Writing Samples test asks the student to write single sentences according to different instructions for each sentence. Anita generated and wrote single sentences but did not always follow the instructions. For example, when asked to write a sentence using a specific word in the middle, she started her sentence with that word. Many of her sentences had words that were spelled incorrectly or had grammatical errors, but she was not penalized for this nor for the time it took her to think of a sentence to write, which was substantial.

The Sentence Writing Fluency test is timed. Anita's score was extremely low, with an RPI of 0/90. Because she was so slow in thinking up each sentence, even though she was given some of the words and was told to write short, simple sentences, she did not get many written. Again, spelling and minor grammatical errors were not penalized. Overall, Anita demonstrated difficulty with basic writing skills but also with generating language to fit specific requirements.

CLUSTER/Test GE AE SS (68% CI) PR RPI Proficiency
MATH CALCULATION SKILLS 1.9 7–4 42 (37–46) <0.1 0/90 Extremely limited
Calculation 2.9 8–4 58 (54–62) 0.3 1/90 Extremely limited
Math Facts Fluency 1.3 6–8 31 (24–39) <0.1 0/90 Extremely limited
MATH PROBLEM SOLVING 2.1 7–6 55 (51–59) 0.1 3/90 Extremely limited
Applied Problems 2.3 7–9 56 (51–61) 0.2 2/90 Extremely limited
Number Matrices 1.7 7–2 63 (58–68) 1 4/90 Very limited

Math

Math is Anita's academic area of greatest difficulty. She does not know her math facts and counted on her fingers; consequently, her speed of responding to simple math facts was equal to or higher than less than one in 1,000 age-peers. She was able to add 1-digit and two 2-digit numbers if they did not require regrouping. She subtracted only by 0 or 1. She answered only very simple word problems that were accompanied by pictures, which she used to count the pictured objects for addition and subtraction. She was not able to answer problems without pictures, nor was she able to tell time to the hour on an analog clock

.
CLUSTER/Test GE AE SS (68% CI) PR RPI Proficiency
ACADEMIC KNOWLEDGE 4.1 9–6 73 (70–77) 4 33/90 Limited
Science 6.2 11–7 89 (83–94) 23 69/90 Limited to average
Social Studies 3.5 8–11 69 (64–75) 2 16/90 Very limited
Humanities 3.1 8–6 71 (66–76) 3 21/90 Very limited

Academic Knowledge

Anita's responses on the tests of Science, Social Studies, and Humanities (e.g., music, literature) suggested that she is most knowledgeable about science. She could name animals, tell what sounds they make, explain what was happening in certain pictures, and recognize emergency vehicles. She did not know the functions of the organs of the body. She responded to only a few questions regarding social studies or humanities.

Adaptive Skills

Anita's mother completed rating scales to estimate Anita's current adaptive skills and the ability to function independently. The General Adaptive Composite represents a comprehensive estimate of an individual's adaptive functioning. According to her mother's responses, Anita's general adaptive skills are in the 2nd percentile.

Conceptual Composite

All of Anita's scores in this composite fell into the 5th percentile or below. The areas subsumed under this composite are:

  • Communication abilities, such as speech, vocabulary, listening, conversation, and nonverbal communication
  • Basic Academic Skills, such as reading, writing, and mathematics
  • Functional Skills, such as measurement and telling time
  • Self-Direction, such as the ability to make independent choices, exhibit self-control, and take responsibility when appropriate

Social Composite

In the Social Composite, Anita's scores were both in the 1st percentile. This composite encompasses:

  • Leisure, which includes the skills needed for participating in enjoyable activities and planning recreational activities
  • Social, which includes the ability to interact socially, initiate and maintain friendships, express and recognize emotions, and assist others when needed

Practical Composite

In the Practical Composite, Anita's competences are varied. Her strongest area of competence is Self-Care (PR 74, Average/High Average), followed by Home Living (PR 50, Average). Self-Care includes activities necessary for taking care of one's basic needs such as eating, dressing, and taking care of personal hygiene. Home Living encompasses functioning inside the home, including cleaning, food preparation, performing chores, and taking care of personal possessions. Anita's mother stressed that she has worked hard with Anita to make sure that she would be able to take care of herself, at least inside their home. In contrast, Anita's competence in the area of Health and Safety is Low (PR 5) and in Community Use, Extremely Low (PR <1). Health and Safety includes Anita's ability to protect her physical well-being and prevent and respond to injuries, including following safety rules, showing caution, and using medicine when appropriate. Community Use includes the ability to function and get around in the community, including shopping, using transportation, and using community resources.

Summary and Conclusions

Anita is an 18-year-old adolescent about to enter twelfth grade. She has a history of developmental delays, including speech and academic difficulties. Her overall conceptual ability is in the Very Low range. She demonstrated mild impairment on language-based tasks and severe impairment on nonverbal and fluid reasoning tasks. Cognitive processing speed and cognitive efficiency were in the Low range. Anita's overall abilities on memory and retrieval were in the Below Average to Low range. Her ability to visualize and reason about spatial relationships was in the Low range. She performed in the Below Average range on auditory processing tasks.

Anita's overall abilities in reading were in the Low range. Her ability in basic writing skills and written expression were in the Very Low range. Her abilities in math calculation were in the Very Low range; her ability to comprehend and complete applied math problems was in the Extremely Low range.

In the area of basic adaptive skills, ratings placed Anita in the Extremely Low range, including conceptual, social, and practical domains. A history of developmental delays also document Anita's impairment. (See School History.) Although the ABAS II completed in 2011 showed extreme differences between the mother's ratings and the teacher's ratings, it is clear that teacher's ratings were a considerable overestimate of Anita's true adaptive abilities. She has demonstrated lower adaptive skills by consistently failing course work and the Florida State testing, and requiring additional special education classes for instruction in basic skills and speech/language services for limited communication skills. Furthermore, her mother's ratings on the ABAS II done in 2011 and the current evaluation show a slight improvement in some skills but are consistent.

Based on her current level of intellectual functioning, multiple cognitive abilities, academics, and adaptive skills, Anita meets the diagnostic criteria for intellectual disability (mild). Intellectual disability, as defined by the Diagnostic and Statistical Manual of Mental Disorders, Fifth Edition, involves impairments of general mental abilities that impact adaptive functioning in three domains. Anita demonstrates significant impairment in the cognitive domain (e.g., skills in language, reading, writing, math, and reasoning), the social domain (e.g., social judgment, interpersonal communication skills), and the practical domain (e.g., skills in community use and health and safety). These impairments provide an accurate description of Anita's current functioning.

Although Anita is now 18 years of age, and individuals with developmental disabilities are typically recognized before their eighteenth birthday, it is important to note that much of Anita's prior test results indicate and support a previous diagnosis of developmental disability. The school failed to recognize a more accurate category of disability. It is this examiner's experience (as a licensed psychologist) that often schools are reluctant to categorize an individual as having intellectual disabilities. Instead, schools categorize individuals with profiles such as Anita's as “specific learning disability” as a means for access to special education services without the “stigma” of intellectual disability. Presently, Anita meets the criteria for intellectual disability (mild).

Impressions

  1. Intellectual disability (mild)

Recommendations

  1. Given the diagnosis of intellectual disability (mild), application for services from the Department of Developmental Disabilities may be pursued. Information on resources and the application process are available on the Florida Department of Economic Security: Division of Developmental Disabilities website.
  2. Anita's school should be contacted to request a multidisciplinary meeting to address her needs in regard to the academic impact her learning difficulties are having on her ability to complete school tasks with the same level of success as her age-peers. Continued special education services would be beneficial to address her limitations in tasks requiring reading, writing, and math, including speech-language and occupational therapy. Eligibility for appropriate accommodations in district/state testing should also be addressed (e.g., out-of-level testing) as well as consideration for change of category from specific learning disability to intellectual disability. Anita's current special education services appear to address her needs at this time; however, any state-required tests to graduate from high school should be modified to reflect Anita's current level of functioning.
  3. Work with a case manager/advocate to assist in the management and implementation of accommodations necessary for Anita to be successful. Anita may experience difficulty in communicating her needs, understanding school requirements, and interacting with others (i.e., teachers, peers, group projects, etc.).
  4. Difficulty with attention has been noted in Anita's records since her early school years. Although difficulty in learning can cause a child to “tune out” of a classroom activity, a primary problem in self-regulation of attention will exacerbate her difficulty with almost any learning or task that is not intrinsically engaging to her. She should be evaluated specifically for attention deficit hyperactivity disorder, inattentive type.

Transition Planning

  1. Because Anita has already reached 18 years of age, the MET/IEP team from her home school should consider a transitional plan. She also may seek transitional services from other sources if the family does not want to work through the public school system. Resources and ideas for developing an appropriate transition plan for Anita include the following:
  2. As Anita considers her future goals, she will need assistance in acquiring the skills needed for transition, including appropriate work behaviors, as well as social, independent living, and vocational skills. Information on resources in vocational rehabilitation counseling can be found on the following websites:

Summary of Test Results

Woodcock-Johnson IV Tests of Cognitive Abilities (Norms based on age 18–1)

CLUSTER/Test W AE EASY to DIFF RPI SS (68% Band) PR
GEN INTELLECTUAL ABIL 494 9–3 7–11 to 11–3 39/90 66 (62–70) 1
Oral Vocabulary 502 10–10 8–9 to 13–9 56/90 78 (73–83) 7
Number Series 461 7–4 6–10 to 7–10 1/90 54 (48–59) 0.1
Verbal Attention 501 10–10 9–0 to 13–11 60/90 83 (77–89) 13
Letter-Pattern Matching 517 11–2 10–0 to 12–5 24/90 81 (72–90) 10
Phonological Processing 498 9–6 7–6 to 13–7 64/90 81 (76–86) 10
Story Recall 493 9–5 7–0 to 20 76/90 84 (78–89) 14
Visualization 492 8–8 6–9 to 12–6 62/90 78 (73–84) 7
Gf-Gc COMPOSITE 487 8–6 7–5 to 10–0 22/90 65 (62–68) 1
Oral Vocabulary 502 10–10 8–9 to 13–9 56/90 78 (73–83) 7
General Information 500 10–4 8–6 to 13–0 44/90 79 (74–84) 8
Concept Formation 487 8–1 6–10 to 9–9 30/90 75 (70–79) 5
Number Series 461 7–4 6–10 to 7–10 1/90 54 (48–59) 0.1
COMP-KNOWLEDGE (Gc) 501 10–7 8–8 to 13–5 50/90 78 (74–81) 7
Oral Vocabulary 502 10–10 8–9 to 13–9 56/90 78 (73–83) 7
General Information 500 10–4 8–6 to 13–0 44/90 79 (74–84) 8
FLUID REASONING (Gf) 474 7–6 6–10 to 8–5 8/90 59 (55–63) 0.3
Number Series 461 7–4 6–10 to 7–10 1/90 54 (48–59) 0.1
Concept Formation 487 8–1 6–10 to 9–9 30/90 75 (70–79) 5
S-TERM WORK MEM (Gwm) 494 9–5 8–0 to 11–7 39/90 75 (70–80) 5
Verbal Attention 501 10–10 9–0 to 13–11 60/90 83 (77–89) 13
Numbers Reversed 487 8–5 7–2 to 10–0 21/90 74 (68–80) 4
COG PROCESS SPEED (Gs) 507 10–5 9–5 to 11–6 10/90 73 (67–79) 4
Letter-Pattern Matching 517 11–2 10–0 to 12–5 24/90 81 (72–90) 10
Pair Cancellation 496 9–9 8–11 to 10–9 4/90 73 (69–78) 4
NUMBER FACILITY 499 10–6 9–5 to 11–10 29/90 75 (69–81) 5
Numbers Reversed 487 8–5 7–2 to 10–0 21/90 74 (68–80) 4
Number-Pattern Matching 511 11–8 10–8 to 12–11 38/90 83 (75–91) 13
PERCEPTUAL SPEED 514 11–5 10–5 to 12–8 30/90 78 (71–86) 7
Letter-Pattern Matching 517 11–2 10–0 to 12–5 24/90 81 (72–90) 10
Number-Pattern Matching 511 11–8 10–8 to 12–11 38/90 83 (75–91) 13
COG EFFICIENCY (Ext) 504 10–9 9–6 to 12–3 35/90 74 (69–79) 4
Verbal Attention 501 10–10 9–0 to 13–11 60/90 83 (77–89) 13
Letter-Pattern Matching 517 11–2 10–0 to 12–5 24/90 81 (72–90) 10
Numbers Reversed 487 8–5 7–2 to 10–0 21/90 74 (68–80) 4
Number-Pattern Matching 511 11–8 10–8 to 12–11 38/90 83 (75–91) 13

Woodcock-Johnson IV Tests of Achievement (Norms based on age 18–1)

CLUSTER/Test W AE EASY to DIFF RPI SS (68% Band) PR
BROAD READING 498 10–1 9–2 to 11–2 13/90 73 (70–76) 4
Passage Comprehension 502 10–11 9–2 to 13–3 43/90 81 (77–86) 11
Letter-Word Identification 491 9–5 8–7 to 10–5 9/90 70 (66–74) 2
Sentence Reading Fluency 500 10–3 9–7 to 11–0 5/90 77 (71–82) 6
BASIC READING SKILLS 494 9–7 8–6 to 11–2 28/90 75 (72–78) 5
Word Attack 496 10–1 8–4 to 13–5 61/90 84 (79–89) 14
Letter-Word Identification 491 9–5 8–7 to 10–5 9/90 70 (66–74) 2
READING COMP (Ext) 496 9–10 8–4 to 12–5 49/90 74 (71–77) 4
Passage Comprehension 502 10–11 9–2 to 13–3 43/90 81 (77–86) 11
Reading Recall 490 8–7 7–5 to 11–5 61/90 76 (71–82) 6
Reading Vocabulary 496 9–10 8–6 to 12–0 41/90 75 (70–80) 5
READING FLUENCY 500 10–3 9–5 to 11–5 17/90 76 (72–80) 6
Oral Reading 500 10–5 9–0 to 12–10 45/90 81 (78–85) 11
Sentence Reading Fluency 500 10–3 9–7 to 11–0 5/90 77 (71–82) 6
READING RATE 485 9–8 9–1 to 10–3 2/90 68 (63–74) 2
Sentence Reading Fluency 500 10–3 9–7 to 11–0 5/90 77 (71–82) 6
Word Reading Fluency 470 9–1 8–8 to 9–8 1/90 67 (58–75) 1
PHONEME-GRAPHEME KNOW 491 8–10 7–8 to 11–4 51/90 78 (74–82) 7
Word Attack 496 10–1 8–4 to 13–5 61/90 84 (79–89) 14
Spelling of Sounds 485 8–0 7–0 to 9–7 42/90 73 (68–79) 4
BROAD MATHEMATICS 460 7–5 7–0 to 8–0 0/90 40 (36–44) <0.1
Applied Problems 470 7–9 7–1 to 8–5 2/90 56 (51–61) 0.2
Calculation 473 8–4 7–10 to 9–0 1/90 58 (54–62) 0.3
Math Facts Fluency 438 6–8 6–4 to 7–0 0/90 31 (24–39) <0.1
MATH CALCULATION SKILLS 455 7–4 6–11 to 7–10 0/90 42 (37–46) <0.1
Calculation 473 8–4 7–10 to 9–0 1/90 58 (54–62) 0.3
Math Facts Fluency 438 6–8 6–4 to 7–0 0/90 31 (24–39) <0.1
MATH PROBLEM SOLVING 470 7–6 6–9 to 8–4 3/90 55 (51–59) 0.1
Applied Problems 470 7–9 7–1 to 8–5 2/90 56 (51–61) 0.2
Number Matrices 470 7–2 6–2 to 8–2 4/90 63 (58–68) 1
BROAD WRITTEN LANGUAGE 491 9–2 8–2 to 10–6 24/90 64 (60–68) 1
Spelling 498 10–0 9–1 to 11–0 17/90 73 (69–77) 4
Writing Samples 497 10–1 8–6 to 12–7 49/90 80 (75–85) 9
Sentence Writing Fluency 478 7–7 6–11 to 8–7 14/90 51 (43–59) <0.1
BASIC WRITING SKILLS 500 10–5 9–4 to 11–11 30/90 76 (72–79) 5
Spelling 498 10–0 9–1 to 11–0 17/90 73 (69–77) 4
Editing 503 11–2 9–8 to 13–6 47/90 82 (78–87) 12
WRITTEN EXPRESSION 488 8–6 7–7 to 10–1 28/90 64 (59–69) 1
Writing Samples 497 10–1 8–6 to 12–7 49/90 80 (75–85) 9
Sentence Writing Fluency 478 7–7 6–11 to 8–7 14/90 51 (43–59) <0.1
ACADEMIC SKILLS 487 9–2 8–5 to 10–1 6/90 64 (62–66) 1
ACADEMIC APPLICATIONS 490 9–0 8–1 to 10–5 21/90 68 (65–71) 2
ACADEMIC FLUENCY 472 8–2 7–8 to 8–10 1/90 52 (48–56) <0.1
ACADEMIC KNOWLEDGE 497 9–6 8–2 to 11–4 33/90 73 (70–77) 4
Science 506 11–7 9–7 to 15–8 69/90 89 (83–94) 23
Social Studies 495 8–11 7–10 to 10–5 16/90 69 (64–75) 2
Humanities 491 8–6 7–2 to 10–3 21/90 71 (66–76) 3
BROAD ACHIEVEMENT 483 8–8 8–0 to 9–7 5/90 58 (56–60) 0.3

Adaptive Behavior Assessment System, 2nd Edition (ABAS-II) Parent Form (Ages 5–21)

Skill Area Composite (95% CI)* Scaled Score* Percentile Rank
General Adaptive Composite 68 (65–71) 2
Conceptual Composite 61 (57–65) 0.5
Communication 5 5
Functional Academics 1 < 1
Self-Direction 3 1
Social Composite 66 (60–72) 3
Leisure 3 1
Social 3 1
Practical Composite 82 (78–86) 12
Community Use 1 < 1
Home Living 10 50
Health and Safety 5 5
Self-Care 12 75
*Mean = 10, SD = 3. CI = confidence interval.

Psychoeducational Evaluation

Name: Robert Maddox
Date of Birth: March 3, 1993
Age: 22
Date of Testing: October 27, 2015
School: University of Fresno
Grade: 16.1

Reason for Evaluation

Robert requested a follow-up psychoeducational evaluation so that he could apply for accommodations on the Graduate Record Exam (GRE). Because Robert's previous evaluation was quite comprehensive, this testing was intended as a brief reevaluation of his status as a person with a learning disability and to ascertain his need for accommodations on high-stakes exams.

Background Information

Robert was evaluated as a high school junior in 2009 by this evaluator and Earnest Crawford, Ph.D., psychologist. The findings of both evaluations were that Robert qualified as gifted based on his Full Scale IQ on the Wechsler Intelligence Scale for Children–IV and his General Intellectual Ability (GIA) on the Woodcock-Johnson III Normative Update Tests of Cognitive Abilities. Test results also indicated that Robert had learning disabilities that significantly affected his reading fluency and reading comprehension.

Robert is a senior at the University of Fresno, where he is majoring in Optical Science and Engineering. He stated that he chose this major based on his interest in the field as well as the fact that the work does not require a lot of reading. Robert would like to pursue a graduate degree in this field at the University of California at Irvine. Robert is currently registered with the University of Fresno Disability Resource Center (DRC). Since his enrollment, he has been allowed extra time on exams, which he considers both necessary and sufficient to allow him to complete his exams. His accommodation history, provided by the DRC, indicates that he has consistently used this accommodation until this school year. Robert explained that if he takes his exams in the DRC, where he is allowed extra time, he misses the clarification of questions and other information that the professor may give to the class during the exam. Consequently, he now takes his exams in the classroom. Because the exams are mostly math-related, he usually does not need extra time.

Robert reported that he has had vision and hearing exams within the last two years, with no problems indicated.

Behavioral Observations

Robert is as an extremely pleasant, articulate, and self-aware young man. He met with this evaluator for one 4-hour session, with breaks as needed. In an interview before testing, Robert was candid about his difficulties with reading speed and comprehension and how these difficulties impact his work in his classes. He reported that he still reads slowly and, although he can understand what he reads, he has to reread often, which further slows his reading. He appeared to give good, consistent effort on all of the tasks presented.

Tests Administered and Description of Scores

  1. Woodcock-Johnson IV Tests of Cognitive Ability (WJ IV COG)
  2. Woodcock-Johnson IV Tests of Oral Language (WJ IV OL)
  3. Woodcock-Johnson IV Tests of Achievement (WJ IV ACH)
  4. Gray Oral Reading Tests, Fifth Edition (GORT-5): Form B

All of the tests administered were appropriate for Robert's ethnicity and primary language. The following were considered and ruled out as contributing factors to the referral concerns: educational disadvantage, environmental deprivation, economic factors, and primary language of the home.

The tests of the WJ IV were scored according to grade norms. Accordingly, Robert's abilities and achievement are compared with those of other college seniors. Robert's performance on the tests of the WJ IV are described as standard scores (SS), by verbal labels for SS ranges, and as percentile ranks (PR). His scores on the GORT-5 are described as scaled scores (ScS) and percentile ranks. Standard scores used in this report have a mean of 100 and a standard deviation of 15. Scaled scores have a mean of 10 and a standard deviation of 3.

Verbal Label Very Low Low Low Average Average High Average Superior Very Superior
SS Range <69 70–79 80–89 90–110 111–120 121–130 >130
ScS 1–3 4–5 6–7 8–12 13–14 15–16 17–20
Percentile <2 2–8 9–24 25–75 76–91 92–98 >98

Occasionally, the Relative Proficiency Index (RPI) is used to describe Robert's performance on a test. The RPI indicates the level of success a person is predicted to have on tasks similar to those used in the test when a typical grade-peer would have 90% success. For example, Robert's Calculation RPI of 99/90 indicates that Robert will be able do math computation problems with 99% success whereas a typical college senior would be 90% successful.

Ceiling effect: On 10 of the WJ IV tests, Robert did not reach a ceiling. A test ceiling is the difficulty level past which a person is statistically unlikely to answer any of the questions correctly. Typically, this is delineated by a specified number of consecutive errors or a time limit. If a person does not attain a ceiling, the test has not measured the upper limit of his or her ability or achievement in the area assessed.

When the scores on a test within a cluster are similar, it is appropriate to discuss the more general ability represented by the cluster. When a significant discrepancy exists between the two tests in a cluster, each test and the reason for the discrepancy will be discussed.

Cognitive Abilities

Strengths

According to the WJ IV COG results, Robert's General Intellectual Ability (GIA) is in the superior range (SS = 123), equal to or above 93% of other college seniors. The tests that make up this cluster (group of tests) span a range of 2 standard deviations and include his weakest cognitive ability. Consequently, the GIA may be an underestimate of his intellectual ability. On four of the seven tests, Robert did not reach a ceiling, indicating that he did not have the opportunity to demonstrate the upper limits of his abilities and knowledge in logical reasoning, spatial relations, and vocabulary knowledge.

Robert's test results indicated a specific strength in fluid reasoning, the ability to solve problems of a type that one has not encountered previously and that require inductive and deductive logical reasoning. His Fluid Reasoning cluster score was in the Superior to Very Superior range (SS 129, PR 97). He made only one error on each of these two tests.

Robert demonstrated a similar strength in working with visual patterns—mentally visualizing images from different perspectives and integrating parts to make a whole (Visualization, SS 124, PR 94, RPI 99/90). He missed only one and two items on each of the subtests of this test, indicating that, again, the test did not assess the upper limits of his abilities.

Additionally, Robert's performance on the Vocabulary cluster was in the Superior range (SS 122, PR 94, RPI 99/90), indicating that he has a broad and flexible vocabulary. Within the Oral Vocabulary test, he easily provided synonyms and antonyms for given words and did not reach a ceiling on either subtest.

Robert's performance on the Perceptual Speed cluster was in the High Average range (SS 115, PR 84, RPI 97/90). These tests assess the ability to rapidly scan symbols to identify visual similarities and differences, as one does when reading.

All of Robert's scores were similar to the scores he obtained when tested 5 years ago.

Weaknesses

Robert's performance on the Long-Term Retrieval cluster, assessing the ability to retrieve information from long-term memory, was significantly lower than his GIA by more than 2 standard deviations (SS 93, PR 32, RPI 87/90). His performance was similar on the Speed of Lexical Access cluster (SS 95, PR 47, RPI 87/90) involving rapid recall from long-term memory of names of pictures and of names of items within a given category. These scores are also similar to those he obtained on testing 5 years ago.

Academic Achievement

Consistent with strong reasoning abilities but relatively slow word retrieval, Robert's application of his academic skills (reading comprehension, math problem solving) was significantly higher than his academic fluency, speed in implementing the more basic skills (i.e., reading rate, rapid retrieval of math facts).

Reading

The WJ IV Letter-Word Identification test assesses recognition of whole words. Robert's standard score on this test was in the Average range (SS 105, PR 64). In contrast, his performance on the Passage Comprehension test was in the Superior range (SS 122, PR 93). Robert did not reach a ceiling on this test. In contrast, Robert's Reading Rate cluster score was in the Low to Low Average range, his lowest score (SS 80, PR 9). His RPI (29/90) indicates that Robert is predicted to be 29% successful on the same type of speeded reading tasks in which other college seniors are 90% successful.

Robert was also administered the GORT-5, a test of reading comprehension, rate, and accuracy. The GORT-5 is normed only by age. Robert's comprehension score on the GORT-5 (ScS 10, PR 50) was significantly lower than on the WJ IV. The discrepancy is likely attributable to the nature of the task. On the WJ IV Passage Comprehension test, Robert read silently a series of one- to two-sentence passages that increased in length and complexity, and demonstrated comprehension by supplying a missing key word. There were no time constraints. On the GORT-5, Robert was instructed to read aloud “as quickly and as carefully” as he could; he could not go back to reread as he normally would or read more slowly for comprehension. His comprehension varied from passage to passage, unrelated to the sophistication of the vocabulary and concepts. Clearly, reading aloud and reading quickly interfered with his comprehension. The GORT-5 Accuracy and WJ IV Letter-Word Identification tests are not comparable; on the GORT-5, any deviation from the print, including repetitions and self-corrections, are considered errors. Robert's Accuracy score was at the low end of the Average range, indicating that 75% of 22-year-olds in the norm sample read with fewer deviations from the print than he did.

Compared with college seniors on the WJ IV and 22-year-olds on the GORT-5, Robert's reading rate earned his lowest scores (WJ IV Sentence Reading Fluency, PR 14; GORT-5 Rate, PR 16). On the GORT-5, as the passages increased in length, vocabulary, and concepts, Robert's reading rate slowed from a raw score of 5 (maximum points) to 0. His performance on both tests were 1 or more standard deviations below his comprehension, and both were significantly lower than his GIA. On the WJ IV Sentence Reading Fluency test, Robert's Relative Proficiency Index (RPI) was 55/90. This indicates that when a typical college senior reads and understand brief, simple sentences at 90% proficiency, Robert would be 55% proficient. This is consistent with Robert's self-evaluation.

Math

Robert's performance on the WJ IV math tests indicated a similar profile of strengths on both computation and the application of math skills to practical problems (Brief Math: SS 118, PR 88), with significantly lower ability in speed of retrieving math facts from memory (Math Facts Fluency: SS 96, PR 41). His RPIs on the Applied Problems and Calculation tests were 99/90 and on Number Matrices, 100/90. This is not surprising, because Robert is majoring in a math-related field. What is surprising is that his RPI in the speed of solving simple math facts was 88/90. Again, Robert did not reach a ceiling on any of the other math tests. Likely his scores would have been higher had there been additional, more advanced problems.

Summary and Conclusions

Test results indicate that Robert has general intellectual abilities at least in the Superior range, the same as or higher than 93% of college seniors. This is likely an underestimate of his intellect because of the ceiling effect found on four of the seven tests that make up the GIA.

Compared with other college seniors of his intellectual ability, Robert demonstrates a significant weakness in retrieval of information from long-term memory, more than 2 standard deviations below his GIA. Additionally, Robert demonstrates a similar weakness in the speed with which he can retrieve specific words from long-term memory. The size of the discrepancy between the GIA and these two clusters is likely to occur in less than 1% of the population of college seniors. These weaknesses in these key cognitive abilities appear to have a significant, negative effects on Robert's ability to comprehend reading material in time-restricted settings. His relatively slow retrieval of basic math facts was further evidence of this difficulty. In addition, on the WJ IV ACH, Robert's performance on the Reading Rate cluster was more than 2 standard deviations below his performance in other reading skills. As evidenced by his variable comprehension on the passages of the GORT-5, Robert's comprehension is impaired when he cannot read slowly and reread as needed.

Recommendation

Robert will need extended time to demonstrate his knowledge on any high-stakes exams that involve reading, such as the GRE. He has needed and used this accommodation throughout his high school and college careers. This accommodation will allow him to demonstrate his aptitude and knowledge without penalizing him for his slow reading speed. He will not require extended time on tests such as math exams that do not require reading.

Scores

Gray Oral Reading Test–5

SS ScS PR Age Equiv. Grade Equiv.
Rate 7 16 14–6 8.4
Accuracy 8 25 17–6 11.7
Fluency 8 25 16–0 10.4
Comprehension 10 50 >18.0 >13.0
Oral Reading Quotient 94 34
ScS: Mean = 10, Standard deviation = 3; SS: Mean = 100, Standard deviation = 15.

In the following tables, clusters are bolded; tests are not.

Tests in which Robert reached the last item without reaching a ceiling are marked with an asterisk (*).

WJ IV Cognitive Abilities: Standard Scores

Very Low <70 Low 70–79 Low Avg 80–89 Average 90–110 High Avg 111–120 Superior 121–130 Very Superior >130
Long-term RetrievalLexical Access Percept. Speed *Fluid Reasoning*Visualization*Vocabulary

WJ IV Academic Achievement: Standard Scores

Very Low <70 Low 70–79 Low Avg 80–89 Average 90–110 High Avg 111–120 Superior 121–130 Very Superior >130
Reading Rate Letter-Word IdentificationMath Facts Fluency *Brief Math *Number Matrices Passage Comprehension

Cognitive and Educational Evaluation

Name: Damon Seldon
Date of Birth: November 25, 1970
Age: 44
Date of Testing: August 1, 2015
Grade: 14
School: Solano Community College

Reason for Referral

Damon referred himself for an evaluation because of his lifelong difficulties with mathematics. Although he reports that he does very well on group projects and public speaking, he has trouble absorbing information in math classes and memorizing information for exams. Damon wanted to gain a deeper understanding of his strengths and weaknesses, and find out whether he has a learning disability that would qualify him for specific accommodations in school to help compensate for his severe difficulties with mathematics.

Background Information

Damon recalls always having some difficulties in school, particularly with the mastery of math concepts. Beginning in second grade, Damon received Speech and Language services for articulation difficulties. Damon has a mild hearing loss and has difficulty hearing certain speech sounds. Beginning in sixth grade and for several years after, he had a math tutor, but he never received special education services. Damon is currently a student at Solano Community College, where he is studying to obtain a degree in social work. He plans to complete Solana in the fall of 2016. Presently, he has completed 37 hours, with a grade point average (GPA) of 2.34. His relatively low GPA is the result of repeated failures in math classes. When Damon finishes his degree at Solano, he plans to attend Snyder State University for a Bachelor's degree in a social science program. Currently, he receives social security disability because of a work-related accident in baggage handling with an airline several years ago. In the last two years, he completed an internship at a local food bank and has continued to volunteer one day a week.

During his school career, Damon has attempted to take Algebra a total of seven times in high school and at Solano and has earned failing grades each time. He noted that even though he failed the final algebra exam in his senior year of high school, his math teacher passed him so that he could graduate and receive a high school diploma. At Solano, Damon has attempted and failed algebra courses in Spring 2013, Fall 2014, and Spring 2015.

In July of 2013, Damon took the math placement test at Solano. He noted that the only problems he got right were the ones on which he could use a calculator. He obtained a score of 30 on Pre-Algebra (41 is passing) and a score of 15 on Algebra (26 is passing). Damon reported that he has learned addition and subtraction but still has little idea about how to solve problems involving multiplication, division, and fractions.

Tests Administered

  1. Woodcock-Johnson IV Tests of Cognitive Abilities (WJ IV COG)
  2. Woodcock-Johnson IV Tests of Achievement (WJ IV ACH) Standard and Extended

The WJ IV tests were scored by age norms.

Test Behavior

Damon was extremely cooperative during a two-and-a-half hour testing session. He was attentive and did not need a break for the entire session.

WJ IV Cognitive Battery

Although many of Damon's scores fell within the Average range, a significant pattern of strengths and weaknesses was noted across the different types of abilities. Damon had average proficiency on tasks that involved knowledge of general information and vocabulary, as well as perceptual speed, the ability to complete simple symbolic tasks quickly. His verbal ability exceeded 53% of his age peers. In contrast, his proficiency was limited on tasks involving memory and reasoning, both critical abilities for success in mathematics. On measures of short-term working memory, Damon's performance exceeded that of only 13% of his peers, and on measures of deductive and inductive reasoning, his performance exceeded 22% of peers. Regarding the reasoning tests, Damon explained that the tasks were like those involved in math problem solving and that he just could not understand what he was supposed to do. Damon also had difficulty on a task involving storing and retrieving associations (Visual-Auditory Learning), where his performance exceeded that of only 13% of peers. Memory for associations is an ability that is related to memorization and retrieval of math facts.

WJ IV Achievement

On the WJ IV ACH, Damon obtained Average proficiency on most of the reading and spelling tests. His proficiency was Limited, however, on all measures of mathematics. Damon's math reasoning abilities exceeded only 14% of his age peers. A significant difference existed, however, among his scores on the Calculation test (a measure of computational skills), Number Matrices (a measure of math reasoning), and the Math Facts Fluency test (a timed measure of simple addition, subtraction, and multiplication facts). On the tests of Calculation and Number Matrices, his scores exceeded only 15% and 6% of his age peers, respectively. In contrast, on the Math Facts Fluency, his score exceeded 34% of his age peers. Even though this score was in the Average range, Damon made several errors on simple calculations (e.g., 6 + 7 = 30). On the Calculation test, Damon was able to solve simple addition, subtraction, and multiplication problems but was unable to solve problems that involved division, addition and subtraction of fractions, or simple algebraic equations. On the Number Matrices test, Damon stated: “I don't really understand how to solve these kinds of problems in a box.”

Conclusions

Damon is a hard-working, capable man with verbal abilities and knowledge commensurate with age-peers and a well-established history of extreme difficulties in mathematics, despite tutoring and support. The underlying causes of this problem relate to difficulties in memory and reasoning with abstract concepts. These specific cognitive weaknesses have impacted his ability to memorize and understand mathematical computations and concepts. His history of failure in math courses provides additional support for this conclusion. Damon exhibits a specific pattern of strengths and weaknesses that indicates the presence of a specific learning disability in mathematics.

DSM-5 Diagnosis

  1. 315.1 With Impairment in Mathematics (Dyscalculia):
    1. Number sense
    2. Memorization of arithmetic facts
    3. Accurate and fluent calculation
    4. Accurate math reasoning
    5. Current severity: Severe

Recommendations

Postsecondary

  1. Damon should receive a waiver or course substitutions for all math classes. He will be unable to pass these courses even with tutoring.
  2. Damon should provide a copy of this evaluation to the Disability Resource Center at Solano College as documentation of his disability.

Damon is a personable, responsible, hard-working man who is anxious to succeed. With a waiver of math requirements, he should be able to complete his coursework at Solano College and then begin study at Snyder State University.